Exam 6 - Comprehensive Final of all chapters

अब Quizwiz के साथ अपने होमवर्क और परीक्षाओं को एस करें!

The nurse notes that a trauma patient has multiple tangle in the hair. Which of the following actions taken by the nurse is apporiate? Select all that apply A.) Work on tangles to the ends of the hair, then trim with scissors B.) Apply warm water and conditioner C.) Apply detangler as available D.) Use a comb or fingers to work through tangles

B, C, and D

How can the nurse be a patient advocate?

By coordinating care and supporting changes necessary to improve patient conditions and outcomes.

Place the following examples of patient needs in priority order (first to fifth), according to Maslow's hierarchy of needs. A.) Autonomy B.) Compassion for care provider C.) Oxygen Level D.) Ability to perform role functions E.) Physical safety

C, E, B, D, and A.

Which of the following are independent nursing interventions? Select all that apply. A.) Administering an IV antibiotic B.) Cleaning a wound and applying a dressing C.) Providing emotional support D.) Administering oxygen E.) Performing oral care F.) Repositioning the patient in bed

C, E, and F.

The patient tells the nurse that she feels lightheaded when she goes from lying down to sitting up. What further information will the nurse want to obtain to validate this finding?

Checking the patients blood pressure upon position changes.

A group of researchers are conducting research on asthma patients based on Orem's theory. What is more probable topic of research? A.) Balance between stress of asthma and equilibrium B.) Asthma's effect on the 14 basic needs C.) Effect nurse-patient communication D.) Self-administration of inhalation medication

D

The nurse asks the patient to provide more details about her pain experience. This is an example of the nurse specifically using critical thinking standard of? A.) Clarity B.) Accuracy C.) Relevance D.) Precision

D

The nurse recognizes which intervention is not a form of mechanical debridement? A.) Wet to dry dressings B.) Whirlpool baths C.) Wet to damp dressing D.) Enzymatic dressing

D Enzymatic debridement is achieved through the application of topical agents containing enzymes that work by breaking down the fibrin, collagen, or elastin present in devitalized tissue, thus allowing for its removal. Mechanical debridement is a nonselective form of debridement because it not only removes the necrotic tissue, but also can remove or disturb exposed viable tissue that may be in the wound. The main forms of mechanical debridement are wet/damp-to-dry dressings and whirlpools.

The nurse understands "First, do no harm" defines what ethical principle? A.) Beneficence B.) Justice C.) Fidelity D.) Nonmaleficence

D First, do no harm is the colloquial definition of nonmaleficence. Unlike beneficence, which requires actively doing good, nonmaleficence requires only the avoidance of harm. In its simplest form, beneficence can be defined as doing good. To do justice is to act fairly and equitably. Keeping promises or agreements made with others constitutes fidelity.

The nurse is caring for a postoperative patient who has had abdominal surgery and whose wound has completely eviscerated when the nurse walks into the room. In addition to notifying the surgeon, what should the nurse do? A.) Cover the wound with a sterile gauze pad. B.) Cover the wound with a transparent dressing. C.) Put pressure on the wound with a sterile gauze pad. D.) Cover the wound with gauze soaked with normal saline.

D If dehiscence or evisceration occurs, cover the wound with gauze moistened with a sterile normal saline, and notify the surgeon immediately. Putting pressure on the wound could cause further complications. Transparent films are used for autolytic debridement. A gauze pad will allow the wound to become dry and cause further complications.

When the nurse is preparing to provide preoperative teaching to a deaf patient, what action by the nurse is best? A.) Use printed materials. B.) Provide recorded materials. C.) Use a family member to interpret. D.) Provide an interpreter.

D Patients who are deaf or have low English proficiency are entitled to professional interpretation by federal law. Printed material may be helpful but not if the patient has low literacy/low health literacy. Recorded material may be an option is the patient has some hearing and the recordings are amplified. Family members are not used as interpreters.

The nursing student learns that the function of the hypothalamus is to do which of the following? A.) Cause lactation to begin. B.) Produce spermatozoa. C.) Release follicle-stimulating hormone. D.) Release gonadotropin-releasing hormone.

D The menstrual cycle is under the influence of the hypothalamus and gonadotropin-releasing hormone.

A patient's blood pressure is 142/76 mm Hg. What does the nurse chart as the pulse pressure? A.) 28 B.) 42 C.) 58 D.) 66

D The pulse pressure is the difference between the systolic and diastolic blood pressure readings. In this case, 142 − 76 = 66.

True/False: Carrying out standing orders limits the responsibility of the nurse.

False

clinical assessment for williebrand diease

GI bleeding epistaxis gingival bleeding

The nurse is reviewing the laboratory results from a postmenopausal woman being evaluated for a breast mass. What type of metastasis does the increased serum calcium and alkaline phosphatase levels suggest? a. Brain b. Bone c. Lung d. Liver

b. Bone

aplastic anemia secondary?

viral invasion, medications, chemicals, radiation and chemo

When mediators remain localized

wheal-and-flare reaction

Type 1 reactions...

•Smooth muscle contraction •Increased vascular permeability •Vasodilation •Hypotension •Increased secretion of mucus •Itching

Many health care facilities use the fire emergency response defined by the acronym: A.) RACE B.) PASS C.) PACE D.) QSEN

A

sickle cell trait ?

heterozygous

The nurse is caring for a woman who has a cyst in her breast that was found at her recent mammogram. The physician wants to make sure that the cyst is not malignant. Which test will be used to determine this? A.) Needle aspiration with biospy B.) Paracentesis C.) Thoracentesis D.) Fiberoptic endoscopy

A

The nurse knows the following would be classified as a closed wound: A.) A large bruise on the side of the face B.) A surgical incision that is sutured closed C.) A puncture wound on the leg that is healing D.) An abrasion on the leg

A

The patient works in a tailor shop and is having surgery to correct bilateral cataracts. If all of the following are realistic, what is the long-term goal for the patient? A.) Return to his occupation B.) Prevention of ocular infection C.) Independent performance of hygienic care D.) Self-administration of eye drops postoperatively

A

Which of the following actions are appropriate to use during communication with a patient who is hearing impaired? A.) Facing the patient B.) Standing over the patient C.) Turning 3/4 away from the patient D.) Raising the voice significantly.

A

Which of the following actual nursing diagnoses best meet the criteria for diagnostic statement? A.) Impaired active ROM associated with knee and ankle discomfort as evidenced by hesitant, unsteady gait B.) Increased fluid volume associated with loss of body weight. C.) Potential for constipation associated with fluid intake and movement D.) Readiness for learning associated with a lack of knowledge.

A

Which of the following behaviors is associated with defense mechanism of displacement? A.) After having a hard time at work, an individual comes homes and yells at their spouse. B.) The patient refuses to speak to the nurse about her problem. C.) The individual takes on more responsibility at home. D.) The patient does not believe that he has an addiction.

A

Which one of the following interventions is considered independent or nurse initiated? A.) Teaching the patient about the therapeutic diet B.) Giving an enema in preparation of radiological testing C.) Providing analgesics for postoperative discomfort D.) Administering Wound Care

A

The nurse knows which statement indicates an appropriate understanding of ethical practice by the student nurse? A.) "I will be held to the same ethical standards as professional nurses." B.) "I will not be held ethically accountable until I graduate." C.) "My nurse educators are responsible for my ethical standards." D.) "Ethics are not important as a student."

A The Code of Ethics for Nurses is "a succinct statement of the ethical obligations and duties of every individual (not just nurse educators) who enters the nursing profession," the profession's "nonnegotiable ethical standard," and "an expression of nursing's own understanding of its commitment to society." This is a powerful mandate for all nurses to communicate and act professionally to prevent inflicting physical or emotional pain on others while pursuing nursing education and engaging in nursing practice.

The nurse is caring for a patient who has urinary retention resulting from benign prostatic hyperplasia (BPH). The patient requires catheterization in order to drain the urine from his bladder. Which action will the nurse take to facilitate this procedure? A.) Obtain a Coudé catheter for insertion. B.) Attach a leg bag to the catheter prior to insertion. C.) Trim the pubic hair before cleaning the perineal area. D.) Wait until the bladder is full to perform catheterization.

A A Coudé catheter is used when there is narrowing or constriction of the urethra, making insertion of a regular indwelling catheter difficult. The Coudé catheter has a special tip on the end that is designed to facilitate insertion of the catheter through the narrowed urethra caused by BPH. Coudé catheters may need to be placed using a metal wire introducer. Placement using an introducer typically is performed by a provider or the patient's urologist, to avoid damaging urethral tissue. Trimming the pubic hair will not facilitate catheterization. Attaching a leg bag to the catheter prior to insertion is not needed because a bedside colle ction bag will usually be used at first.

The nurse is caring for a patient who has been prescribed warfarin (Coumadin) therapy after being diagnosed with atrial fibrillation. The patient asks the nurse what could happen if the prescription doesn't get filled. What is the nurse's best response? A.) "You could have a stroke." B.) "Your kidneys could fail." C.) "You could develop heart failure." D.) "You could go into respiratory failure."

A A major complication of chronic atrial fibrillation is formation of blood clots within the atria due to sluggish blood flow. Anticoagulation therapy is common to prevent blood clot formation that could travel to the brain, causing a stroke.

A nurse wishes to incorporate an assessment of patient sexuality into all patient encounters but is concerned about appearing inappropriate. What action by the nurse is best? A.)State, "I always ask my patients permission to discuss sexuality. Is this alright?" B.) Wait for the patient to bring the subject of sexuality up to the nurse. C.) Give the patient written material on sexuality, then ask if he/she has questions. D.) Tell patients that if they have any sexual concerns, you would be happy to discuss them.

A A matter-of-fact, organized approach to sexuality will decrease anxiety in both patient and nurse. Stating that all patients are asked about this topic makes it appear to be a normal part of the assessment. The other options put the responsibility for discussing sexuality on the patient when the nurse should be directing the process.

The nurse is caring for a patient with a history of type 1 diabetes. Which assessment finding indicates to the nurse that the patient may not be compliant with the diabetic treatment regimen? A.) The patient is always thirsty and frequently voids very large amounts of urine. B.) The patient's urine is very concentrated with a dark amber color. C.) The patient complains of throbbing flank pain and burning with urination. D.) The patient has urinary hesitancy and difficulty initiating a stream of urine.

A A noncompliant diabetic patient will have elevated blood sugars that cause thirst and polyuria. Concentrated urine indicates dehydration. Throbbing flank pain and burning with urination are indicative of urinary tract infection. Urinary hesitancy and difficulty initiating urine stream are not indicative of elevated blood sugar levels.

The nurse who plans, organizes, delivers, and evaluates nursing care for patients is functioning in what role? A.) Patient care provider B.) Patient advocate C.) Case manager D.) Clinical nurse leader

A A nurse does not have to be a manager to be a leader. Even at the bedside, nurses use leadership skills, although possibly in different ways than a nurse manager. The patient care provider must be able to plan, organize, deliver, and evaluate nursing care for patients. An advocate is someone who supports and promotes the interests of others. The RN acts as a patient advocate during treatment. Although many health care organizations have case managers to aid in moving the patient through the health care system, the bedside nurse also acts as a case manager. One important way a nurse can do this is by beginning discharge planning on admission. The clinical nurse leader (CNL) has a master's degree and certification from the American Association of Colleges of Nursing (AACN) Commission on Nurse Certification.

The nurse assesses a patient's pulse and finds it hard to obliterate with palpation. What action by the nurse is the most appropriate? A.) Assess the patient for fluid volume overload. B.) Assess the patient for fluid volume deficit. C.) Assess the patient's apical heart rate. D.) Assess the patient's pulse deficit.

A A pulse that is hard to obliterate (a bounding pulse) can be caused by fluid volume overload, or overhydration. The nurse should assess for this situation. The other actions are not necessary.

The nurse is caring for a patient who is admitted with a serum sodium level of 120 mEq/L. Which is the most important intervention for the nurse to perform? A.) Perform regular neurologic checks and institute seizure precautions. B.) Encourage the patient to eat foods that are high in sodium. C.) Administer hypotonic IV solutions as ordered by the physician. D.) Assess for signs and symptoms of digoxin (Lanoxin) toxicity.

A A serum sodium level of 124 mEq/L is dangerously low and may cause neurologic problems including seizures, confusion, and weakness. Regular neurologic checks should be performed and the patient should be placed on seizure precautions until the sodium level is corrected. Encouraging the patient to eat high-sodium foods is fine, but it is not as important as the patient's safety. A hypotonic saline solution will further lower the patient's sodium level. Digoxin toxicity is seen with hypokalemia rather than hyponatremia.

The nurse is caring for a patient who has been taking ibuprofen (Motrin) 800 mg TID for the last several months to relieve knee pain from arthritis. Which assessment finding must be reported by the nurse to the provider promptly? A.) The patient has abdominal pain and pale skin. B.) The patient has constipation and takes stool softeners daily. C.) The patient enjoys a glass of wine every Friday and Saturday evening. D.) The patient has gained 15 lb in the last 3 months.

A A side effect of ibuprofen and other NSAIDs is the risk of gastrointestinal bleeding, especially with long-term use. Abdominal pain with pale skin in this patient may be indicative of a bleeding ulcer and should be reported to the provider promptly.

The nurse is working with a patient who has a sexual dysfunction. What statement by the patient indicates progress toward an important goal? A.) "I am beginning to enjoy sex more these days." B.) "I'm glad my partner is understanding of the lack of sex." C.) "I wish I didn't need these pills but I know they are important." D.) "I hope one day to have a sexual partner again."

A According to NANDA, sexual dysfunction occurs when a person has a change in sexual function that the person finds "unsatisfying, unrewarding, or inadequate." To show that a goal has been met, the patient would state that sexual activity is more satisfying, rewarding, or adequate. The patient stating that he/she is beginning to enjoy sex more shows progress toward that goal.

The nurse is caring for a woman who had a right-sided mastectomy 2 years ago. What action by the nurse is most appropriate? A.) Place a sign above the bed: "No blood pressures on the right arm." B.) Place a sign above the bed: "No continuous blood pressures on the right arm." C.) Place a sign above the bed: "Blood pressures in legs only." D.) No specific action is needed for this situation.

A After a mastectomy or after lymph nodes have been removed, the patient should not have blood pressures taken on the operative side. Doing so can cause lymphedema. The nurse communicates this to all staff with a sign stating that no blood pressures are to be taken on the right side. The other actions are not warranted.

A nurse has conducted an Allen's test on a patient and the result was 8 seconds. What action by the nurse is best? A.) Document the findings and continue the assessment. B.) Notify the health care provider immediately. C.) Elevate the patient's arm above the level of the heart. D.) Assess the patient for other signs of circulatory problems.

A After the hand blanches in an Allen's test, when the nurse releases the pressure, normal color should return within 10 seconds. This patient's findings were normal, so the nurse should document the results and continue with the assessment. The other actions are not needed.

The nurse knows which law protects health care professionals from charges of negligence when providing emergency care at the scene of an accident? A.) Good Samaritan Act B.) HIPAA C.) Licensure D.) Living wills

A All 50 states have enacted Good Samaritan laws offering protection for physicians and other health care professionals who provide emergency care at the scene of a disaster, emergency, or accident. Good Samaritan laws protect health care professionals from charges of negligence in providing emergency care if: (1) the care is within the professional's scope of knowledge and standards of care and (2) no fee is received or charged for services. The Health Insurance Portability and Accountability Act (HIPAA) was enacted in 1996 to protect the privacy of health care information. Licensure and certification of nurses seek to ensure professional competence. The laws of each state require graduates of accredited nursing schools and colleges pass the National Council Licensure Examination (NCLEX) before beginning professional practice. A living will specifies the treatment a person wants to receive when he/she is unconscious or no longer capable of making decisions independently.

A patient is found unresponsive and pulseless. The nurse begins cardiopulmonary resuscitation (CPR) and calls for help. When help arrives, the nurse should take on which role? A.) Autocratic leader B.) Democratic leader C.) Laissez-faire leader D.) Bureaucratic leader

A Although autocratic leadership is a strict form of leadership, it is useful in crisis situations. A nurse may act as an autocratic leader when taking charge after a patient is found unresponsive. In this situation, it is helpful to have a leader who takes control and directs other members of the health care team. Democratic leaders may see themselves as equals with other team members and may consult with other nurses, exhibiting a democratic form of leadership. This style of leadership can be used in unit council meetings where nurses collaborate to identify solutions to common problems. A nurse in a leadership position who uses the laissez-faire style of leadership assigns patient care and expects all team members to set goals for the day and manage their time to complete the assignment. Successful implementation of this leadership style in nursing requires a highly efficient and reliable staff, such as seen in some specialized OR nursing teams with a history of working together on a set type of cases. The bureaucratic leader relies on policies and procedures to direct goals and work processes. The nurse using bureaucratic leadership tends to relate impersonally to staff and exercises power on the basis of established rules.

The nurse is caring for a patient who had a colonoscopy earlier that day. The patient states that he still feels very bloated after the procedure. What is the best action of the nurse? A.) Assist the patient to ambulate in the hall. B.) Insert a rectal tube to remove retained flatus. C.) Administer an enema to stimulate peristalsis. D.) Encourage oral intake of fluids and high-fiber foods.

A Ambulation is a good way to promote peristalsis and relieve bloating. An enema should not be used after colonoscopy. A rectal tube is not needed. Eating high-fiber foods soon after colonoscopy may increase gas and bloating.

The nurse is caring for a patient with a peripheral IV who tells the nurse that the IV site is painful and puffy. What is the nurse's best action? A.) Discontinue the IV and start another line in the other arm. B.) Aspirate to check for blood return and flush the IV with sterile saline. C.) Clean the IV site with chlorhexidine and apply a new sterile dressing. D.) Change the IV tubing and administer prescribed pain medication.

A An IV site that is puffy and painful should be discontinued promptly because the fluid has infiltrated outside the vein and is causing localized irritation. The IV should be restarted in the other arm if possible. The other actions are inappropriate.

The nurse is caring for a patient whose immune system is destroying red blood cells at a very rapid rate. Which test result will the nurse expect to see in the patient's chart as a result? A.) Bilirubin level 4 mg/dL B.) Platelet count 450,000/mm3 C.) Serum uric acid level 1.7 mg/dL D.) Partial thromboplastin time 45 seconds

A An elevated bilirubin level is the result of increased red blood cell destruction. Normal bilirubin levels are 0.3 to 1.0 mg/dL. Increased platelet count, decreased serum acid level, and decreased partial thromboplastin times are not indicative of increased red blood cell destruction.

The nurse knows which is the best pain medication option for a patient to manage severe long-term cancer pain at home? A.) Fentanyl (Duragesic) 50 mcg transdermal patch q 72 hours B.) Meperidine (Demerol) 50 mg IM q 6 hours C.) Hydromorphone (Dilaudid) 0.2 mg q 10 minutes IV via PCA pump D.) Hydromorphone (Dilaudid) 0.08 mg/hour infusion through epidural catheter

A An opioid transdermal patch is the best pain management option for home use with patients who have long-term, severe cancer pain as no injections are required and the opioid is slowly released over 72 hours. Epidurals and PCA pumps are intended for hospital use. Frequent IM injections require nursing administration, are not comfortable for the patient and are not optimal for chronic long-term pain.

The nurse is caring for a patient who is slow to awaken following general anesthesia. The patient is breathing spontaneously but is minimally responsive and having difficulty maintaining a patent airway. Which intervention is the most appropriate for the patient to improve oxygenation? A.) Insert an oral airway. B.) Lower the head of the bed. C.) Turn the patient's head to the side. D.) Monitor the patient's pulse oximetry.

A An oral airway will prevent the patient's tongue from falling back and occluding the airway. Lowering the head of the bed will only increase airway occlusion and risk of aspiration. Turning the patient's head to the side will not clear the back of the patient's tongue from the airway. Monitoring the patient's pulse oximetry will not improve oxygenation or clear the airway.

The public health nurse volunteers for a missionary group caring for Ebola patients in Africa. The nurse is reviewing the data using analytic epidemiology methods. What information does the nurse collect as the priority? A.) Cultural norms in burial practices B.) Genetic variables in disease acquisition C.) Statistics related to incidence and prevalence D.) Autopsy data on direct cause of death

A Analytic epidemiology hypothesizes why a disease is occurring in a community and looks at cultural practices, nutrition, and extrinsic factors such as the environment for links. Genetic variables and direct cause of death data are more related to epidemiology.

The nurse is caring for a patient who is to have a noncontrast MRI scan performed. Which assessment finding leads the nurse to report that the patient may not be able to have the test? A.) The patient has an implanted insulin pump. B.) The patient is breastfeeding her newborn infant. C.) The patient is severely allergic to iodine and latex. D.) The patient has profound hearing loss.

A Any metal implants are a contraindication for an MRI scan because the scan uses powerful magnets. Insulin pumps often contain metal that can react with the strong magnets in the MRI machine. Breastfeeding is not a contraindication to MRI because there is no radiation exposure. No latex or iodine is used during MRI testing. Profound hearing loss will not be a problem, although MRI scanning is very loud.

In determining patient goals, the nurse should complete which action? A.) Allow patients to identify what is most important to them. B.) Take the lead and determine what is best for the patient. C.) Focus on health promotion and staying healthy. D.) Explain the importance of avoiding complications.

A As health care educators, nurses should allow patients to identify what is most important to them. If a newly diagnosed diabetic patient is interested in learning techniques of care that will allow discharge to home rather than to an extended care facility, the patient is more likely to be receptive to learning about self-monitoring blood sugar levels. After the learning goals related to the issues that the patient feels are a priority have been met, the patient may then be able to focus on health promotion and avoiding complications.

A new nurse is delegating care of a chronic, nonsterile wound to a UAP. What action by the new nurse causes the preceptor to intervene? A.) The nurse asks the UAP to assess the wound. B.) The nurse asks the UAP to report increased wound drainage. C.) The nurse asks the UAP to observe changes in dietary intake. D.) The nurse asks the UAP to change the dressing.

A Assessment and evaluation of a patient's skin and wounds, and the effectiveness of the treatment plan, are a nurse's responsibility and cannot be delegated to unlicensed assistive personnel (UAP). UAP should report to the nurse any changes in skin condition or integrity; elevation in temperature; complaints of pain; increased wound drainage or incontinence; and observed changes in dietary intake. Some dressing changes can be performed by UAP in some situations.

The nurse is caring for a patient whose family does not want the patient to be told about the new diagnosis of cancer because of the poor prognosis. Keeping this secret from the patient is in direct conflict with which ethical concepts? A.) Autonomy and veracity B.) Veracity and advocacy C.) Justice and nonmaleficence D.) Confidentiality and justice

A Autonomy, or self-determination, is the freedom to make decisions supported by knowledge and self-confidence. Truthfulness defines the ethical concept of veracity. Supporting or promoting the interests of others or to do so for a cause greater than ourselves defines advocacy. To do justice is to act fairly and equitably. First, do no harm is the colloquial definition of nonmaleficence. Unlike beneficence, which requires actively doing good, nonmaleficence requires only the avoidance of harm. Confidentiality is the ethical concept that limits sharing private patient information.

The nurse is caring for a patient who periodically has small streaks of fresh red blood in the stool. The patient denies abdominal pain or loss of appetite. The nurse identifies what to be the most likely cause of this patient's bleeding? A.) Hemorrhoids B.) Bleeding gastric ulcer C.) Colon polyps D.) Perforated colon

A Bleeding hemorrhoids can lead to small streaks of fresh red blood in the stool. Bleeding gastric ulcer would lead to black, tarry stools as the blood is digested. Colon polyps do not cause bleeding.

The nurse is caring for a patient who has a deep leg wound that is badly infected. Which laboratory test results will the nurse expect to find in the patient's chart? A.) Elevated C-reactive protein (CRP) 6.5 mg/dL B.) Decreased serum creatinine 0.8 mg/dL C.) Elevated serum bilirubin 0.5 mg/dL D.) Prothrombin time (PT) 11.5 sec

A C-reactive protein (CRP) is produced by the liver in response to inflammation, tissue damage, and infection. Blood levels of CRP have been used as a marker for inflammatory and autoimmune disorders. The nurse would expect to see an elevated CRP in a patient with an infected wound. Creatinine is an indicator of kidney function, and bilirubin is an indicator of liver function. Prothrombin time indicates clotting ability of the blood, particularly when the patient is taking warfarin (Coumadin).

The nurse is caring for a patient who will be returning to the nursing unit following a cardiac catheterization via the right femoral artery. Which assessment is the highest priority for the nurse to perform when the patient arrives on the unit? A.) Checking the patient's right pedal pulse and warmth of the right leg B.) Checking pulse oximetry and listening to the patient's lung sounds C.) Checking bilateral radial pulses to check for a pulse deficit D.) Estimating the patient's jugular venous pressure

A Cardiac catheterization includes the insertion of a large IV needle into the patient's femoral artery. Occlusion of the femoral artery may develop after the procedure leading to faint or absent pedal pulses and loss of warmth to the right leg. The nurse should check the patient's right pedal pulses and leg warmth to ensure that the femoral artery has not become occluded. The other assessments may be performed once the patient's right leg is found to be warm with strong pulses.

The nurse is caring for a patient who has presented to the ER with chest pain. Which diagnostic test will best indicate if there is significant blockage of important blood vessels that provide oxygen to the heart muscle? A.) Cardiac catheterization B.) Chest x-ray C.) Echocardiogram D.) Electrocardiogram

A Cardiac catheterization includes the use of contrast dye to visualize the coronary arteries and determine blood flow to cardiac muscle. The other tests will not allow the physician to determine which (if any) coronary arteries are occluded.

The nurse is caring for a patient from a different culture. After assessing the patient and formulating the care plan, what action by the nurse is best? A.) Review the care plan for acceptance by the patient. B.) Delegate appropriate tasks to unlicensed assistive personnel. C.) Go over the care plan with the charge nurse. D.) Begin implementing the planned interventions.

A Care plans, with their goals and interventions, should always be validated by the patient. This is especially true when the patient is from a different culture than the nurse. The charge nurse may or may not need to view the care plan, but after validation with the patient, the nurse can begin implementing the plan, including delegating appropriate tasks.

The nurse is caring for a patient with a history of hyperparathyroidism who presents with a serum calcium level of 14.5 mg/dL. What is the highest priority Nursing diagnosis for this patient? A.) Risk for injury related to weakened bones that may easily fracture B.) Lack of knowledge related to need for supplemental calcium in diet C.) Risk for constipation caused by decreased gastrointestinal motility D.) Activity intolerance related to muscle cramping and spasms

A Chronic hypercalcemia can lead to weakened bones as strengthening calcium is removed over time. Pathologic fractures can easily result, so risk for injury is a high priority Nursing diagnosis for this patient. The other Nursing diagnoses apply but are less important than the safety of the patient.

The nurse is caring for a terminally ill patient who is actively dying and refuses to eat anything other than a few bites of ice cream. The patient's family member approaches the nurse and requests that a feeding tube be inserted so that her loved one will not starve to death. What is the best response of the nurse? A.) "Loss of appetite is a natural part of the dying process. Tube feedings would be uncomfortable and cause nausea." B.) "I will contact the provider to obtain an order to insert the tube and start tube feedings." C.) "Intravenous fluids would be more comfortable for the patient than a tube feeding. I will call the doctor to get the order." D.) "I will listen to the patient's abdomen to make sure that bowel sounds are present and try encouraging oral fluids."

A Common physical symptoms at the end of life include anorexia and cachexia. Tube feedings will cause discomfort as the tube is inserted and nausea as the GI tract is given food that it cannot handle. Encouraging oral intake will lead to increased secretions and congestion as well as possible aspiration of fluids. Intravenous fluids will increase congestion and edema. The nurse would educate the family on this part of the dying process.

The nurse is caring for a patient who will undergo colonoscopy testing. Which intervention will the nurse include in the patient's plan of care for the day before the test? A.) Provide the patient with zinc oxide skin barrier cream for the perineal area. B.) Obtain an order for a gentle laxative to be given once the test is completed. C.) Carefully assess the patient's ability to swallow liquids through a straw. D.) Check the patient for allergies to shellfish and iodine-based contrast dyes.

A Complete bowel evacuation is required prior to colonoscopy so that the physician can visualize the interior of the large intestine. The patient will have multiple soft-liquid bowel movements as part of the bowel prep for the test, so skin barrier cream will be helpful to prevent perineal irritation. Laxatives will not be needed after the colonoscopy, and no contrast dyes are used.

The nurse is caring for a patient with severe COPD who is becoming increasingly confused and disoriented. What is the priority action of the nurse? A.) Obtain an arterial blood gas to check for carbon dioxide retention. B.) Increase the patient's oxygen until the pulse oximetry is greater than 98%. C.) Lower the head of the patient's bed and insert a nasal airway. D.) Administer a mild sedative and reorient the patient as needed.

A Confusion and disorientation in a patient with severe COPD may likely be due to carbon dioxide retention. An arterial blood gas should be drawn to determine if this is the case. COPD patients should be kept on low oxygen flow rates whenever possible to avoid impeding the drive to breathe. Lowering the head of the bed will increase the difficulty of breathing as the abdominal contents press on the diaphragm. A sedative will cause respiratory depression and should be avoided.

A nurse is assessing a patient's cranial nerves and notes an abnormal response to testing cranial nerve VI. What action by the nurse is best? A.) Ask the patient about recent facial trauma. B.) Inform the provider immediately. C.) Document findings in the patient's chart. D.) Have the patient frown and lift the eyebrows.

A Cranial nerve VI (abducens) is responsible for outward gaze of the eyes. Abnormal findings could indicate a fracture of the orbit or a brain tumor. The nurse asks the patient questions related to these two conditions. The provider needs to be informed and the nurse must document, but first the nurse conducts a thorough assessment. Frowning and lifting the eyes assesses cranial nerve VII.

The nurse is caring for a patient from a different cultural background. What action by the nurse best demonstrates cultural maintenance? A.) Assist the patient with a healing ritual. B.) Teach the patient a heart healthy diet. C.) Instruct the patient on monitoring blood glucose. D.) Discuss what self-care activities the patient is willing to do.

A Cultural maintenance maintains and preserves relevant cultural care values pertaining to health care. Assisting the patient with a healing ritual important to him/her is an example. Teaching a heart-healthy diet and blood glucose monitoring falls into cultural care repatterning. Discussing what changes the patient is willing to accommodate is an example of cultural care accommodation.

The nurse is ready to analyze the data obtained through a qualitative study. What approach to data analysis should the nurse use? A.) Content analysis B.) Statistical analysis C.) Coding of themes D.) Dissemination

A Data analysis techniques are procedures used to summarize words or numbers and create a meaningful result for interpretation. Qualitative analysis involves content analysis. The qualitative data may contain quotations and require their interpretation. Quotations from study participants support the evidence that is provided by the study. Quantitative analysis involves statistical analysis. Many types of statistical analyses may be performed on the data, and the appropriate technique needs to be applied. This process requires coding of themes and analysis of the narrative content. Dissemination is communication and distribution of the information.

The nurse is providing discharge education for a patient with restless leg syndrome. Which statement by the patient indicates a need for further instruction? A.) "I should avoid all caffeine." B.) "I can do leg massage and knee bends." C.) "Taking magnesium supplements may be helpful." D.) "Taking a walk regularly may be helpful."

A Decreasing caffeine is helpful but does not need to be eliminated. Massaging the legs, walking, or doing deep knee bends may temporarily relieve symptoms. Supplements to correct deficiencies in iron, folate, and magnesium may be helpful.

What action by the nurse is inappropriate regarding denture care? A.) Carrying the dentures to the sink wrapped in a paper towel B.) Placing a towel in the sink and brushing the dentures over the towel C.) Brushing the dentures as the nurse would the teeth of a conscious patient D.) Applying adhesive, then inserting upper and then lower dentures

A Dentures should not be wrapped in a paper towel; they should be placed in the denture cup to carry them to the sink. The towel prevents the dentures from being damaged if the teeth are dropped. The nurse can brush the dentures as she would the teeth of a conscious patient. Apply denture adhesive (if used) and insert the dentures, inserting first the upper and then the lower plates, using 4 × 4 inch gauze.

The nurse is providing discharge education to the patient with diabetes regarding foot care. Which statement by the patient indicates a need for further education? A.) "I can go barefoot outside only in the summer." B.) "I should wear good fitting shoes." C.) "I cannot soak my feet in a hot tub." D.) "I can use lotion on my feet."

A Diabetic patients should not go barefoot outside even in the summer as they often have neuropathy, which decreases the patient's ability to discern touch, especially in the lower extremities. This can lead to foot injuries that can become infected and are slow to heal. The patient should wear good fitting shoes, should avoid extreme temperatures, and can use lotion to keep their skin moist to avoid overly dry skin.

The nurse is caring for a patient who has just completed 2 weeks of IV antibiotics for a severe infection. The patient now has frequent loose watery stools and a low-grade temperature. What is the most likely cause of the patient's new symptoms? A.) Clostridium difficile infection B.) Paralytic ileus C.) Fecal impaction D.) Salmonella food poisoning

A Diarrhea, abdominal pain, and low-grade temperature after completing IV antibiotics are often caused by C. difficile infection.

The new nurse tells the preceptor that since she is not prejudiced against ethnic minorities, they will not be discriminated against while in the hospital. What statement by the preceptor is most appropriate? A.) Discrimination can occur at the societal level. B.) The hospital needs more nurses like her. C.) Prejudice and discrimination are not the same thing. D.) There is always some discrimination against minorities.

A Discrimination can occur at the societal level, so even though this nurse is not prejudiced, patients from ethnic and cultural minorities can still suffer from discrimination. The other answers do not explain how discrimination can occur.

The nurse is caring for a male patient who is having open heart surgery. The patient's chest is covered with thick hair, so the surgical technician begins to shave the patient's skin near the operative site. Which action by the technician requires intervention by the nurse to correct the technique? A.) A straight safety razor and antibiotic foam is used. B.) Disposable electric trimmers are used to trim the hair. C.) Antibacterial soap is used prior to hair removal. D.) Only the hair directly around the surgical site is removed.

A Disposable electric trimmers should be used to remove excess hair from operative sites. Antibacterial soap is commonly used to clean the skin before surgical procedures. Only the hair around the surgical site is removed. A straight safety razor would never be used because small nicks in the skin can occur, increasing infection risk.

Which nurse has committed a serious documentation error? A.) The nurse who documents all medications for assigned patients prior to administration. B.) The nurse who documents medication administration as the medications are given. C.) The nurse who documents assessments as soon as they are completed. D.) The nurse who documents meal intake as meal trays are picked up.

A Documentation must be accurate to provide a realistic view of a patient's condition. Serious documentation errors include: (1) omitting documentation from patient records, (2) recording assessment findings obtained by another nurse or unlicensed assistive personnel (UAP), and (3) recording care not yet provided. Nurses sometimes document that a patient has received medication before its administration; this is a serious violation of the law and becomes a medication error of omission if the nurse is distracted before administering the patient's medication.

The nurse is caring for a patient who is to receive dopamine (Intropin) through the IV line. Which intervention has the highest priority when administering this medication? A.) Check for IV blood return prior to administration. B.) Use a new IV tubing set each time the medication is administered. C.) Document the date, time, and nurse's initials after each dose is administered. D.) Use sterile gloves when drawing up and administering the medication.

A Dopamine is a vesicant and can cause significant irritation to blood vessels and tissues when administered via IV. For this reason, the nurse must ensure that the IV catheter is located correctly in the vein by checking for a blood return prior to administration

When the patient is diagnosed with pertussis, which isolation precaution should the nurse implement? A.) Droplet B.) Airborne C.) Contact D.) Protective

A Droplet precautions are used when known or suspected contagious diseases can be transmitted through large droplets suspended in the air. Contact precautions are used when a known or suspected contagious disease may be present and is transmitted through direct contact with the patient or indirect contact with items in the patient's environment. Airborne precautions are used when known or suspected contagious diseases can be transmitted by means of small droplets or particles that can remain suspended in the air for prolonged periods.

The nurse is caring for a cancer patient with ongoing pain from widespread metastasis to the bones. The nurse notes that the patient's morphine dosage had to be increased to sufficiently manage the discomfort. What is the nurse's interpretation of this assessment finding? A.) The patient became tolerant to the previous morphine dosage. B.) The patient is becoming addicted to the pain medication. C.) The patient has been abusing the prescribed pain medications. D.) The patient may be seeking to end life with an overdose of morphine.

A Drug tolerance is an adaptation to the medication, which eventually leads to less effective pain relief. The patient is requiring higher doses of narcotic pain medication because of this tolerance. This is common when patients require long-term pain medication. Since the patient is taking morphine to control ongoing pain, the patient is not addicted to it. Need for increased morphine dosage is not indicative of drug abuse or a wish to die.

The nurse is ambulating a patient back from the bathroom when the patient begins to have a seizure. Which action should the nurse do first? A.) Lower the patient to the floor if standing. B.) Move sharp or hard objects away from the patient. C.) Turn the patient's head to the side to prevent aspiration. D.) Attempt to place a tongue blade to prevent choking.

A During a seizure, a patient should be protected from injury by first lowering the patient to the ground if standing. The nurse should then place the head on a soft surface and turn it to the side to prevent aspiration and move sharp or hard objects out of the way. The nurse should never attempt to force any object into a seizing patient's mouth.

The nurse is caring for a patient who has had severe acid reflux. Which test will allow the physician to directly check for damage to the esophagus? A.) Esophagogastroduodenoscopy (EGD) B.) MRI scan with contrast C.) Abdominal ultrasound D.) Positron emission tomography (PET) scan

A EGD is performed using a lighted tube that allows for direct visualization of the esophagus, stomach, and upper duodenum. MRI, ultrasound, and PET scanning do not allow physicians to see the esophagus directly.

After general anesthesia is administered, the patient is carefully placed in the prone position. What is the primary consideration of the nursing staff as the patient is positioned? A.) Making sure that the patient's endotracheal tube does not become kinked B.) Ensuring that the patient's head is positioned to prevent cervical nerve injury C.) Carefully taping the patient's eyes shut to avoid corneal abrasions D.) Padding the operating table carefully and keeping linens free of wrinkles

A Ensuring that the patient's respiratory status is not compromised is the priority of all staff during repositioning after general anesthesia has been administered. Positioning and protection of eyes is important but less than protection of the endotracheal tube patency.

The nurse prepares to administer the following medication to the patient. Which instruction will the nurse be sure to give before the patient takes the medication? MS Contin Morphine sulfate Extended release tablets, USP 15 mg CII only A.) "Be sure to swallow the pill whole." B.) "Crush the medication and place the powder in applesauce." C.) "Place the pill under your tongue." D.) "Let the pill slowly dissolve in your mouth."

A Extended release medications must always be swallowed whole without crushing or dissolving the tablet. They are not given sublingually or allowed to dissolve in the mouth.

The nurse is caring for a patient who is scheduled for a needle aspiration and biopsy to rule out cancer. Which Nursing diagnosis is appropriate and important for this patient? A.) Anxiety related to potential for cancer diagnosis depending on biopsy results B.) Impaired health maintenance related to delayed insurance coverage for procedure C.) Powerlessness related to lengthy wait for diagnosis D.) Ineffective coping related to patient stated she is a little nervous about the test results

A Fear is an emotion commonly experienced by patients waiting for diagnostic tests and biopsy results. Impaired health maintenance related to delayed insurance coverage is not a priority diagnosis for this patient at this time. Powerlessness is about the patient's ability to control an outcome and is not related to the wait for test results. The patient statement of feeling a little nervous about the test results is not indicative of ineffective coping.

A home health care nurse has been working with a patient who has the Nursing diagnosis Spiritual Distress. After a few weeks of implementing the care plan, what method is best for the nurse to determine if goals have been met? A.) Ask the patient to what extent he/she feels goals have been met. B.) Ask the patient to rate the distress on a scale of 1 to 10. C.) Assess for objective data to support goal attainment. D.) Determine if the patient thinks the interventions are helpful.

A For a diagnosis with a large subjective component, getting the patient's feedback on goal attainment is best. There may be no objective data the nurse can use to rate goal attainment. Using a scale can be a part of the evaluation, but the patient's determination is best.

The nurse is caring for a patient who will be using a hydromorphone (Dilaudid) PCA analgesia pump following surgery. Which intervention is the highest priority for the nurse to include in the patient's care plan related to this pump? A.) Assess the patient's respiratory status frequently after PCA pump started. B.) Review patient's medication profile to check for interactions with hydromorphone. C.) Teach the patient how to use PCA pump when the pain level is still tolerable. D.) Keep naloxone (Narcan) available at the bedside in case of respiratory depression.

A For patient safety, the nurse would check the patient's respirations frequently after the pump has been initiated due to possible respiratory depression. Reviewing the medication profile would occur prior to initiating the pump. Teaching the patient how to use the pump is important, but not the priority. Naloxone should be close by to treat respiratory depression but monitoring the respirations frequently would hopefully prevent depression.

The nurse has established a teaching plan including goals and identifies this type of education is termed by what term? A.) Formal teaching B.) Informal teaching C.) Psychomotor teaching D.) Affective teaching

A Formal patient education is delivered throughout the community in the form of media, in a variety of educational and group settings, or in a planned, goal-directed, one-on-one session with a patient in the acute care setting. Informal education is usually learner or patient directed. The psychomotor domain incorporates physical movement and the use of motor skills in learning. Teaching the newly diagnosed diabetic how to check blood sugar is an example of a psychomotor skill. Affective domain learning recognizes the emotional component of integrating new knowledge. Successful education in this domain takes into account the patient's feelings, values, motivations, and attitudes.

The nurse has established a teaching plan including goals. This type of education is termed: A.) Formal teaching B.) Informal Teaching C.) Psychomotor Teaching D.) Affective Teaching

A Formal patient education is delivered throughout the community in the form of media, in a variety of educational and group settings, or in a planned, goal-directed, one-one-one session with a patient in the acute care setting. Informal education is usually learner or patient directed. The psychomotor domain incorporates physical movement and the use of motor skills in learning. Teaching the new diagnosed diabetic on how to check their blood sugar is an example of a psychomotor skill. Affective domain learning recognizes the emotional component of integrating new knowledge. Successful education in this domain takes into account the patient's feelings, values, motivations, and attitudes.

The nurse is caring for a patient whose ABG results reveal the following: pH 7.56, PaCO2 32 mm Hg, HCO3 42 mEq/L, PaO2 90 mm Hg. Which condition will the nurse expect to see in the patient's chart as the underlying cause of these results? A.) Gastroenteritis with severe nausea, vomiting, and diarrhea B.) Widespread tissue ischemia caused by cardiogenic shock C.) Respiratory failure caused by pneumonia with pleural effusions D.) Hyperventilation after a panic attack

A Gastroenteritis with nausea, vomiting, and diarrhea will lead to a metabolic alkalosis resulting from loss of electrolytes and acids through emesis and loose stools. Metabolic alkalosis features the elevated pH of 7.56, elevated HCO3 42 mEq/L, and normal PaCO2 of 32 mm Hg. Widespread tissue ischemia would lead to metabolic acidosis with low pH resulting from release of lactic acid from the tissues. Respiratory failure leads to a respiratory acidosis with a low pH and elevated PaCO2 level. Hyperventilation leads to respiratory alkalosis with an elevated pH and elevated HCO3 level.

A nurse is discharging a patient and is planning on what material to give the patient to take home. What action by the nurse is best? A.) Assess the patient's ability to read and understand. B.) Determine if the patient wants to take written material home. C.) Give the patient the same material as other patients get. D.) Ask the patient if he/she has a need for written material.

A Health literacy in an important concept in health. If the patient cannot read or comprehend written material, it will be of limited use. The nurse first assesses the patient's ability to read and comprehend written material before choosing the material with which to send him/her home. Patients may or may not realize what they need for discharge, if anything. Giving the patient the same material other patients get does not acknowledge their need for holistic and individualized care.

The nurse is caring for a patient who has diabetes. The patient reports compliance with the medical regime. Which test result indicates to the nurse that the patient has not been compliant with the treatment plan? A.) Hemoglobin A1c 16% B.) Random blood sugar (RBS) 112 mg/dL C.) Lactate dehydrogenase (LDH) 55 units/L D.) Erythrocyte sedimentation rate (ESR) 14 mm/hr

A Hemoglobin A1c (Hgb A1c), or glycosylated hemoglobin, testing evaluates blood sugar levels over a period of 2 to 3 months This blood test is performed to provide the primary care provider (PCP) with information about long-term blood sugar control. The normal value of Hgb A1c in patients without diabetes is 4% to 5.9%. The American Diabetes Association (2016) states that diabetes is diagnosed for Hgb A1c levels greater than 6.5%. A higher level indicates that the patient has had poor blood glucose control during the past few weeks, and increases the patient's risk of long-term complications from hyperglycemia. The other tests are not related to long-term diabetes control.

The nurse knows that the antigen-antibody reaction is an example of what type of immunity? A.) Humoral B.) Cellular C.) Innate D.) Passive

A Humoral immunity is a defense system that involves antibodies and white blood cells that are produced to fight antigens. Cellular immunity involves defense by white blood cells against any microorganisms that the body does not recognize as its own. The innate (nonspecific) immune system provides immediate defense against foreign antigens. Passive immunity occurs when a person receives an antibody produced in another body.

The nurse is assisting a patient to insert contacts and a contact is dropped. What action should occur next? A.) Moisten the finger with lens solution and gently touch it to pick it up. B.) Moisten the contact lens with tap water and pick it up. C.) Pick it up and insert the contact lens. D.) Discard the contact lens.

A If a lens is dropped, do the following: (1) moisten a finger with the lens solution, and then gently touch the lens with the moistened finger to pick it up. (2) Clean, rinse, and disinfect the lens to avoid a potential eye infection from any microorganisms that might have adhered to the lens. The contact lens does not need to be discarded and tap water should not be used for contact lens.

The nurse knows which description would be classified as a closed wound? A.) A large bruise on the side of the face B.) A surgical incision that is sutured closed C.) A puncture wound that is healing D.) An abrasion on the leg

A In a closed wound, as seen with bruising, the skin is still intact. An open wound is characterized by an actual break in the skin's surface. For example, an abrasion, a puncture wound, and a surgical incision are types of open wounds.

A nurse has been asked to care for a patient who is an inmate from a nearby prison. During shift report, the nurse asks, "Why was the man convicted and imprisoned?" Another nurse responds that this is not important since nurses are required to provide compassionate care for all people in all circumstances. The responding nurse has displayed what concept? A.) Beneficence B.) Advocacy C.) Confidentiality D.) Autonomy

A In its simplest form, beneficence can be defined as doing good. Nurses demonstrate beneficence by acting on behalf of others and placing a priority on the needs of others rather than on personal thoughts and feelings. The ethical concept of beneficence necessitates providing care for the prisoner without reproach and provide compassionate care for all people in all circumstances. Supporting or promoting the interests of others or doing so for a cause greater than ourselves defines advocacy. Confidentiality is the ethical concept that limits sharing private patient information. Autonomy, or self-determination, is the freedom to make decisions supported by knowledge and self-confidence.

The nurse is visiting a patient with cardiac disease who has been experiencing increased episodes of shortness of breath when exercise is attempted. The nurse is concerned that the patient's decrease in activity may lead to which outcome? A.) Orthostatic hypotension B.) Increase risk of heart disease C.) Loss of short-term memory D.) Worsening shortness of breath

A Inactivity in patients with cardiopulmonary disease can lead to an unsafe drop in blood pressure with position changes, or orthostatic hypotension. The patient already has heart disease. Loss of short-term memory is not related to the shortness of breath. The lack of activity is not likely to worsen the shortness of breath; improving activity level may help things eventually.

The nurse is caring for a postoperative patient who has a history of COPD. What is the priority Nursing diagnosis for this patient? A.) Ineffective airway clearance B.) Readiness for enhanced knowledge C.) Risk for delayed surgical recovery D.) Activity intolerance

A Ineffective airway clearance is the priority diagnosis for the COPD patient undergoing surgery because the patient is at high risk for bronchoconstriction, increased mucus, and ineffective cough, which may easily become worse after the patient has received general anesthesia. The other Nursing diagnoses are applicable to the patient but are not as high priority.

The nurse uses what term to identify a disease-causing organism? A.) Pathogen B.) Normal flora C.) Germ D.) Microorganism

A Infectious agents include any disease-causing agent and are called pathogens. They include bacteria, fungi, viruses, and parasites. Normal flora is a group of non-disease-causing microorganisms that live in or on the body. Germ is a term used for microorganism. A microorganism is bacteria, fungi, or protozoa.

The nurse leader recognizes that to deliver quality care, focus needs to be placed on which participant? A.) Patient B.) Self C.) Other staff members D.) Health care provider

A It is important for nurse leaders to be focused on the patients rather than themselves to deliver good patient-focused care. Nurses must desire to improve the status quo to provide higher levels of quality in the care delivered. These qualities are also discussed in other works concerning effective managers (Delgado & Mitchell, 2016; Feather, Ebright, & Bakas, 2015).

The nurse is conducting a literature review to determine the statistical results of all related studies and identifies t his to be what type of review? A.) A meta-analysis B.) An integrative literature review C.) A systematic review D.) Grounded theory research

A Literature reviews include scholarly analyses of research. A meta-analysis merges statistical results from related studies to discover similarities and differences in their findings. An integrative literature review synthesizes research findings and formulates ideas about future research. A systematic review of the literature provides a comprehensive, unbiased analysis using a strict scientific design to select and assess each of the studies. Grounded theory research derives theories from the data collected in studies.

The nurse has been involved sexually with a patient. The nurse manages becomes aware of this situation and tells the nurse this behavior is a which type of crime? A.) Malpractice B.) Libel C.) Slander D.) Battery

A Malpractice may occur when a professional such as nurse acts unethically, demonstrates deficient skills, or fails to meet standards of care required for safe practice. Examples of these types of malpractice include engaging in sexual activity with a patient and administering penicillin to a patient with a documented penicillin allergy, resulting in the patient's death from a severe allergic (anaphylactic) reaction. Written forms of defamation of character are considered libel. Broadcasting or reading statements aloud that have the potential to hurt the reputation of another person is considered libel. Oral defamation of character is slander. Actual physical harm caused to another person is battery.

The nurse knows which skill does not require the use of sterile technique? A.) NG tube insertion B.) Foley catheterization C.) Tracheostomy care D.) PICC line insertion

A NG tube insertion requires a clean, not sterile, technique as the gastrointestinal tract is not sterile. Use strict asep tic technique when inserting an intravenous (IV) or Foley catheter and when performing suctioning of the lower airway.

The nurse is caring for a woman who has a cyst in her breast that was found at her recent mammogram. The physician wants to make sure that the cyst is not malignant. Which test will be used to determine this? A.) Needle aspiration with biopsy B.) Paracentesis C.) Thoracentesis D.) Fiberoptic endoscopy

A Needle aspirations are procedures that are used to remove fluid and tissue for testing. A biopsy involves removing a larger collection of cells, as in a tumor or mass, and may be used to detect cancer in the skin, breast, or liver. Paracentesis is drainage of fluid from the abdomen, and thoracentesis is drainage of fluid from the pleural cavity. Fiberoptic endoscopy allows the physician to see inside the upper and/or lower GI tract.

The nurse is caring for a patient who is headed to the operating room for abdominal surgery. Which goal is appropriate for the Nursing diagnosis risk for perioperative positioning injury? A.) Patient will deny numbness or tingling in extremities after surgical procedure. B.) Patient will maintain urine output of at least 30 mL/hr during and after surgery. C.) Patient will maintain elastic skin turgor as well as moist tongue and mucus membranes. D.) Patient will have no emesis and deny nausea following arousal from general anesthesia.

A Numbness and tingling of extremities may be indicative of injury following improper positioning for surgery. The other goals do not relate to positioning injury but relate to Nursing diagnoses of fluid balance and nausea.

The nurse manager is developing a training guide and identifies which organization that is the best for resources to help develop guidelines to prevent exposure to hazardous situations and decrease the risk of injury in the workplace? A.) OSHA (Occupational Safety and Health Administration) B.) CDC (Centers for Disease Control and Prevention) C.) QSEN (Quality and Safety Education for Nurses) D.) NIOSH (National Institute for Occupational Safety and Health)

A Occupational Safety and Health Administration (OSHA) was established in 1970 to provide employers with guidelines for preventing exposure to hazardous chemicals and hazardous situations and reducing the risk of injury in the workplace. The CDC is the Centers for Disease Control and Prevention and provides information to address exposure to infectious diseases. QSEN, or the Quality and Safety Education for Nurses, was funded by the RWJ to focus on preparing nurses of the future with the knowledge, skills, and attitudes to advance quality and safety on the job. NIOSH, or the National Institute for Occupational Safety and Health, is a federal agency within the CDC that was established to conduct research and recommend interventions for the prevention of work-related injury and illness.

The nurse understands ongoing evaluation of patient education occurs by which team member? A.) Each member of the health care team who provides teaching B.) The nurse who evaluates the patient's physical abilities C.) The patient stating that he understands the instruction D.) Not allowing review from the provider so the focus remains forward

A Ongoing evaluation of patient education occurs by each member of the health care team who provides teaching according to the patient's teaching plan. Having the learner repeat what has been learned can help the nurse evaluate the teaching plan and adjust the plan for future patient education sessions. Future sessions should review what was learned previously and continue to add to what has been taught. Health care team members can view documentation on the electronic health record (EHR) before beginning an education session to determine the patient's progress in meeting educational goals.

The nurse leader is conducting a staff meeting. During the meeting, staff members have verbalized dissatisfaction with the staffing pattern created by the nurse leader. The nurse listens intently as the staff come up with other options. The staff members recognize that the nurse leader is demonstrating which quality? A.) Openness B.) Integrity C.) Dedication D.) Magnanimity

A Openness refers to the leader's ability to listen to other points of view without prejudging or discouraging them. An effective leader considers others' opinions with an open mind because a wider variety of solutions to problems is offered. Openness by the nurse leader encourages creative solutions by providing an environment in which people feel comfortable "thinking outside the box." Integrity is the quality of having clear ethical principles and aligning one's actions with the stated values. Dedication is the ability to spend the time necessary to accomplish a task. Magnanimity means giving credit where credit is due. Good leaders reflect the work and success of accomplishing a goal by crediting those who helped reach it.

A patient who claims to be very involved in church is near death. What action by the nurse is best? A.) Get permission to contact the religious leader. B.) Allow the family to stay at the patient's bedside. C.) Call the hospital chaplain to come to the bedside. D.) Ask if the patient and family want to pray.

A Organized religions use rituals to mark important life events such as birth, marriage, and death. This patient would most likely want end-of-life rituals as practiced in his/her church. The nurse's best action is to contact the religious leader (with permission) of that church or institution. Allowing the family to remain at the bedside is important but not the best option to care for the patient's spirituality needs. The hospital chaplain is a valuable resource, but the patient's own religious leader would be better. Praying with the family is always acceptable, but it is best to let the family take the lead in prayer.

The nurse is caring for a patient who has a history of congestive heart failure. The nurse includes the diagnosis hypervolemia in the patient's care plan. Which goal statement has the highest priority for the patient and nurse? A.) The patient's lung sounds will remain clear. B.) The patient will have urine output of at least 30 mL/hr. C.) The patient will verbalize understanding of fluid restrictions. D.) The patient's pitting pedal edema will resolve within 72 hours.

A Oxygenation is the highest priority for the patient with congestive heart failure and hypervolemia. Keeping the patient's lungs clear is the most important goal for the nurse to consider when caring for this patient.

The nurse is caring for a patient who developed a pulmonary embolism after surgery. Which goal statement is the highest priority for the nurse to include in the patient's care plan for the diagnosis impaired gas exchange r/t impaired pulmonary blood flow from embolus? A.) The patient will maintain pulse oximetry values of at least 95% on room air. B.) The patient will verbalize understanding of ordered anticoagulants. C.) The patient will report chest pain of no greater than 3 on a 1 to 10 scale. D.) The patient will ambulate 50 feet in hallway without shortness of breath.

A Oxygenation is the most important human need, so adequate oxygenation of tissues as evidenced by pulse oximetry values of at least 95% on room air is the highest priority goal. The other goals may be addressed once the oxygenation goal has been met.

Which assessment question helps the nurse determine the character of the patient's pain? A.) "What does the pain feel like?" B.) "When did the pain first start?" C.) "What interventions make the pain better?" D.) "Is there any pattern to when the pain occurs?"

A Pain character should be assessed using questions to learn more about what the pain feels like. Examples like stabbing, aching, burning may be used so that patients can understand what the nurse is requesting. Onset is determined by asking when the pain started. Exacerbating/relieving factors are determined by asking which interventions make the pain better. Time course is determined by asking if there is a pattern to when the pain occurs.

The nurse is caring for a patient with advanced colon cancer. The patient is to have surgery to relieve a bowel obstruction that has been causing unrelenting vomiting and abdominal pain. What type of surgery will this patient undergo? A.) Palliative B.) Reconstructive C.) Diagnostic D.) Ablative

A Palliative surgery is performed to alleviate distressing symptoms such as unrelenting pain, nausea, and vomiting. Palliative surgery will not attempt to cure the underlying disease process but will make the patient more comfortable. Reconstructive surgery restores function or appearance of traumatized tissue. A diagnostic surgical procedure establishes or confirms a diagnosis. Ablative surgery removes diseased tissue.

When the nurse is preparing to teach a 5-year-old child postoperative care that will be anticipated after a tonsillectomy, the nurse would incorporate what concept? A.) Use pictures and simple words to describe care to the patient. B.) Teach the parents alone to reduce fear in the patient. C.) Exclude the parents to reduce parental anxiety. D.) Use clear simple explanations to convey information.

A Patient education provided for children should be age specific. Use pictures and simple words for young children. Use clear, simple explanations for school-age children. The patient's age directly affects the instructional methods and materials used. Effective patient education involving a child requires the presence of a parent or caregiver, who is likely the target of teaching. Children should not be excluded from the learning session unless exclusion is deemed appropriate by the parent or caregiver; a presentation using an age-appropriate strategy may complement the instructions reviewed with the adult. The stages of development should be explored as the foundation for the choice of educational materials.

The nurse is caring for a patient who has developed kidney failure. Which test finding leads the nurse to contact the nephrologist and arrange for emergency hemodialysis? A.) Potassium level 6.8 mmol/L B.) Serum creatinine level of 2.8 mg/dL C.) Large amounts of protein in the urine D.) 1500 mL of retained urine in the bladder

A Patients in renal failure often require dialysis to reduce serum potassium levels to less than 5.5 mmol/L . Critically high serum potassium levels can lead to lethal arrhythmias and must be corrected promptly. Patients with advanced renal failure may require emergency hemodialysis if the potassium level does not lower with other methods (insulin and 50% dextrose, kayexalate). An elevated creatinine is consistent with kidney dysfunction. Large amounts of protein in the urine occurs in some diseases. 1500 mL of retained urine requires straight catheterization.

A patient has the Nursing diagnosis Spiritual Distress. What assessment by the patient best indicates that an important goal has been met? A.) Observed praying quietly. B.) Indecisive about treatment. C.) Asks nurse if God exists. D.) Executes living will.

A Patients may have spiritual distress when facing situations that threaten their meaning and purpose in life, such as in the face of a terminal diagnosis. Patients often express anger, frustration, neediness, or crying. The patient who has worked through this situation and is able to pray has best shown goal attainment. Indecision and questioning do not indicate the resolution of this diagnosis. Executing a living will may be an indication of pragmatism.

The nurse is caring for a patient who is to have a cleansing enema. Which assessment finding b the nurse indicates a need to contact the prescriber and question the order? A.) The patient is recovering from a traumatic brain injury. B.) The patient has not had a bowel movement for 3 days. C.) The patient is to have a lower GI series the following morning. D.) The patient had an upper GI series performed the previous day.

A Patients with a traumatic brain injury often have increased intracranial pressure, which can be worsened with enema administration, thus putting the patient at risk for additional neurologic damage. The provider should be contacted and the order should be questioned. Constipation, preparation for a lower GI series, and removal of barium from the colon after upper GI series are all indications for a cleansing enema.

A patient refuses to take his blood pressure medication because "I feel totally fine and don't need it." What action should the nurse take first? A.) Assess the patient's time orientation. B.) Document the patient's noncompliance. C.) Educate the patient about the medication. D.) Warn the patient about possible complications.

A People with a present time orientation typically live in the "here and now" and may not see the benefit of adhering to medical regimens when they are not symptomatic. The nurse should assess the patient's time orientation. Documentation and education are both important but are not likely to secure the patient's cooperation.

The nurse is caring for a patient who has been intubated with an oral endotracheal tube for several weeks. The physicians predict that the patient will need to remain on a ventilator for at least several more weeks before he will be able to maintain his airway and breathe on his own. What procedure does the nurse anticipate will be planned for the patient to facilitate recovery? A.) Placement of a tracheostomy tube B.) Diagnostic thoracentesis C.) Pulmonary angiogram D.) Lung transplantation surgery

A Placement of a tracheostomy tube will secure the patient's airway directly through the trachea, eliminating the need for the endotracheal tube. This will make the patient more comfortable and may allow eating while minimizing damage to the oropharynx from the endotracheal tube.

The nurse teaches the patient what information about polysomnograpy? A.) This is the recording of brain waves and other variables. B.) This is the relay of motor impulse to the hypothalamus. C.) This is the patterns of biological functioning. D.) This is the recording of seizure activity in the brain.

A Polysomnography is the recording of brain waves and other physiologic variables, such as muscle activity and eye movements, during sleep. The reticular activating system (RAS) receives sensory impulses from the spinal cord and relays motor impulses to the thalamus. The circadian rhythms influence patterns of biological and behavioral functions. An electroencephalogram is used to record seizure activity in the brain.

The nurse is caring for a patient who is recovering from septic shock. While in the ICU, the patient developed renal failure. The nurse recognizes which type of renal failure the patient most likely developed? A.) Prerenal B.) Renal C.) Postrenal D.) Mixed

A Prerenal failure occurs as a result of reduction in blood flow to the kidneys, which would occur with septic shock. Causes of prerenal failure include dehydration, vascular collapse, and low cardiac output. Structural issues with the kidneys, from primary glomerular diseases or vascular lesions, result in renal failure. Postrenal failure is related to a mechanical or functional obstruction of the flow of urine.

The nurse is providing care for a patient who demands discharge from the hospital against the physician's orders. What action by the nurse is most appropriate? A.) Have the patient sign an "Against medical advice" form. B.) Follow the guidelines as presented in the code of Academic and Clinical Conduct. C.) Review the ANA's Nursing Code of Ethics for guidance. D.) Permit the patient to leave after an informed consent form is signed.

A Preventing patients from leaving a health care facility at their request may be considered false imprisonment. To prevent health care providers and institutions from being held liable if a patient chooses to leave a facility when physicians and nurses think that it is in the patient's best interest to remain hospitalized, the patient is asked to sign an against medical advice (AMA) form. A signed AMA form documents that the patient has chosen to leave the facility when leaving could jeopardize the patient's condition. The National Student Nurses Association adopted the Code of Academic and Clinical Conduct, in which students agree to "promote the highest level of moral and ethical principles" and "promote an environment that respects human rights, values, and choice of cultural and spiritual beliefs." This document does not apply to the issue at hand. The Code of Ethics for Nurses is "a succinct statement of the ethical obligations and duties of every individual who enters the nursing profession." While this is resource for nurses the described situation requires nurses to follow facility policy. Informed consent is permission granted by a patient after discussing each of the following topics with the physician, surgeon, or advanced practice nurse who will perform the surgery or procedure: (1) exact details of the treatment, (2) necessity of the treatment, (3) all known benefits and risks involved, (4) available alternatives, and (5) risks of treatment refusal. This does not apply to the stated situation.

A nurse is working with a patient who has limited English proficiency. What action by the nurse is best? A.) Use a qualified interpreter. B.) Ask family members to translate. C.) Use drawings and pictures. D.) Speak in simple sentences.

A Qualified interpreters should be utilized when working with non- or limited-English speaking persons. Using a family member to interpret can upset the balance of power within the family, cause embarrassment, and lead to inaccuracies. Using drawings and pictures or speaking in simple sentences is not as effective as using an interpreter.

The nurse recognizes that when conducting a qualitative research study, what concept is a basis of this type of research? A.) Qualitative research is based on a constructivist philosophy. B.) Qualitative research assumes that reality is the same for everyone. C.) Qualitative research is deductive in nature and approach. D.) Qualitative research proceeds from specific facts to generalizations.

A Qualitative research is based on a constructivist philosophy, which assumes that reality is composed of multiple socially constructed realities of each person or group and is therefore value laden, focusing on personal beliefs, thoughts, and feelings. Constructivism assumes an approach that is inductive (Creswell, 2014). Inductive reasoning generalizes from specific facts. Qualitative research usually results in data expressed in words, often in the form of a narrative.

When the nurse is conducting a quantitative research study, what concept is implemented? A.) Quantitative research assumes that reality is fixed and stable. B.) Quantitative research is based on an inductive approach. C.) Quantitative research seeks to gain knowledge through observation. D.) Quantitative research usually produces data in narrative format.

A Quantitative research is based on a postpositivist philosophy, which assumes that reality is objective, fixed, stable, observable, measurable, and value free. Positivism assumes that the approach is deductive in nature, and it seeks to gain knowledge through scientific and experimental research. Quantitative research usually produces data in the form of numbers.

The staff nurse knows that many health care facilities use the fire emergency response defined by which acronym? A.) RACE B.) PASS C.) PACE D.) QSEN

A RACE stands for rescue, alarm, contain, and extinguish. QSEN is the Quality and Safety Education for Nurses. PASS is pull, aim, squeeze, and sweep for fire extinguishers. PACE is not a health care acronym.

The nurse is caring for a patient who has just died in a motor vehicle accident. What is the priority action of the nurse before the patient's family arrives to see the patient's body? A.) Gently wash the body and provide perineal care. B.) Remove the patient's dentures and jewelry. C.) Ensure that the death certificate has been signed. D.) Determine which funeral home will pick up the body.

A Release of bowel and bladder contents often occur at the time of death, and the perineal care is a priority before the family arrives. The body should be gently cleaned to remove blood and debris from the accident. The patient's dentures and jewelry should not be removed from the body. The death certificate does not need to be signed before the family arrives. The family can decide which funeral home will be used and notify the nurse after their arrival.

The nurse knows testing the application of theories in different situations with different populations is what type of research? A.) Applied research B.) Clinical research C.) Basic research D.) Quantitative research

A Research conducted to generate theories is basic research. These theories help to provide explanations for phenomena. Testing theories in different situations with different populations is applied research. Clinical research is conducted to test theories about the effectiveness of interventions. Each type of research contributes to the theoretical base for the practice of nursing. Quantitative research usually produces data in the form of numbers.

The nurse is planning to educate four patients on preventing skin cancer and early warning signs. Which patient is the priority for this education? A.) Adolescent who uses a tanning bed B.) Middle-aged adult who walks for fitness C.) Older woman who sits in the sun for 10 minutes daily D.) Person who works indoors under fluorescent lights

A Research indicates that indoor tanning before the age of 35 increases a person's risk for the deadliest form of skin cancer, melanoma, by 59% with each exposure. The adolescent who tans is the highest priority for this education. The others do not have as high a risk.

The nurse is preparing to conduct a study involving the "postprandial" blood sugars in patients who have received intensive diabetic rehabilitation versus diabetics undergoing "usual care." For the consent to be valid, the nurse would have to carry out which action? A.) Change the language of the consent. B.) Keep explanations to a minimum to reduce stress. C.) Keep potential risks undisclosed. D.) Insist that the participant sign the consent right away.

A Research participants require an explanation of the study in which they are subjects. Any information provided needs to be in a language that is understandable to them. Procedures and the purpose of the study need to be explained. The way subject anonymity and confidentiality will be protected needs to be explained. Any potential harm, including physical or mental discomfort, and possible benefits from participation should be explained. Questions should be answered so that participants fully understand the research and their part in the process. All subjects need to be given time to decide about participation. Study participants are voluntary, may withdraw at any time, or may choose not to complete tasks.

A nurse is caring for a refugee patient who wants the community shaman to perform a healing ritual at the bedside. What action by the nurse is best? A.) Work with the patient to allow the shaman to perform the ritual. B.) Investigate whether the ritual will harm the patient. C.) Check to see if the ritual breaks laws or policies. D.) Offer to call the hospital chaplain instead.

A Rituals are deeply powerful and have great meaning for individuals who practice them. The nurse should work with the patient to facilitate the ritual. Investigating the ritual for patient harm or illegality is ethnocentric; the nurse's first though ts should not be on the potential negative aspects of a deeply meaningful activity. The patient has not requested the chaplain; offering to call the chaplain shows ethnocentrism and lack of respect for the patient. While working to facilitate the ritual, the nurse will discover if any aspect of it might be problematic and can collaborate with the patient and shaman to resolve the situation (e.g., if lighted candles are needed but prohibited by policy and fire code).

The nurse knows that the third phase of evidence-based research involves what action? A.) Searching for evidence and evaluating B.) Assessing the problem C.) Developing a question D.) Performing a critical appraisal

A Searching the database for the evidence begins the third phase. The nurse may need to consult three categories of information resources, which are reviewed in sequential order depending on need and applicability. The categories are general information (background) resources, filtered resources, and unfiltered resources. The second phase of evidence-based research consists of formulating a specific research question so that the nurse can effectively search the literature databases. The first phase of evidence-based research consists of assessing the need for change in practice by identifying a problem. After identifying an article or systematic review resource that seems appropriate to the question, the nurse must critically appraise the information.

A parent confides to the nurse that the parent's 3-year-old son seems to be touching his genitals frequently. What response by the nurse is best? A.) "This is normal behavior at his age." B.) "Why do you think he is doing that?" C.) "Does he complain of burning with urination?" D.) "I'd ignore that behavior; it's attention-seeking."

A Self-exploration of the body is a normal behavior at this age. The other responses are not necessary.

The nurse is acting as a leader in the role of charge nurse and notes that the unlicensed assistive personnel (UAP) on the floor are stressed related to their increased workload. The nurse changes the original planned approach based on the presenting situation. Which theory of leadership is the nurse implementing with this action? A.) Situational B.) Transactional C.) Transformational D.) Autocratic

A Situational theories suggest that leaders change their approach depending on the situation. Transactional leaders use reward and punishment to gain the cooperation of followers. Transformational leaders use methods that inspire people to follow their lead. Transformational leaders work toward transforming an organization with the help of others. The authoritarian or autocratic leader exercises strong control over subordinates

The nurse knows that which patient has a teaching need based on statements by the patient's parents? A.) "My 6-month-old daughter only sleeps with me when she's ill." B.) "I do not put pillows in the bed with my 3-month-old son." C.) "I do not feed popcorn to my 2-year-old." D.) "I have discussed the risks of the 'choking game' with my 16-year-old."

A Small children should never sleep in the bed with others because of the risk of suffocation. The rest of the statements are appropriate. Pillows do present a hazard to a 3-month-old, and popcorn is a choking risk for a 2-year-old. The choking game is a risk to any adolescent.

A patient has been diagnosed with a sexually transmitted disease (STD) and the patient's partner is angry, saying, "She must have cheated on me." What response by the nurse is most appropriate? A.) "This infection may have been present for a long time." B.) "You need to be tested for this disease too." C.) "Yes, you're right; if you don't have the STD, she cheated." D.) "Now, now, getting angry will not help anything."

A Some STD symptoms may go unnoticed for a long time. Telling the partner to get tested as well without further explanation is likely to cause defensiveness. The nurse has no way of knowing if the patient "cheated" on the partner or not. Telling the partner not to get angry is dismissive of his/her concerns.

When planning interventions for a community, what action by the nurse is best? A.) Involve community leaders in planning. B.) Create a plan of action addressing priorities. C.) Determine what resources are available. D.) Attempt to find funding for the plan.

A Stakeholders need to be involved in planning to ensure buy-in from the community. The stakeholders could be community or business leaders. The other actions are important, but if the community leaders are not committed to the plan, the plan is unlikely to work.

The nurse administered 100 mcg sublingual fentanyl spray (Subsys) at 10:00 a.m. to a patient experiencing severe breakthrough pain. At what time will the nurse ask the patient if pain relief was obtained? A.) 10:30 a.m. B.) 11:00 a.m. C.) 11:30 a.m. D.) 12:00 noon

A Sublingual pain medications should be working well 15 to 30 minutes after administration, so the nurse should reassess the patient's pain at 10:30 a.m.

A nurse assesses a patient's radial pulse rate to be 110 beats/min and regular. What action by the nurse is best? A.) Assess the patient for causes of tachycardia. B.) Take an apical heart rate and compare the two. C.) Document the findings in the patient's chart. D.) Notify the patient's health care provider.

A Tachycardia (rapid heart rate) is often caused by factors such as pain, anxiety, fever, or fluid volume alterations. The nurse should assess the patient thoroughly for possible causative factors. Since the pulse is regular, there is no reason to take an apical pulse. The findings should be documented, but the nurse needs to do more. The provider may or may not need to be notified, depending on the outcome of the nurse's assessment.

The nurse correctly identifies which patient as having the greatest risk for infection? A.) An 80-year-old male with an enlarged prostate B.) A 24-year-old female long-distance runner C.) A 50-year-old obese male D.) A 40-year-old sexually active female

A The 80-year-old male has more risk factors because he is elderly and has increased risk of urinary tract infection related to prostate enlargement, so he has two risk factors. A 24-year-old female runner is likely healthy with no additional risk factors. The 50-year-old obese male has one additional risk factor. The 40-year-old sexually active female may not have additional risk factors if she is using protection and does not have multiple partners.

Which tool is used by the nurse to determine risk for impaired skin integrity? A.) Braden scale B.) Glasgow scale C.) Vanderbilt scale D.) MMSE scale

A The Braden scale is used to determine risk for impaired skin integrity: The Glasgow is a coma scale, the Vanderbilt is a behavior scale, and the MMSE is the mini-mental exam to determine cognitive status.

The nurse knows that which assessment tool is not used to assess fall risk? A.) Glasgow Falls Scale B.) Johns Hopkins Hospital Fall Assessment Tool C.) Morse Fall Scale D.) Hendrich II Fall Risk Model

A The Glasgow is a coma scale used to measure level of consciousness, not falls. The rest are scales used to assess the risk for falls in patients.

A nurse is working with a patient using the PLISSIT model. In the LI phase, what is an appropriate activity? A.) Educate the patient on water-based lubricants. B.) Ask the patient for permission to discuss sexuality. C.) Instruct the patient on positions acceptable after knee replacement. D.) Refer the patient and partner to a licensed therapist.

A The LI phase of the PLISSIT model stands for limited information, which would be information the patient needs to function sexually. Asking permission is P. Discussing specific concerns related to a specific medical condition is SS (specific suggestions). Referral to a therapist is IT (intensive therapy).

According to Fayol, controlling is a function of management. The nurse understands controlling compares to what phase of the nursing process? A.) Evaluation B.) Diagnosis C.) Assessment D.) Implementation

A The act of controlling involves comparing expected results of the planned work with the actual results. In the nursing process, evaluation is comparable to controlling. The planning function of a manager is comparable to the assessment, diagnosis, and planning portions of the nursing process.

Upon entering a patient's room, the nurse notes that the patient is unresponsive. The nurse takes control and begins to direct other members of the health care team during this crisis. The nurse is demonstrating characteristics of which type of nursing leadership? A.) Autocratic B.) Democratic C.) Laissez-faire D.) Bureaucratic

A The authoritarian or autocratic leader exercises strong control over subordinates. In this scenario, the nurse takes charge and gives directions that others will follow. The participative or democratic leader believes that employees are motivated by internal means and want to participate in decision making. The primary function of the leader in this situation is to foster communication and develop relationships with followers. Like the democratic leader, the permissive or laissez-faire leader thinks that employees are motivated by their own desire to do well. The laissez-faire leader provides little or no direction to followers, who develop their own goals and make their own. Like the autocratic leader, the bureaucratic leader assumes that employees are motivated by external forces. This type of leader relies on policies and procedures to direct goals and work processes. The nurse using bureaucratic leadership tends to relate impersonally to staff and exercises power on the basis of established rules.

The nurse knows that which statement is true regarding the importance of hygiene? A.) The nurse can assess other body systems during the bath. B.) UAPs perform hygiene because there is no benefit of nurses doing this care. C.) The mucous membranes of the lips, nostrils, anus, vagina, and urethra are not a part of the integumentary system when providing hygiene. D.) The main purpose of bathing is to decrease the patient's body odor.

A The bath is an excellent opportunity for the nurse to assess multiple body systems. Although the UAP can perform hygiene, there is benefit to the nurse doing it because of the ability to assess the patient. The mucous membranes are a part of the integumentary system, and bathing cleanses the skin, reduces odor, provides comfort, and contributes to the patient's health and well-being.

The nurse is caring for a patient who only speaks a foreign language. What is the best method for the nurse to assess the patient's pain level? A.) Perform a pain assessment using a translator. B.) Check the patient's vital signs and pulse oximetry. C.) Check the patient's respiratory rate, depth, and rhythm. D.)Look to see if the patient appears to be resting comfortably.

A The best method to determine pain in a patient who speaks only a foreign language is to use an interpreter. The Universal Pain Assessment Tool is available with foreign-language phrases that may also be used. Measuring vital signs and seeing if the patient is resting comfortably are not accurate pain assessment techniques.

The nurse is caring for a patient with a chest tube who was transported to radiology for testing. When the patient returns to the nursing unit, the transporter shows the nurse the patient's chest tube collection device, which was badly damaged after being caught in the elevator door. What is the priority action of the nurse? A.) Clamp the chest tube until the collection device is replaced. B.) Cover the insertion site with a new occlusive dressing. C.) Ensure that there is gentle bubbling in the water seal chamber. D.) Check the patient's lung sounds and pulse oximetry.

A The broken collection device may no longer be used to collect chest tube drainage. Clamping the chest tube until the collection device is replaced will prevent air from entering the lung space until the new collection device is attached.

A patient in the hospital is an adherent Muslim. Which of the five pillars of Islam can the nurse assist the patient in meeting? A.) Praying five times a day B.) Having privacy C.) Personal cleanliness D.) Giving alms E.) Maintaining modesty

A The five pillars of Islam are: believe in one God, pray five times a day facing Mecca, giving alms to the less fortunate, fasting during Ramadan, and making a pilgrimage to Mecca. The nurse is best able to help the patient maintain the practice of praying five times a day while hospitalized.

The home care nurse is caring for a patient who has severe COPD and home oxygen therapy. The patent tells the nurse that she feels much better after increasing the oxygen flowmeter from 2 L to 5 L/min. The patient's pulse oximetry is 98%. What is the priority action of the nurse? A.) Reduce the oxygen flow rate until the patient's pulse oximetry value is more than 90%. B.) Inform the patient's physician and obtain an order for oxygen at 5 L/min. C.) Document the intervention and findings in the patient's medical record. D.) Listen to the patient's lung fields and reinforce pursed-lip breathing techniques.

A The goal of long-term therapy for the patient with COPD is usually to have an oxygen saturation level of more than 90%, which represents adequate delivery of oxygen to the tissues. Oxygen saturation may decrease during exercise, sleep, or deterioration of the respiratory status. For the patient with COPD, use low-flow oxygen delivery only(≤ 2 L/min) unless a higher level of oxygen administration is indicated by low oxygen saturation levels. High-flow oxygen may lead to respiratory suppression caused by loss of the patient's drive to breathe. The nurse should reduce the oxygen flow rate until the patient's pulse oximetry is more than 90% and educate the patient about oxygen therapy for COPD.

The nurse is caring for a patient who is very dehydrated. Which goal best indicates that dehydration has been corrected and that the patient's fluid balance has been restored? A.) The patient had 1300 mL of light yellow urine in the last 24 hours. B.) The patient's lung sounds are clear bilaterally. C.) The patient has no jugular venous distention. D.) The patient verbalizes need for adequate daily fluid intake.

A The goal that best indicates that the patient's dehydration has been corrected is output of 1300 mL of clear yellow urine in the last 24 hours. Dark concentrated urine is a symptom of dehydration. Jugular venous distention and presence of crackles in the lungs are both indicative of fluid volume overload.

The nurse is caring for a patient who has an ileostomy. Which Nursing diagnosis has the highest priority for the patient? A.) Impaired skin integrity r/t localized skin irritation from liquid stool B.) Social isolation r/t potential leakage of stool from ostomy appliance C.) Lack of knowledge r/t care and maintenance of ostomy appliance D.) Disturbed body image r/t presence of stoma and altered elimination

A The highest priority Nursing diagnosis for this patient is impaired skin integrity because the liquid stool from the ileostomy quickly leads to breakdown when in contact with the skin. Open sores can lead to bacterial infection and significant discomfort for the patient. In addition, ostomy appliances do not adhere well to open wounds, increasing the risk for continuing skin breakdown. The other nursing diagnoses are appropriate for this patient but are not the highest priority.

The nurse is caring for a patient who is having difficulty coping after being in a motor vehicle crash in which her brother was killed. The patient was driving the car and blames herself for the accident. What is the priority nursing intervention of the nurse? A.) Check to make sure that the patient does not want to hurt or kill herself. B.) Educate the patient about available support systems for grief resolution. C.) Enhance the patient's coping skills to alleviate depression and anxiety. D.) Encourage the patient to meet with a spiritual leader for guidance.

A The highest priority for the nurse is to ensure the safety of the patient, so assessment of potential suicidal tendencies is paramount. The other interventions can take place once the nurse is confident that the patient will not try to hurt or kill herself.

The nurse is caring for a patient who has had a severe stroke and requires assistance to use the toilet. Which goal is the highest priority for this patient? A.) The patient will remain continent with no perineal skin breakdown. B.) The patient will state satisfaction with use of gait belt for toilet transfers. C.) The patient will regain ability to pull up clothing after using the toilet. D.) The patient will have privacy once properly positioned on the toilet.

A The highest priority goal for this patient is continence with no perineal skin breakdown to maintain skin integrity and self-esteem. Patient statements of satisfaction and the ability to pull up clothing are important but not the priority over preventing skin breakdown. Privacy is an intervention to be performed by the staff rather than a goal for the patient.

The nurse is caring for a patient who is recovering after hip surgery. The patient requires assistance to use the bathroom because no weight bearing is allowed on the right leg. Which goal is most important for the nurse to include for the diagnosis Impaired self-toileting? A.) The patient will demonstrate safe transfer technique between wheelchair and toilet. B.) The call light will be answered promptly when the patient needs to use the toilet. C.) Toileting will be scheduled in the morning when the patient needs to defecate. D.) Toilet paper and handwashing items will be kept within easy reach of the patient.

A The highest priority goal for this patient is the demonstration of safe transfer technique between the chair and the toilet. The other statements are interventions performed by staff rather than goals that will be accomplished by the patient.

The nurse is caring for a patient who lost her husband 1 year ago after 55 years of marriage. The patient no longer takes care of herself or cooks and rarely eats, stating she has no appetite. The nurse determines that the Nursing diagnosis of complicated grieving applies to the patient. Which is the priority goal for the patient? A.) The patient will shower every other day and eat at least two meals a day. B.) The patient will identify personal strengths that will increase coping ability. C.) The patient will discuss the meaning of her loss with a family member or friend. D.) The patient will be provided with phone numbers for local community resources.

A The highest priority goal of this patient is self-care including showering and eating in order to protect her health and safety. The other goals are lower priority after the patient's necessary activities of daily living are addressed. Goals should also reflect what the patient accomplishes; so the goal of being provided with phone numbers is actually something for the nurse to do.

The nurse is caring for a patient who is recovering from thoracotomy surgery and notes that the patient's pain is rated 9/10 and is unable to focus on anything. Which intervention by the nurse is the highest priority? A.) Administer prescribed IV pain medication and evaluate impact in 30 minutes. B.) Ask the patient to describe prior pain experiences and methods used to manage pain. C.) Explain that comfort is a priority goal of nursing care in the postoperative period. D.) Assist the patient to minimize the effects of pain on interpersonal relationships with family members.

A The highest priority intervention for a patient in acute pain is to provide pain relief. The other interventions do not address acute pain relief.

A nurse is assisting a patient who is having an examination of the female genitalia. What action by the nurse is best? A.) Get the provider; assist patient into lithotomy position. B.) Assist the patient into lithotomy position; get the provider. C.) Get the provider; assist patient into Sims position. D.) Assist the patient into Sims position; get the provider.

A The lithotomy position is used to examine female genitalia. It is an uncomfortable and embarrassing position, so the nurse ensures time spent in that position is limited. The nurse gets the provider, then assists the patient into the position. The Sims position is used to examine the rectal and perineal areas.

The nurse identifies which medication order to be administered PRN? A.) Zolpidem (Ambien) 10 mg PO tonight if the patient cannot sleep B.) Prednisone 10 mg PO today, then taper down 1 mg each day for the next 10 days C.) Humulin R 10 units subcutaneously before each meal and at bedtime D.) Kefzol (Ancef) 1 g IVPB 30 minutes prior to surgery

A The nurse is to give the zolpidine (Ambien) if the patient cannot sleep. Therefore, this is the PRN (as needed) medication order. The other orders have specific time frames.

The nurse is caring for a patient who has just had an intravenous pyelography (IVP) completed. Which assessment is the nurse's highest priority after the patient returns from the test? A.) Calculate the patient's intake and output. B.) Monitor for discoloration of the patient's urine. C.) Assess for possible iodine or shellfish allergies. D.) Inquire if the patient has burning or pain with urination.

A The nurse must carefully monitor the patient's intake and output after IVP testing to ensure that the patient's kidneys were not damaged by the contrast dye. PO fluid intake should be encouraged to facilitate excretion of the contrast dye. Urine is not discolored from the IVP. Burning or pain with urination should not occur after IVP testing because there is no instrumentation of the urinary tract. Assessment of allergies must be done before the IVP is done because iodine-based contrast is used.

The nurse is caring for a postoperative patient on his first day after surgery. The nurse informs the patient that the plan is to sit in the chair and ambulate in the hallway. The patient states that he is in pain and has no intention of getting out of bed. What is the nurse's best response? A.) "It's important to move around so you don't get a blood clot in your leg." B.) "Your doctor ordered that you are to get out of bed at least twice every day." C.) "I understand. You can rest in bed until tomorrow when the pain is better." D.) "I will call the doctor and let him know that you do not want to get up."

A The nurse should teach the patient why it is important to ambulate after surgery. Early ambulation helps to prevent many complications postoperatively, including constipation, deep vein thrombosis, atelectasis, pneumonia, and urinary stasis. Many patients who experience abdominal distention and gas pain obtain some relief from ambulating. Simply telling the patient that the physician ordered ambulation is not sufficient. Allowing the patient to stay in bed will increase the risk of DVT.

The nurse begins a shift on a busy medical-surgical unit and will be caring for multiple patients. Which patient does the nurse assess first? A.) A patient who would like some acetaminophen (Tylenol) for a mild headache. B.) A patient who has a question about her daily medications. C.) A patient who needs discharge teaching about an antibiotic. D.) A patient who just received nitroglycerin for chest pain.

A The nurse's first priority is always: ABCs—Airway, Breathing, and Circulation. This includes any patients who are having chest pain and/or difficulty breathing. The nurse needs to see this patient first to determine if the chest pain has been relieved or not and to determine if the patient is now stable or if additional interventions need to be done. The other patients' needs are less critical and can be met after this patient is assessed.

A nurse has assessed a community and has found many areas in which health can be improved. As a result, the nurse has multiple ideas for programming. What action by the nurse is best? A.) Determine what the community thinks is most important. B.) Use vital statistics to determine which is most important. C.) See what other communities are focusing programming on. D.) Choose the easiest problem to address first.

A The nurse's priorities may be very different from the community's. For programming to be successful, there must be buy-in from members of the community. Unless programming addresses a need the community thinks is important, it is unlikely to be successful.

The nurse is caring for a terminally ill patient whose children have come home to be with their mother during her last few days. They spend time looking through picture albums, watching old home movies, and remembering fun times spent together. The nurse identifies which term that best describes the activity of the patient's children? A.) Anticipatory grieving B.) Bereavement C.) Caregiver role strain D.) Death anxiety

A The patient and her children are experiencing anticipatory grief as they prepare for the expected death of the patient. Reminiscence and life review are used to assist those experiencing anticipatory grief with the realization that death is approaching.

The nurse is caring for a patient who suffered a miscarriage at 24 weeks of pregnancy. The patient is devastated by the loss but her husband minimizes her grief by stating, "Quit crying. It's not like you lost a real baby." What term best describes the anguish felt by the patient? A.) Disenfranchised grief B.) Delayed grief C.) Moral distress D.) Masked grief

A The patient is experiencing disenfranchised grief because she cannot share the pain of her loss with her husband. The husband is not willing to support his wife as she mourns the loss of her pregnancy or recognize the grief that she is going through. Delayed grief is suppression of the grief process. Moral distress occurs when people cannot act according to their moral values. Masked grief occurs when a person's bereavement behaviors interfere with his or her life, but the person does not notice this.

The nurse is caring for a patient who is recovering from chest surgery. Which action by the patient indicates that additional teaching is needed about how to use the ordered incentive spirometer correctly? A.) The patient breathes into the spirometer so that the marker rises slowly. B.) The patient uses the spirometer 5 to 12 times every 1 to 2 hours while awake. C.) The patient seals his lips tightly around the spirometer mouthpiece. D.) The patient should hold each inhaled breath 3 to 5 seconds.

A The patient must take in a deep breath while holding the spirometer to the mouth so that the device can indicate how much air is being inhaled into the lungs. The remaining responses are correct components of the procedure.

A patient has Sleep deprivation. What finding by the nurse best indicates goal achievement? A.) Remains asleep for 6 to 7 hours consistently for 1 week. B.) Falls asleep within 15 minutes of going to bed. C.) Reports an ability to concentrate on tasks. D.) Verbalizes understanding of medication instructions.

A The patient remaining asleep for 6 to 7 hours consistently for 1 week is an appropriate goal for Sleep deprivation. The patient falling asleep within 15 minutes of going to bed is a goal for Insomnia. The patient reporting an ability to concentrate on tasks is a goal for Anxiety. The patient repeating medication instructions is an appropriate goal for any patient.

The nurse is caring for a patient who will be undergoing bone marrow biopsy. Which statement by the patient indicates that additional teaching is needed? A.) "I will count the ceiling tiles when the doctor inserts the numbing medicine." B.) "I will take acetaminophen later today if the site becomes uncomfortable." C.) "I will squeeze your hand to help calm my fears about the test." D.) "I will keep the biopsy site clean and dry for the next 24 hours."

A The patient will be positioned in the prone or lateral position for the test, so the patient will not be able to count ceiling tiles as a distraction during the numbing step of the test. The patient may take acetaminophen as needed for discomfort afterward. The biopsy site must be kept clean and dry for 24 hours after the biopsy to prevent infection. Holding the nurse's hand will help calm the patient before and during the procedure.

The nurse is caring for a patient who will be undergoing flexible sigmoidoscopy testing to screen for colon cancer. What goal will the nurse include in the patient's plan of care? A.) Patient will verbalize understanding of preprocedure preparation to be completed at home the day before the test. B.) Patient will feel comfortable about the upcoming test and have trust in the health care providers. C.) Patient will learn common side effects of the medications used to prepare the GI tract for endoscopy testing. D.) Patient will realize how important regular sigmoidoscopy testing is in the prevention of colon cancer.

A The patient will need to complete colon preparation prior to the sigmoidoscopy testing. The nurse must determine that the patient understands how and when to complete the prep. Having the patient verbalize understanding of the prep procedure is an objective goal so that the nurse can readily determine whether or not it has been met. The other goals are not objective or measurable, so the nurse cannot determine whether or not they have been met.

The nurse is caring for a patient who is admitted to the hospital with dehydration and gastroenteritis. The patient attempted to walk to the bathroom and fainted right after getting out of bed. The nurse knows which condition to be the most likely cause of the patient's collapse? A.) Orthostatic hypotension B.) Circulatory overload C.) Hemolytic reaction D.) Catheter embolism

A The patient with dehydration is at risk for orthostatic hypotension or falling of the blood pressure when the patient rises to a standing position. When the blood pressure falls sufficiently, fainting may occur. The patient should be assisted to rise slowly from a supine to a sitting position first before slowly getting to his feet. Circulatory overload, hemolytic anemia, and catheter embolism are unlikely to be causative factors.

The nurse is caring for a hospitalized patient with hyperparathyroid disease and a serum calcium level of 14.2 mg/dL. What is the priority intervention of the nurse? A.) Instruct the patient to always call for assistance before getting out of bed. B.) Assist the patient to change into dry clothing after episodes of diaphoresis. C.) Teach stress-relieving techniques, including progressive muscle relaxation. D.) Notify the provider if urine output is less than 30 mL/hr.

A The patient with hypercalcemia should always call for assistance before getting out of bed because of the risk of falling due to muscle weakness, soft bones, and lethargy. Diaphoresis and decreased urine output are not common symptoms of hypercalcemia. Teaching stress-relieving techniques is not a priority.

The nurse is caring for a patient who reports an urgent need to urinate but is unable to pass more than a few drops of urine in the toilet. Which is the priority assessment to be performed by the nurse? A.) Bladder scan to determine the amount of urine in the bladder B.) Auscultation to assess circulation through the right and left renal arteries C.) Bimanual palpation to assess for possible enlargement of the kidneys D.) Calculate the patient's intake and output to check for fluid volume deficit

A The patient with suspected urinary retention should have a bladder scan performed to determine the amount of urine in the bladder. If a significant amount of urine is found in the bladder, the provider may be notified to obtain an order for straight catheterization.

The hospice nurse is caring for a terminally ill patient who will probably die within the next hour or two. The patient's daughter is keeping a vigil by the bedside and asks what she can do to help her father at this time. What is the appropriate response of the nurse? A.) "Just let him know you are here, talk to him, and let him know that you love him." B.) "You can try to feed him a few bites of ice cream to keep his mouth from getting dry." C.) "You can take this time to ensure that arrangements are set with the funeral home." D.) "You should let me know when your father's breathing pattern changes."

A The patient's daughter should be encouraged to spend the last moments of her father's life with him, reassuring him with her presence. The daughter should be encouraged to continue talking with him because the patient may still hear her even if his eyes are closed and he does not speak. The nurse is responsible for monitoring the patient for breathing changes. Oral intake will lead to nausea and/or aspiration. This is not the time to make arrangements with the funeral home.

The nurse is caring for a patient who is terminally ill with metastatic bone cancer. The patient tells the nurse that he is not afraid of death but does not want to be in pain and suffer before he dies. Which intervention by the nurse will be most appropriate to meet this patient's wishes? A.) Establish around-the-clock dosing for pain medications with additional doses for breakthrough pain. B.) Assist the patient to reminisce and review his life, spending as much time as possible with loved ones. C.) Use therapeutic touch, guided imagery, and soft music to put the patient at ease and relieve anxiety. D.) Encourage the patient to participate in prayer and meditation along with preferred religious practices.

A The patient's primary wish is to die without pain, and the best intervention to meet this goal is administration of pain medication around the clock with extra doses for breakthrough pain. The other interventions may make the patient more comfortable but will not address his primary desire for adequate pain management.

Which delegation of tasks would require the nurse manager to intervene? A.) The UAP re-delegates vital signs to the student nurse. B.) The RN delegates assistance with bathing to the student nurse. C.) The RN delegates monitoring of intake and output to the UAP. D.) The RN delegates assistance with mobility to the UAP.

A The person to whom the assignment was delegated cannot delegate that assignment to someone else. If the person cannot carry out the assignment, the individual needs to notify the delegating RN so that the task may be reassigned or completed by the RN. The RN must remember to delegate tasks that do not require nursing judgment. Only tasks that are routine such as bathing, monitoring intake and output, and assisting with mobility, and do not require variation from a standardized procedure should be delegated.

The nurse is caring for a patient who is recovering from bowel surgery. Which assessment finding best indicates that the bowel is starting to resume function and the patient will be able to resume oral intake soon? A.) The patient has bowel sounds x 4 quadrants and is passing gas. B.) The patient has no nausea, and abdominal pain is minimal. C.) The patient feels hungry for chicken soup and hot tea. D.) The patient's nasogastric tube was discontinued the previous day.

A The presence of bowel sounds and passage of flatus indicate that the patient's bowels are starting to resume function and the patient will be able to resume oral intake soon. Hunger, discontinuation of the NG tube, or absence of nausea are not definite indicators of readiness to resume oral feedings.

The nurse is caring for a postoperative patient who underwent bowel resection surgery that morning. The nurse assesses the patient's abdomen and notes that there are hypoactive bowel sounds. The patient is resting quietly without nausea or vomiting. What is the appropriate action of the nurse? A.) Keep the patient NPO and document the findings in the chart. B.) Administer a laxative suppository to stimulate peristalsis. C.) Insert a Salem sump nasogastric tube to low continuous suction. D.) Notify the surgeon and prepare the patient to return to surgery.

A The presence of hypoactive bowel sounds is an expected finding for the first hours after abdominal surgery. The patient should be kept NPO to prevent nausea and vomiting. A laxative should not be administered. A nasogastric tube is not needed unless the patient starts vomiting or a paralytic ileus develops.

What is the priority nursing assessment for a patient who is receiving postoperative epidural analgesia with hydromorphone (Dilaudid)? A.) Respiratory rate, depth, and pattern B.) Skin underneath the epidural dressing C.) Bladder scanning to check for urinary retention D.) Itching on the trunk and/or extremities

A The respiratory system is the priority nursing assessment for patients receiving narcotic pain medication via any route. This is because narcotics can cause respiratory suppression. The other assessments are a lower priority and may be done after a respiratory assessment is completed.

A nurse is wondering if home health care nursing is a good fit. What characteristic or ability does the experienced home health care nurse suggest is most important? A.) Clinical reasoning B.) Organization C.) Assessment skills D.) Time management

A The role of the registered nurse in home health care is essentially autonomous in that the nurse must be highly proficient in health assessment (physical and psychosocial), be well versed in complex technical and clinical skills, possess strong critical-thinking and clinical reasoning abilities, and demonstrate excellent organizational skills.All choices are important characteristics or abilities of home health care nurses. However, since the nurse working out in the community may not have the resources (personnel or materiel) available in an acute care facility and often must improvise, clinical reasoning would be the most important of the choices provided.

The nurse is caring for a postoperative patient who is very sleepy following general anesthesia and administration of pain medication. The nurse notes that the patient is making snoring sounds and his pulse oximetry has dropped to 88%. What is the best action of the nurse? A.) Assess the airway and administer oxygen. B.) Call for anesthesia to immediately reintubate the patient. C.) Remove the pillow from behind the patient's head. D.) Elevate the head of the patient's bed.

A The snoring sounds made by the patient and low oximetry levels indicate that the patient's airway is partially occluded because of anesthesia and pain medication. Patency of the airway should be assessed and oxygen administered to maintain the airway and oxygenation. Reintubation should be performed if the oral airway alone does not maintain the airway and improve oxygenation. Removing the pillow from the head of the patient's bed or elevating the head of the patient's bed will not be sufficient to open the patient's airway.

A nurse performs orthostatic blood pressure readings on a patient with the following results: lying 148/76 mm Hg, standing 110/60 mm Hg. What action by the nurse is best? A.) Instruct the patient not to get up without help. B.) Document the findings and continue to monitor. C.) Reassure the patient that these findings are normal. D.) Reassess the blood pressures in 1 hour.

A This patient has orthostatic hypotension, which is a drop of 20 mm Hg in systolic reading and 10 mm Hg in diastolic reading when the patient stands up from a sitting or lying position. The patient's cardiovascular system does not compensate for this, so the patient is at risk of becoming dizzy and fainting. The nurse instructs the patient to call for assistance before getting up to prevent a fall. The nurse should document the findings but needs to do more. These findings are not normal, so the nurse should not tell the patient that they are. The patient may need to be assessed sooner than 1 hour.

A nurse observes a patient sitting up in bed, leaning forward with the arms braced against the over-the-bed table. What action by the nurse is best? A.) Assess the patient for a barrel-chest appearance. B.) Palpate the patient's abdomen for tenderness. C.) Inspect the patient's spine for deformities. D.) Ask the patient if he/she is experiencing dizziness.

A This patient is sitting in a tripod position, often seen in patients with chronic obstructive pulmonary disease. These patients also often have a barrel-chest appearance, so the nurse assesses for this finding. The other actions are not related to a tripod position.

A nurse is concerned about not consistently meeting the spiritual needs of patients. What action by the nurse is best? A.) Care for own spiritual needs. B.) Begin a meditation practice. C.) Consult the chaplain. D.) Read books on the subject.

A To avoid burnout and a decreased ability to attend to the spiritual needs of patients, nurses must take care of their own spiritual needs first. This may include meditation, consultations, and reading, but other activities can guide the nurse into a reflective practice that will allow better spiritual care.

The nurse is caring for a patient with the nursing diagnosis of Urge incontinence of urine related to urinary tract infection. Which statement is appropriate for the "as evidenced by" portion of the patient's diagnosis? A.) Sudden leakage of urine when patient is unable to get to the toilet in time B.) Continuous urine flow from the bladder regardless of attempts to use the toilet C.) Leakage of urine from the bladder when the patient coughs, sneezes, or laughs D.) Leakage of urine because the patient is unable to indicate need to use the toilet

A Urge incontinence of urine occurs when the patient has a sudden need to urinate but cannot get to the toilet in time. Continuous flow of urine is deemed total urinary incontinence. Leakage of urine when sneezing or coughing is stress incontinence. Functional incontinence occurs when the patient cannot indicate need to use the toilet.

The nurse is teaching a group of patients about diseases that are transmitted by ticks. Which term would the nurse use when identifying the function of a tick in spreading disease? A.) Vectors B.) Bacteria C.) Viruses D.) Fungi

A Vectors carry pathogens from one host to another. Bacteria are single-cell organisms. Viruses are the smallest organisms. Fungi are single-cell organisms that can cause infection.

The nurse is caring for a patient who has severe abdominal pain caused by acute cholecystitis. The nurse recognizes which type of pain is this patient experiencing? A.) Visceral pain B.) Somatic pain C.) Radiating pain D.) Referred pain

A Visceral pain arises from the organs of the body and occurs when inflammation and tissue damage occur, such as with cholecystitis. Somatic pain occurs when there is tissue damage to skin, muscle, joints, and bones. Referred pain occurs when the discomfort is felt at a location other than the origin of the pain. Radiating pain extends to another area of the body.

The UAP asks why the arms are washed from distal to proximal. Which response by the nurse is appropriate? A.) To promote circulation B.) To maintain asepsis C.) To maintain comfort D.) To maintain tradition

A Washing from distal to proximal promotes circulation and blood return. Asepsis is the state of being free from disease-causing contaminates. There is no difference in comfort. Tradition is a custom.

When teaching a student nurse about removing PPE, the nurse would include which correct order of equipment removal? A.) Gloves, eyewear, gown, and mask B.)Mask, eyewear, gown, and gloves C.) Gown, mask, eyewear, and gloves D.) Gloves, gown, mask, and eyewear

A When removing PPE, gloves, which are contaminated, are removed first to prevent contamination of the face and eyes during removal of the mask and to prevent spread of microorganisms. Eyewear should then be removed, followed by the gown and finally the mask.

If the nurse is trying to determine the best treatment or course of action and wants to incorporate the most reliable evidence into the decision, the nurse will use what filtered resource? A.) Cochrane Reviews B.) UpToDate C.) STAT!Ref D.) MD Consult

A When trying to determine the best treatment or course of action and wanting to incorporate the most reliable evidence into the decision, the nurse can use a filtered resource such as the Cochrane Reviews or the Joanna Briggs Institute Library of Systematic Reviews. The filtered resource provides the best available evidence. In filtered resources, clinical and subject experts have asked a question and then synthesized evidence to establish conclusions based on the research. This pre-evaluation process is already completed for nurses and allows the resources to be used while caring for patients. The conclusions from filtered resources still need to be evaluated by clinicians in terms of a specific patient. Filtered resources produce systematic reviews of the literature. Nurses may encounter conditions outside their specialty area and need an overview. Background resources provide detailed information. If the nurse is looking for a presentation of information or types of therapies, the best source is a background resource. Background resources include UpToDate, STAT!Ref, and MD Consult, which are web-based databases. Another source of background information is a current nursing textbook.

The nurse is providing patient care and pays special attention to meeting the needs of the patient while maintaining the patient's right to privacy, confidentiality, autonomy, and dignity. This nurse is applying what ethical theory? A.) Deontology B.) Utilitarianism C.)Autonomy D.) Accountability

A Deontology is an ethical theory that stresses the rightness or wrongness of individual behaviors, duties, and obligations without concern for the consequences of specific actions. Meeting the needs of patients while maintaining their right to privacy, confidentiality, autonomy, and dignity is consistent with the tenets of deontology. Compared with deontology, utilitarianism is on the opposite end of the ethical theory continuum. Utilitarianism maintains that behaviors are determined to be right or wrong solely based on their consequences. Autonomy, or self-determination, is the freedom to make decisions supported by knowledge and self-confidence. Accountability is the willingness to accept responsibility for one's actions.

The nurse is to teach an 84-year-old Spanish-speaking patient newly diagnosed with diabetes how to self-administer insulin. The patient has hearing and visual impairments. To be effective as a teacher, the nurse should carry out which tasks? (Select all that apply.) A.) Assess reading level and learning style. B.) Determine readiness to learn. C.) Use family members as interpreters. D.) Provide written instruction in English. E.) Place the patient in group classes.

A & B Before health care teaching sessions for adults, assess reading level, learning styles, and readiness to learn. Family members should not be used as interpreters of specific medical information to maintain the patient's right to privacy and to avoid possible misinterpretation of medical terminology. Access to interpretation or translation for deaf and limited English proficiency (LEP) patients is required by Title VI of the Civil Rights Act of 1964, which mandates equal rights for people regardless of race, color, or national origin. Use photos, drawings, or video to enhance understanding. A patient whose cultural beliefs and values are considered is more likely to demonstrate compliance. Patients with learning disabilities or cognitive alterations need individualized instruction geared to their special needs.

A nurse is teaching patients about their medications and implications for sexuality. Which combinations are correct? (Select all that apply.) A.) Antipsychotics: erectile dysfunction B.) Phenytoin: decreased desire C.) Antihistamines: increased vaginal lubrication D.) SSRIs: prolonged orgasm E.) Marijuana: chronic use—reduced inhibitions

A & B. Antipsychotics can lead to erectile dysfunction. Phenytoin can lead to decreased desire and function. Antihistamines can cause decreased vaginal lubrication. SSRIs can lead to absent or delayed orgasm. Chronic marijuana used can lead to decreased desire.

The nurse is caring for a patient who is anemic. Which CBC test results demonstrate that the patient's treatment plan is effective and the anemia is resolving? (Select all that apply.) A.) Red blood cell count (RBC) 5.8 million/mm3 B.) Hematocrit (HCT) 25% C.) Hemoglobin (HGB) 14 g/dL D.) White blood cell count (WBC) 4500/mm3 E.) Platelet count (PLT) 255,000/mm3

A & C. Red blood cell count of 5.8 million and hemoglobin value of 14 g/dL are both normal. Hematocrit level of 25% is very low and indicative of ongoing anemia. White blood cell and platelet counts are not checked for anemia.

The nurse is caring for a patient who has been having abdominal pain. The doctor suspects that the patient may have an abdominal aortic aneurysm. Which tests would confirm the doctor's suspicion? (Select all that apply.) A.) Magnetic resonance imaging (MRI) scan B.) Needle aspiration with biopsy C.) Fiberoptic endoscopy D.) Computed tomography (CT) scan E.) Flexible sigmoidoscopy F.) Thoracentesis

A & D. CT scan and MRI may be used to determine the presence of an abdominal aortic aneurysm. Endoscopy, needle aspiration, sigmoidoscopy, and thoracentesis will not help make this diagnosis.

The nurse is performing a health history to determine the patient's cognitive status. Which questions will be best suited to elicit the information needed? (Select all that apply.) A.) "Are you able to drive to the store or do errands?" B.) "Do you have any pain?" C.) "Is your vision blurry?" D.) "Are you able to smell different foods?" E.) "Have you noticed any difficulty adding up numbers?"

A & E. Driving and adding numbers relates to cognitive ability. The remaining three options have a sensory focus.

Revise the following goal statements to make them more precise, time orientated and measurable. A.) Temperature will be normal B.) The patient will walk more. C.) The diet will be understood. D.) Ankle wound will heal.

A - Body temperature will range between 98.3 and 99 during the day. (Outcome indicator is Vital Signs - Body Temperature) B - Patient will ambulate up and down the hallway to room at least tid during hospital stay (Outcome indicator is Activity - Mobility) C - Patient will be able to verbalize the foods allowed on the therapeutic diet by tomorrow. (Outcome indicator is Appetite - Knowledge) D - Ankle wound will be clean and dry, reducing the size by 1/2 cm within 2 days. (Outcome indicator is Wound Healing)

Deontology

the study of the nature of duty and obligation.

when red blood cells are 120 days old what happens to them?

they are destroyed by the liver

3 critical functions of blood

transport o2 regulates pH balance protects clotting mechanism

medical management of DIC

treat underling cause cryopercipiate heparin protect from bleeding and trauma

Thrombocytopenia signs and symptoms

1.) Mucosal bleeding 2.) Petechiae (tiny brown/purple spots due to bleeding under skin) 3.) Easy bruising 4.) Hematuria 5.) Gingival bleeding

Volume of blood

10-12 pints

Consistency of blood

45% blood cells 55% plasma

Where is apical pulse found?

5th intercostal space left midsclavicular line

pH of blood

7.35-7.45

A nurse administers a polio vaccine to an infant. What is this level of prevention called? A.) Primary Prevention B.) Tertiary Prevention C.) Rehabilitation D.) Secondary Prevention

A

Beliefs

A mental representation of reality or an individual's perceptions about what is right, true, or real, or what the individual expects to happen in a given situation.

Referent

An event or thought initiating the communication

Promotion of human welfare

Humanistic

Physiological disease or mental disorder

Illness

White blood cells

Leukocytes/ no form

What is conventional morality?

Moves from egocentric viewpoint toward the expectations of society; loyalty and obedience to the organization (society) become paramount

Assertiveness

The ability to express ideas and concerns clearly while respecting the thoughts of others

Channel

The method of communication

Feedback

The response of the receiver

Decode

To sort out the meaning of what is being communicated

Where is the dorsalis pedis pulse?

Top of foot

A patient is prescribed trastuzumab for breast cancer. What is the priority nursing intervention for this patient? a. Assess for signs of cardiotoxicity b. Assess for signs of bleeding c. Premedicate with an antiemetic d. Rotate injection sites

a. Assess for signs of cardiotoxicity

What information can be obtained by surgical staging? Select all a. Assessment of tumor size b. Number of tumors c. Sites of tumors d. Types of tumors e. Pattern spread of tumors f. Pain related to tumors

a. Assessment of tumor size b. Number of tumors c. Sites of tumors e. Pattern spread of tumors

Pernicious Anemia

b12 cannot be absorbed from food

what is the treatment for pernicious anemia?

b12 injections for life folic acid and fe suppliements

pancytopenic mean?

decreased or absent RBC, WBC, and platelets

What is the id?

the basic instinct principle in Freudian theory. It is the seat of aggression and sexual impulse. It is devoid of logic and time orientation. It is chaotic and bodily focused.

What is teratogens?

Agents, such as chemicals and viruses, that can reach the embryo or fetus during prenatal development and cause harm

The nurse receives the patient assignment in the morning. Which one of the patients should she see first? The patient who: A.) Takes a hypnotic medication at bedtime B.) Has fluctuations in blood sugar readings C.) Needs assistance with morning care D.) Has an order for a daily dressing change.

B

The nurse researcher understands a human subject is defined as a living individual about whom an investigator conducting research obtains what information? A.) Data without direct or indirect interaction or intervention B.) Information that is not expected to be made public C.) No diagnostic information and does not manipulate the subject environment D.) Information without any communication/contact during the research

B A human subject is defined as "a living individual about whom an investigator conducting research obtains: (1) data through intervention or interaction with the individual or (2) identifiable private information" (Office of Human Research Protection (OHRP), 2016). Interventions may include procedures such as gathering diagnostic information or manipulating the subject's environment. Interaction refers to any communication or contact during the research. Private information includes anything not expected to be made public, such as a medical record.

The nurse knows while leadership behaviors and management skills often complement each other they differ in which way? A.) Managers focus on relationships. B.) A manager may not possess leadership traits. C.) Leadership focuses on coordinating and directing others. D.) A manager is a visionary who sets the direction for a group

B A manager may not possess leadership traits, and a leader may lack management skills. Management is the process of coordinating others and directing them toward a common goal. Leadership focuses on relationships, using interpersonal skills to persuade others to work toward a common goal. Leaders are visionaries who set the overall direction for a group or organization

The nurse has applied a pulse oximeter to the finger of a patient who is hypothermic. The pulse oximeter does not provide a good reading. What action by the nurse is best? A.) Move the oximeter probe to another finger. B.) Assess the fingers for good circulation. C.) Document that the reading cannot be obtained. D.) Remove any fingernail polish present on the fingernail.

B A patient who is hypothermic may not have good circulation to the extremities. The nurse should assess the patient's circulation, and if it is poor to the extremities, choose another spot at which to measure the oxygen saturation. Moving the probe to another finger or removing nail polish will not help if the problem is poor circulation. The nurse should document appropriately but needs to do more than just charting that the reading could not be obtained.

The nurse identifies what goal to be the most appropriate goal for a patient with a stage 3 pressure ulcer who has a Nursing diagnosis of Impaired skin integrity? A.) Wound will be completely healed in 72 hours. B.) Wound will show signs of healing within 2 weeks. C.) Patient will develop no new pressure ulcers. D.) Patient will ambulate twice a day.

B A stage 3 pressure ulcer is a more extensive wound and will take time to heal, so the most appropriate goal will be to show signs of healing in 2 weeks. It will not heal in 72 hours. The goal of no new pressure ulcers is good, but not the most appropriate, and ambulating twice a day is more of an intervention.

The nurse knows the most appropriate goal for a patient with stage III pressure ulcer who has a nursing diagnosis of Impaired Skin Integrity is: A.) The wound will completely heal in 72 hours B.) The wound show signs of healing within 2 weeks C.) The patient will develop no new pressure ulcers D.) The patient will ambulate twice a day

B A stage III pressure ulcer is more extensive wound and will take time to heal, so the most appropriate goal will be to show signs of healing in 2 weeks. It will not heal in 72 hours. The goal of no new pressure ulcers is good, but not the most appropriate, and ambulating twice a day is more of an intervention.

The nurse correctly selects which intervention to avoid causing shear or friction when moving a patient in bed? A.) Using an airflow bed B.) Using a slide board C.) Using a trochanter roll D.) Using a gel mattress

B A transfer or slide board is made of plastic-like material that reduces friction. Linens easily slide over the board, facilitating bed linen changes. Patients can be repositioned or transferred with a minimum of force required. A trochanter roll prevents outward rolling of the hip when a patient is lying on his/her back. An air-fluidized bed uses airflow to move silicone particles in the bed, creating a watery, fluid-like movement and resulting in lower pressure to avoid or alleviate decubitus ulcers. A foam or gel combination mattress reduces pressure.

The nurse is working with a patient from an unfamiliar culture. After assessing the patient and the patient's cultural beliefs related to health care, what action by the nurse is best? A.) Create a nursing plan of care for the patient. B.) Recheck cultural beliefs with the patient. C.) Use a standard plan of care for consistency. D.) Have an interpreter validate the information.

B According to Leininger, the nurse should recheck assumptions and findings related to culture with the patient. This is an important step prior to creating a care plan. A standard plan will not be culturally congruent. The stem does not indicate that the patient has limited English, but if he did, using an interpreter would be important.

Which explanation by the nurse best describes active assistive range of motion? A.) The patient independently moves all joints. B.) The patient to partially moves all joints. C.) The caregiver must move the patient's joints. D.) The patient performs isotonic exercises.

B Active assistive range of motion occurs when the caregiver minimally assists the patient, or the patient minimally assists himself/herself in the movement of joints through a full motion. Active range of motion occurs when the patient has full independent movement of all joints; this is also known as isotonic exercise. Passive range of motion occurs when the caregiver moves the patient's joints through a full motion. This exercise does not maintain or improve strength but maintains flexibility and prevents contractures and atrophy.

The nurse recognizes that starting an intravenous (IV) infusion line on a patient against his will may be classified as which wrongdoing? a. Assault B.) Battery C.) Felony D.) Misdemeanor

B Actual physical harm caused to another person is battery. Battery may involve angry, forceful touching of people, their clothes, or anything attached to them. Performing a surgical procedure without informed consent is an example of battery. Actions much more subtle, such as inserting an intravenous catheter or urinary catheter against the will of a patient, also may be classified as battery. Assault is a threat of bodily harm or violence caused by a demonstration of force by the perpetrator. A feeling of imminent harm or feeling of immediate danger must exist for assault to be claimed. A misdemeanor is a crime of lesser consequence that is punishable by a fine or incarceration in a local or county jail for up to 1 year. A felony is a more serious crime that results in the perpetrator's being imprisoned in a state or federal facility for more than 1 year.

The nurse is caring for an Islamic patient who has just died. The family is traveling from overseas. Which action is the priority for the nurse to take right after the patient dies? A.) Arranging for embalming to preserve the body until burial B.) Rearrange the furniture so the bed can face Mecca C.) Arranging for transportation of the body to the crematorium D.) Bringing in fruit for the patient's journey to the other world

B After death, a patient's body can be turned to face Mecca which is the holy site for Muslims. The nurse would need to find out which direction that is. The family will work with the funeral home to determine when and where burial will take place. Buddhists often bring fruit when someone dies.

The nurse is caring for a confused, combative patient. Which action would be considered last by the nurse to control behavior of the client? A.) Orient the patient frequently. B.) Apply restraints. C.) Move the patient to a room close to the nurse's station. D.) Encourage the family to spend time with the patient.

B All alternatives to physical restraints should be considered prior to their use.

The nurse is educating the patient about alternative therapies. Which statement by the patient indicates a need for more information? A.) Alternative therapies can include relaxation techniques. B.) Alternative therapies are used in conjunction with medical therapies. C.) Alternative therapies can be used when patients are experiencing stress. D.) Some alternative therapists require certification.

B Alternative therapies are used in place of medical treatment. These types of interventions are useful when patients are experiencing physiologic and psychological responses to stress. Some complementary and alternative therapies such as therapeutic touch, Reiki, biofeedback, and massage therapy require additional certification and training, whereas muscle relaxation and guided imagery do not.

The nurse is educating the patient about the signs and symptoms of a wound infection. Which statement indicates a need for further education? A.) "The wound will be red." B.) "The wound will have pus." C.) "The wound will be warm." D.) "The wound will need to be treated."

B An infected wound shows clinical signs of infection, including redness, warmth, and increased drainage that may or may not be purulent (contain pus), and has a bacterial count in the tissue of at least 105/g of tissue sampled when cultured. The wound will need to be treated for the infection.

A patient has recently been given a terminal diagnosis. When family members offer to help, the patient snaps and yells at them, but then angrily accuses them of not helping. The patient's spouse is frustrated and asks the hospice nurse what to do about this situation. What response by the nurse is best? A.) "Don't worry. Your spouse will get over this phase soon." B.) "Anger is an expected part of the grieving process." C.) "Would your spouse be open to professional counseling?" D.) "This diagnosis is difficult to handle; just be patient."

B Anger is one of the stages of grief as identified by Elizabeth Kubler-Ross. The nurse would first explain this to the spouse. Telling the spouse the patient will get over the phase soon or that the diagnosis is difficult to handle is false reassurance and dismissive of the concerns. It is too early to consider counseling although the patient may need it later. This is also a yes/no question which is not therapeutic.

The nurse understands that as the health care community explores the concept of health literacy, many organizations recognize what concept? A.) Consumers need to understand has no governmental support. B.) Improvements are dependent on developing operational definitions. C.) Low literacy and low health literacy are interchangeable terms. D.) Interest in effective patient education is unique to the United States.

B As the health care community explores the concept of health literacy, many organizations recognize that before improvements can be made, operational definitions are imperative. The realization that consumers need to be able to understand the medical information delivered by health care providers has gained recognition at many governmental levels. The Healthy People 2020 publication describes a national movement that addresses the priorities of prevention and public health in the United States. Health literacy with its impact on this initiative is being recognized and has become a key component of the project. Although low literacy and low health literacy are related terms, they are not interchangeable. Low health literacy is content-specific, meaning that the individual may not have difficulty reading and writing outside the health care arena. Interest in effective patient education is not a phenomenon unique to the United States. The Institute of Medicine Roundtable on Health Literacy held a workshop in 2012 focused on international health literacy.

The nurse suspects that the patient is experiencing a drug toxicity rather than a side effect. Which question will the nurse ask to help confirm this suspicion? A.) "When did you take your last dose of the medication?" B.) "Have you been taking extra doses of the medication?" C.) "Are you taking any other medications?" D.) "Have you ever taken this medication in the past?"

B Asking if the patient has been taking extra doses of the medication will allow the nurse to determine if the patient has been taking too much of the drug or more than was prescribed. Toxicity occurs when the patient receives/takes excessive amounts of the drug.

The nurse and UAP are making an occupied bed together. Which action by the nurse is incorrect? A.) The nurse asks and assists the patient to turn toward the UAP and loosens the fitted sheet and rolls it in toward the patient. B.) The nurse rolls dirty linens to the side then places the linens on the floor while finishing. C.) The nurse tucks the clean bottom sheet under the cleaner underside of the dirty linens. D.) The nurse wears gloves to remove dirty linens.

B Bed linens should be placed in the linen hamper, not on the floor, after they are removed from the bed. The patient turns to each side while the bed linens are changed, and the nurse wears gloves.

The nurse is caring for a patient who is to receive a transfusion of packed red blood cells. The patient has a 22-gauge IV in his arm with 0.9% normal saline infusing. What intervention will the nurse perform before obtaining the packed red blood cells from the blood bank? A.) Identify the blood group, type, and expiration date with another nurse. B.) Assess the patency of the current IV site for the administration of the packed RBCs. C.) Program the IV infusion pump so that the transfusion will complete within 4 hours. D.) Obtain a new microdrip tubing and extension tubing from the clean utility room.

B Before obtaining the blood from the blood bank, the nurse ensures the patient's IV is patent. Research confirms blood can be safely infused in as small as a 24 gauge IV. Then the nurse obtains the blood and double checks the blood group, type, expiration date, and patient ID with a second nurse. Next the nurse sets up the IV and IV pump. Blood is not run through a microdrip tubing set.

The nurse is educating a patient about taking measures to help avoid disruption to the circadian rhythm. Which statement by the patient indicates a need for further education? A.) "I know the circadian rhythm influences biological functions." B.) "I know the circadian rhythm exists only in humans." C.) "I know the sleep-wake circadian rhythm is impacted by the light-dark cycle." D.) "The most familiar circadian rhythm is the day-night 24-hour cycle."

B Biological rhythms exist in plants, animals, and humans. In humans, these biorhythms, along with internal and external factors, affect sleep. The most familiar rhythm is the day-night, 24-hour circadian rhythm cycle. Circadian rhythms influence patterns of biological and behavioral functions. Some creatures are diurnal, or primarily active during the day, whereas others are nocturnal, with most of their activity during the night.

The nurse is caring for a patient who is to undergo computed tomography (CT) of the kidneys and ureters. Which assessment finding by the nurse must be reported to the provider and radiologist before the patient has the procedure? A.) The patient is allergic to bananas and latex. B.) The patient thinks that she might be pregnant. C.) The patient has a family history of bladder cancer. D.) The patient currently has a urinary tract infection.

B CT requires exposure to radiation similar to an x-ray, so the patient's provider and radiologist should be notified promptly of the possibility of pregnancy. The other conditions do not preclude CT scan examination for the patient.

The nurse is caring for a patient who is at risk for fluid overload due to a history of congestive heart failure. Which intervention will the nurse teach the patient to perform at home to monitor fluid balance? A.) "Check to make sure that your urine is a bright yellow color." B.) "Weigh yourself every morning before breakfast." C.) "Count your heart rate every evening before you go to bed." D.) "Drink plain water rather than soda, coffee, or fruit juice."

B Checking the weight every morning before breakfast is a sensitive indicator of the patient's fluid volume status. Weight gain of 2 kg in 3 days generally indicates fluid retention and should be reported to the physician.

A nurse is caring for a patient who has an elevated temperature. The nurse plans to help the patient regain a normal temperature through conduction. What technique does the nurse use? A.) Placing a cooling fan in the patient's room B.) Putting ice packs in the patient's axillae C.) Spraying the patient with a fine mist of water D.) Turning the temperature down in the room

B Conduction is the transfer of heat through direct contact with another object, such as an ice pack. A cooling fan would help lower temperature by convection. Spraying the patient with a mist of water would lead to evaporative cooling. Turning the temperature down is an example of radiation.

Which isolation precaution should the nurse implement for the patient who has been diagnosed with hepatitis A? A.) Airborne B.) Contact C.) Droplet D.) Protective

B Contact precautions are used when a known or suspected contagious disease may be present and is transmitted through direct contact with the patient or indirect contact with items in the patient's environment. Airborne precautions are used when known or suspected contagious diseases can be transmitted by means of small droplets or particles that can remain suspended in the air for prolonged periods. Droplet precautions are used when known or suspected contagious diseases can be transmitted through large droplets suspended in the air. Protective isolation is used for patients who have compromised immune systems.

The nurse is assessing a patient's cranial nerve III. What technique is best? A.) Have patient identify a common scent with closed eyes. B.) Shine a light into the patient's eyes to assess pupil response. C.) Have the patient read a newspaper or use the Snellen chart. D.) Assess if patient can hear both spoken and whispered words.

B Cranial nerve III (oculomotor nerve) is assessed by observing the patient's pupil size and reaction to light and the direction of gaze. Identifying a common scent would test cranial nerve I. Assessing the patient's visual acuity tests cranial nerve II. Assessing hearing is cranial nerve VIII.

A new graduate nurse tells the manager that she does not believe she needs more in-service training on culturally congruent care because she already recognizes that there are significant differences among cultures to consider when providing care. What response by the manager is best? A.) "You have done a great job becoming culturally competent." B.) "Providing culturally congruent care takes ongoing work and effort." C.) "That is a great start but be sure to sign up for the in-service." D.) "Cultural sensitivity and cultural competence are not the same."

B Cultural sensitivity is the recognition that there are profound differences among cultures that can affect health care. But to provide culturally congruent care, the nurse must do more than just recognize these differences. This is an ongoing process. Option B is the only one that provides useful information to the nurse as to why she must continue to work on this aspect of her profession.

The nurse is caring for a patient with a urinary tract infection. Which test will indicate which antibiotics will be effective to treat the infection? A.) Complete blood count (CBC) B.) Culture and sensitivity (C&S) C.) Renal scan and angiography D.) Radioreceptor assay for HCG

B Culture and sensitivity are performed on specimens to determine which bacteria are causing the infection and which antibiotics will be effective treatment. CBC, renal scan, and radioreceptor assay for HCG will not indicate which antibiotics may be used to treat an infection.

A nursing faculty member is contrasting culture and ethnicity to students. Which statement is most accurate? A.) Culture is biologically determined; ethnicity is chosen. B.) Culture is socially transmitted; ethnicity is identification with a group. C.) Culture is a chosen identity whereas ethnicity is biologically based. D.) Culture and ethnicity are similar constructs used interchangeably.

B Culture refers to the learned, shared, and transmitted knowledge of values, beliefs, and ways of life of a group that generally are transmitted from one generation to another and influence the individual person's thinking, decisions, and actions in patterned or certain ways. Ethnicity is the person's identification with or membership in a racial, national, or cultural group and observation of the group's customs, beliefs, and language. The words may be used interchangeably by some people, but this is not correct

The nurse is caring for a patient with depression. Which statement by the patient indicates a need for further education? A.) "Depression can be caused by chemical changes in the brain." B.) "Depression is always treated with medication." C.) "Depression is a mood disorder." D.) "Depression can have a rapid onset."

B Depression is usually reversible with treatment either by eliminating the underlying cause, providing counseling, or prescribing antidepressive agents. Depression is a mood disorder and is believed to be caused by chemical changes in the brain. Depression usually has a rapid onset, and the patient's mood is constant.

The nurse is caring for a patient who is prescribed diphenoxylate-atropine (Lomotil). Which assessment finding by the nurse indicates a need to contact the prescriber and question the order? A.) The patient has skin breakdown from loose stools. B.) The patient is constipated with last BM 3 days ago. C.) The patient is on a low-fiber, gluten-free diet. D.) The patient has painful bleeding hemorrhoids.

B Diphenoxylate-atropine is an antidiarrheal medication. It should not be given to patients who are constipated until the patient is checked for impaction. The other assessment findings are not contraindications.

The nurse is preparing to insert a nasogastric (NG) tube in a patient. Which step in the process indicates a need for further education? A.) The nurse lubricates 4 inches of the tube prior to insertion. B.) The nurse marks the length of the tube with a marker for insertion. C.) The nurse measures the length of tube needed using the nose-earlobe-xiphoid process. D.) The nurse applies clean gloves for the procedure.

B Document the length of the tube to be used if the tube has a preprinted measurement scale. For any tube (with or without a preprinted scale), mark the measurement on the tube using a small piece of tape to ensure proper placement of the tube; fold the ends of the tape for easy removal. Do not use a permanent marker to mark the tube at this point of the procedure. When placement of the tube is confirmed as correct, then remove the measurement marking tape, mark the exit location on the tube with permanent marker, and proceed with the ordered treatment. Lubricate 4 inches of the tube tip with a water-soluble lubricant. For an NG tube, measure the length of tube needed for the patient by placing the tip of the tube at the tip of the patient's nose and extending it to the patient's earlobe and then to the patient's xiphoid process. Clean gloves are used.

The nurse is caring for a seriously ill patient whose laboratory results show a serum creatinine level of 3.5 mg/dL and a serum BUN of 35 mg/dL. Which conclusion can the nurse draw from these test results? A.) The patient is severely dehydrated. B.) The patient's kidneys have been damaged. C.) The patient has a urinary tract infection. D.) The patient has developed a renal calculus.

B Elevated BUN and creatinine are found in laboratory test results when the kidneys have been damaged and are unable to sufficiently clear metabolic wastes from the bloodstream. A dehydrated patient may have an elevated BUN, but the serum creatinine should be normal. Urinary tract infection and kidney stone (renal calculus) would not cause elevated BUN and creatinine levels.

The nurse identifies which patient would most likely need to have adjustments made to the education plan for discharge because of role function? A.) A 67-year-old married female who lives with her retired husband B.) A 32-year-old single mother of a toddler following hysterectomy C.) A 13-year-old who lives at home with his parents after appendectomy D.) A 50-year-old married mother with two children in college and teenager at home

B Exploration of the patient's roles is an important task that must be done before development of a patient education plan. For example, a 32-year-old, single mother of five young children who has just undergone a hysterectomy may require a different perspective in her discharge instructions than that in the instructions of a 67-year-old female living with her husband who recently retired after 35 years as a family practice physician. The first patient may have less support and less flexibility regarding rest, lifting limitations, and cost of prescriptions than the second. It is important not to stereotype and assign roles but rather to develop a plan in collaboration with the individual. The patient's support system should be taken into consideration when the nurse plans patient education.

The nurse is educating parents about firearm safety. Which parent statement indicates to the nurse a need for further education? A.) "I should make sure I obtain the proper permits." B.) "It is okay to store firearms with ammunition loaded." C.) "I should store all firearms without ammunition." D.) "I should make sure all firearms have trigger locks in place."

B Firearms should be stored in a secure location with trigger locks in place. Ammunition should be stored in a separate location also locked. Proper permits should be obtained as appropriate. Loaded firearms should never be stored where children can access them.

A clinic nurse is examining an older, confused patient on an examination table and realizes a piece of needed equipment was left outside in the hall. What action by the nurse is best? A.) Tell the patient to lie still and go get the equipment. B.) Call for another staff member to bring the equipment. C.)Have the patient get into a chair and get the equipment. D.) Finish the rest of the exam, get the equipment, and use it.

B For patient safety, some patients should never be left alone on an examination table: infants; small children; older adults who are confused, combative, or uncooperative, and people who are physically or chemically restrained. The nurse calls for another staff member to get the missing equipment. Getting up and down off the table is inconvenient and may be difficult. Finishing the exam and then retrieving the piece of equipment also involves the patient changing locations and is inconvenient for the patient.

The nurse is caring for a trauma patient with the Nursing diagnosis of acute pain r/t fracture and muscle spasms. Which is an appropriate goal for this Nursing diagnosis? A.) The patient will experience less pain when participating in physical therapy. B.) The patient will describe meditation techniques that can be used to cope with pain. C.) Nursing staff will explain the ordered pain management approach to the patient. D.) The patient will feel less pain each day when range-of-motion therapy is performed.

B Goals must be measurable and objective so that nursing staff can determine when each of the goals has been met. Having the patient describe meditation techniques is measurable because the nursing staff can determine whether he can actually describe them. Goals are achieved by the patient, not nursing staff. The nursing staff cannot accurately measure whether the patient is experiencing or feeling less pain. The goal statements "The patient will report less pain ... or state that he has less pain ..." are not measurable and appropriate.

The nurse is providing discharge instructions to a patient who has had sleep alterations. Which statement by the patient indicates further education is needed? A.) "I should avoid drinking caffeine too close to bedtime." B.) "I should not eat anything too close to bedtime." C.) "I should exercise regularly to help with sleeping." D.) "I can gain weight if I don't sleep enough."

B Going to bed hungry or eating a large, heavy, or spicy meal just before going to bed can interfere with sleep. Bedtime snacks that contain complex carbohydrates are recommended to promote calmness and relaxation. Highly caffeinated substances such as coffee, cola, and chocolate are central nervous system stimulants that can disrupt the sleep cycle. Exercise can assist with weight-loss efforts and promote fatigue and relaxation, but excessive exercise, especially in the evening, interferes with sleep. Both children and adults who sleep less than the recommended number of hours each night are more likely to be overweight.

A new UAP is measuring a patient's height. Which step of the procedure indicates a need for the registered nurse to provide further education on this skill? A.) The UAP instructs the patient to remove shoes. B.) The UAP measures from the top of the patient's head to the bottom of the patient's foot arch. C.) The UAP positions the head against the headboard or measuring device. D.) The UAP makes sure the patient is standing erect.

B Height is measured from the top of the head to the bottom of the heel. The patient is instructed to remove shoes, stand erect, and position the top of the patient's head against the headboard or measuring device for accuracy.

The nurse is providing discharge instructions for a patient with multiple sclerosis. Which discharge instruction is aimed at preventing a future exacerbation? A.) Engage in some form of exercise as tolerated. B.) Avoid highly stressful situations. C.) Check your skin regularly for pressure sores. D.) Eat a diet with lots of fiber.

B High stress levels are known to exacerbate multiple sclerosis and other autoimmune diseases. Exercise helps keep muscles loose and helps with balance. Assessing skin for pressure sores and eating a diet with high fiber prevents complications from multiple sclerosis.

Which statement by the patient indicates to the nurse that it may be an appropriate time to consider hospice care rather than further aggressive measures to treat his terminal illness? A.) "I am praying every day that this last round of chemotherapy will work." B.) "I want to spend what time I have left at home with my grandchildren." C.) "I need to meet with my financial planner to make sure my life insurance is all set." D.) "I am concerned that my wife won't be able to live on her own after my death."

B Hospice care is provided to patients who are terminally ill and wish to have no further aggressive treatment in attempt to cure the disease. The patient's statement that she just wants to be home with her grandchildren indicates a readiness for hospice care.

A nurse works in a pediatric oncology unit and is feeling depressed and discouraged. What initial action by the nurse is best? A.) Apply for a job transfer to another unit. B.) Consult with the hospital chaplain. C.) Make an appointment with Employee Assistance. D.) Ask other nurses how they deal with the stress.

B Hospital chaplains are great resources for nurses experiencing burnout, moral distress, or spiritual distress. The nurse can take all options, but a consultation with the chaplain is the best place to start to see if the issue can be resolved. The chaplain has a wider range of perceptions and tools than do the other staff nurses.

The nurse is caring for a patient who is in agonizing pain. All the following options are listed on the patient's medication order sheet to relive pain. The nurse knows which option that will provide the most rapid pain relief for the patient? A.) Morphine (MSContin) 10 mg PO B.) Hydromorphone (Dilaudid) 1 mg IV push C.) Meperidine (Demerol) 75 mg IM D.) Fentanyl (Duragesic) 50 mcg transdermal patch

B IV administration has the most rapid onset of action and will provide the patient with the quickest pain relief.

The nurse knows that when coordination between multiple health care disciplines is needed, which role should be utilized? A.) Pastoral care B.) Case manager C.) Social worker D.) Dietitian

B If coordination of care between multiple health care disciplines is needed, a case manager is used. Pastoral care plays a significant role in addressing stress and anxiety issues when the patient has a preferred religion or strong faith background. A social worker identifies appropriate services and resources. A dietitian can provide education regarding dietary needs and food choices.

The nurse is implementing generalized falls precautions for patients who are at risk for falls. Which intervention indicates a lack of understanding of these precautions? A.) The bed is placed in the low position. B.) The patient is wearing socks. C.) The patient's cell phone is by the bedside. D.) The patient's call light is within reach.

B If the patient is ambulatory, require the use of nonskid footwear. Socks can be slippery unless they have a grip surface on them. Keep patient belongings (e.g., tissues, water, urinals, personal items) within the patient's reach. Keep the call light in reach and remind the patient to use it and keep the bed in the low position.

In researching the effectiveness of an antihypertensive medication, the nurse knows that the medication would be what type of variable? A.) Dependent B.) Independent C.) Treatment D.) Controlled

B In experimental research, the independent variable is referred to as an experimental variable or treatment variable. An independent variable is a concept or idea whose value determines the value of other (dependent) variables. In research, the independent variable comprises the experimental treatment or intervention, and it is manipulated by the researcher to yield various outcomes. The dependent variable is the outcome that is affected by manipulation of the independent variable. For example, in researching the effectiveness of an antihypertensive medication, the medication is the independent variable and the person's blood pressure is the dependent variable. In a controlled study, some of the participants are assigned to the treatment group, and others are assigned to the control group by a random process. The control group does not receive the treatment. In the clinical trial of a medication, the control group receives a placebo. The purpose of a control group is to prevent bias and ensure that the outcome results from the treatment rather than some other factor.

The nurse is providing discharge instructions to an older adult who is being discharged with orthostatic hypotension. Which response by the patient indicates a need for further education? A.) "I should take my blood pressure once a day at home." B.) "I should get up quickly to avoid my blood pressure dropping." C.) "I should drink plenty of water during the day." D.) "I should get up slowly and carefully."

B In orthostatic hypotension, dizziness and loss of consciousness may occur if a patient changes position too quickly. Instead they should change positions slowly. A patient can take their blood pressure at home to monitor it. Drinking water will keep them hydrated.

A patient in the emergency department needs an emergency operation. The patient refuses to consent and wants the nurse to call a respected elder in the community for consent. What action by the nurse is best? A.) Explain that this violates privacy laws. B.) Call the elder to get consent for the operation. C.) Tell the woman she has the right to consent. D.) Arrange for admission without the operation.

B In some cultures, decisions are made by men or community leaders. Although the patient may have the legal right to consent, if she comes from a culture in which gender and/or social roles do not permit decision making, she will likely refuse to consent. The best action is for the nurse to contact the elder and have him participate in the decision-making process per the patient's wishes. If the patient has given permission to share the information, doing so does not violate privacy laws. Admitting the patient without the operation does not help her medically.

A patient is recovering from colostomy surgery and states, "I guess I'll never be able to have sex again ... who would want me?" What Nursing diagnosis is most important for this patient? A.) Sexual dysfunction B.) Ineffective sexuality pattern C.) Knowledge deficit D.) Ineffective coping

B Ineffective sexuality patterns refer to a patient who expresses concern about his/her own sexuality. This patient is concerned about the effect of this surgery on his/her attractiveness and desirability. Sexual dysfunction relates more to the physical problems. The patient may have a knowledge deficit or ineffective coping, but these are not apparent from the question.

The nurse recognizes which goal to be appropriate for the patient with a Nursing diagnosis of social isolation? A.) The patient will participate in cognitive exercises. B.) The patient will interact with other residents during activities. C.) The patient will communicate basic needs through use of photos. D.) The patient will remain within the unit while in long-term care.

B Interacting with others during activities is an appropriate goal to help the patient not feel so alone. Cognitive exerci se is a goal for a patient with disturbed thought processes. Communication of basic needs through the use of photos is a goal for a patient with a diagnosis of impaired verbal communication and remaining in the unit is appropriate for chronic confusion.

The nurse is to administer 1 mL of prochlorperazine (Compazine) 10 mg IM to an adult patient. Which syringe will the nurse select to administer the medication? A.) 1 mL tuberculin syringe with 27 gauge, 1/2 inch needle B.) 3 mL syringe with 23 gauge, 1 1/2 inch needle C.) 1 mL syringe with 27 gauge, 5/8 inch needle D.) 3 mL syringe with 18 gauge, 1 inch needle

B Intramuscular injections for adults are usually administered with a 3 mL syringe and a 1 to 3 inch, 19 to 25 gauge needle. Tuberculin syringes are typically used for subcutaneous injections. The inch needles are too short for intramuscular injections into adults. The 18 and 27 gauge needles are too small for adult intramuscular injections.

The nurse correctly defines leadership when making which statement? A.) "Leadership is coordinating others toward a common goal." B.) "Leadership is the ability to influence others." C.) "Leadership focuses on the task at hand." D.) "Leadership is based in formal authority."

B Leaders have the ability influence and motivate others while maintaining relationships to accomplish a goal. Management is the process of coordinating others and directing them toward a common goal. Management is focused on the task at hand. A manager holds a formal position of authority in an organization; that position includes accountability and responsibility for accomplishing the tasks within the work environment. Managers demonstrate accountability when they are answerable for their own actions and the actions of those under their direction

The nurse is implementing a patient teaching plan regarding diabetes mellitus. One of the short-term goals of the plan is that the patient will be able to verbalize three symptoms of hypoglycemia. The nurse recognizes that this is what type of teaching? A.) Psychomotor teaching B.) Cognitive teaching C.) Affective teaching D.) VARK teaching

B Learners in the cognitive domain integrate new knowledge through first learning and then recalling the information. They then categorize and evaluate, making comparisons with previous knowledge that result in conclusions related to the new content. The psychomotor domain incorporates physical movement and the use of motor skills in learning. Teaching the newly diagnosed diabetic how to check blood sugar is an example of a psychomotor skill. Affective domain learning recognizes the emotional component of integrating new knowledge. Successful education in this domain takes into account the patient's feelings, values, motivations, and attitudes. Tools have been developed to help health care workers evaluate the health literacy of their patients. One such tool is the VARK (verbal, aural, read/write, kinesthetic) assessment of learning styles of people who are having difficulty learning.

The nurse is caring for a patient who has a family history of reactions to general anesthesia. Which medication will the nurse anesthetist have ready as a precautionary measure before the patient's surgery is started? A.) Protamine sulfate B.) Dantrolene sodium (Dantrium) C.) Activated charcoal with sorbitol D.) Folinic acid (Leucovorin)

B Malignant hyperthermia is a dangerous anesthesia reaction caused by a genetic defect that may be passed down via family history. Knowing this, the anesthesiologist would have dantrolene ready as a precaution because it is a mainstay of treatment for malignant hyperthermia. The other medications are not related.

The nurse knows that manual lifting should only be done in which situation? A.) Patients who are less than 150 lb B.) Life-threatening situations C.) Postsurgical patients D.) Patients who are less than 200 lb

B Many manual patient handling tasks are unsafe, because the weights lifted and movements required are beyond the ability of most caregivers. The key is to identify the task to be accomplished, and then use the required equipment and personnel so that the task fits the capabilities of the staff (U.S. Department of Veterans Affairs, 2016). The patient's level of cooperation is taken into consideration when using the safe patient-handling and mobility (SPHM) algorithms to decide the best method of moving the patient. The patient's weight, medical conditions, and ability to assist are also considered (U.S. Department of Veterans Affairs, 2016). Postsurgical patients as well as patients less than 150 or 200 lb may not fit the criteria.

The nurse is caring for a patient who will undergo ultrasound testing of the bladder and kidneys the next morning. Which instruction will the nurse provide to the patient about the test? A.) "A small IV will be inserted into your arm to inject the contrast dye." B.) "You will need to drink lots of water but not use the toilet." C.) "You should not have anything to eat or drink after midnight." D.) "You will receive a cleansing enema before you have the test."

B No preparation is needed for kidney and bladder ultrasound other than having the patient drink lots of fluid beforehand. The patient is instructed not to use the toilet so that the bladder will be filled and easy to visualize. No contrast dye, enemas, or fasting is required.

The nurse is caring for a patient who has just undergone bronchoscopy. The patient requests a drink of water. What is the nurse's best action? A.) Provide ice chips. B.) Check the patient for a gag reflex. C.) Provide a small cup of ice water with a straw. D.) Keep the patient NPO.

B Numbing medication is applied to the back of the throat just before bronchoscopy. Thismay lead to swallowing difficulty and risk for aspiration until the gag reflex returns. The nurse should keep patient NPO until swallow, gag, and cough reflexes have returned.The nurse does not need to keep the patient NPO after the gag reflex returns so it should be checked in order to allow the patient to have fluids as soon as possible to relieve thirst.

The nurse is caring for a terminally ill patient whose family is insistent that additional chemotherapy be administered even though the patient will most likely die within the next few days. What is the best response of the nurse? A.) "The insurance company will not pay for chemotherapy at this stage." B.) "The focus right now needs to be on keeping your loved one comfortable." C.) "I will call the provider and relay your wishes." D.) "The patient needs to get stronger first before chemotherapy can be administered."

B Nurses advocate for patients to ensure that they are aware of their options for care that include interventions, treatments, anticipated outcomes, as well as risk and benefits of any decision made concerning medical care. The nurse must function as the patient's advocate and encourage what is in the best interest of the patient. Chemotherapy will not extend the patient's life when death is expected within the next few days and will only make the patient suffer needlessly when it is administered. The patient will not get stronger over the next few days, and this criterion for chemotherapy will never be met.

A charge nurse works on an inpatient unit in a diverse city. To provide culturally congruent care to the patient, which action by the nurse would be most appropriate? A.) Using puns and sarcasm to help draw the patient into sharing information B.) Working to understand the socioeconomic status of the patient so teaching is culturally sensitive and appropriate C.) Assuming a patient from a minority population does not have the economic means to pay for home care follow-up D.) Admonishing a Hispanic patient for showing up for a preoperative teaching class 15 minutes late

B Nurses need to be cognizant of the impact of a patient's socioeconomic status to health care practices. The use of puns, sarcasm, and colloquialisms are not easily comprehended or interpreted by those who speak a different primary language. While the level of poverty in minority populations within all cultures is disproportionally higher, it is inappropriate to base an action on an assumption. According to research, some Hispanics believe that time is flexible and events will begin when they arrive. However, admonishment is not the best approach to dealing with this behavior.

A home health care nurse is visiting the home of a patient whose culture is totally unfamiliar to the nurse. What action by the nurse is best? A.) Perform nursing care with a high degree of professionalism. B.) Watch family interaction patterns closely and try to copy them. C.) Tell the family you need to learn about their culture. D.) Apologize after performing tasks that make the patient uncomfortable.

B Nurses should observe family dynamics carefully, including communication, and try to copy them as much as possible. For instance, if the family does not make eye contact with the nurse, he/she should avoid trying to make direct eye contact with the family. The other options are reasonable, although telling the family you need to learn about their culture may place the burden of educating the nurse on them.

The nurse teaches the family member to provide the patient with how much dietary fiber per day? A.) 25 to 35 g B.) 20 to 35 g C.) 25 to 40 g D.) 20 to 40 g

B Older children, adolescents, and adults should consume 20 to 35 g of fiber a day. Food sources include whole grains, wheat bran, cereals, fresh fruits, vegetables, and legumes.

A nurse has referred a patient to a community agency. When talking to the patient later, he states that he did not find the agency helpful. What action by the nurse is best? A.) Determine what the patient would find helpful. B.) Review the agency's mission and scope. C.) Make another appointment with the agency. D.) Warn the patient that nonadherence affects payment.

B One of the most important aspects of a community health nurse's role is to be familiar with referral agencies. Awareness of the scope of an agency's influence and services helps the community nurse to pinpoint which agencies are best able to address specific needs. The nurse may have sent this patient to an agency that did not meet his needs. The nurse should ask the patient's opinion about what services are needed. Making another appointment without ensuring that this is the right agency for the patient will not solve the problem. Telling the patient that payment might not be ensured for nonadherence is not therapeutic communication.

The nurse is caring for a terminally ill patient who appears to be calmly having a conversation with someone even though there is nobody else in the room. The patient reaches out and appears to take something out of thin air and hold it close. Which is the appropriate action of the nurse? A.) Reorient the patient and reassure that nobody else is in the room. B.) Be present but quiet and let the patient continue the conversation. C.) Carefully assess the patient's mental status and level of attention. D.) Obtain a set of vital signs and check the patient's pulse oximetry.

B Patients who are near death sometimes have a special communication with loved ones who have already died. It is important to recognize that these experiences can be comforting to the dying patient, and nurses would not contradict or argue with the person. It is imperative to simply be present with the person, listen, and be open to any attempts to communicate. It is acceptable to ask gentle questions such as "What are you seeing?" or "How does that make you feel?" Having an open discussion with the family while describing what is occurring may provide further insight to the nurse as the health care provider, as well as promoting a sense of understanding and acceptance for the family. As long as the patient is calm and content, the best action of the nurse is to be present but let the patient continue the conversation undisturbed.

The nurse recognizes which goal to be appropriate for the patient who is postoperative day one from a hip fracture with the nursing diagnosis Impaired mobility? A.) Patient will interact with others. B.) Patient will ambulate to the bathroom with assistance. C.) Patient will have no skin breakdown. D.) Patient will have a physical therapy consult.

B Patients with a diagnosis of Impaired mobility should have a goal aimed at improving their mobility. Although immobility can impact social isolation and skin breakdown, those goals are not appropriate for this diagnosis. Have a physical therapy consult is not a goal but an intervention.

A patient died suddenly in the emergency department. Which action by the nurse best provides the family connection with others? A.) Offering the family written information on grief support groups. B.) Asking the family if there is someone the nurse can call for them. C.) Having the hospital social worker or chaplain sit with the family. D.) Offering to stay with the family during this difficult time.

B Promoting connectedness means recognizing that family and friends are providing at least some of the patient's spiritual care. The nurse best assists when offering to call someone for the patient or family. The other options may be appropriate but are not directly related to connectedness.

What does the nursing student learn about race? A.) It is biologically based. B.) It is a social construct. C.) It is chosen by the person. D.) It helps establish superiority.

B Race is often thought to be inherited and biologically based, but this is not true. Race is a social construct that is used to group people together based on common physical characteristics, heredity, or common descent. People are placed into racial categories by the larger society. One race is not superior to any other.

The nurse identifies which instruction to be appropriate to delegate to the UAP (Unlicensed assistive personnel)? A.) Assess the patient's skin during a bath. B.) Reposition the patient using the trapeze. C.) Assess the patient's ability to perform range-of-motion exercises. D.) Notify the health care provider of any changes.

B Repositioning a patient can be delegated to unlicensed assistive personnel (UAP); the nurse should provide proper instruction regarding specific positioning techniques, individualized patient concerns, and circumstances that require notifying a nurse. UAP may not perform assessments or evaluations but should notify the nurse about any skin or musculoskeletal issues (not the health care provider).

The nurse is teaching a student nurse about restraint use in patients. Which statement by the student nurse indicates a learning need regarding restraints? A.) "Having all four side rails up on the bed is considered a restraint." B.) "The use of restraints has been shown to decrease fall-related injuries." C.) "Death has been associated with the use of restraints." D.) "Medications administered to control behavior are considered a chemical restraint."

B Restraints may be physical or chemical. A physical restraint is a mechanical or physical device, such as material or equipment attached or adjacent to the patient's body, used to restrict movement. Examples of physical restraints are wrist or ankle restraints, a jacket or vest, and side rails. A medication that is administered to a patient to control behavior is a chemical restraint. The use of restraints has been associated with patient injury including death and does not prevent patient falls.

The nurse is caring for a patient who requires emergency surgery for injuries sustained in a motor vehicle accident. The patient was on his way back to work after having lunch with colleagues when the accident happened. What is the highest priority Nursing diagnosis for this patient? A.) Risk for imbalanced body temperature B.) Risk for aspiration C.) Risk for perioperative positioning injury D.) Risk for delayed surgical recovery

B Risk for aspiration is the highest priority because the patient has not been NPO and his stomach is filled with food after lunching with his colleagues. The patient may easily aspirate stomach contents into the airway when general anesthesia is administered, so precautions must be taken to prevent this from happening. The other Nursing diagnoses certainly apply but are not as important as risk for aspiration.

A nurse wants to volunteer for a community group providing secondary prevention. What activity would the nurse attend? A.) Stroke rehabilitation support group B.) Blood pressure screening at the mall C.) Bicycle safety class at the elementary school D.) Drop by nutrition station at the grocery store

B Secondary prevention activities are aimed at early diagnosis and prompt intervention. Blood pressure screening events are a good example. Stroke rehabilitation is tertiary prevention. Bicycle safety classes and nutrition education are examples of primary prevention.

The nurse knows the one theory explaining the variation in response to stress among individuals is identified by which term? A.) Stress appraisal B.) Sense of coherence C.) Allostasis D.) Homeostasis

B Sense of coherence (SOC) is a characteristic of personality that references one's perception of the world as comprehensible, meaningful, and manageable. Stress appraisal is the automatic, often unconscious assessment of a demand or stressor. Allostasis is an alternative term for the stress response. Homeostasis is the tendency of the body to seek and maintain a condition of balance or equilibrium.

The nurse is caring for a patient with renal failure who has a serum potassium level of 7.1 mEq/L and serum magnesium level of 3.5 mEq/L. The nurse prepares to administer 10 units of insulin and an ampule of 50% dextrose to the patient. The patient asks why he will be receiving insulin when he is not diabetic. What is the nurse's best answer? A.) "The doctor has prescribed these medications for you to help heal your kidneys." B.) "These medications will lower your potassium level and prevent an irregular heart rate." C.) "These medications will prevent you from having a seizure from too little magnesium." D.) "These medications will increase your urine output until your kidneys recover."

B Serum potassium levels above 7.0 mEq/L can lead to dangerous cardiac arrhythmias, so the potassium level must be lowered promptly. Administration of IV insulin with 50% dextrose will push potassium into the cells to avoid hyperkalemia symptoms.

The nurse is providing education to a patient about the difference between simple and complex carbohydrates. Which statement by the patient indicates a need for further education? A.) "Simple carbohydrates give me quick energy." B.) "Complex carbohydrates come from fruit." C.) "Complex carbohydrates take longer to break down." D.) "Simple carbohydrates come from milk products."

B Simple carbohydrates are broken down and absorbed quickly, providing a quick source of energy. Examples are sugars such as those derived from fruit (fructose), table sugar (sucrose), milk products (lactose), and blood sugar (glucose). Complex carbohydrates are composed of starches, glycogen, and fiber. They take longer to break down prior to absorption and utilization by the body's cells.

The nurse recognizes which situation to be inappropriate to use alcohol-based hand sanitizer? A.) Patient with pneumonia B.) Patient with Clostridium difficile C.) Status post-appendectomy D.) Patient with HIV

B Soap and water must be used to thoroughly clean hands if there is any visible soiling or dirt and with certain infections such as Clostridium difficile and vancomycin-resistant enterococci when preparing for a sterile or surgical procedure, before and after eating, and after using the restroom. In the other situations, a hand sanitizer is as effective as soap and water.

The nurse recognizes that a patient is using a portable generator in the house as a power source. What source of poisoning does the nurse appropriately identify? A.) Lead B.) Carbon monoxide C.) Antifreeze D.) Pesticide

B Sources of carbon monoxide include automobiles, stoves, gas ranges, portable generators, lanterns, the burning of charcoal and wood, and heating systems. Lead is found in lead-based paints in toys, buildings, and ceramic dishes; sources of lead include water from lead pipes or pipes soldered with lead, gasoline or soil contaminated by gasoline, and household dust that may contain paint chips or soil. Antifreeze and pesticides are liquids.

he student nurse asks why spirituality is important in health care. What response by the registered nurse is best? A.) "All people have a spiritual aspect to their beings." B.) "Spirituality affects behavior, which also affects health." C.) "Knowledge of it is needed to understand a patient holistically." D) "People who are less spiritual have worse outcomes."

B Spirituality affects behavior, which has a direct impact on health. Spirituality is a universal concept, but all people may not recognize it in themselves. Holistic knowledge is indeed based in part on spirituality, but that does not give the student information on a concrete link. Less spiritual people may or may not have worse outcomes.

The nurse knows a stage III pressure ulcer is: A.) A pressure ulcer that involves exposure of bone and connective tissue B.) A pressure ulcer that does not extend through the fascia C.) A pressure ulcer that does not include tunneling D.) A partial-thick wound that involves the epidermis

B Stage III pressure ulcers are full-thickness wounds that extend into the subcutaneous tissue but does not extend through the fascia to muscle, bone, or connective tissue. There may be undermining or tunneling present in the wound. Stage IV pressure ulcers involve exposure of muscle, bone, or connective tissue such as tendons or cartilage. Stage II pressure ulcers are partial-thickness wounds that involve the epidermis and/or dermis.

The nurse understands state legislatures give authority to administrative bodies, such as state boards of nursing, to carry out what action? A.) Create statutory laws. B.) Establish regulatory laws. C.) Try case law cases. D.) Create laws based on social mores

B Statutory law is created by legislative bodies such as the U.S. Congress and state legislatures. Statutory laws are often referred to as statutes. State legislatures give authority to administrative bodies, such as state boards of nursing, to establish regulatory law, which outlines how the requirements of statutory law will be met. Judicial decisions from individual court cases determine case law. Case law was historically referred to as common law because it originally was determined by customs or social mores that were common at the time.

The nurse is caring for a patient who has a central venous catheter (CVC). Which nursing intervention is the most important for the nurse to include in the patient's plan of care? A.) Carefully document all assessments of the catheter site. B.) Use strict sterile procedure when performing dressing changes. C.) Label each new dressing with the date, time, and nurse's initials. D.) Ensure that the CVC is discontinued as soon as possible.

B Strict sterile procedure is mandatory when changing CVC dressings because of the high risk of septicemia and/or sepsis. The other actions are appropriate, but not of the highest priority.

The nurse is working with a patient who has been advised to take 2 20-minute naps during the day for fatigue. After a week, the patient states the naps have not helped. What response by the nurse is best? A.) "Maybe that is too much sleep for you during the day." B.) "Why don't you try one 40-minute nap instead?" C.) "Let's explore some sleeping medications for you to try." D.) "It often takes a few weeks for napping to help."

B Taking 2 20-minute naps is an appropriate action for the patient with fatigue to implement. Many non-pharmacological interventions take weeks to become effective, so the nurse would explain that fact and encourage the patient to continue the napping.

The American Nurses Association (ANA) standards of professional performance require nurses to use research findings in practice. How do these standards impact nurses in the workplace? A.) Nurses need to regulate their practice according to the latest journal articles. B.) Nurses need to use the best available evidence to guide practice decisions. C.) Nurses only need to participate in research while in advanced practice. D.) Nurses may use evidence-based practice to develop procedures but not policies.

B The American Nurses Association (ANA) standards of professional performance require nurses to use research findings in practice. Two criteria are measured. The first criterion is that nurses need to use the best available evidence, which includes research findings, to guide their practice decisions (ANA, 2015). The second criterion is that nurses participate in research activities that are appropriate for their position and level of education. Activities may include identifying problems in the clinical setting that may be researched; participating in data collection; participating as a member of a research committee or a research program; sharing research findings the nurse has found with others; conducting research; critiquing research that may be used in practice; using research findings to develop policies, procedures, and standards for patient care at health care facilities; and incorporating research as part of ongoing learning as a nurse. Nurses may participate in one or more of these activities during their careers. The ANA (2017) Research Toolkit was developed to help nurses to provide evidence-based care that improves patient outcomes. The ANA (2017) Research Toolkit was developed to help nurses to provide evidence-based care that improves patient outcomes.

A nurse is working with a patient after the patient had a heart attack and is using the PLISSIT model to address sexuality needs. For the SS phase, what action by the nurse is best? A.) Ask the patient if he wants to discuss sexuality. B.) Teach the patient positions that require less stress. C.) Offer the patient a referral to a sex therapist. D.) Direct the patient to speak with the doctor about sex.

B The PLISSIT model is a framework for addressing sexuality. In the SS (specific suggestions) phase, the nurse provides information that allows the patient to proceed with sexual relations. Informing the patient about sexual positions that are less stressful on the heart is an example. P stands for permission. LI stands for basic, limited information, and IT stands for intensive therapy.

The nurse recognizes what function of the reticular activating system (RAS)? A.) Records brain waves and other variables. B.) Relays motor impulse to the hypothalamus. C.) Influences patterns of biological functioning. D.) Is affected by the light-dark cycle.

B The RAS receives sensory impulses from the spinal cord and relays motor impulses to the thalamus and all parts of the cerebral cortex. Polysomnography is the recording of brain waves and other physiologic variables, such as muscle activity and eye movements, during sleep. The circadian rhythms influence patterns of biological and behavioral functions, and the sleep-wake circadian rhythm is affected by the light-dark cycle.

The nurse is checking on the patient after administering pain medication 30 minutes previously. Which assessment finding best indicates to the nurse that the pain medication was effective? A.) The patient is sleeping quietly. B.) The patient states a reduction of the pain. C.) The patient's respirations are slow and regular. D.) The patient's blood pressure has returned to baseline.

B The best way for the nurse to determine that the pain medication was effective is for the patient to state a reduction of the pain. The other assessment findings cannot definitively determine whether the patient is still in pain.

A nursing student is caring for a patient with metabolic acidosis. The student asks the registered nurse why the patient's respiratory rate is so high. What response by the nurse is best? A.) "The patient's metabolic rate is increased from being ill." B.) "The lungs are trying to rid the body of extra carbon dioxide." C.) "The patient is trying to reduce his temperature through panting." D.) "Patients who are acutely ill often have abnormal vital signs."

B The body tries to compensate for excess carbon dioxide (seen in acidosis) by increasing the rate and depth of respirations to "blow off" the carbon dioxide.

A nurse is a case manager for a home health care agency. The nurse often orders supplies for patients seen by the agency. What action by the nurse is best? A.) Negotiate for cheaper prices from suppliers. B.) Investigate what each patient's insurance will cover. C.) Refer the patient to the closest supply source. D.) Use the same supplier for all patients' needs.

B The case manager in home health care must be a well-versed financial steward and understand what each patient's insurance will cover to maximize the patient's benefit. The home health care nurse serves as a case manager (coordinator) of client care, needed services, and needed supplies in the home setting. The nurse must be well versed as a financial resource manager, who needs to be aware of what is or is not covered on the client's insurance plan.

The preceptor is working with a new nurse to provide care for a patient with a chest tub to relieve a pneumothorax. Which action by the new nurse indicates need for additional teaching about chest tube care? A.) The suction is discontinued when the patient is ambulated to the bathroom. B.) The collection device is emptied at the end of the shift and output recorded in the chart. C.) The patient's bed is placed in the semi-Fowler's position to facilitate lung reexpansion. D.) The patient is encouraged to use his incentive spirometer at least 10 times every hour.

B The chest tube collection device is not emptied at the end of the shift. Instead, the amount of drainage present at the end of the shift (or specified time) is marked on the collection device and the amount of drainage is documented in the patient's chart

When the nurse measures the patient's blood glucose levels after an acute myocardial infarction (MI), the nurse knows this action is based on which rationale? A.) Damaged muscle tissue releases glucose. B.) Corticosteroids increase glucose. C.) Myocardial infarctions are often seen in diabetics. D.) All patients should have their blood glucose checked.

B The endocrine system responds to stress on the body such as what happens during an acute MI. Corticosteroids are important in the stress response because they increase serum glucose levels and inhibit the inflammatory response. Although MIs can be seen in diabetics, there is nothing to indicate this patient is diabetic. All patients do not routinely have their blood glucose checked regularly.

A nurse is interested in epidemiology. What work activity would best fit this role? A.) Studying census data to determine common causes of death B.) Researching population variables that contribute to disease C.) Developing sanitary measures to prevent foodborne illness D.) Designing research to determine the connection between pollution and cancer

B The epidemiologist works to develop programs to prevent the development and spread of disease. Studying census data, researching population variables, and designing studies do not fall in this field.

The family of a patient who was in a motor vehicle accident tells the nurse "I'm just not the person I was before the crash." The nurse recognizes this is likely because of the injury to what area of brain? A.) Parietal lobes B.) Frontal lobes C.) Occipital lobes D.) Temporal lobes

B The frontal lobes of the cerebrum are the areas of the brain responsible for voluntary motor function, concentration, communication, decision making, and personality. The parietal lobes are responsible for the sense of touch, distinguishing the shape and texture of objects. The temporal lobes are concerned with the senses of hearing and smell. The occipital lobes process visual information.

The nurse is caring for a postoperative patient who has just been diagnosed with a deep vein thrombosis (DVT) in the right leg. Which focused assessment question has the highest priority for this patient? A.) "Do you have a headache or any dizziness?" B.) "Do you have any chest pain or shortness of breath?" C.) "When did you first notice the swelling and redness in your leg?" D.) "Do you have any cramping or muscle spasms in your leg?"

B The highest risk of a DVT is the potential for the clot to break free and travel through the bloodstream to cause a pulmonary embolus (PE). The nurse should ask the patient about chest pain or shortness of breath to assess if a PE may have occurred.

A nurse is conducting a physical assessment in a clinic with a partly undressed patient. What action by the nurse is most appropriate? A.) Offer the patient a small pillow for under his/her head. B.) Provide a method for ensuring the patient stays warm. C.) Raise the head of the bed to about 30 degrees. D.) Ensure there is enough lighting for an adequate examination.

B The important fact in this question is that the patient is partly undressed, and the nurse provides a means to keep the patient warm. All answers are appropriate for any examination but keeping the patient warm is specific to this situation.

The nurse is walking a postoperative patient in the hallway when she notices a large red stain of fresh blood on the patient's gown over the abdominal incision. The patient states, "I felt something just ripped open." What is the priority action of the nurse? A.) Lift up the patient's gown and assess the incision. B.) Assist the patient to the floor and call for assistance. C.) Return the patient to bed and irrigate the wound with sterile saline. D.) Check the patient's vital signs and pulse oximetry.

B The large red blood stain over the incision and feeling of ripping open most likely indicates that the patient's wound has dehisced or eviscerated. The nurse should immediately lower the patient to the floor to reduce tension on the wound. Patient modesty and privacy should be maintained, so the incision should be assessed once the patient is transported back to his room. Checking the patient's vital signs and pulse oximetry can be performed once the patient has been lowered to the floor.

Which patient assessment result would require the nurse to assess that patient further? A.) A 40-year-old woman with a radial pulse of 68 B.) A 65-year-old man with a respiratory rate of 10 C.) A 12-year-old with a pulse of 92 after ambulating in the hallway D.) A 50-year-old man with a BP of 112/60 upon awakening in the morning

B The normal respiratory rate is 12 to 20 breaths/min for an adult, so a rate of 10 would require further assessment. The other options are all within normal limits.

Which task is inappropriate for the nurse to delegate to the unlicensed assistive personnel (UAP)? A.) Providing oral care B.) Evaluating sleep patterns C.) Providing bedtime routines D.) Documenting sleep hours

B The nurse may not delegate assessment activities such as evaluating sleeping patterns. The nurse may delegate activities to help the patient prepare for sleep such as oral care and other bedtime routines. The UAP may document the hours slept.

The nurse manager is considered a "great communicator." She can be found on the unit talking with staff, keeping them informed and asking their opinions. She believes that nurses are motivated by internal means and that they want to participate in making decisions about the unit although the final decision always rests with her. The nurses recognize that this nurse manager is what type of leader? A.) Autocratic B.) Democratic C.) Bureaucratic D.) Laissez-faire

B The participative or democratic leader believes that employees are motivated by internal means and want to participate in decision making. The primary function of the leader in this situation is to foster communication and develop relationships with followers. The authoritarian or autocratic leader exercises strong control over subordinates. Like the autocratic leader, the bureaucratic leader assumes that employees are motivated by external forces. This type of leader relies on policies and procedures to direct goals and work processes. The nurse using bureaucratic leadership tends to relate impersonally to staff and exercises power on the basis of established rules. Like the democratic leader, the permissive or laissez-faire leader thinks that employees are motivated by their own desire to do well. The laissez-faire leader provides little or no direction to followers, who develop their own goals and make their own decisions

The nurse notices her 50-year-old patient is holding the lunch menu at arm's length while trying to read the choices. The nurse knows this is an indication of which condition? A.) Retinopathy B.) Presbyopia C.) Cataracts D.) Macular degeneration

B The patient demonstrates presbyopia by holding reading materials at a distance or by being unable to read normal-sized or small print. Retinopathy is damage to the retina and occurs in diabetics. Cataracts are a clouding of the lens. Macular degeneration is a chronic condition that causes loss of vision in the center of your field of vision.

The nurse is caring for a patient who has a 1200 mL daily fluid restriction. The patient has consumed 250 mL with each of the three meals and had another 150 mL with medication administration. The patient has received 150 mL of IV fluids during the day. How many mL of fluid may the patient still consume to stay within the prescribed fluid restriction? A.) 100 mL B.) 150 mL C.) 250 mL D.) 300 mL

B The patient has had an oral fluid intake of 900 mL and an IV fluid intake of 150 mL, giving a total of 1050 mL. This leaves 150 mL that the patient may consume for the rest of the evening to stay within the prescribed fluid restriction.

A nurse assesses a patient's lungs and notes the presence of low-pitched snoring sounds that clear with coughing. What action by the nurse is best? A.) Prepare to treat the patient for asthma. B.) Prepare to treat the patient for pneumonia. C.) Teach the parent how to prevent croup. D.) Assess the patient for heart failure.

B The patient has rhonchi. Rhonchi are caused by increased secretions in large airways and can be seen in pneumonia or in other conditions, leading to increased mucus production. The nurse prepares to treat the patient for pneumonia. Asthma would manifest with wheezing, croup with stridor, and heart failure with rales or crackles.

The nurse sees a young child in the clinic whose mother has only a few weeks to live. The child has been misbehaving at school recently and is suspended after picking fights with other students and defying teachers. The nurse identifies which stage of grieving that the patient is experiencing? A.) Denial B.) Anger C.) Bargaining D.) Depression

B The patient is angry over the impending death of the mother and is acting out this anger at school by picking fights and defying his teachers. Denial is a temporary defense while processing the information. Bargaining is negotiation to change the predicted outcome. Depression includes crying and sadness.

The nurse is caring for a patient with multiple chronic illnesses who is having difficulty remembering to take multiple medications at the correct times. Which is the appropriate Nursing diagnosis for this patient? A.) Activity intolerance related to inability to take medications on time B.) Impaired health maintenance related to complexity of medication schedule C.) Risk for aspiration related to need to swallow many pills during day D.) Powerlessness related to inability to figure out medication dose times

B The patient is not able to manage the prescribed medication regimen because of the complexity of the schedule, so Impaired health maintenance is an appropriate diagnosis. Activity intolerance does not relate to the ability to take multiple medications at once and manage medication times. The patient does not state any difficulty swallowing pills, so risk for aspiration is not applicable. Inability to figure out medication dose times does not constitute powerlessness.

The nurse is caring for a patient who had prostate surgery the previous day. The patient has had significantly decreased urine output over the last shift despite ample oral and IV fluid intake. The patient's urine from the indwelling catheter is cherry red with occasional small clots. What is the appropriate action of the nurse? A.) Remove the urinary catheter and replace it with a new one. B.) Gently irrigate the catheter using warmed sterile normal saline. C.) Send a sample of the patient's urine to the laboratory for analysis. D.) Call the provider and obtain an order for kidney and bladder ultrasound.

B The patient most likely has decreased urine output caused by clot formation that is blocking urine from draining through the catheter. The catheter should be gently irrigated using sterile technique and warmed sterile saline to loosen clots and facilitate urinary drainage. The catheter should not be removed. Ultrasound and urinalysis are not necessary.

The nurse is caring for a patient who has a history of congestive heart failure and takes once-daily furosemide (Lasix) to prevent fluid overload and pulmonary edema. The patient admits to stopping the medication due to nocturia. What is the nurse's best response? A.) "You should ask your doctor to decrease the dose." B.) "Take the diuretic early in the morning before breakfast." C.) "Eat foods high in potassium and limit your salt intake." D.) "Restrict your fluid intake after dinner and in the evening."

B The patient should be instructed to take the diuretic early in the morning so that the effects will wear off before the patient goes to bed at night. Decreasing the dose could lead to fluid overload and pulmonary edema.

The nurse is caring for a patient who is experiencing stress incontinence. The nurse identifies which goal to be the most important for this patient? A.) The patient will carefully complete a voiding diary for the duration of 2 weeks. B.) The patient will not experience involuntary urination during coughing or sneezing. C.) The patient will be able to recognize and effectively manage perineal dermatitis. D.) The patient will demonstrate how to appropriately use urinary incontinence products.

B The patient with stress incontinence experiences loss of urine when coughing, sneezing, laughing, or exercising. The highest priority goal for this patient is to not experience incontinence at all and remain continent through all daily activities. If the patient remains continent, perineal dermatitis will not be a problem and urinary incontinence products will not be needed.

The nurse is caring for a patient who will be having surgery. The patient has just signed the consent form for the operation. What does the patient's signature indicate? A.) The patient agrees with the doctor's diagnosis. B.) The patient gives permission for the surgery to be performed. C.) The patient has agreed to pay for any costs not covered by insurance. D.) The patient has been told of all the available treatment options.

B The patient's signature on the consent form indicates that the patient gives permission for the surgery to be performed. It does not indicate that the patient agrees with the physician's diagnosis, agrees to pay for costs not covered by insurance, or has been informed of all the possible treatment options.

The hospice nurse is caring for a terminally ill patient. The patient's son is distraught because the patient will probably die within the next few days and there is nothing he can do about it. What is the most appropriate nursing diagnosis for the patient's son currently? A.) Chronic grief related to impending death of mother B.) Death anxiety related to feeling powerless over situation C.) Powerlessness related to progression of mother's terminal illness D.) Complicated grieving related to desired avoidance of mourning

B The patient's son is experiencing death anxiety because he is unable to change the outcome of his mother's imminent death. The son makes no mention of religious beliefs, so impaired religiosity is not appropriate. Complicated grieving is applicable to individuals who have recently experienced a loss. Chronic grief is grief that continues for a long period of time.

The nurse is caring for a patient who is sedated following a colonoscopy. Which is the priority action of the nurse? A.) Provide a quiet, dark environment so that the patient can rest comfortably. B.) Monitor the patient's pulse oximetry and respirations closely. C.) Inform the patient that the procedure has been completed. D.) Assess the patient's bowel sounds and passage of flatus.

B The priority intervention for sedated patients is to monitor pulse oximetry andrespirations closely because sedation may suppress the respiratory drive. The nurse should monitor vital signs until the patient is fully awake and observe stools forvisible blood. The nurse should also instruct the patient to report any abdominal pain as these assessment findings are alerts for possible perforation of bowel, hypotension, and hemorrhage. Providing a quiet environment is nice for the patient, but dim lighting may impair the nurse's ability to assess the patient. Informing the patient that the procedure has been completed is not a priority. Assessing the patient's bowel sounds and passage of flatus is not as important as careful respiratory monitoring.

The nurse finds the patient in cardiopulmonary arrest with no pulse or respirations. Which oxygen delivery device will the nurse use for this patient? A.) Non-rebreather mask B.) Bag-valve-mask unit C.) Continuous positive airway pressure (CPAP) D.) High-flow nasal cannula

B The priority of the nurse is to ventilate the patient manually using a bag-valve-mask unit (also called by the proprietary name Ambu bag). This allows air to be forced into the patient's lungs when there are no spontaneous respirations. The non-rebreather mask and nasal cannula require the patient to breathe on his or her own. CPAP is used for patients who are awake, oriented, and in respiratory failure.

The nurse carefully reviews the patient's medication list. Which observation about the list indicates the highest risk for serious drug-drug interactions? A.) The patient has been taking the same medications for a long time. B.) The patient is taking a large number of medications. C.) Most of the drugs on the list are prescribed at high doses. D.) The patient takes oral, injected, and inhaled medications.

B The risk of drug-drug interactions increases when a patient takes many drugs. One of the most important ways to prevent adverse drug interactions is to minimize the number of drugs that the patient is taking. The other options do not show a high likelihood of drug-drug interactions.

The nurse recognizes which term to identify the second line of defense that leads to local capillary dilation and leukocyte infiltration? A.) Normal flora B.) Inflammatory response C.) Immune response D.) Humoral immunity

B The second line of defense is the inflammatory response. Inflammation is a local response to cellular injury or infection that includes capillary dilation and leukocyte infiltration. Normal flora is the body's first line of defense. The immune response is the body's attempt to protect itself from foreign and harmful substances. Humoral immunity is a defense system that involves white blood cells (B lymphocytes) that produce antibodies in response to antigens or pathogens circulating in the lymph and blood.

The nurse is providing discharge education for a patient with narcolepsy. Which statement by the patient indicates a need for further education? A.) "Daytime naps are helpful." B.) "Taking the medication will cure it." C.) "High protein meals are helpful." D.) "I need to avoid alcohol."

B There is no cure currently for narcolepsy. Treatment includes a regular exercise routine, a regular sleep routine, daytime naps if possible, light meals high in protein to maintain alertness, and vitamins. Avoiding alcohol, heavy meals, long-distance driving, and long periods of sitting is helpful.

A nurse is assessing a patient's abdomen and hears bowel sounds every 20 to 25 seconds. What action by the nurse is best? A.) Avoid palpating this patient's abdomen. B.) Document the findings in the patient's chart. C.) Have another nurse verify the findings. D.) Ask the patient when the last food intake was.

B These findings are normal; it may take up to 30 seconds of listening to hear bowel sounds. The nurse documents the findings; no other action is needed.

A nurse is caring for a homeless patient and tells the manager, "I will make sure he doesn't steal food from our nourishment center." What action by the manager is best? A.) Tell the nurse she is right to monitor the patient's activity. B.) Inform the nurse that not all homeless people will steal. C.) Educate the nurse that hunger might make the patient steal. D.) Remind the nurse to initiate a social work consultation.

B This nurse is guilty of being prejudiced against the patient, who is a member of the homeless culture. Although hunger might drive a homeless person to steal, prejudice leads the nurse to believe that all homeless people steal. The manager informs the nurse of this information, gently pointing out the nurse's bias. A social work consultation may be a good idea for the patient but does not address the prejudiced nurse.

A woman complains that her partner threatens her and berates her in front of the children. She denies being in an abusive relationship or being the victim of physical violence. What action by the nurse is best? A.) Tell the woman to leave the abusive partner. B.) Educate the woman on forms of domestic abuse. C.) Help the woman work on a physical safety plan. D.) Insist the woman take written information.

B This woman first needs to understand she is indeed in an abusive relationship. The nurse gently educates her on the type of abuse that is possible. Telling the woman what to do is likely to be met with resistance, plus the time of leaving is the most dangerous part of the relationship. The woman may not be accepting of a physical safety plan, since she states there is no physical violence. The woman may be fearful of taking written information because the abuser may find it.

The student nurse asks if it matters whether a healthy eye or a diseased eye should be examined first. What response by the faculty is best? A.) Diseased eye first because it is the priority B.) Healthy eye first to prevent spread of disease C.) It does not matter if both eyes are examined D.) Start with the eye the patient wants you to start with

B To prevent cross contamination, the healthy eye is examined before the diseased eye.

The nurse is preparing a teaching plan and is applying evidence-based practice. To promote involvement, the nurse must include which concept? A.) Provide the latest professional literature to the patient. B.) Ensure that the patient understands relevant information. C.) Use only one teaching method to reduce confusion. D.) Not review previously learned information.

B To promote involvement, nurses must ensure that patients understand the information relevant to their care. Nurses need to provide patients with easy-to-understand information and speak in a clear, distinct voice, using short sentences and understandable terminology. Multiple teaching methods should be used to meet the needs of all types of learners. Patient education sessions should be reassessed after two to three key points to ensure that the patient is still engaged in learning and ready to assimilate more information. Information taught at previous sessions can be reviewed before proceeding with new key points.

To teach effectively, nurses must recognize which concept? A.) Age and socioeconomic status play a large role in understanding. B.) 90% of Americans possess rudimentary literary skills. C.) The ability to comprehend is a very new concept in health care. D.) Most health care teaching is effective and understood.

B To teach effectively, nurses must recognize that patients of all ages come from diverse cultural and socioeconomic backgrounds. Each has a different ability to comprehend health care information. Results of the NAAL research indicate that among American adults, 30 million (14%) had below basic health literacy in English and 47 million (22%) had basic health literacy. This means that 77 million (36%) American adults possessed very rudimentary literacy skills that allowed them to read only short, simple printed and written materials. Although discussion of Nightingale's work often focuses on her efforts to distinguish nursing as a profession and address the impact of sanitation on health, she advocated exploring all aspects of the patient. She thought that patients needed care that is "delicate and decent" and that demonstrates "the power of giving real interests to the patient." Exploring patients' interests and abilities was an early acknowledgment that nurses must be aware of patients' ability to comprehend the health care information provided. Often, health care professionals assume that the explanations and instructions given to patients and families are readily understood. In reality, research has shown that these instructions are frequently misunderstood, sometimes resulting in serious errors.

The nurse is caring for a patient with severe chronic pain and applied the first 50 mcg transdermal fetanyl (Duragesic) patch 2 hours ago. The patient states that the pain is presently rated at 9 on a 1 to 10 scale. What is the nurse's best action? A.) Instruct the patient that the fentanyl patch will start to work soon. B.) Check the provider's orders for a short-acting narcotic medication to administer for breakthrough pain. C.) Give the patient a gentle back rub and encourage guided imagery. D.) Apply a second 25-mcg transdermal fentanyl patch now.

B Transdermal administration of medication does not become effective for 12 to 16 hours after application. Short-acting narcotic medication should be given in the meantime to make the patient comfortable.

The nurse is providing discharge instructions for the patient with sleep pattern disturbances. Which statement by the patient indicates a need for further education? A.) "It is a good idea to have a bedtime routine." B.) "My bedtime routine can include watching TV in bed until I fall asleep." C.) "I will keep my regular sleep pattern on the weekend." D.) "If I can't fall asleep, I will get out of bed and do something relaxing."

B Use the bedroom for sleep and sex only; do not watch television, read, study, or eat in there. A consistent bedtime routine signals to the mind and body that it is time to go to sleep. Maintaining a regular sleep pattern helps maintain circadian rhythms. Continue the regular sleep pattern on weekends and on holidays. Getting out of bed if unable to sleep trains the mind to sleep when in bed. If unable to fall asleep within 20 minutes, get out of bed and engage in a relaxing activity such as reading or listening to music until sleepy.

The student studying community health nursing learns that vulnerable populations can be best assisted by which activity? A.) Researching their genetic risk for health problems B.) Working with the community to decrease health risks C.) Studying vital statistics to determine their causes of death D.) Making sure the population maintains immunizations

B Vulnerable populations have some characteristic that puts them at higher risk for identified health problems. The nurse can best assist vulnerable populations by identifying and working with them to decrease their risks. Researching genetic risks, studying vital statistics, and improving immunizations are all part of the solution, but the overarching priority action is to help the community decrease its risks.

The student nurse learning about ethics expresses good knowledge when making which appropriate statement? A.) "Ethics are internal values developed outside the influence of societal norms." B.) "Ethics are influenced by many variables including family and friends." C.)"Ethics are societal in nature and do not involve personal influences." D.) "Ethics are totally independent from a person's character."

B Family, friends, beliefs, education, culture, and socioeconomic status influence the development of ethical behavior. The study of ethics considers the standards of moral conduct in a society. Personal ethics are influenced by values, societal norms, and practices. Behaviors that are judged as ethical or unethical, right or wrong, reflect a person's character.

The nurse knows which findings indicate orthostatic hypotension? (Select all that apply.) A.) A decrease in systolic blood pressure by 30 mm Hg B.) A decrease in diastolic blood pressure by 10 mm Hg C.) An increase in heart rate by 30 beats/min D.) An increase in systolic blood pressure by 20 mm Hg E.) A decrease in heart rate by 20 beats/min

B & D. A drop in systolic blood pressure of 20 mm Hg, an increase in heart rate of 20 beats/min, or a drop of diastolic blood pressure of 10 mm Hg when a patient stands is classified as orthostatic hypotension.

The nurse is educating the patient about the effects of immobility on the body. Which statements by the patient indicate a need for further education? (Select all that apply.) A.) "I can become very weak." B.) "I will gain weight." C.) "I will lose muscle tone." D.) "I can get bed sores." E.) "I won't have any lung problems."

B & E. Immobility may cause weakness, instability, anorexia, elimination alterations, decreased muscle tone, circulatory stasis, DVTs, pulmonary embolism, and skin breakdown. Knowing the effects of immobility on various body systems allows the nurse to quickly assess a patient's risk and recognize signs of impending complications.

The nurse is correctly demonstrating the use of a transfer belt when engaging in which actions? (Select all that apply.) A.) The belt is placed around the patient's hips. B.) The belt is secure, leaving only enough room for the nurse to grasp the belt. C.) The nurse stands on the weaker side. D.) The nurse holds the belt on the side of the patient. E.) The nurse stands behind the patient while ambulating.

B and C. Transfer belts are used for patients with an unsteady gait or generalized weakness. Canvas transfer or gait belts are applied snugly around the patient's waist, leaving only enough room for the nurse to grasp the belt firmly during ambulation. Some belts may have handles. If the patient has a weaker side, the nurse should stand on that side and hold the gait belt firmly at the back of the patient's waist while ambulating.

The nurse is caring for a patient who has severe burning pain in the right arm caused by a compressed nerve in the neck. Which medications can be used along with a narcotic pain reliever to relieve the patient's pain until surgery can be performed to release the nerve? (Select all that apply.) A.) Diphenhydramine (Benadryl) 50 mg PO daily B.) Amitriptyline (Elavil) 50 mg PO BID C.) Ondansetron (Zofran) 8 mg PO q 4 hours PRN D.) Gabapentin (Neurontin) 400 mg PO BID E.) Senna (Senokot) 8.6 mg PO daily F.) Naloxone (Narcan) 0.4 mg IV now, may repeat in 1 hour PRN

B and D Tricyclic antidepressants like amitriptyline and anticonvulsants like gabapentin are often used to treat neuropathic pain because they work directly on the nervous system. They may be given along with narcotic pain medication to make the patient comfortable. Senna will relieve constipation and diphenhydramine will relieve itching. Ondansetron is used to relieve nausea and vomiting, whereas naloxone will reverse narcotic-induced respiratory suppression.

The nurse is providing education to a cardiac patient who has multiple life stressors that are impacting the patient's health. Which statements by the patient indicate a good understanding of actions that can be taken to reduce stressors? (Select all that apply.) A.) "I should change my job." B.) "I should plan some downtime." C.) "I should meet with a financial counselor." D.) "I should talk with my family about my situation." E.) "I should make my family go to counseling with me."

B, C and D. In adulthood, life stressors such as financial concerns, work-related demands, and efforts to balance work with family life are common challenges that can take a physical toll on the body. Individuals should plan relaxation periods or vacations. Meeting with financial counselors and talking with family can help to achieve that balance. Changing jobs may be beneficial but could also create more stress and forcing family to go to counseling may also not be a wise choice.

The nurse is demonstrating cultural sensitivity in performing perineal care when carrying out which actions? (Select all that apply.) A.) The male nurse delegates perineal care of a female patient to the female UAP. B.) The male nurse asks a female patient if she would prefer a female to perform care. C.) The nurse approaches the care in a sensitive, professional manner. D.) The nurse assesses cultural preferences of the patient prior to care. E.) The nurse provides care quickly and in a matter of fact manner.

B, C and D. The nurse assesses patient backgrounds and provides hygienic care in a manner that is sensitive to the differences in habits and customs. This includes asking the patient about their preferences and not assuming what their preferences will be. A female patient may be comfortable with a male nurse performing perineal care. The nurse should not perform the care without asking first and should not preform the task quickly.

The nurse is providing education to the family of a patient being discharged with dementia. Which statement by the family indicates an appropriate level of understanding of dementia? (Select all that apply.) A.) "The condition is permanent and has an acute onset." B.) "Alzheimer is the most common type of dementia." C.) "The condition worsens over time." D.) "I should observe for wandering behavior." E.) "Agitation can be worse in the evening."

B, C, D, and E. Dementia, which is a permanent decline in mental function, has a subtle onset. The most common type of dementia is Alzheimer disease. Dementia is not reversible and worsens over time. Behavioral problems that arise in dementia patients include wandering, agitation, repetitive behaviors, and sundowning, or worsening of agitation and confusion in the evening.

The nurse knows that which areas of the patient's body are at increased risk of excoriation? (Select all that apply.) A.) Exposed areas such as the face B.) Areas exposed to stool C.) Skin on skin areas D.) Area under pendulous breasts E.) Under an abdominal fold

B, C, D, and E. Excoriation (red, scaly areas with surface loss of skin tissue) occurs in patients whose skin is exposed to bodily fluids such as stool, urine, or gastric juices. Excoriation also occurs in areas where skin rests on skin, such as in the axilla (armpit); under large, pendulous breasts; or in abdominal folds. Exposed areas are more likely to become sun burned or wind burned.

The nurse is bathing a patient and notes reddened skin above the coccyx. Which actions by the nurse are appropriate? (Select all that apply.) A.) Apply a barrier cream and massage the area. B.) Document the findings. C.) Position the patient to relieve pressure on coccyx. D.) Report the area to the charge nurse. E.) Report the new finding to the provider.

B, C, D, and E. Gently wash any reddened or swollen areas and pat them dry. Use clean, nonsterile gloves as needed to comply with standard precautions. Document the findings from the assessment and report them to the provider, charge nurse, or other appropriate personnel per agency policies. Avoid massaging reddened areas on the skin during the bath. Further tissue breakdown can occur if reddened areas are massaged.

The nurse must provide patient education to a patient who has just been given the diagnosis of stage III cancer. The patient is complaining of chest and bone discomfort. Before providing the needed education, the nurse will complete which tasks? (Select all that apply.) A.) Draw the curtain in the semi-private room. B.) Medicate the patient to ease the pain. C.) Place the patient in a private room if possible. D.) Wait until later in the day. E.) Attend to any other personal needs first.

B, C, D, and E. The location of patient education influences the outcome. The setting should be quiet, and the session should have minimal interruptions. Providing privacy is difficult in settings such as emergency rooms, outpatient surgery centers, and semi-private inpatient rooms, but the nurse should make every effort to ensure confidentiality. Environmental considerations such as good lighting and the availability of resources should be explored to enhance the outcome of patient education. The nurse should examine the patient's situation and comfort level before beginning teaching. For example, a postoperative patient who is rating pain at 7 of 10 will be much more receptive to learning after being medicated for pain. A patient who just received a diagnosis of metastatic cancer will learn and assimilate more information later in the day or perhaps the next day. The nurse must also take care of any other personal needs first, such as the need to use the bathroom.

For the following, indicate which are examples of objective data. Select all that apply. A.) Patient tells the nurse, "I feel hungry." B.) Blood pressure 140/90 mm Hg C.) Rash noted on lower right abdomen D.) Patient informs the nurse that she did not eat breakfast. E.) Uneven gait on ambulation F.) Potassium level noted in electronic diagnosis record as 4.3

B, C, E, and F.

The nurse is caring for a patient who is to complete a 24-hour urine collection to measure creatinine clearance. Which tasks related to this test may be delegated to the nursing assistant? (Select all that apply.) A.) Teaching the patient about sterile specimen collection B.) Keeping the urine collection container cool on ice C.) Dumping the urine from the patient's first void D.) Restricting the patient's oral fluid intake during the test E.) Transporting the specimen to the laboratory for testing F.) Reminding the patient not to put toilet paper in the urine

B, C, E, and F. The nurse assistant can help the nurse by keeping the urine collection container cool on ice, dumping the urine from the patient's first void, and reminding the patient not to put toilet tissue in the urine specimen. The nurse assistant can also transport the specimen to the laboratory after the urine has been collected for 24 hours. Fluid intake should be encouraged during the test. Teaching the patient about the testing procedure is done by the nurse, although creatinine clearance testing does not require sterile technique.

A nurse is caring for a patient who is quadriplegic. According to Maslow's hierarchy of needs, which patient needs should be given priority over others? (Select all that apply). A.) Allowing the family members to visit B.) Feeding the patient C.) Ensuring adequate fluid intake D.) Assisting with bladder or bowel elimination E.)Reassuring the patient

B, C, and D

The nurse is demonstrating cultural sensitivity in performing perineal care when he/she does the following: Select all that apply A.) The male nurse delegates perineal care of a female patient to the female UAP B.) The male nurse asks a female patient if she would prefer a female to perform care C.) The nurse approaches the care in a sensitive, professional manner D.) The nurse assess cultural preferences of the patient prior to care.

B, C, and D.

The nurse notes that a trauma patient has multiple tangles in the hair. Which actions taken by the nurse are appropriate? (Select all that apply.) A.) Work the tangles to the ends of the hair, then trim with scissors. B.) Apply warm water and conditioner. C.) Apply detangler as available. D.) Use a comb or fingers to work through tangles. E.) Cut the tangles out if working on them agitates the patient.

B, C, and D. Apply warm water and a conditioner or a detangler, if available, to release tangles and avoid injury to the scalp. Use a comb and/or fingers to work through the tangles individually before shampooing. The nurse avoids cutting the patient's hair unless first asking the patient's permission.

The nurse is performing a morning assessment and notes the patient to be experiencing dyspnea. Which patient assessment findings would most indicate this respiratory condition? (Select all that apply.) A.) Occasional productive cough B.) Pulse oximetry 89% C.) Patient in orthopneic position D.) Respirations 26 & shallow E.) Temperature 100.1 °F

B, C, and D. Dyspnea is difficult, labored breathing, usually with a rapid, shallow pattern, that may be painful. Anxiety usually is present as well. Accessory muscles in the chest and neck are used in dyspneic breathing. Many patients experiencing dyspnea find it easier to breath in an upright position. Difficulty breathing experienced in positions other than sitting or standing is termed orthopnea. Occasional productive cough and slight temperature elevation are not indicators of dyspnea.

When teaching children, the nurse should include which concepts? (Select all that apply.) A.) Exclude the children from teaching. B.) Encourage parents or caregivers to be present. C.) Use age-specific strategies. D.) Consider the stages of development. E.) Remember that parents are not the targets of the teaching.

B, C, and D. Patient education provided for children should be age specific. Effective patient education involving a child requires the presence of a parent or caregiver, who is likely the target of teaching. Children should not be excluded from the learning session unless exclusion is deemed appropriate by the parent or caregiver; a presentation using an age-appropriate strategy may complement the instructions reviewed with the adult. The stages of development should be explored as the foundation for the choice of educational materials.

The nurse examining a patient's skin correlates which conditions with which underlying pathology? (Select all that apply.) A.) Albinism: Full-thickness burns B.) Peripheral cyanosis: poor circulation C.) Purpura: clotting disorders D.) Jaundice: liver disease E.) Vitiligo: skin infestation

B, C, and D. Peripheral cyanosis can result from poor circulation. Purpura can be seen in patients with clotting disorders. Jaundice often indicates liver disorders such as liver failure. Albinism is genetically determined. Vitiligo is thought to be an autoimmune response.

The nurse is explaining the National Patient Safety Goals (NPSG) to the student nurse. Which answers indicate that the student has a good understanding of these goals? (Select all that apply.) A.) The NPSG's focus on treating chronic infections quickly B.) The NPGS's focus on improving staff communication C.) The NPGS's focus on using medications safely D.) The NPGS's focus on identifying patients correctly

B, C, and D. The NPSG focus on specific goals each year. The goals for 2018 included: identify patients correctly, improve staff communication, improve the safety of using medications, reduce harm associated with clinical alarm systems, reduce risk of health care-associated infections and the hospital identifies safety risks inherent in its patient population. Although treating chronic infections quickly is important, it is not an NPSG.

The nurse knows when the body responds to the release of hormones during "fight or flight," that response includes which physiological signs? (Select all that apply.) A.) Decreased respiratory rate B.) Slowing of the digestive process C.) Glucose being mobilized from the liver D.) Pupils dilating E.) Smooth muscles in the bronchi constricting

B, C, and D. The release of hormones increases the heart rate, resulting in increased cardiac output, and elevated blood pressure. There is an increase in the flow of blood to muscles at the expense of the digestive and other systems not immediately needed in the fight-or-flight response. Smooth muscles in the bronchi relax and dilate the bronchi and smaller airways, and the respiratory rate increases, allowing for an enhanced flow of well-oxygenated blood to muscles and other organs. The motility of the digestive tract is decreased, slowing digestive processes, but glucose and fatty acids are mobilized from the liver and other stores to support increased mental activities (alertness) and skeletal muscle function. Pupillary dilation produces a larger visual field.

The nurse is caring for a patient who underwent abdominal surgery the previous day. Which assessment findings indicate to the nurse that the patient may be experiencing serious internal bleeding? (Select all that apply.) A.) The patient's urinary output increased to 40 mL/hr. B.) The patient's pulse has risen from 76 to 112 beats/min. C.) The patient states that his abdominal pain is worse than yesterday. D.) The patient complains of generalized itching. E.) The patient's hematocrit dropped from 14.6 to 11.0 g/dL. F.) The patient has not been able to have a bowel movement since before surgery.

B, C, and E. Signs of internal bleeding include tachycardia, increased abdominal pain and a drop in hematocrit/hemoglobin. Urinary output would decrease with internal bleeding because the kidneys work to conserve fluids. Itching and constipation are not signs of internal bleeding.

The nurse recognizes barriers to the use of evidence-based practice (EBP) include what points? (Select all that apply.) A.) Nurses critiquing research B.) Difficulty communicating how to conduct EBP C.) Copious amount of literature available D.) Short time between research and practice E.) Reluctance of organizations to fund research

B, C, and E. To adequately integrate EBP into patient care, nurses must critique research to differentiate between opinion and evidence and must regularly read current professional journals. Some barriers are common to research use and EBP, including the difficulty of communicating how to conduct EBP and the individual nurse's skills in determining the quality of research available for review. Another limitation is the reluctance of organizations to fund research and subsequently make potentially costly practice changes based on the best evidence. Because of the copious amount of literature on a specific topic, it is difficult to analyze the literature in an efficient and effective manner. Health care literature with clinically applicable findings is published at a rate that is impossible for individual health care professionals to keep up with. There are delays of approximately 17 years for implementation of clinical research into practice.

The nurse working with older adults wants to support healthy coping strategies. What actions by the nurse are most appropriate? (Select all that apply.) A.) Installing boxing equipment in the recreation room B.) Provide reminiscing sessions for the adults to share personal stories C.) Arrange for gentle yoga to be provided at the senior center D.) Create activities designed to distract them from their losses E.) Encourage the adults to eat frequent, healthy snacks

B, C, and E. To promote health coping in older adults, the nurse would provide reminiscing sessions, yoga, and would encourage small healthy snacks as this population frequently loses their appetite when stressed. Boxing equipment might cause the adults to focus on anger. Distraction can be a negative or positive coping mechanism.

For the diagnostic label of Insufficient Gas Exchange, indicate which defining characteristics are most appropriate to include in the diagnostic statement. Select all that apply. A.) Emphysema B.) Diminished lung sounds to R lower lung base C.) Pulse oximetry of 89% D.) Weight gain-5% in 2 weeks E) Pain in lower R leg on ambulation F.) Crackles heard on auscultation

B, C, and F.

The nurse is providing community education on sudden infant death syndrome (SIDS). What information does the nurse include? (Select all that apply.) A.) SIDS is the second most common cause of death among infants (1 to 12 months). B.) The etiology remains largely unknown. C.) The most modifiable risk factor is sleeping supine. D.) Risk factors include being exposed to cigarette smoke. E.) It is defined as sudden unexpected death.

B, D, and E. SIDS is the leading cause of death among infants 1 to 12 months of age. The etiology remains largely unknown. The most important modifiable SIDS risk factor appears to be prone sleeping. Risk factors include the infant's being exposed to cigarette smoke. Sudden infant death syndrome (SIDS) is defined as the sudden unexpected death of an infant younger than 1 year of age that remains unexplained after a thorough postmortem investigation.

A nurse is teaching a patient and the patient's family about self-care measures for hypertension. Which topics should the nurse include? (Select all that apply.) A.) Increase exercise on most days. B.) Maintain a normal body weight. C.) Abstain from any alcohol. D.) Reduce dietary sodium to 2.4 g/day. E.) Follow the DASH diet.

B, D, and E. Self-care measures for hypertension include 30 minutes of aerobic exercise on most days of the week, maintaining a normal body weight, limiting alcohol to two drinks/day for men and one drink/day for women, reducing sodium intake to 2.4 g/day, and following the DASH diet.

The nurse is working with a nursing assistant to care for several postoperative patients. Which interventions can the nurse delegate to the assistant for completion? (Select all that apply.) A.) Assess patients' comfort levels and need for pain medication. B.) Empty urinary catheter bags and record urine output. C.) Teach patients how to use incentive spirometers hourly. D.) Provide ice chips and juice to patients who are no longer NPO. E.) Monitor incisions for signs of infection. F.) Apply TED hose and assist with oral care.

B, D, and F. Basic patient care tasks that do not require assessment or critical thinking may be assigned to the nursing assistant for completion. These include emptying drainage bags, providing ice chips to patients who are allowed oral intake, and applying TED hose. Teaching, monitoring, and assessing patients are done by the nurse.

The preceptor is working with a new nurse to suction a patient through a new tracheostomy. Which actions by the new nurse indicate need for additional teaching about the procedure? (Select all that apply.) A.) The suction is not applied to the catheter until it is being withdrawn. B.) The patient is placed in the supine position prior to suctioning. C.) The suction catheter is twirled side to side as it is being withdrawn. D.) Suction is applied continuously as the catheter is withdrawn. E.) The patient's oxygen is reapplied between suction attempts. F.) Water-soluble lubricant is applied to the suction catheter before insertion.

B, D, and F. The head of the patient's bed should be elevated prior to suctioning to facilitate coughing out secretions. Suction is always applied intermittently as the catheter is withdrawn. Water-soluble lubricant is used when suctioning the naris but not a tracheostomy because the secretions negate the need for additional lubrication.

Sterotype

Belief about an individual, a group, or an event that is thought to be typical of all others in that category.

In planning for the patient assignment, the nurse prioritizes a schedule on the basis of the patient's needs and conditions. In reviewing the nursing diagnoses, which of the following patients should be seen first in the morning? A.) Altered urinary elimination B.) Change in sleep pattern C.) Reduced cardiac output D.) Inability to perform self-care; grooming

C

The nurse is caring for an adolescent patient with anorexia nervosa. She knows the best treatment option is: A.) Hospitalization with skill nursing care B.) Compulsory tube feedings C.) Individually determined by a collaborative team D.) Outpatient treatment

C

The nurse is providing education to an older adult around diet to support the challenges related to aging. Which statement indicates a need for further education? A.) "I should choose foods that are nutrient dense." B.) "High-fiber foods minimize the risk for constipation." C.) "I should eat more calories to avoid malnutrition." D.) "I can add spices to enhance the taste of food."

C

The nurse is to administer 45 mg of phenobarbital to the patient. How many tablets will the patient receive? Phenobarbital tablets, USP 15mg CIVRXonly A.) 1 tablet B.) 2 tablets C.) 3 tablets D.) 4 tablets

C

The nurse is working with a postoperative patient on the surgical unit. Which aspect of care demonstrates a manipulation of the patient's environment? A.) Providing wound care B.) Administering analgesic medication C.) Making sure the room is warm enough D.) Measuring the patient's vital signs

C

The nurse keeps working with the patient to help him ambulate, motivating him to reach his goal of being independent. The nurse is demonstrating which critical thinking trait? A.) Confidence B.) Humility C.) Perseverance D.) Fairness

C

The nurse knows an appropriate outcome statement for the nursing diagnosis Impaired Swallowing is: A.) The patient will consume 50% of their meal B.) The patient will gain 2lb a week C.) The patient will show signs of aspiration during meals D.) The patient will demonstrate using an assistive device to feed himself

C

What measure does the nurse take to ensure effective planning of patient care? A.) Ensure that the patient cooperates to achieve the set goals B.) Realize that the outcome depends entirely on the nurse. C.) Identify the most urgent needs of the patient. D.) Involve the primary health care provider in setting the goals

C

Which essential critical thinking trait is the nurse using when she tries out the new way to apply a dressing? A.) Curiosity B.) Discipline C.) Creativity D.) Perseverance

C

Which of the following nursing interventions is most clearly stated and will assist other staff members to provide safe care? A.) Provide extra fluids B.) Increase ambulation in hallway C.) Reinforce use of incentive spirometer tid D.) Complete assessment with patient in the am.

C

Which patient would require the nurse to relate the position of food on a plate to the position of numbers on an analog clock as a reference while communicating? A.) Physically impaired patients B.) Cognitively impaired patients C.) Visually impaired patients D.) Hearing impaired patients

C

Which statement by the nurse identifies UAP role in a patient restraint use? A.) "The UAP can perform initial assessment." B.) "The UAP can apply a restraint." C.) "The UAP can assist with applying and monitoring of a physical restraint." D.) "The UAP can contact the physician and request an order for restraints."

C

The nurse is assessing a patient whose chart indicates a Grade 3 heart murmur. What action is best to hear the murmur? A.) Ensure that the room is extremely quiet. B.) Use a specialized stethoscope with amplification. C.) Auscultate the patient's chest with a stethoscope. D.) Place the stethoscope diaphragm on the patient's back.

C A Grade 3 murmur should be readily heard with a regular stethoscope. Although the room should be quiet for all auscultation tasks, an "extremely" quiet room and an amplification stethoscope should not be necessary. The bell of the stethoscope is usually used to listen to heart murmurs, but the stethoscope needs to be on the patient's chest.

The nurse correctly devises a dissemination plan at what point during the research process? A.) Conclusion of the study B.) After the literature review C.) The beginning of the research process D.) While conducting research

C A dissemination plan should be devised at the beginning of the research.

The nurse reads in a chart that a patient has a paronychia. What assessment technique is most appropriate? A.) Auscultate the patient's bowel sounds. B.) Test the cranial nerves for sensory function. C.) Inspect the patient's nails and surrounding skin. D.) Inspect the skin using the ABCDE mnemonic.

C A paronychia is inflammation at the base of the nail, so the nurse assesses the patient's nails and the surrounding skin. The other assessments are not related to this diagnosis.

The nurse has assisted the patient to wash the hands, face, axillae, and perineal area. What type of bath does the nurse chart? A.) Sink bath B.) Complete bed bath C.) Partial bed bath D.) Shower

C A partial bed bath is performed when only part of the body is washed. A complete bed bath is for patients who are completely bedridden or are totally dependent on others for care. A shower is usually for patients who are strong enough to shower independently. A sink bath is when the patient washes while standing or sitting in front of a bath basin or sink.

The student learns that which is the best definition of a public health nurse? A.) Works with the public. B.) Works in public areas. C.) Works with the greater community. D.) Works with public funding.

C A public health nurse works with communities as a larger whole and is concerned with specific target or vulnerable groups within that community. The other options are inaccurate.

The nurse receives a hand-off report on four patients. Which patient finding should the nurse assess first? A.) Pulse oximetry 96% B.) Blood pressure 102/62 mm Hg C.) Pulse 42 beats/min D.) Respiratory rate 18 breaths/min

C A pulse of 42 beats/min is considered bradycardia and the patient should be assessed first because perfusion could be compromised. The blood pressure, pulse oximetry, and respiratory rate are normal.

The nurse is caring for a patient who is receiving a blood transfusion. Fifteen minutes into the transfusion, the patient's blood pressure decreases significantly, and the patient complains of a severe headache. What is the priority action of the nurse? A.) Check the patient's temperature and administer acetaminophen (Tylenol) if higher than 101 °F. B.) Recheck the patient's blood pressure in 15 minutes after administering pain medication. C.) Stop the blood transfusion and administer 0.9% normal saline through new IV tubing. D.) Double-check that the transfusion blood type is an exact match to the patient.

C A significant drop in blood pressure and a severe headache are signs that the patient may be experiencing a transfusion reaction. Also, most reactions to a transfusion occur within the first 15 minutes of initiation. Therefore the nurse should remain with the patient at the bedside during this time to observe for signs of a reaction. The transfusion should be stopped and 0.9% normal saline should be administered through new IV tubing to prevent infusion of additional blood through the tubing used for the transfusion. The physician should be notified immediately to evaluate the patient. Ensuring that the transfusion blood type is an exact match to the patient is done before the transfusion is begun.

Which action by the nurse best demonstrates accountability in the operating room? A.) Applying warm blankets when the patient reports feeling chilly B.) Holding the patient's hand to allay anxiety before anesthesia is administered C.) Double-checking that the surgical site is clearly marked and visible after draping D.) Using calming speech with a reassuring tone of voice when speaking with the patient

C Accountability is accomplished by ensuring that proper precautions are taken to prevent errors from happening. The nurse can prevent wrong-site surgery by making sure that the surgical site is clearly marked and visible after the draping is completed. The surgeon could inadvertently operate on the incorrect site if the markings are covered by the surgical drapes. The other actions of the nurse are appropriate but do not demonstrate accountability and error prevention.

The nurse is caring for a patient who is hospitalized for pneumonia. Which Nursing diagnosis has the highest priority? A.) Activity intolerance r/t generalized weakness and hypoxemia B.) Impaired nutritional intake r/t poor appetite and increased metabolic needs C.) Impaired airway clearance r/t thick secretions in trachea and bronchi D.) Lack of knowledge r/t use of nebulizer and inhaled bronchodilators

C Airway maintenance and patency is the highest priority for all patients, especially patients with respiratory disorders. Oxygenation is the most important human need. The other diagnoses can apply once the patient's airway is kept patent.

A patient asks the nurse to pray with him. The nurse is an atheist and uncomfortable with this request. What action by the nurse is best? A.) Deny the request because of atheistic beliefs. B.) Offer to call the chaplain instead. C.) Agree to sit with the patient while he prays. D.) Ask the patient if he will meditate instead.

C Although the nurse is uncomfortable with the request, the patient's needs (not the nurse's) come first. The nurse should attempt to honor the request while not imposing his/her ideas of religion and spirituality on the patient. The best option is to agree to sit with the patient while he prays himself. This is consistent with caring behaviors and fulfilling the patient's needs. Denying the request does nothing to address the patient's needs. The nurse can offer to call the chaplain in addition to sitting with the patient. Asking the patient to change his practices is unethical.

The student nurse asks why he needs to assess a patient's spirituality when he can call the chaplain. What response by the nurse is best? A.) "This way you learn what is involved in a spiritual assessment." B.) "Students need to perform all aspects of patient care." C.) "Regulatory organizations list this as a required BSN competence." D.) "All patients should have a spirituality assessment."

C Although there is some truth to all options, several regulatory groups list conducting a spiritual assessment as a vital skill for nurses, including the American Association of Colleges of Nursing, The Joint Commission, and the American Nurses Association.

The nurse identifies which goal to be appropriate for the nursing diagnosis of Difficulty coping? A.) The patient will report an ability to remember discharge instructions. B.) The patient's family will understand how to access respite care services. C.) The patient will discuss possible coping strategies during weekly counseling sessions. D.) The patient will attend an online support group weekly.

C An appropriate goal for Difficulty coping would be to discuss coping strategies. Remembering discharge instructions is an appropriate goal for Anxiety. Understanding how to access respite care services is an appropriate goal for Caregiver stress. Attending a support group is an appropriate goal for Difficulty coping.

The nurse recognizes which outcome statement to be appropriate for the nursing diagnosis Impaired swallowing? A.) Patient will consume 50% of each meal. B.) Patient will gain 2 lb a week. C.) Patient will not show any signs of aspiration during meals. D.) Patient will demonstrate using an assistive device to feed self.

C An appropriate goal statement for impaired swallowing is that the patient will not exhibit any signs or symptoms of aspiration during this hospitalization (e.g., lungs clear, respiratory rate within normal range for patient). Consuming 50% of meals and gaining weight are appropriate goals for Impaired nutritional intake. Using assistive devices is an appropriate goal for Impaired self-feeding.

Excessively dry skin can lead to cracks and openings in the integumentary system. Based on this, what is the most applicable Nursing diagnosis for a patient with excessively dry skin? A.) Impaired Health Maintenance B.) Risk for Injury C.) Risk for infection D.) Acute pain

C Any interruption in the skin, which is the body's first line of defense, can potentially lead to infection. Impaired health maintenance could have dry skin as a symptom. Acute pain and risk for injury are not appropriate.

The nurse is caring for a patient who has a bleeding gastric ulcer. How will the nurse expect the patient's stool to appear? A.) Soft and formed with bright red streaks B.) Watery with particles of undigested food C.) Sticky and black D.) Hard lumps that are difficult to pass

C Bleeding anywhere along the GI tract results in blood in the stool. Bleeding that occurs in the upper GI tract produces stools that are black and tarry in appearance. Bleeding within the lower GI tract presents with soft stools that have bright red streaks. Watery stool with particles of food is indicative of gastroenteritis. Hard lumps that are difficult to pass indicate constipation, often from medications or lack of fiber in the diet.

The nurse is providing discharge instructions to a patient with visual alterations. Which statement by the patient indicates a need for further education? A.) "I should make sure the passageways are wide." B.) "I should remove all the throw rugs." C.) "I should keep the lights dim." D.) "I can use a cane to feel for objects in front of me."

C Bright lighting in hallways and stairways prevents falls by the patient who has limited vision. Furniture is placed to allow wide passageways. Throw rugs, which are a tripping hazard, are removed. If vision is severely limited, use of a cane or walking stick held slightly in front helps the patient feel objects in his/her path.

The nurse is providing education to an older adult around a healthy diet to support the challenges related to aging. Which statement indicates a need for further education? A.) "I should choose foods that are nutrient dense." B.) "High-fiber foods minimize the risk of constipation." C.) "I should eat more calories to avoid malnutrition." D.) "I can add spices to enhance the taste of food."

C Calorie needs change with aging because of more body fat and less lean muscle. Less activity further decreases calorie requirements. Eating whole-grain foods and a variety of fruits and vegetables and drinking water may minimize the risk of constipation. The challenge for older adults is to choose foods that are nutrient dense; these foods are high in nutrients in relation to their calories. Older adults may experience a decreased sense of smell or taste, so the addition of spices and herbs may enhance the taste of foods.

When the nurse is explaining cataplexy to the patient, which description should be included? A.) It is an uncontrolled desire to sleep. B.) It is falling asleep for several minutes. C.) It is loss of voluntary muscle tone. D.) It is a sleep cycle that begins with NREM.

C Cataplexy is characterized by the sudden loss of voluntary muscle tone; vivid hallucinations during sleep onset or on awakening; and brief episodes of total paralysis at the beginning or end of sleep. An uncontrolled desire to sleep and falling asleep for several minutes define narcolepsy. Narcolepsy begins with REM sleep.

The hospice nurse is caring for a several adult children shortly after the death of a parent. They have various reactions as they deal with their loss. The nurse recognizes which reactions to be in the cognitive domain? A.) They let the house get filthy because they can't be bothered to clean it. B.) They are tossing and turning all night and are unable to get a good night's sleep. C.) They are easily distracted and often lose train of thought during conversation. D.) They have lost their appetites and have no desire to eat anything.

C Cognitive deficits include the inability to concentrate and follow a conversation. Letting the house get filthy is a sign of apathy, which is in the behavioral domain. Insomnia falls within the behavioral and physical domains. Loss of appetite is within the physical domain.

A patient from an unfamiliar culture appears disinterested when the physician is telling her about options for treatment of a new diagnosis. After the physician leaves, the nurse attempts to talk to the patient and notices the same behavior. What action by the nurse is best? A.) Give the patient the information in writing to read later. B.) Ask the patient about the meaning of the patient's behavior. C.) Investigate nonverbal communication patterns of this group. D.) Leave the patient alone to come to terms with the diagnosis.

C Communication differences can lead to misunderstandings and possible medical errors. Many cultural groups have verbal and nonverbal communication patterns that differ from other groups. Variations can occur due to personal or social situations. The nurse should attempt to learn about the cultural group's communication patterns. Giving the patient written material and leaving the patient alone do not help solve this dilemma, and the patient may not have the literacy skills to understand the material. Asking the patient the meaning of behavior is unlikely to elicit useful information because the patient herself may not totally understand it or be able to articulate it. This may be a deeply seated cultural custom that is simply part of who the patient is.

The nurse is caring for a patient recently diagnosed with cancer that is being asked to participate in a new chemotherapy trial. How would the nurse respond if working under the ethical principle of utilitarianism? A.) "The patient should be allowed to decide." B.) "As your nurse, I'll support your right to refuse." C.) "You should do this because many could benefit from it." D.) "If this is against your beliefs, you should not do it."

C Compared with deontology, utilitarianism is on the opposite end of the ethical theory continuum. Utilitarianism maintains that behaviors are determined to be right or wrong solely based on their consequences. Deontology is an ethical theory that stresses the rightness or wrongness of individual behaviors, duties, and obligations without concern for the consequences of specific actions. Meeting the needs of patients while maintaining their right to privacy, confidentiality, autonomy, and dignity is consistent with the tenets of deontology. Autonomy, or self-determination, is the freedom to make decisions supported by knowledge and self-confidence. The remaining responses are examples of either deontology or autonomy.

The nurse is caring for an incontinent male patient who has a deep decubitus ulcer on his sacrum. Which intervention will best manage the patient's urinary incontinence and facilitate healing of the ulcer? A.) Use of disposable absorbable incontinence briefs B.) Daily application of perineal barrier cream containing zinc oxide C.) Careful perineal care and application of a condom catheter D.) Insertion of a single-lumen straight urinary catheter

C Condom catheters allow for collection of urine in the incontinent patient without the infection risks of an indwelling catheter. The condom catheter is applied to the outside of the penis like a condom instead of being inserted into the urethra. Careful perineal care is performed prior to application of the condom catheter and regularly thereafter. Use of disposable briefs or perineal barrier cream will not facilitate healing of the sacral ulcer. A single-lumen straight urinary catheter is used to drain the bladder to relieve urinary retention or to obtain a urine sample for testing. A straight catheter is not used for management of incontinence.

The nurse is educating the family of a patient in the intensive care unit about the patient's cognitive status, including the current problem of delirium. Which statement by the family indicates a need for further education? A.) "The delirium can be caused by sensory overload." B.) "The delirium is reversible." C.) "The delirium is a mood disorder." D.) "The delirium is a state of confusion."

C Delirium is a reversible state of acute confusion. It is characterized by a disturbance in consciousness or a change in cognition that develops over 1 to 2 days and is caused by a medical condition. Delirium may occur in intensive care patients as a result of sensory overload. It is not a mood disorder.

In practice, the nurse has identified an observable phenomenon and wants to conduct research to generate a hypothesis through observation of the situation. The nurse knows what approach to be the best way for the nurse to conduct this type of investigation? A.) Correlational research study B.) Experimental research study C.) Descriptive research study D.) Quasi-experimental research study

C Descriptive research identifies data and characteristics about the population or phenomenon. Correlational research is used to explore a relationship between two variables. Experimental research explores the causal relationships between variables. Experimental research examines whether one variable has a cause-and-effect relationship with another. Quasi-experimental research examines a causal relationship between variables, but it may not meet the strict guidelines of experimental research.

The nurse hears a loud murmur when listening to the patient's heart. Which diagnostic test will best display the condition of the valves and structures within the patient's heart that could be causing the murmur? A.) Chest x-ray B.) Cardiac catheterization C.) Echocardiogram D.) Electrocardiogram

C Echocardiograms allow for ultrasound visualization of the structures of the heart along with function of the heart valves and cardiac musculature.

The nurse is educating the patient about the proper disposal of medications in the home. Which statement by the patient indicates a good understanding of the information? A.) "Remove the label from the bottle and throw in the trash." B.) "Flush the medication down the disposal." C.) "Mix the medications with kitty litter, place the mixture in a jar, and put the jar in the trash." D.) "Dissolve the medication in water and pour down the drain."

C Flushing or pouring the medication down the drain can contaminate the water system. Throwing the medication in the trash poses potential for someone to remove the medication and use it. This can be avoided by mixing it with an undesirable substance like kitty litter or coffee grounds.

A patient has hypertension and is on a very-low-sodium diet. However, the patient is going to celebrate an important religious holiday soon that includes many food items high in sodium. What action by the nurse is best? A.) Tell the patient you are so sorry she can't have any of these foods. B.) Consult with the prescriber about increasing the blood pressure medications. C.) Collaborate with the patient and dietitian to include some of these foods. D.) Tell the patient eating these foods once won't hurt her condition.

C Food has important meaning to many people, especially when they are part of celebrations, religious, or cultural activities. The nurse should collaborate with the patient and dietitian and try to find ways to incorporate some of these items. The nurse should not just tell the patient she can't have them. Increasing the medications or encouraging the patient to be nonadherent could lead to adverse outcomes.

The nurse is explaining to the UAP that the patient is on a full-liquid diet. Which statement by the UAP indicates a need for reorientation? A.) "I can give the patient orange juice." B.) "I can give the patient yogurt." C.) "I can give the patient oatmeal." D.) "I can give the patient milk."

C Full-liquid diets consist of foods that are or may become liquid at room or body temperature. Full-liquid diets include juices with and without pulp, milk and milk products, yogurt, strained cream soups, and liquid dietary supplements. Oatmeal is not considered part of a full-liquid diet.

A patient states, "I just don't conform to my gender role." What does the nurse understand about this statement? A.) The patient is a homosexual. B.) The patient's behaviors are abnormal. C.) The patient's actions differ from what is expected. D.) The patient is having a gender crisis.

C Gender roles are socially imposed "rules" about the behavior appropriate for men and women. When someone does not conform to gender role expectations, his/her behaviors are at odds with those expected by society. The patient is not necessarily homosexual, abnormal, or having a crisis.

The nurse is performing a wet/damp to dry dressing change when the patient begins to complain of severe pain. What does the nurse do first? A.) Notify the provider. B.) Notify the wound care nurse. C.) Stop the procedure. D.) Give the patient pain medication.

C If the patient is complaining of severe pain, the nurse should first stop the procedure and then determine if the pain is new or preexisting. Then the nurse can determine what to do next based on the patient's response.

The nurse is caring for a patient who is undergoing a major cardiac procedure. When the patient complains of a racing heart and nausea, the nurse recognizes these complaints as part of what hormone response? A.) Sense of coherence B.) Stress appraisal C.) Fight or flight D.) Sympathoadrenal response

C In the "fight or flight" response, the corticotropin-releasing hormone (CRH) released by the hypothalamus stimulates the pituitary to release adrenocorticotropic hormone (ACTH). These hormones increase the heart rate, resulting in increased cardiac output, and the motility of the digestive tract is decreased, slowing digestive processes that could result in abdominal distress. Sense of coherence (SOC) is a characteristic of personality that references one's perception of the world as comprehensible, meaningful, and manageable. Stress appraisal is the automatic, often unconscious, assessment of a demand, or stressor. The sympathoadrenal response is a consequence of hypothalamic activation in sympathetic stimulation, which triggers epinephrine and norepinephrine release from the adrenal medulla.

The nurse is admitting a patient who has cystic fibrosis. During the admission interview, it is apparent that the patient is well versed in most aspects of his illness. When asked about where he learned so much, the patient responds, "I learned most of it myself. I looked things up on the Internet and read books. You have to know what's wrong with you to be sure that you're being treated right." The nurse knows this is an example of what type of education/learning? A.) Formal education B.) Psychomotor learning C.) Informal education D.) Affective learning

C Informal education is usually learner or patient directed. Formal patient education is delivered throughout the community in the form of media, in a variety of educational and group settings, or in a planned, goal-directed, one-on-one session with a patient in the acute care setting. The psychomotor domain incorporates physical movement and the use of motor skills in learning. Teaching the newly diagnosed diabetic how to check blood sugar is an example of a psychomotor skill. Affective domain learning recognizes the emotional component of integrating new knowledge. Successful education in this domain takes into account the patient's feelings, values, motivations, and attitudes.

The nurse is providing education to the patient about isometric exercises. Which statement by the patient indicates a good understanding of these exercises? A.) "An example of this type of exercise is walking." B.) "An example of this type of exercise is running." C.) "An example of this type of exercise is Kegels." D.)"An example of this type of exercise is weight lifting."

C Isometric exercise requires tension and relaxation of muscles without joint movement. An example is tension and relaxation of pelvic floor muscles (i.e., Kegel exercise). Isotonic exercise involves active movement with constant muscle contraction, such as walking, turning in bed, and self-feeding. Aerobic exercise requires oxygen metabolism to produce energy. Patients may engage in rigorous walking or repeated stair climbing to achieve the positive effects of aerobic exercise. Anaerobic exercise builds power and body mass. Without oxygen to produce energy for activity, anaerobic exercise takes place, such as heavy weight lifting.

The nurse manager of the emergency room believes that efficiency is the expected standard for the department and believes that efficiency lies in following established rules, policies, and guidelines. The only way to change procedures is to changes rules, policies, and guidelines. To run the emergency room with this philosophy, the nurse manager must take on which role? A.) Laissez-faire leader B.) Democratic leader C.) Bureaucratic leader D.) Autocratic leader

C Like the autocratic leader, the bureaucratic leader assumes that employees are motivated by external forces. This type of leader relies on policies and procedures to direct goals and work processes. The nurse using bureaucratic leadership tends to relate impersonally to staff and exercises power on the basis of established rules. The permissive or laissez-faire leader thinks that employees are motivated by their own desire to do well. The laissez-faire leader provides little or no direction to followers, who develop their own goals and make their own. The participative or democratic leader believes that employees are motivated by internal means and want to participate in decision making. The primary function of the leader in this situation is to foster communication and develop relationships with followers. The authoritarian or autocratic leader exercises strong control over subordinates

The manager of the intensive care unit is accepting an award for excellence and efficiency in the provision of patient care. The manager accepts the award for the unit and cites the contributions of the staff since, without their expertise and dedication, the award may not have been achieved. The staff nurse recognizes the nurse manager is demonstrating which quality? A.) Dedication B.) Openness C.) Magnanimity D.) Creativity

C Magnanimity means giving credit where credit is due. Good leaders reflect the work and success of accomplishing a goal by crediting those who helped reach it. Dedication is the ability to spend the time necessary to accomplish a task. Effective leaders persist in working toward accomplishment of a goal even when doing so is difficult. Openness refers to the leader's ability to listen to other points of view without prejudging or discouraging them. An effective leader considers others' opinions with an open mind because a wider variety of solutions to problems is offered. Openness by the nurse leader encourages creative solutions by providing an environment in which people feel comfortable "thinking outside the box." Creativity is the ability to think differently. A creative leader examines all possible solutions to a problem even if at first glance they appear to be unrealistic or outside the norm. This ability allows the nurse leader to inspire followers to consider broader visions and goals.

A patient is hesitating to accept a blood transfusion as a course of treatment. What Nursing diagnosis is most appropriate for this patient? A.) Spiritual distress B.) Anxiety C.) Moral distress D.) Decisional conflict

C Moral distress is cultural conflict between medical treatment and religious beliefs, expressions of concern about rejection by religious community, hesitation in accepting blood transfusion. The other diagnoses are not related.

The nurse is reviewing the patient's laboratory results. Which result must be communicated to the physician immediately? A.) Serum chloride level 85 mEq/L B.) Serum sodium level 134 mEq/L C.) Serum potassium level 6.8 mEq/L D.) Serum magnesium level 2.3 mEq/L

C Normal serum potassium level is 3.5 to 5.0 mEq/L. A serum potassium level of 6.8 mEq/L is very high and puts the patient at risk for cardiac arrhythmias. The potassium level should be reported to the physician immediately. The chlorine and sodium levels are slightly low and the magnesium level is slightly elevated.

The hospice nurse is caring for a patient who is terminally ill. The patient's spouse is the primary caregiver, providing constant care and spending all his or her time meeting the patient's needs. The spouse says to the nurse "After my spouse dies, I will finally get that colonoscopy my provider has been bugging me about." What does the nurse understand about this statement? A.) The spouse is looking forward to being freed from the caretaker role. B.) The spouse has neglected his or her own physical needs for too long. C.) The spouse is making some realistic plans for life after the death. D.) The spouse is in denial that the patient is dying and the important role of caregiver will end.

C Often caregivers neglect their own needs while in the caregiver role. The spouse understands the patient will die soon and is being realistic in understanding his or her own physical needs have been neglected. This shows healthy coping.

The acronym PICO assists in remembering the steps to constructing a good research question and the nurse identifies that the "O" in the acronym refers to what term? A.) Objectivity B.) Ordinal approach C.) Outcome D.) Observer

C One method of formulating a research question is identified by the acronym PICO (i.e., patient, population, or problem; intervention; comparison intervention; and outcomes).

The nurse is caring for a patient who is comatose. When preforming oral hygiene, which interval is most appropriate? A.) Every shift B.) Twice daily C.) Every 4 hours D.) Daily

C Oral care should be performed every 4 hours to prevent the colonization of bacteria. Less often than every 4 hours is not effective.

The nurse is preparing to assist the patient to walk to the bathroom after medicating the patient with a narcotic for pain management. What possible adverse effect should the nurse be immediately aware? A.) Constipation B.) Depression C.) Dizziness D.) Pain relief

C Potential adverse side effects of narcotics include respiratory depression, hypotension, confusion, sedation, constipation, and dizziness. The nurse should be immediately aware of dizziness during ambulation because of the safety risks. Pain relief is expected. Depression is not an immediate adverse side effect. Constipation will not impact the nurse's ability to safely ambulate the patient.

The nurse knows practicing nursing without a license is what wrongdoing? A.) Misdemeanor B.) Statute C.) Felony D.) Tort

C Practicing nursing without a license is a felony. A misdemeanor is a minor crime, such as stealing an item from a patient that does not have much value. A statute is a law created by legislative bodies. Torts are crimes committed against another person. An intentional tort example is assault and battery. Negligence and malpractice are examples of unintentional torts.

The nurse is caring for a patient who is having blood drawn as part of preoperative testing. Which step is the most important to ensure the safety of the patient and the nurse? A.) Ensuring that the tourniquet is not left in place for too long B.) Using the smallest possible needle for venipuncture C.) Properly disposing of the needle after the specimen is obtained D.) Making sure that all of the collection tubes are filled completely

C Proper disposal of needles and sharps after procedures is essential for safe nursing practice to ensure the safety of staff as well as patients. Ensuring that the tourniquet is not left on too long, using the smallest needle possible, and making sure that all of the vials are filled are important steps in venipuncture, but only proper sharps disposal will help ensure the safety of the patient and the nurse.

The nurse understands which rationale to be appropriate for drying a wound after irrigation? A.) Ensure the new dressing adheres to the wound. B.) Ensure the new dressing remains occlusive. C.) Prevent skin breakdown from moisture. D.) Prevent infection from irrigate solution.

C Proper drying prevents further skin breakdown from moisture. Patting (rather than rubbing) prevents healthy tissue from being removed and reduces trauma to the wound. The type of dressing will determine how it lays in the wound and whether it is occlusive. The drying does not prevent infection.

The nurse is caring for a patient who has been belligerent and is in 4-point "leather" restraints. When the patient continues to be verbally abusive and still tries to kick and punch staff even though he is restrained, the nurse should carry out which action? A.) Do not attempt to meet patient needs until the patient has calmed down. B.) Only provide care while security is in the room. C.) Continue to attempt to meet the patient's needs. D.) Inform the patient the police will be called if the patient's behavior does not stop.

C Provision 1.5 (of the Nursing Code of Ethics) states, "The principle of respect for persons extends to all individuals with whom the nurse interacts. The nurse maintains compassionate and caring relationships with colleagues and others with a commitment to the fair treatment of individuals, to integrity-preserving compromise, and to resolving conflict. The nurse should make all attempts to provide for the patient's needs. It is unrealistic to only provide care if security is present. Telling the patient that the police will be called is threatening.

The nursing instructor asks the student nurse to identify what Robert Wood Johnson Foundation funded project that focuses on nurses' increased attention to patient safety? A.) OSHA (Occupational Safety and Health Agency) B.) MSDS (material safety data sheets) C.) QSEN (Quality and Safety Education for Nurses) D.) ADA (Americans with Disability Act)

C QSEN, or the Quality and Safety Education for Nurses, was funded by the RWJ to focus on preparing nurses of the future with the knowledge, skills, and attitudes to advance quality and safety on the job. MSDS are material safety data sheets, OSHA is the Occupational Safety and Health Agency, and ADA is the Americans with Disability Act.

The nurse is educating a patient about a renal diet. Which statement by the patient indicates a need for further education? A.) "I need to eat a low-sodium diet." B.) "I can have limited amounts of meat." C.) "I can drink unlimited cola if it is diet." D.) "I should avoid or limit bananas."

C Renal diets restrict potassium, sodium, protein, and phosphorous intake. Fresh fruits (except bananas) and vegetables are excellent dietary choices for individuals on a renal diet. Meats, processed foods, peanut butter, cheese, nuts, caramels, ice cream, and colas are typically allowed in limited quantities or contraindicated.

What response would the nurse give the patient when questioned about the effect of rheumatoid arthritis on the musculoskeletal system? A.) Muscle weakness B.) Muscle wasting C.) Joint inflammation D.) Joint spasticity

C Rheumatoid arthritis and osteoarthritis cause inflammation of joints, resulting in pain and limited joint mobility, not muscle mobility. Genetic disorders such as muscular dystrophy result in muscle weakness and gradual muscle wasting. Spasticity (increased muscle tone) occurs in developmental disorders, such as cerebral palsy, and results in reduced range of motion (ROM) and abnormal movement patterns.

The nurse is caring for a preoperative patient who has just received sedation prior to general anesthesia in the OR. What is the priority action of the nurse? A.) Check to make sure that the consent form was signed. B.) Turn off the lights and provide a quiet environment. C.) Raise the side rails on the patient's stretcher. D.) Indicate the surgical site with an indelible marker.

C Safety of the preoperative patient is a priority after sedation has been administered. Raising the side rails of the stretcher will help prevent the patient from falling. Turning off the lights is nice but is not a priority. Marking the surgical site and signing the consent form must be performed prior to administration of sedation.

The nurse recognizes conversations about safe sexual practices, including the consequences of unprotected sex such as pregnancy and sexually transmitted infections, are important to begin in what patient population? A.) Adults B.) School-aged children C.) Adolescents D.) Older adults

C Sexual curiosity and experimentation occur in the adolescent patient population. Conversations about safe sexual practices, including the consequences of unprotected sex, such as pregnancy and sexually transmitted infections, are important. These conversations are also important for adults and older adults but are handled differently in context with their age-related needs. School-aged children may be too young depending on their age and their environment. The nurse must use judgment on when to have the conversation.

During patient teaching led by the nurse with goals established through cooperation of the nurse and patient, the patient asks questions as needed and the nurse answers. The nurse understands that this is what type of teaching? A.) Formal teaching B.) Informal teaching C.) Both formal and informal teaching D.) Psychomotor teaching

C Some patient education sessions have formal and informal elements, because the nurse and patient may set goals together before the nurse formulates and implements the plan of care, and the patient is free to ask questions that may direct the session. The health care information is considered informal because it is situation and patient specific. Formal patient education is delivered throughout the community in the form of media, in a variety of educational and group settings, or in a planned, goal-directed, one-on-one session with a patient in the acute care setting. Informal education is usually learner or patient directed. The psychomotor domain incorporates physical movement and the use of motor skills in learning. Teaching the newly diagnosed diabetic how to check blood sugar is an example of a psychomotor skill.

The nurse concerned about a patient's spiritual needs can best address this by which action? A.) Leaving a note on the chart for other professionals B.) Calling the chaplain to come see the patient C.) Collaborating during interdisciplinary rounds D.) Informing the provider of the patient's needs

C Spiritual care must be multidisciplinary to be most effective. The nurse best addresses patients' spiritual needs by discussing them during interdisciplinary rounds.

A nurse has been told he has many obvious stereotypes about a specific cultural group. What action by the nurse is best? A.) Ask to not care for members of this cultural group. B.) Ask to take care of as many members of this group as possible. C.) Begin to educate himself on aspects of this cultural group. D.) Vow to not allow his stereotypes to show when providing care.

C Stereotypes are fixed ideas, often unfavorable, about groups of people. They occur because of being unwilling to gather all the information needed to make fair determinations. The nurse would benefit most from beginning to learn about this cultural group. Caring or not caring for members of this group will not help him obtain new information. The nurse should not let stereotypes show, but this is not the best option.

The nurse is performing passive range-of-motion exercises on a patient when the patient begins to complain of pain. What is the first thing the nurse should do? A.) Notify the health care provider. B.) Hyperextend the joint. C.) Stop the range of motion. D.) Switch to active range of motion.

C Stop range-of-motion exercises if the patient begins to complain of pain or if resistance to movement is experienced. Never hyperextend or flex a patient's joints beyond the position of comfort. Active range of motion is when the patient moves the joint. Notifying the health care provider would happen later.

The nurse is providing education to a patient around anger management strategies. Which statement indicates a need for further education by the patient? A.) "Exercise can help me deal with the anger." B.) "I can use humor." C.) "I can punch things." D.) "I can take a time-out."

C Strategies should focus on nonviolent methods. Some anger management interventions include expressing feelings in a calm, non-confrontational manner; exercising; identifying potential solutions; taking a time-out; forgiving; diffusing the situation with humor; owning one's feelings; and breathing deeply.

The nurse knows that initial verification of a nasogastric placement is important. Which method is considered the only reliable method to determine enteral tube placement? A.) Auscultation of air bolus B.) Measurement of pH of the aspirate C.) Radiographic image D.) Aspirate contents to visually inspect appearance

C Studies support the use of radiographic confirmation as the only reliable method to date of confirming enteral tube placement. Using only pH and the appearance of aspirate from the newly inserted tube is not a safe method of verifying proper gastric tube placement, especially in patients receiving antacid medications. Auscultation of an air bolus to assess tube placement is no longer recognized as a reliable source in determining gastric tube placement.

The nurse is noting an order for a medication to be given TID. Which times will the nurse plan to administer the medication to the patient? A.) 9 a.m., 1 p.m., 5 p.m., and 10 p.m. B.) 9 a.m. and 9 p.m. C.) 9 a.m., 1 p.m., and 5 p.m. D.) Nightly before the patient goes to sleep

C TID indicates that the medication is to be administered three times daily. Common times for TID medications are 9 a.m., 1 p.m., and 5 p.m.

The nurse is caring for a patient who is complaining of tingling in the hands and fingers. The nurse knows this is a sign of what electrolyte imbalance? A.) Hyponatremia B.) Hypernatremia C.) Hypocalcemia D.) Hypercalcemia

C Tactile disturbances, such as tingling and numbness around the mouth and in the fingers, are signs of hypocalcemia. Mental changes are associated with both hypercalcemia and hypocalcemia. Both hypernatremia and hyponatremia have symptoms of central nervous system disorder.

The nurse is preparing to teach a 90-year-old patient. In teaching an elderly patient, the nurse realizes what information? A.) Most elderly patients are highly literate. B.) Cognitive abilities always decline with age. C.) Sensory alterations often occur with aging. D.) Teaching methods are the same as for the middle aged.

C Teaching should be tailored to elderly patients. Reports indicate that two-thirds of U.S. adults 66 years old and older have inadequate or marginal literacy skills, and 81% of patients 60 years old and older at a public hospital could not read or understand basic materials such as prescription labels. Although each patient must assessed individually, cognitive and sensory alterations often occur with aging, and the teaching materials should be adjusted accordingly.

A nurse is orienting to a new job in a home health care agency and is told that most of her patients need tertiary prevention. What activity does the nurse plan to include in the daily routine? A.) Household safety checks B.) Well-baby checkups C.) Antibiotic administration D.) Monthly blood pressure assessments

C Tertiary care is aimed at people who are already experiencing a health alteration, such as those with an infection who need antibiotics. The other options are secondary prevention.

When the nurse is caring for a patient with a Penrose drain, what care needs to be carried out? A.) The drain must be compressed after emptying to work properly. B.) The drain must be connected to suction if ordered. C.) The drain is not sutured in place so care is taken to not dislodge it. D.) The suction pulls drainage away from the wound as it re-expands.

C The Penrose drain, an open drain that is a flexible piece of tubing, is usually not sutured into place and is not connected to suction. Closed drains are compressed or connected to suction if ordered and pull drainage away as they expand.

Which statement by the nurse correctly identifies the UAP role in patient restraint use? A.) "The UAP can perform initial assessment." B.) "The UAP can apply a restraint." C.) "The UAP can assist with applying and monitoring of a physical restraint." D.) "The UAP can contact the health care provider and request an order for restraints."

C The UAP cannot perform the initial assessment, and most facilities require that a registered nurse or licensed practical nurse. Applying a restraint. The health care provider should be contacted by the nurse, not the UAP. The UAP can assist with applying the restraint and can perform monitoring checks under the direction of a Registered Nursing.

The nursing faculty member is observing a student taking a patient's carotid pulse. What action by the student requires intervention by the faculty member? A.) Counts pulse for 30 seconds and multiplies by two. B.) Performs hand hygiene prior to patient contact. C.) Compares pulses in both carotid arteries at the same time. D.) Assesses pulse on one side then assesses the other side.

C The carotid arteries are the main supply route of blood to the brain. Compressing both sides of the carotid arteries at the same time can lead to ischemia. The other actions are appropriate.

A student nurse is preparing to auscultate a patient's lungs. What action by the student leads the instructor to intervene? A.) Student asks to turn the television volume down. B.) Student warms the bell of the stethoscope before use. C.) Student uses the stethoscope bell to listen to bowel sounds. D.) Student places the stethoscope diaphragm on the patient's skin.

C The diaphragm is used to listen to bowel sounds. The other actions are appropriate.

The nurse is assessing level of stress in a patient from another culture. Which question is the most appropriate in helping the nurse understand the impact of the patient's belief system? A.) "Do you engage in prayer to help you during times of stress?" B.) "Do you go to church or other form of organized worship?" C.) "Do you have certain beliefs that are helpful during times of stress?" D.) "Do you want spiritual counseling while you are here?"

C The nurse needs to obtain a knowledge base of the patient's culture as well as identify health beliefs and cultural values from the patient's worldview. Asking the patient specific questions about prayer or church or spiritual counseling is inappropriate until the nurse first understands what the patient's own beliefs and practices are.

The nurse is caring for a patient who was just made NPO. The nurse is to administer carvedilol (Coreg) 25 mg PO to the patient for control of high blood pressure. What is the best action of the nurse? A.) Crush the medication and administer it to the patient mixed with applesauce. B.) Administer the medication to the patient with a small sip of water. C.) Contact the patient's provider to clarify the order. D.) Administer the equivalent medication dose through the patient's IV.

C The nurse should contact the patient's provider to clarify the order. Oral medications should never be administered to NPO patients without specific orders to do so from the provider. Not all medications can be administered intravenously.

A nurse is caring for a patient with a stroke that has altered her ability to see. The nurse knows which area of the brain was likely impacted by the stroke that is responsible for visual function? A.) Parietal lobes B.) Frontal lobes C.) Occipital lobes D.) Temporal lobes

C The occipital lobes process visual information. The frontal lobes of the cerebrum are the areas of the brain responsible for voluntary motor function, concentration, communication, decision making, and personality. The parietal lobes are responsible for the sense of touch, distinguishing the shape and texture of objects. The temporal lobes are concerned with the senses of hearing and smell.

The nurse identifies which goal to be appropriate for the patient who is postoperative day one from abdominal surgery and on bed rest with the nursing diagnosis impaired skin integrity? A.) Patient will ambulate twice a day. B.) Patient will eat 50% of meals. C.) Patient will have no further skin breakdown. D.) Patient will interact with others.

C The patient already has a wound, so the goal is focused on no further skin breakdown as a result of the bed rest and immobility. Although nutrition is important to wound healing, it is not the focus of this Nursing diagnosis. Ambulating and interacting with others are not goals for this diagnosis.

The nurse identifies which goal to be most appropriate for the Nursing diagnosis of acute confusion? A.) The patient will use the call light before getting out of bed within 48 hours. B.) The patient will use a calendar to remember the date within 48 hours. C.) The patient will respond appropriately to questions about place within 48 hours. D.) The patient will remain within the unit while in long-term care.

C The patient has acute confusion and therefore an appropriate early goal as the confusion resolves is to remember where they are. Remembering to use a call light would be appropriate for risk for falls. Using a calendar is appropriate for impaired memory and remaining in the unit is appropriate for chronic confusion.

The nurse is caring for a patient with benign prostatic hypertrophy who states that he feels a constant urge to urinate but cannot pass more than 30 to 60 mL of urine at a time. The nurse performs a bladder scan and finds that there are 1100 mL of urine in the patient's bladder. What is the priority nursing diagnosis for this patient? A.) Anxiety r/t continual urge to urinate B.) Reflex incontinence of urine r/t over-distention of the bladder C.) Impaired urination r/t obstruction of urinary bladder outlet D.) Impaired self-toileting r/t inability to pass urine into the toilet

C The patient has acute urinary retention with overflow as evidenced by 1100 mL of urine in the bladder and frequent passage of small amounts of urine. The priority nursing diagnosis is thus Impaired urination r/t obstruction of urinary bladder outlet. Urinary retention is the cause of the patient's discomfort and drainage of the bladder will result in relief of the patient's symptoms. The patient is able to get himself on and off the toilet so toileting self-care deficit is not a problem. Reflex incontinence of urine r/t over-distention of the bladder is not as specific to this scenario as the nursing diagnosis of impaired urination.

The nurse is caring for a patient who will be undergoing upper GI series testing the next day. Which instruction will the nurse provide to the patient about the upcoming exam? A.) "The back of your throat will be sprayed with numbing medicine." B.) "You will need to have a clear liquid diet and take a laxative tonight." C.) "You will be given a milky liquid to drink shortly before the test starts." D.) "You should not take your dose of warfarin (Coumadin) tonight."

C The patient is given a milky barium liquid to drink as part of the upper GI series, so the patient should be informed of this. The back of the throat is numbed for upper GI endoscopy, not an upper GI series. Warfarin is not contraindicated prior to an upper GI series, and no bowel prep is required.

A student nurse is working with a preceptor to administer an enema to the patient. Which action by the student prompts intervention and redirection by the preceptor? A.) Water-soluble lubricant is applied to the end of the enema tubing. B.) The enema tubing is primed with solution that has been warmed. C.) The patient is positioned comfortably in the right side-lying Sims position. D.) The patient's bedpan is put at the bedside in preparation for use.

C The patient should be placed in the left side-lying Sims position prior to enema administration so that the enema fluid will readily flow through the colon without having to go uphill. The other actions demonstrate correct enema administration steps.

The nurse is caring for an immobile patient who has abdominal pain and frequent small, liquid stools. The patient vomited his breakfast and is still nauseated. Which action by the nurse is the highest priority? A.) Provide oral care after each episode of emesis. B.) Apply a skin barrier to the patient's perineal area. C.) Check the patient for a fecal impaction. D.) Administer antiemetic medication with a sip of water.

C The patient who has abdominal pain and frequent small liquid stools should be checked for fecal impaction, especially since the patient is vomiting. Immobility is a risk factor for the development of fecal impaction. The other actions can be performed once fecal impaction is ruled out.

The nurse knows which goal to be appropriate for the nursing diagnosis of Caregiver stress? A.) The patient will report an ability to focus on discharge instructions. B.) The caregiver will attend a coping skills class on a weekly basis. C.) Caregiver will use respite care for the family loved one once a week for the next month. D.) The patient will discuss strategies for coping with relationship violence within 24 hours.

C The patient will discuss possible coping strategies during weekly office visits is an appropriate goal for Difficulty coping. The patient will report an ability to focus on discharge instructions is an appropriate goal for Anxiety. Relationship violence is not related.

The nurse is caring for a patient who recently underwent ileal conduit surgery. Which nursing diagnosis is the highest priority for this patient? A.) Impaired sexual function related to changed body structure B.) Social isolation related to potential for accidental leakage of urine C.) Lack of knowledge related to care and maintenance of ostomy appliance D.) Disturbed body image related to presence of stoma and appliance

C The patient with a new ileal conduit needs to learn how to care for the urinary stoma and appliance prior to discharge from the hospital. If the appliance is not used and applied correctly, the patient may experience urinary leakage and significant skin breakdown from exposure to urine. The other diagnoses are less important than the patient's lack of knowledge about ostomy care

The nurse is caring for a terminally ill patient who will probably die within the next 2 weeks. What is the priority nursing intervention? A.) Encouraging the patient to limit fluid intake to minimize congestion B.) Limiting the use of pain medications so that the patient can visit with family C.) Helping the patient to identify and complete desired tasks and activities D.) Completing funeral arrangements with the patient's next of kin

C The priority intervention for the nurse currently is to help the patient identify and complete desired tasks and activities while the patient is still able to do so. Pain management is a high priority at this time, so analgesics should never be limited unless requested by the patient. The patient can drink as much or as little fluid as desired.

The nurse is performing a physical assessment of patient who is undergoing a bone marrow biopsy. What finding by the nurse indicates the patient is experiencing stress? A.) Blood pressure of 120/84 B.) Temperature of 99.5 °F (37.5 °C) C.) Heart rate of 110 beats/min D.) Respiratory rate of 10 breaths/min

C The release of hormones increases the heart rate, resulting in increased cardiac output and elevated blood pressure. A reading of 120/84 is a normal blood pressure, and temperature is elevated is indicative of an infection. The respiratory rate increases in stress not decreases.

The nurse is caring for a patient with swallowing concerns and decreased level of consciousness. The nurse knows to put the patient in what position for oral care? A.) High Fowler's B.) Prone C.) Side-lying D.) Low Fowler's

C The side-lying position should be used to prevent aspiration. The high Fowler's, low Fowler's, and prone position will not prevent aspiration.

The nurse identifies which skin layer that delivers the blood supply to the dermis, provides insulation, and has a cushioning effect? A.) Stratum germinativum B.) Epidermis C.) Subcutaneous layer D.) Stratum corneum

C The subcutaneous layer delivers the blood supply to the dermis, provides insulation, and has a cushioning effect. The stratum germinativum constantly produces new cells that are pushed upward through the other layers of the epidermis toward the stratum corneum, where they flatten, die, and are eventually sloughed off and replaced by new cells. The epidermis is the outermost layer of the skin and the thinnest of the layers. The stratum corneum is made up of flattened dead cells.

The nurse is assessing the patient's use of coping skills in response to stressful situations. The nurse identifies which question to be the most useful? A.) "Have you been evaluated for stress?" B.) "Do you have someone you can go to for help when you are stressed?" C.) "How have you managed stressful situations in the past?" D.) "Does stress cause you to experience muscle tension or headaches?"

C The use of open-ended questions assists in obtaining accurate information regarding the patient's stressors and coping skills. Questions that elicit yes/no answers will not allow the patient to provide as much information. Asking the patient about headaches and tension is asking about physical symptoms, not coping skills.

A community was devastated by a tornado several months ago. What nursing diagnosis would be most appropriate for the nurse to consider? A.) Social isolation B.) Deficient community resources C.)Ineffective community coping D.) Deficient community health

C This diagnosis considers those in a community who may be feeling helpless, hopeless, or frustrated because of an extraordinary event. Financial and physical resources may not be available for rebuilding. Social isolation refers to unacceptable social behavior. Deficient community resources is not an approved diagnosis. Deficient community health may become a problem if sanitary conditions lead to an outbreak of disease.

The nurse is educating the patient on the use of relaxation therapy. Which statement by the patient indicates a need for further education? A.) "I should relax my muscles from head to toe." B.) "I visual the relaxed muscle." C.) "I should do this three times a week." D.) "I focus on muscles that are tense."

C This technique should be done daily. Typically, relaxation progresses from head to toe. With practice, the patient visualizes an image of the relaxed muscles and will be able to relax muscles from the mental image. Progressive relaxation is implemented by having patient's focus on muscles that are tensed and then intentionally relax those muscle groups.

The nurse is caring for a patient who is to have testing for fecal occult blood. What step will the nurse perform during this testing? A.) Keep the patient on a clear liquid diet for 72 hours. B.) Send the samples to the laboratory while they are still warm. C.) Inform the patient that several stool samples will be needed. D.) Use a sterile container when collecting the stool samples.

C Three stool samples are required for fecal occult testing to avoid missing blood that appears intermittently. A sterile container is not required, and the patient does not need to be on a clear liquid diet for the test. Stool samples for culture and sensitivity should be sent to the laboratory when they are fresh and warm.

Which statement by the patient indicates to the nurse a teaching need regarding safety in the home? A.) "I will put a night-light in every room." B.) "I will not use an extension cord to plug in multiple items." C.) "I will wash my throw rugs in the bathroom regularly." D.) "I will keep all cleaning supplies out of reach of children."

C Throw rugs present a fall or tripping hazard. Night-lights help light halls to prevent falls, extension cords can present a trip hazard, and cleaning supplies can contain poisonous materials.

The nurse is caring for a 6-month-old infant who has just undergone surgery. The infant's facial muscles are tight with a furrowed brow and the infant's respirations are shallow and irregular. The infant is mildly fussy and softly crying without muscular rigidity in the arms and legs. What score will the nurse give to the infant on the Neonatal Infant Pain Scale? A.) 2 B.) 3 C.) 4 D.) 5

C Tight muscles and furrowed brow = 1 point. Softly crying = 1 point. Shallow, irregular respirations = 1 point. Relaxed arms and legs = 0 points. Mild fussiness = 1 point. Total = 4 points.

The nurse is delegating care to an unlicensed assistive personnel (UAP) to a patient who has sensory overload. Which statement by the UAP indicates a need for further orientation? A.) "I should keep the noise levels low." B.) "I should schedule all the care together." C.) "I should keep the room well lit." D.) "I should allow the family to visit."

C To prevent or alleviate overload, the nurse reduces sensory stimuli, dimming unnecessary lights and turning down the sound on alarms if possible. Nursing care is planned so that the patient is not constantly disturbed. Visitation by family provides reality orientation and a soothing, recognizable presence for some patients experiencing overload.

The nurse has received an order from the health care provider to discontinue the nasogastric tube. Which action by the nurse indicates a need for further education? A.) The nurse clears the tube with air prior to discontinuing. B.) The nurse stops the tube feeding. C.) The nurse instructs the patient to cough while pulling out the tube. D.) The nurse clamps the tube while pulling it out.

C To remove the tube, instruct the patient to take a deep breath and hold it; pinch the tube, and pull it out smoothly and quickly. The nurse should stop any feedings, and suction and flush the tube with water and/or air as appropriate. The nurse should not ask the patient to cough while pulling out the tube. Coughing during tube insertion may indicate the tube is entering the patient's lungs.

The unit charge nurse uses reward and punishment to gain the cooperation of the nurses assigned to the unit. What type of leader is this charge nurse? A.) Transformation B.) Autocratic C.) Transactional D.) Situational

C Transactional leaders use reward and punishment to gain the cooperation of followers. Transformational leaders use methods that inspire people to follow their lead. Transformational leaders work toward transforming an organization with the help of others. The authoritarian or autocratic leader exercises strong control over subordinates. Situational theories suggest that leaders change their approach depending on the situation

When the nurse is caring for a patient who is receiving total parenteral nutrition (TPN), the nurse will change the tubing at which interval? A.) Every 72 hours B.) Every 48 hours C.) Every 24 hours D.) Every 12 hours

C Tubing should be changed every 24 hours, with aseptic technique used to minimize the risk of contamination, and the dressing over the site should be changed every 48 hours, with assessment for signs and symptoms of infection (redness, swelling, or drainage).

When caring for an adolescent patient with anorexia nervosa, the nurse knows what would be the best treatment option for this patient? A.) Hospitalization with skill nursing care B.) Compulsory tube feedings C.) Individually determined by a collaborative team D.) Outpatient treatment

C Ultimately, the decision on how best to ethically treat an adolescent suffering from an eating disorder needs to be one of collaboration among the child's physician, nurse, counselor, spiritual adviser, parents, and other concerned adults. Highly skilled nursing care with hospitalization is preferred prior to a drop in BMI levels below 13 kg/m2. Compulsory tube feedings are not always the best option. Although most adolescents with eating disorders can be treated on an outpatient basis, those who exhibit severe depression, extreme physical complications resulting from electrolyte imbalances, or suicidal tendencies may require extensive inpatient treatment.

The nurse is caring for a patient with a neurological condition that causes constant severe thirst, drinking fluids continuously, and voiding 3 to 4 L of clear yellow urine daily. Which term will the nurse use in the record to describe this patient's urinary output? A.) Anuria B.) Oliguria C.) Polyuria D.) Enuresis

C Urinary output greater than 2500 mL/day is polyuria. Insufficient urine output is oliguria, whereas absence of urine is anuria. Enuresis is commonly known as "bedwetting" at night.

The nurse is assigned to care for several patients on the surgical unit. Which patient need will the nurse address first? A.) A patient who is waiting for discharge teaching before going home. B.) A patient who needs to be ambulated for the first time postoperatively. C.) A patient who has not voided since the catheter was removed 8 hours ago. D.) A patient who requires a daily dressing change to the surgical incision.

C Urinary retention is common after removal of urinary catheters and must be addressed promptly. The nurse should address the patient with urinary retention first because it is the highest priority. The other patients may wait until later.

The nurse is educating the family to care for a patient at home with cognitive alterations. Which statement by the family indicates a need for further education? A.) "I should keep the home free of scissors." B.) "I should minimize the number of visitors." C.) "I should use push-button door locks." D.) "24-hour supervision may become necessary."

C Use of door locks that require a key may be necessary if the patient wanders. Keep the environment free of hazards such as sharp objects and minimize distractions. If the patient is not safe to be left alone, 24-hour supervision may be necessary.

The nurse is caring for a patient on a medical-surgical inpatient unit when the patient tells the nurse he is very sad and is considering suicide. What is the first thing the nurse should do? A.) Notify the health care provider. B.) Make a referral to psychiatric services. C.) Implement one-on-one observations. D.) Document in the electronic medical record.

C Verbalization of suicidal ideation or a suicide plan must be taken seriously. In the case of a hospitalized patient, one-on-one observation should be implemented to ensure patient safety. Once the patient is under observation, the health care provider is notified to put in the referral; nurses generally do not put in the referral. Documentation is always done after the patient's safety is ensured.

The nurse makes a medication error. Which action will the nurse take first? A.) Prepare an incident report. B.) Explain to the patient that a medication error has occurred. C.) Assess the patient for any adverse reactions. D.) Document the medication given, the response, and corrective actions taken.

C When a medication error occurs, the nurse's priorities are to determine the effect on the patient and intervene to offset any adverse effects of the error. Actions include immediate and ongoing assessment, notification of the prescribing health care provider, initiation of interventions as prescribed to offset any adverse effects, and documentation related to the event. Error reporting is an essential component of patient safety and should be completed as soon as the patient is assessed and stable. The nurse should follow facility guidelines for medication error reporting.

The nurse is performing an assessment of the patient's sleep patterns. Which question by the nurse will elicit the best response? A.) "Do you feel rested when you awaken?" B.) "What is your normal eating pattern?" C.) "Do you awaken during the night?" D.) "Do you drink beverages with caffeine?"

C When gathering information about the patient's sleep patterns, the best questions will be to ask if they awaken during the night. Asking the patient if they feel rested evaluates quality of sleep. Normal eating pattern and drinking beverages with caffeine establishes if they have any habits that might interfere with sleep.

The nurse is repositioning the patient in the side-lying position. To avoid putting the patient at risk for pressure ulcers, the nurse should place the head of the bed in which position? A.) Flat B.) 90 degrees C.) 30 degrees D.) 45 degrees

C When side-lying, patients should be positioned at 30 degrees, as opposed to 90 degrees, to avoid positioning the patient directly on bony prominences such as the head of the trochanter.

The nurse is caring for a patient with a new diagnosis of diabetes type 2. Which statement indicates a negative coping response? A.) "I will look up information on the Internet about diabetes." B.) "I will join a support group." C.) "I will only focus on learning to manage my medication first." D.) "I will make changes slowly so I can adapt to each change."

C When the patient puts limits on learning by stating he/she will only learn about medication, he/she is using avoidance strategies to alleviate stress. Using strategies such as information gathering (seeking information about diabetes) is positive. Joining support groups and making changes slowly to adapt is also taking direct action by moving forward.

The nurse is correctly assisting the patient in using a cane when the patient demonstrates which activities? (Select all that apply.) A.) The top of the cane is level with the patient's bent elbow. B.) The patient holds the cane on his/her weaker side. C.) The patient moves the cane forward first. D.) The patient's arm is comfortably bent when walking. E.) The patient moves the strong leg forward first.

C and D. The top of the cane should be level with the hip joint, and the patient's arm should be comfortably bent when the patient is walking. The patient should hold the cane on his/her stronger side and move the cane forward first, followed by the weaker leg and then the stronger leg. This ensures that another point of support is always on the ground when the weaker leg is bearing weight and gives the patient a wide base of support. A patient using a cane should be encouraged to stand up straight and look forward. Leaning to one side or looking down can jeopardize safety and cause poor posture.

The community health nurse knows that which are standards of professional performance for home care nurses according to the ANA? (Select all that apply.) A.) Collegiality B.) Performance appraisal C.) Ethical behavior D.) Outcome identification E.) Resource utilization

C and E. The ANA's Public Health Nursing: Scope and Standards of Practice (2013) requires participation in research, responsible resource utilization, ethical behavior, leadership, and advocacy similar to the standards of practice for all nurses.

A 52-year-old patient is admitted to the hospital for treatment of lymphoma. The nurse enters the patient's room to find the patient worried and depressed. What should the nurse do according to the caring principles? Select all that apply. A.) Do not disturb the patient B.) Do necessary work and leave without speaking C.) Greet the patient and gently touch her shoulder D.) Make good eye contact and sit next to the patient. E.) Ask the patient about her concerns and listen to her complaints patiently.

C, D and E.

The nurse is caring for a patient with expressive aphasia. Which interventions will assist the nurse in communicating with the patient? (Select all that apply.) A.) Use simple phrases. B.) Speak loudly. C.) Use yes/no questions. D.) Use a picture board. E.) Be patient and unrushed.

C, D and E. If a patient has expressive aphasia, he or she understands language but is unable to answer questions, name common objects, or state simple ideas. The patient can answer yes/no questions by shaking the head. The patient might be able to point to pictures to express needs. For any type of aphasia, being patient and not rushing will make communication less stressful.

The nurse is caring for a patient who suffered a stroke on the right side of the brain. The nurse is careful to implement what safety measures? (Select all that apply.) A.) Puts a picture board in the room to communicate with the patient. B.) Places the call light on the patient's left side. C.) Leaves a light on in the bathroom at night for good visibility. D.) Places the call light on the patient's right side. E.) Makes sure there are no trip hazards in the patient's room.

C, D and E. If the damage is on the right side of the brain, there is loss of sensation and motor function in the extremities on the left side of the body and visual-spatial problems occur. Therefore, placing the call light on the side where the patient is likely to be strong is important. The patient will not necessarily have communication problems but might have visual problems, so the bathroom light is helpful. If the damage is on the left side of the brain, there is loss of sensation and motor function in the extremities on the right side of the body and problems with speech occur.

The nurse is caring for a patient who has B-positive blood. The patient is severely anemic and requires a blood transfusion. The nurse knows which types of blood can the patient receive? (Select all that apply.) A.) AB positive B.) AB negative C.) B negative D.) B positive E.) O positive F.) O negative

C, D, E, and F. If a person produces the B antigen, the blood type is classified as B. Type O blood is classified as universal donors because their blood cells contain no antigens. Rh positive (Rh+) blood which means the person has the Rh factor on the surface of the red blood cells. Those who do not have the Rh factor are considered Rh negative (Rh−). A person who is B positive can receive B or O blood, and it can be positive or negative Rh factor.

A nurse decides to use Swanson's theory of caring in clinical practice. What are the caring processes of this theory which the nurse should implement when caring for patients? Select all that apply. A.) Randomization B.) Bias C.) Being with D.) Doing for E.) Knowing

C, D, and E.

The nurse is preparing discharge instructions for a patient who has tactile alterations in his legs. Which instructions would be included? (Select all that apply.) A.) Verify bath water temperature is approximately 39.5 °C. B.) Do not use hot or cold therapy on any extremity. C.) Use sturdy shoes when walking outside or on hard surfaces. D.) Report any changes in skin color on your legs to your health care provider. E.) Set your water heater so that scalding is not possible.

C, D, and E. Bath water temperature should be approximately 37.8 °C (100 °F), so 39.5 (103.1 °F) is too hot. Hot and cold therapy should not be used on the affected extremities, although it can be used on other areas of the body. Sturdy shoes can prevent foot injuries when there is decreased sensation in the lower extremities. Any decrease in sensation, change in the color of the skin, or wounds are reported to the health care provider. Water heaters are set so that scalding is not possible.

The nurse identifies what decisional roles that are included in Mintzburg's description of management in terms of behavior? (Select all that apply.) A.) Figurehead B.) Spokesperson C.) Entrepreneur D.) Resource allocator E.) Negotiator

C, D, and E. Mintzberg described management in terms of behaviors. Underlying his descriptions were two assumptions: much of a manager's time is spent in human relations, and managers are more reactive than proactive. These assumptions provided the basis for three categories of behaviors: interpersonal roles, informational roles, and decisional roles. Mintzberg described three interpersonal roles: figurehead, leader, and liaison. The three informational roles he described are monitor, disseminator, and spokesperson. The third category of Mintzberg's behavioral roles comprises the four decisional roles: entrepreneur, disturbance handler, resource allocator, and negotiator.

The nurse recognizes which changes in sleep patterns occur in the older adult? (Select all that apply.) A.) Sleep increases to approximately 8 to 10 hours a night. B.) REM sleep is shorter. C.) Stage 4 NREM is decreased. D.) The use of medication may interfere with sleep. E.) Older adults awaken more at night.

C, D, and E. Older adults sleep approximately 7 to 8 hours a night. The first REM stage is longer. There is no stage 4 REM. Deeper stages of sleep are shortened, resulting in less restorative sleep. A decline in health or the use of medications may interfere with sleep. Older adults awaken more at night and take longer to go back to sleep.

The student nurse learns the ANA's Scope and Standards of Practice for public health nursing include components? (Select all that apply.) A.) Team membership B.) Developing research C.) Ethical behavior D.) Responsible resource use E.) Advocacy

C, D, and E. The ANA's Scope and Standards of Practice for public health nursing requires participation in research, responsible resource utilization, ethical behavior, leadership, and advocacy like the standards of practice for all nurses. Team membership and developing one's own research are not included.

Which of the following actions indicates the act of refreezing in Lewin's Change Theory? A.) A patient overcomes inertia and changes her mindset. B.) There is time of transition and confusion when change takes place. C.) Change is completed, reinforced, and accepted. D.) The right environment is created for change.

C.

Nursing diagnoses are primarily used to: A.) Make all the patient's problems easier to solve. B.) Assist the medical provider to determine care. C.) Meet accreditation requirements. D.) Facilitate clear communication of patient needs.

D

The patient is reportedly well educated and employed as an engineer, but is struggling to comprehend terms found in health-related literature given to explain his disease process. This is evidence of? A.) Low literacy B.) Psychomotor dysfunction C.) Affect domain deficiency D.) Low health literacy

D

The patient tells the nurse that she is not confident with self-injection of insulin. The nurse should use which of the following to validate this information from the patient? A.) Ask the family how the patient performed the self-injection. B.) Confer with other staff members to see how the technique was taught to the patient. C.) Determine what insulin was prescribed by the provider. D.) Observe the patient giving the insulin injection.

D

What does the nurse recognize as a "related factor" in the nursing diagnosis? A.) Psychological factors B.) Environmental factors C.) Physical factors D.) Etiological factors

D

Which collaborative team member would be most effective in assisting the nurse to identify medication alternatives that are less likely to cause drowsiness and dizziness to reduce the risk of falls in elderly patients? A.) Nursing house manager B.) Charge Nurse C.) Physical Therapist D.) Pharmacist

D

Which of the following is an example of tertiary prevention? A.) Providing information on immunization B.) Screening a family for a hereditary disease C.) Presenting a class on hand hygiene to an elementary school class D.) Working with a patient who is paraplegic to prevent musculoskeletal complications

D

Which of the following is associated with specifically meeting the Quality and Safety Education of Nurses (QSEN) Teamwork and Collaboration competencies? A.) Providing the patient with the schedule for diagnostic testing. B.) Assessing the patient's level of pain C.) Engaging the patient in conversation D.) Working with the patient and nutritionist

D

Which of the following is the best example of a measurable patient goal? The patient will: A.) Sit out of bed in the chair B.) Eat low-sodium foods C.) Verbalize feelings about surgery at some point. D.) Identify the 5 major drug side effects before discharge tomorrow.

D

Which of the following statements is an example of reflecting? A.) "Tell me more about how you feel." B.) "What is your biggest concern?" C.) "Your surgery is scheduled for 10am tomorrow morning." D.) "You feel like the world is closing in on you and making it hard to breathe?"

D

The nurse identifies which syringe to use when irrigating a patient's deep wound? A.) 5-mL syringe B.) 10-mL syringe C.) 3-mL syringe D.) 30-mL syringe

D A deep wound is irrigated with a 30- to 50-mL piston syringe with an 18-gauge angiocath. Unlike the 1 pound per square inch (psi) of pressure or less that is delivered by a standard bulb syringe, the use of a 30- to 50-mL syringe and 18-gauge catheter has been shown to achieve an irrigation force that falls within the recommended 4 to 15 psi.

The nurse is caring for a patient with syndrome of inappropriate antidiuretic hormone secretion (SIADH) who has a serum sodium level of 118 mEq/dL and symptoms of fluid overload. Which IV fluid will the nurse expect to administer to this patient to correct the patient's fluid imbalance? A.) 0.33% normal saline B.) 0.45% normal saline C.) 0.9% normal saline D.) 3% normal saline

D A hypertonic 3% saline solution will be used to correct the patient's hyponatremia and fluid overload that have developed due to SIADH. A 0.9% normal saline solution can be used once the serum sodium level has been raised nearer to normal range. A 0.45% or 0.33% normal saline solution is hypotonic and will only worsen the patient's fluid overload and hyponatremia.

The nurse is caring for a patient who has a serum magnesium level of 0.8 mEqL. Which is the highest priority goal to include in the patient's plan of care? A.) The patient will maintain urine output of at least 30 mL/hr. B.) The patient will verbalize the importance of sufficient dietary intake of magnesium. C.) The patient's oral mucous membranes will remain free of ulceration and pain. D.) The patient will remain alert and oriented ×3 with no confusion or seizure activity.

D A patient with low serum magnesium is at risk for neurologic symptoms including confusion, disorientation, and seizures. The highest priority goal for this patient is to avoid neurologic problems that could lead to injury. The other goals are applicable to the patient with low magnesium but are less important.

A school nurse is planning a sex education activity. What information from research does the nurse apply to this education? A.) Sex education should be taught in high school. B.) The school nurse should be the primary source of sex education. C.) The method of birth control that should be presented is abstinence. D.) Parents' open communication regarding sex education has a positive impact on their children.

D According to research, parents should be encouraged to have open communication with their children regarding sex educations. The study noted sex education initiated in the sixth grade had an overall positive effect in delaying sexual activity. The school-based program is secondary to parental input and supports that input by providing knowledge and skill building. Open communication would encourage discussion of a variety of birth control methods.

The nurse is caring for a patient who has just been brought to the postoperative unit following major surgery and notes that the patient has many tubes and monitors in place. Which will the nurse assess first? A.) The patient's intravenous lines B.) The patient's urinary catheter C.) The patient's nasogastric tube D.) The patient's endotracheal tube

D Airway maintenance and protection is the highest priority for this patient, so the nurse should assess the endotracheal tube first to ensure that it is patent and positioned correctly. The other tubes may be assessed afterward.

The nurse is caring for a diabetic patient who has painful foot neuropathy. The patient asks why the nurse is administering gabapentin (Neurontin) when there is no history of seizure disorder. What is the nurse's best response? A.) "Gabapentin will help you sleep at night so you can deal with the pain more effectively." B.) "Long-term diabetes can put patients at risk for certain type of seizures." C.) "This medication can help relieve your anxiety from being admitted to the hospital." D.) "Gabapentin works on the nervous system to help relieve the burning pain in your feet."

D Anticonvulsant medication like gabapentin and tricyclic antidepressants are often used to relieve neuropathic pain as they work directly on the nervous system. The other statements do not correctly indicate why the patient is receiving this medication.

The nurse researcher audiotaped interviews with subjects and would like to play these tapes during dissemination. The nurse identifies what steps that may be required to play the tapes? A.) Inform the participants that they cannot hear the tapes beforehand. B.) None, if the tape is of a group, since there is no expectation of anonymity. C.) None, since the tape is a direct "quote" and voice recognition is not controllable. D.) A release will need to be obtained from the subjects.

D Any videos, photos, or audiotapes require releases if they are to be shown in the dissemination of research findings. The participants have the right to review these tapes before allowing them to be used for research. If subjects are involved in a group, they should be reminded that their exchange of information and identities must remain confidential. Participants' permission is needed if the data include quotations or can reveal the subject's identity.

The home care nurse is caring for a terminally ill patient who states that he wants to set up a scholarship in his name at the local university before he dies. What is the best action by the nurse? A.) Suggest that the patient think it over and wait a few days before contacting the school. B.) Direct the patient to ask his family about the possibility of starting a scholarship. C.) Assess the patient's mental status to ensure that he is competent to make the decision. D.) Assist the patient to find the necessary information about endowed scholarships.

D As the patient's advocate, the nurse should help provide the necessary information for the patient to set up a scholarship if that is his decision. The patient does not need to discuss the subject with his family first, and assessment of the patient's mental status is not needed. The patient may not have the time to wait a few days before contacting the university.

The nurse wants to help a hospitalized patient get more sleep. What intervention will be most helpful? A.) Allow the patient an hour nap during the day shift. B.) Administer sleeping medication if the patient can't go to sleep after an hour. C.) Place a "do not disturb" sign on the door for the duration of the night shift. D.) Cluster cares so the patient gets at least 90 minutes of uninterrupted sleep at night.

D Being able to sleep for at least 90 minutes without interruption will best help this patient as the normal sleep cycle is approximately 90 minutes long. An hour nap is probably too long even in the hospital. Administering sleeping pills is an alternative if the patient truly can't sleep while hospitalized, but this is not the preferred solution. Not disturbing a hospitalized patient for an entire night shift is unrealistic.

The nurse will be caring for a patient who has just arrived on the medical-surgical unit following surgical repair of his fractured right ankle. Which is the priority action of the nurse when the patient arrives on the unit? A.) Instruct the patient how to call for assistance using the call light. B.) Assess the color and warmth of the toes on the patient's right foot. C.) Determine when the patient's next pain medication is due. D.) Check pulse oximetry and obtain a full set of vital signs.

D Checking pulse oximetry and vital signs is the priority action when the patient first arrives on the medical-surgical unit from the postoperative area. The other actions can wait until the vital signs have been obtained.

The nurse is caring for a patient with rheumatoid arthritis who is in constant severe pain. Which nursing diagnosis is the highest priority for this patient? A.) Impaired mobility r/t patient's need to use a cane or walker with ambulation B.) Impaired health maintenance r/t sedentary lifestyle and poor physical condition C.) Anxiety r/t mistrust of health care personnel D.) Chronic pain r/t ongoing inflammatory tissue damage and joint destruction

D Chronic pain is the highest priority diagnosis for this patient because it is severe. The other diagnoses may be addressed once the patient's pain is controlled.

The nurse is caring for a patient who is recovering from gastroenteritis. The nurse teaches the patient about dietary recommendations as the digestive system recovers. Which menu selection by the patient indicates that additional teaching is needed? A.) Applesauce B.) Orange Popsicle C.) White toast D.) Coffee with cream

D Coffee with cream should be avoided by patients recovering from gastroenteritis because milk proteins are difficult for the digestive system and caffeine increases peristalsis. Caffeine is also a diuretic, which can lead to continued dehydration.

A patient is finding conflict when trying to maintain personal beliefs while making health care decisions. What Nursing diagnosis is a priority as the nurse plans care? A.) Spiritual distress B.) Impaired religiosity C.) Moral distress D.) Decisional conflict

D Decisional conflict is unclear personal beliefs, questioning of personal beliefs while making decisions, delayed decision making. The other diagnoses may exist as well, but they are not manifested by this conflict.

The nurse is caring for a patient who is to collect a 24-hour urine specimen. Which statement by the patient indicates that additional teaching is required? A.) "I will keep the urine container on ice to keep it chilled until I bring it to the lab." B.) "I will start the test over if I forget and urinate into the toilet during the testing time." C.) "I will start the test tomorrow after I urinate first thing in the morning." D.) "I will drink extra fluids so that the lab will have a large specimen to test."

D Drinking extra fluids so that the lab will have an extra-large specimen to test is not done as part of 24-hour urine collection, and it may skew the test results. The specimen should be kept chilled on ice or in a refrigerator until it is brought to the lab. If the patient accidentally urinates in the toilet, the test must be started over again. Urine collection is started after the patient' s first void of the morning into the toilet.

The nurse has delegated to the UAP to assist a patient with ambulating in the hallway with a cane. Which statement by the UAP indicates a need for further education? A.) "I should report any complaints of soreness to the nurse." B.) "I should watch for indications that the patient has difficulties using the cane." C.) "I should let the nurse or PT know if the cane doesn't seem to fit correctly." D.) "I should teach the patient how to walk with the cane."

D Educating patients on how to walk with assistive devices may not be delegated to unlicensed assistive personnel (UAP). UAP should report any of the following: noticeable incorrect usage or fit of assistive devices, complaints of soreness or weakness, difficulties involving balance or strength, or difficulties in performing the procedure or other concerns verbalized by the patient.

The nurse frequently cares for patients who are nearing the end of life. The nurse identifies what strategy that is designed to prolong the time of death rather than restoring life? A.) Establishing a do-not-resuscitate (DNR) order B.) Adherence to living will requests C.) Removal of extraordinary measures already in place D.) Continuance of futile care

D Ethical dilemmas in end-of-life care exist regarding the establishment of do-not-resuscitate (DNR) orders, adherence to living will and organ donation requests, removal of extraordinary measures already initiated, and continuance of futile care (i.e., care that is useless and prolongs the time until death rather than restoring life).

The nurse is caring for a patient from a culture that is unfamiliar. The patient nodded her head "yes" when asked if she will take her prescriptions as ordered, but the nurse discovers the patient does not take the medication but uses herbs for treatment. What action by the nurse is best? A.) Warn the patient of the consequences on noncompliance. B.) Tell the patient how the medication will help the condition. C.) Ask the patient why herbal preparations are preferred. D.) Ask the patient to explain the meaning of the herbal products.

D Ethnocentrism is the belief that one's cultural beliefs are superior to others. To avoid practicing in an ethnocentric manner, the nurse needs to understand the meaning of the herbal preparation to the patient. Warning the patient of bad outcomes will not achieve the desired results if the herbs are culturally important and meaningful to her. Patient education is always important but is not the best answer because it does not allow the nurse to learn from the patient. Asking "why" question is a communication barrier likely to put the patient on the defensive.

A male patient takes a medication known to cause erectile dysfunction. What action by the nurse is best? A.) State, "If this medication has bad side effects, talk to your doctor." B.) Ask, "Are you having any sexual problems in your life right now?" C.) Give the patient written information on the side effects of the drug. D.)State, "Many men have erectile dysfunction on this drug."

D Giving the patient factual information is best. The nurse can follow up on this statement by asking the patient if the medication is affecting his sexuality. "Bad side effects" is vague, as is "any sexual problems." Written information may be helpful, but the patient may not be literate and this does not allow the nurse to be engaged with the patient.

The nurse is caring for a patient whose mother recently passed away. The patient states that she has not been able to concentrate or sleep since the funeral and is consuming increasing amounts of alcohol to get through each day. The nurse knows which goal to be most appropriate for this patient? A.) The patient will be referred to medical social services for evaluation and counseling. B.) The patient will be encouraged to describe previous stressors and coping mechanisms. C.) Nursing staff support patient's coping attempts and encourage verbalization of feelings. D.) The patient will use effective coping strategies with no alcohol consumption.

D Goals are met by the patient rather than nursing or medical staff. The patient's use of effective coping strategies without drinking alcohol is an appropriate goal. Referring the patient for counseling and encouraging the patient to verbalize stressors are interventions rather than goals.

The nurse is assessing the patient's ability to hear and knows which is the correct procedure for the doing this? A.) The nurse whispers to the patient while standing on each side of the patient. B.) The nurse speaks in a normal voice while standing on each side of the patient. C.) The nurse speaks in a normal voice while standing directly in front of the patient. D.) The nurse speaks in a normal voice while standing slightly behind the patient.

D Hearing ability can be determined by observing the patient's conversation and responses and by talking with the patient in a normal conversational tone while standing slightly behind the patient. If the patient does not respond appropriately, a hearing impairment may exist. Standing in front of the patient allows the patient to read your lips and will not detect a hearing loss. A whispered voice will also give a false reading.

The nurse knows that a hydrocolloid dressing is appropriate for use on which type of wound? A.) A wound with a large amount of drainage B.) A wound that is tunneling C.) A postsurgical incision with staples D.) A wound with a moderate amount of drainage

D Hydrocolloids are occlusive, adhesive dressings composed of gelling agents and carboxymethylcellulose. They absorb a small to moderate amount of drainage over a 3- to 7-day period, forming a gel as drainage is absorbed. A wound with a large amount of drainage would require a foam or alginate dressing, a postsurgical incision with staples could use Steri-Strips or gauze, and a wound that is tunneling may require packing.

The nurse is caring for a patient who will be receiving iodine-based contrast medium for a CT scan. Which allergy should be reported to the technician and radiologist before the test is performed? A.) Gluten and lactose B.) Strawberries and blueberries C.) Peanuts and cashews D.) Shrimp and scallops

D If the patient is undergoing an examination that involves an iodine contrast medium, check for a history of adverse reactions or allergies to iodine-containing food (e.g., shellfish, cabbage, kale, iodized salt). The other allergies are not related.

The nurse knows changes in which body system affect overall mobility increasing the propensity of falling? A.) Neurologic B.) Hepatic C.) Cardiopulmonary D.) Musculoskeletal

D Impairments in the musculoskeletal system can impact mobility through restrictions of range of motion and strength, increasing the chances of falling. Changes to the neurologic system can impair cognitive functioning, changes to the hepatic system can affect mental status, and changes to the cardiopulmonary system can affect activity tolerance.

While conducting a controlled research study, the nurse wants greater assurance that the result is due to treatment itself and not another factor. For this purpose, the researcher should include what other component? A.) A treatment group B.) An independent variable C.) A dependent variable D.) A control group

D In a controlled study, some of the participants are assigned to the treatment group, and others are assigned to the control group by a random process. The control group does not receive the treatment. In the clinical trial of a medication, the control group receives a placebo. The purpose of a control group is to prevent bias and ensure that the outcome results from the treatment rather than some other factor. An independent variable is a concept or idea whose value determines the value of other (dependent) variables. In research, the independent variable comprises the experimental treatment or intervention, and it is manipulated by the researcher to yield various outcomes. The dependent variable is the outcome that is affected by manipulation of the independent variable.

The nurse is performing an oral examination on a patient and notices a beefy-red tongue. The nurse identifies this as a characteristic finding for what condition? A.) Anorexia nervosa B.) Malnutrition C.) Bulimia D.) Pernicious anemia

D In conditions such as pernicious anemia, a characteristic finding is a sore, smooth-surfaced, beefy-red tongue, which may interfere with the person's ability to chew certain foods. Anorexia nervosa and bulimia are eating disorders. In malnutrition the oral mucosa may be a darker red than normal with oral lesions and/or the tongue may reveal white irregular areas.

The nurse understands who is ultimately responsible for explaining the content of the informed consent? A.) The registered nurse B.) The hospital social worker C.) Educated family members D.) The provider of the procedure

D Informed consent is permission granted by a patient after discussing each of the following topics with the physician, surgeon, or advanced practice nurse who will perform the surgery or procedure: (1) exact details of the treatment, (2) necessity of the treatment, (3) all known benefits and risks involved, (4) available alternatives, and (5) risks of treatment refusal.

The nurse is caring for a patient with congestive heart failure who requires intermittent IV bolus doses of furosemide (Lasix) to correct fluid volume overload. No continuous IV fluids are ordered. Which type of IV will the nurse insert to administer the patient's medication? A.) Peripherally inserted central catheter B.) Midline inside-the-needle catheter C.) Central venous catheter D.) Over-the-needle catheter

D Intermittent doses of IV diuretics are best administered via an over-the-needle angiocatheter that is connected to a saline lock. The other IV catheter options are used when the patient requires a vesicant drug that could cause significant damage to tissues or when the patient requires weeks of IV therapy.

The nurse administers a medication to the patient. Which symptoms indicate to the nurse that the patient is having an allergic reaction rather than a side effect? A.) Hair loss and sweaty skin B.) Nausea and constipation C.) Heartburn and nasty taste in the mouth D.) Itchy rash and difficulty breathing

D Itchy rash and difficulty breathing are indicative of an allergic reaction to a medication. The other symptoms are common side effects of medications.

The nurse is providing care to a patient experiencing pain. The nurse assesses the pain and promptly administers the ordered analgesics as promised to the patient. This nurse has applied what concept? A.) Autonomy B.) Accountability C.) Confidentiality D.) Fidelity

D Keeping promises or agreements made with others constitutes fidelity. In nursing, fidelity is essential for building trusting relationships with patients and their families. Following through on promises is a critical factor in establishing strong professional relationships with patients and their families. Autonomy, or self-determination, is the freedom to make decisions supported by knowledge and self-confidence. Accountability is the willingness to accept responsibility for one's actions. Confidentiality is the ethical concept that limits sharing private patient information.

After studying legal issues important to nursing, the student shows appropriate understanding with which statement? A.) Laws change often, creating liability issues for nurses. B.) Licensure laws are devised to protect the nurse. C.) The nurse is not responsible for other disciplines' mistakes. D.) Keeping current with changing laws can protect the nurse.

D Laws delineate acceptable nursing practice, provide a basis on which many health care decisions are determined, and protect nurses from liability in cases in which safe practice is maintained. Each state has a nurse practice act that establishes the standards of care required for legal nursing practice. Licensure, laws, rules, and regulations governing nursing practice are enforced to protect the public from harm. In many cases, the nurse is the last line of defense to prevent an error in medication administration or other types of patient care. Keeping current with changing laws related to nursing practice and technology can ensure safety for nurses and their patients.

The nurse has made patient care assignments and expects all team members to set their own goals for the day and manage their time to meet their goals. The nurse is implementing what style of leadership? A.) Autocratic B.) Democratic C.) Bureaucratic D.) Laissez-faire

D Like the democratic leader, the permissive or laissez-faire leader thinks that employees are motivated by their own desire to do well. The laissez-faire leader provides little or no direction to followers, who develop their own goals and make their own decisions. The authoritarian or autocratic leader exercises strong control over subordinates. The participative or democratic leader believes that employees are motivated by internal means and want to participate in decision making. The primary function of the leader in this situation is to foster communication and develop relationships with followers. Like the autocratic leader, the bureaucratic leader assumes that employees are motivated by external forces. This type of leader relies on policies and procedures to direct goals and work processes. The nurse using bureaucratic leadership tends to relate impersonally to staff and exercises power on the basis of on established rules.

The nurse is caring for an emergency room patient who died because of a mishap with a loaded gun. The patient's body will be transported to the coroner's office for an autopsy. Which items will the nursing staff remove from the body before it leaves the hospital? A.) Endotracheal tube B.) Foley catheter and IV line C.) Dentures D.) Necklace and watch

D Medical devices and tubes are not removed from the body if an autopsy is to be performed. The patient's necklace and watch may be removed and given to the patient's family members before the body is transported to the coroner's office for autopsy. Dentures should be left in the patient's mouth.

The nurse is teaching a patient about the difference between mild anxiety and moderate anxiety. Which statement by the patient indicates a need for further education? A.) "Mild anxiety can help me remember things." B.) "Moderate anxiety will narrow my focus." C.) "Mild anxiety will help me be creative." D.) "Moderate anxiety will increase my perception."

D Moderate anxiety narrows a person's focus, dulls perception, and may challenge a person to pay attention or use appropriate problem-solving skills. Mild anxiety can be motivational, foster creativity, and increase a person's ability to think clearly.

A nurse has assessed a patient's capillary refill, which was 5 seconds. What action by the nurse is most appropriate? A.) Document the findings and continue the examination. B.) Ask the patient about the use of artificial nails. C.) Ask the patient about his/her occupation. D.) Assess the patient for signs of hypoxia.

D Normal capillary refill is 2 to 3 seconds. Prolonged capillary refill can indicate hypoxia, anemia, or circulatory insufficiency. The nurse should document the findings, but further action is not needed. Asking about artificial nails and occupation are not warranted.

When applying research to practice, the nurse finds what information? A.) It is usually easy to access information at the bedside. B.) Research articles are clear in defining nursing practice. C.) Bedside care is not directly related to research. D.) Nursing research should be used to improve care.

D One obstacle to applying research to practice is the difficulty in bedside access to information by nurses. Nurses often lack the time to participate in research-related activities. By reading research articles, the nurse may notice discrepancies in what is recommended in current practice and what is found in the literature. Nurses often feel that their bedside nursing care is removed from the research process. However, nurses participate every day in the care of patients, which is based on the nursing process. Nurses should use research to improve the quality of patient care and should understand the research base before initiating nursing interventions.

The nurse is caring for a patient who has just undergone paracentesis. For which complication will the nurse carefully monitor? A.) Collapse of the lung with shortness of breath B.) Fecal impaction from retained barium in the colon C.) Cerebrospinal fluid leak resulting in severe headache D.) Perforation of the bowel resulting in abdominal infection

D Paracentesis is drainage of fluid from the abdominal cavity. Since the needle is near the intestines, bowel perforation can occur, manifested by abdominal pain and fever as infection (peritonitis) sets in. Possible complications do not include lung collapse, CSF leak, or impaction.

The nurse is asked to shave a patient who is taking warfarin (Coumadin). What is the most appropriate action? A.) Refuse to shave the patient because he is on an anticoagulant. B.) Shave as usual with a safety razor. C.) Offer to wax rather than shave the patient. D.) Use an electric razor.

D Patients on anticoagulants should use an electric razor for shaving to avoid bleeding complications. Patients should have the option of shaving if they would like to shave. Waxing may not be an option.

The nurse is caring for a patient with a history of dementia who is incontinent of stool because of the inability to communicate the need to defecate. What is the priority action of the nurse? A.) Administer a daily laxative and take the patient to the toilet afterward. B.) Digitally remove stool from the patient's rectum every other day. C.) Insert a rectal tube to facilitate drainage of soft or liquid stool. D.) Begin a prompted toileting program to facilitate bowel continence.

D Patients who cannot communicate the need to use the toilet often benefit from a prompted toileting program in which the patient is brought to the toilet at the same times each day to promote urinary and bowel continence. A rectal tube should not be used. Digital removal of the impaction should be avoided whenever possible. Laxatives should be used only when necessary because continued use will lead to dependence.

Which member of the collaborative team is most appropriate to cut the toenails of a diabetic patient? A.) Nurse B.) Physical therapist C.) Occupational therapist D.) Podiatrist

D Patients with diabetes are usually seen by a podiatrist or diabetic specialist for foot care needs. Nurses can trim toenails of patients not at risk for infection. Physical therapists provide services that restore function and mobility. Occupational therapists use treatments to maintain or restore daily living and work skills.

The nurse correctly identifies which patient as having the highest risk for injury related to temperature of water when bathing? A.) Patient with asthma B.) Patient with attention deficit hyperactivity disorder C.) Patient with a stroke D.) Patient with diabetes

D Patients with neurologic deficits such as peripheral neuropathy resulting from diabetes may not be able to identify extremes of hot and cold. Patients with attention deficit hyperactivity disorder and asthma are not likely to be injured by temperature extremes. Patients with a stroke may have some alteration in sensation on one side of their body but can compensate by using the other side, and they are at less risk than a patient with diabetes.

The nurse is preparing to conduct a research study and is interested in exploring the lived experiences of nurses responsible for approaching patients and family members about the donation of organs. Which type of research would this be considered? A.) Grounded theory B.) Ethnography C.) Historical D.) Phenomenologic

D Phenomenologic research explores the reactions of a specific group of people who experienced a similar event in their lives. Grounded theory research derives theories from the data collected in studies. Ethnography focuses on the sociology of meaning through close field observation of a sociocultural phenomenon. The term ethnography is sometimes applied to the field notes or case studies produced from ethnographic research. Historical research studies historical documents to determine an accurate picture of a past event or time period.

The nurse identifies which patient who would benefit from postural drainage? A.) A patient with a heart murmur and jugular venous distention B.) A patient with asthma and audible wheezing C.) A patient with right-sided heart failure and pitting edema D.) A patient with chronic bronchitis and congested cough

D Postural drainage is used for patients who have difficulty removing thick secretions from the airway. A patient with chronic bronchitis and a congested, productive cough would benefit from postural drainage because it would help clear the airway.

The nurse is reviewing a research study that includes data in the form of numbers and recognizes that this is likely to be what type of study? A.) Qualitative B.) Experimental C.) Quasi-experimental D.) Quantitative

D Quantitative research usually produces data in the form of numbers. Experimental research explores the causal relationships between variables. Experimental research examines whether one variable has a cause-and-effect relationship with another. Quasi-experimental research examines a causal relationship between variables, but it may not meet the strict guidelines of experimental research. Qualitative research is based on a constructivist philosophy, which assumes that reality is composed of multiple socially constructed realities of each person or group and is therefore value laden, focusing on personal beliefs, thoughts, and feelings.

The nurse is preparing to reposition the patient in bed. What is the first step in this process? A.) Position the patient's arms across his/her chest. B.) Lower the side rails. C.) Grasp the draw sheet. D.) Raise the bed to a working height.

D Raising the bed to a working height is the first step before beginning the procedure. Proper positioning of equipment prevents provider discomfort and reduces the chance of possible injury. Then lower the side rails as appropriate and safe and ensure that the bed wheels are locked. Then you can have the patient position his/her arms and/or grasp the draw sheet.

The nurse is caring for a patient who has urinary frequency. Which nursing diagnosis is the highest priority for this patient? A.) Impaired urination r/t occasional incontinence B.) Anxiety r/t living alone at home with nocturia C.) Risk for infection r/t urine contact with perineal area skin D.) Risk for fall-related injury r/t hurried trips to the bathroom during the day and night

D Risk for falls is the highest priority diagnosis for this patient because rushing to the bathroom can lead to loss of balance and serious injury. Walking to the bathroom at night is even more dangerous because of low lighting conditions and sleepiness. The other nursing diagnosis may be appropriate but not higher than the injury risk.

The nurse is caring for a postoperative patient who is recovering from abdominal surgery. The nurse notes that the patient's breath sounds are clear but diminished, shallow, and slightly labored. The patient's pulse oximetry is 96% on room air. What is the priority action of the nurse? A.) Administer a dose of the prescribed pain medication. B.) Administer 2 L of oxygen via nasal cannula. C.) Obtain an order from the physician for a chest x-ray. D.) Ensure that the patient is using the spirometer 10 times every hour.

D The assessment findings indicate that the patient most likely has atelectasis, so the nurse should ensure that the patient is using the incentive spirometer 10 times every hour to facilitate expansion and reinflation of alveoli. Administering prescribed pain medication is not the priority and may further suppress the patient's respiratory drive. The patient's pulse oximetry is 96% on room air so supplemental oxygen is not needed. A chest x-ray may be ordered if the patient's condition worsens or does not improve with regular spirometer use.

The nurse is performing perineal care for the uncircumcised patient. Which action does the nurse take? A.) Does not move the foreskin. B.) Retracts the foreskin, pulling it away from the body. C.) Leaves the foreskin retracted, allowing it to return to position naturally after care. D.) Retracts the foreskin and returns it to its natural position after cleaning, rinsing, and drying.

D The foreskin must be returned to its normal position after cleaning to prevent contraction and swelling. It is okay to move the foreskin to clean the penis. To retract the foreskin, gently push it toward the body. It should be returned to its position by the nurse, not left to return on its own.

The nurse is caring for a female patient who died a few minutes previously. The patient's family comes in to the room and immediately starts to wash the body in preparation for burial. What is the most appropriate action of the nurse currently? A.) Inform the patient's family that the body must be transported to the morgue. B.) Instruct the patient's family that hospital staff will provide postmortem care. C.) Obtain needed signatures for organ donation and autopsy. D.) Offer to provide any needed supplies and provide privacy for the family

D The most appropriate action of the nurse currently is to allow the family to wash the patient's body in accordance with their wishes and cultural values. The family may wish to participate in this procedure or may complete this procedure in private. Health care personnel should abide by their wishes as much as possible. Signatures may be obtained from the next of kin when washing is complete. The patient's body may be transported to the morgue or funeral home after washing is completed.

The nurse is assessing a patient's alcohol intake. What question is most appropriate? A.) "Do you drink alcohol at all?" B.) "You don't drink much do you?" C.) "When was your last drink?" D.) "How much alcohol do you drink daily?"

D The nurse needs to be nonjudgmental when inquiring about topics that might be sensitive, such as alcohol or drug use. The nurse asks a neutral, objective question such as "How much alcohol do you drink daily?" that allows the patient to quantify the intake. Avoid yes/no questions because they are closed ended and do not lead to further discussion or disclosure. Avoid a negatively charged question such as, "You don't drink much, do you?"; this demonstrates the nurse's displeasure with drinking. Asking when the last drink was is not as important in a general survey as quantifying the amount of intake.

The nurse is caring for a patient who died a few minutes ago. The patient's family is at the bedside and very demonstrative in their grief, weeping loudly and holding on to the patient's body. What is the most appropriate action of the nurse? A.) Inform the family that the patient's body must be taken to the morgue shortly. B.) Ask the family members to step outside while postmortem care is provided. C.) Obtain required signatures for the body to be taken to the funeral home. D.) Provide privacy and allow the patient's family to grieve over the body.

D The nurse should allow the patient's family to grieve in private over the loss of their loved one. Some cultures favor free expression of emotions after death, and the nurse should respect this. Signatures can be obtained, postmortem care can be provided, and the body brought to the morgue after an appropriate time of grieving has been provided to the family.

The nurse is caring for a patient who has not had a bowel movement for 2 days. Which is the priority nursing intervention for this patient? A.) Obtain an order to administer a soap suds cleansing enema. B.) Teach the patient how to use the Valsalva maneuver. C.) Discontinue medications that can cause constipation. D.) Assess the patient's usual pattern of bowel movements.

D The nurse should assess the patient's usual pattern of bowel movements to determine if it is normal for the patient to have a bowel movement every 2 to 3 days. Patients should be taught not to use the Valsalva maneuver because it can lead to bradycardia or death. Medications are not independently discontinued by the nurse and this would require a conversation with the provider.

The nurse is concerned about helping the patient find resources to obtain assistive equipment to be used in the home. Which team member should the nurse contact first? A.) Occupational therapist B.) Physical therapist C.) Health care provider D.) Social worker

D The nurse should collaborate with the social worker to identify community resources for obtaining assistive equipment. The social worker facilitates contact with insurance companies or other agencies to assist with the financing of recommended therapeutic assistive and specialty devices. Occupational therapists evaluate the patient for safe performance of activities of daily living (ADLs) such as bathing, dressing, and grooming, and they make recommendations to enhance safe performance of these activities, such as the use of specialty equipment (e.g., grippers for pants, oversized shoehorns). Physical therapists evaluate the patient's ability to perform and maintain balance during routine activities such as sitting, standing, and walking. They make recommendations for assistive devices such as canes and walkers to promote safe performance of these activities. Health care providers order the equipment.

The charge nurse overhears a new nurse telling a patient that he should no longer follow his vegetarian diet because his protein needs are so high and because "God made animals for us to eat." What action by the charge nurse is best? A.) No action is necessary for the charge nurse to take. B.) Reinforce the nurse's teaching on proper diet. C.) Offer to call the dietitian to work with the patient. D.) Privately speak to the nurse about this conversation.

D The nurse should not share opinions or religious edicts with patients when those beliefs contradict the patient's. The charge nurse should counsel the new nurse about this practice. The patient may hold deep convictions about being a vegetarian and may feel disapproval from the nurse, which will impact the nurse-patient relationship. The other options are not appropriate, although the charge nurse could suggest the new nurse collaborate with the dietitian and patient to determine high-protein foods the patient finds acceptable.

A patient returned from a procedure and has vital sign measurements ordered every hour. The patient's blood pressure has dropped from 132/82 mm Hg an hour ago to 90/66 mm Hg. What priority action by the nurse is most appropriate? A.) Take the vital signs again in another hour. B.) Document the findings in the patient's chart. C.) Have another nurse recheck the vital signs. D.) Plan to take the vital signs more often.

D The nurse uses clinical judgment to determine how often the patient's vital signs should be checked when there is a change in patient condition. The nurse should plan to assess vital signs more often in this patient. Since this is a significant change, the nurse should not wait another hour even though this is what the provider prescribed. It is not necessary for another nurse to double-check the vital signs. Documentation needs to occur, but the priority is to plan to take the vitals more often.

The nurse is caring for a postoperative patient whose urinary catheter was removed 8 hours previously. The patient has not been able to void since the catheter was removed and now reports suprapubic pain. What is the priority action of the nurse? A.) Encourage oral fluid intake and administer a diuretic. B.) Obtain a urine sample to test for culture and sensitivity. C.) Calculate the patient's daily intake and output. D.) Obtain an order to straight-catheterize the patient.

D The patient who has not voided for 6 to 8 hours after urinary catheter removal and is complaining of suprapubic pain has acute urinary retention. The physician should be notified to obtain an order for straight catheterization to drain the bladder. A urine sample for culture and sensitivity is not ordered. Encouraging fluid intake and administering a diuretic will increase the amount of urine in the bladder and make the patient even more uncomfortable.

The nurse is caring for a patient who is constipated and has not had a bowel movement for 3 days. The nurse performs a rectal examination and finds hard dry stool in the rectum. What is the best option to help the patient have a bowel movement? A.) Glass of warmed prune juice B.) Loperamide (Imodium) C.) Oral fiber supplement D.) An oil retention enema

D The patient with hard, dry stool in the rectum will benefit from an oil retention enema because it will soften the stool and make it easier to pass. Imodium is an antidiarrheal that will worsen the constipation. An oral fiber supplement and prune juice should be given after the patient has a bowel movement to prevent constipation from recurring.

he preceptor is watching a nursing student care for a male patient who requires a condom catheter. Which action by the nursing student indicates that the procedure is performed correctly? A.) Sterile gloves are donned before touching the catheter. B.) Adhesive tape is applied securely around the base of the penis. C.) Water-soluble lubricant is applied to the end of the catheter. D.) The foreskin is returned to its natural position before the catheter is applied.

D The patient's penis should be cleaned with soap and water with the foreskin retracted prior to condom catheter application. The foreskin should then be returned to its natural position before the catheter is applied. Adhesive tape should never be applied around the base of the penis because circulation may be compromised. Sterile gloves and lubricant are not needed.

The nurse correctly teaches the patient to rise from a chair using crutches when which intervention is used? A.) Patient starts from the back of the chair. B.) The weak leg is closest to the chair. C.) The hand on the strong side holds the hand bar of the crutch. D.) The strong leg is closest to the chair.

D The patient's strongest leg needs to be closest to the chair. The patient's hand on the weak side holds the hand bar of the crutches, and the hand on the patient's strong side holds onto the armrest of the chair. The patient moves to the front edge of the chair.

When providing end-of-life care, the nurse knows it is essential to carry out which action? A.) Tell the patient what he might like to hear to relieve anxiety. B.) Begin making health care decisions for the patient. C.) Provide the patient with the nurse's personal opinions. D.) Offer unconditional support for the patient and family.

D Two major roles of a nurse caring for a dying patient are: (1) providing accurate information regarding the disease process and treatment options and (2) offering support for the patient and family without interjecting personal opinions. An essential ethical concept is autonomy, which underscores the importance of allowing patients to make their own health care decisions. Limiting information to what will relieve anxiety, providing personal opinions, and making decisions for the patient do not demonstrate respect for patient autonomy.

The nurse identifies which type of wounds heal by tertiary intention? A.) An acute wound in which the patient has sutures placed when it happened. B.) A pressure ulcer that was treated with dressing changes and is healed. C.) An acute wound in which surgical glue was used to close the wound. D.) A wound that was left open initially and closed later with sutures.

D When a delay occurs between injury and closure, the wound healing is said to happen by tertiary intention. Wounds such as surgical incisions or traumatic wounds in which the edges of the wound can be approximated (brought together) to heal are examples of acute wounds. This type of wound is said to heal by primary intention. When a wound heals by secondary intention, new tissue must fill in from the bottom and sides of the wound until the wound bed is filled with new tissue such as a pressure ulcer.

When administering phenytoin (Dilantin) through the patient's IV line, the nurse carefully flushes the IV with normal saline before and afterward to avoid crystal formation of the medication that occurs when it mixes with dextrose in water (D5W) solution. Which type of drug interaction is the nurse being careful to avoid? A.) Antagonism B.) Potentiation C.) Synergism D.) Incompatibility

D When medications combine to form crystals or adverse chemical reactions, the result is a drug incompatibility. Compatibility must be assessed prior to medication preparation and administration.

EHR

Electronic Health Record

Identify the different nursing roles (e.g., advocate) that are identified with the following situations: Nurses provide direction and purpose to others, build a sense of commitment toward common goals, communicate effectively, and assist with addressing challenges.

Leader

A mutually supporting and sensible combination of combined thoughts, whereby the conclusion follows from present facts

Logic

A listing of behaviors or observable items that indicate attainment of a goal

Outcome identification

If blood pressure has to be taken on legs, where is it taken?

Popliteal artery.

What is the Cross-Linking theory?

Proposes that the irreversible aging of proteins such as collagen is responsible for the ultimate failure of tissues and organs.

Identify the tasks that are included in activities of the daily living (ADLs)

Tasks that are undertaken on a regular basis; such as eating, dressings, bathing, toileting, and ambulation.

Who is the primary source of data?

The patient

Sender

The person who initiates and encodes the communication

Reciever

The person who receives and decodes, or interprets the communication

How would you describe the location of the food on this plate to a visually impaired patient?

The plate is described as follows: potato at 12 o'clock, bread at 10 o'clock, butter at 9 o'clock, meet between six and 7 o'clock and vegetables between three and 4 o'clock.

Proxemics

The study of the spatial requirements of human and animals

What are the 4 classic signs of RA?

Ulnar drift ("zig-zag deformity") Boutonnière deformity Hallux valgus Swan neck deformity

The nurse wants to use holistic health strategies with the patient. What types of interventions will be investigated by the nurse?

Use of natural remedies, art, and guided imagery, therapeutic touch, music therapy, relaxation techniques, reminiscence and referral for acupuncture, yoga, and tai chi.

What should happen in goal setting for an alert, involved patient when there is no life-threatening concern present?

When there is no life-threatening concern for the present, goal setting for a patient-centered plan of care must take patient preferences into consideration.

Agranulocytosis

a severe reduction in the number of granulocytes WBC less that 200

Anaphylaxis

a severe response to an allergen in which the symptoms develop quickly, and without help, the patient can die within a few minutes.

Ideally the health care team should encourage primary prevention measures to target which step of carcinogenesis? a. Initiation b. Promotion c. Progression d. Metastatis

a. Initiation

Which factors are used to determine a cancer patient's absorbed radiation dose? Select all. a. Intensity of radiation exposure b. Proximity of radiation source to body c. Type of radiation particle d. Age of the patient during radiation therapy e. Overall health at time of radiation therapy f. Duration of radiation exposure

a. Intensity of radiation exposure b. Proximity of radiation source to body f. Duration of radiation exposure

Because chemotherapy drug dosage is based on total body surface area, the nurse should perform what assessment? a. Measure the patient's height and weight b. Compare the patient's weight to a nomogram c. Calculate the BMI d. Measure abdominal girth

a. Measure the patient's height and weight

Which patient has benign tumor cells that are the result of a small problem with cellular regulation? a. Patient is diagnosed with uterine fibroids b. Patient is advised that she has melanoma c. Patient is advised that he has a G1 tumor d. Patient is diagnosed with hairy cell leukemia

a. Patient is diagnosed with uterine fibroids

The nurse is caring for several patients who are receiving chemo. Which patient is the most likely to need transfer to the ICU? a. Patient receiving interleukin therapy for renal cell carcinoma develops widespread edema. b. Patient receiving estrogen therapy develops calf pain with redness and swelling c. Patient receiving vascular endothelial growth factor/receptor inhibitor has high BP d. Patient receiving an antiandrogen receptor develops gynecomastia

a. Patient receiving interleukin therapy for renal cell carcinoma develops widespread edema.

Why would the nurse encourage a patient to get a vaccine such as Gardasil or Cervarix? a. Protects against human papillomaviruses which are associated with genital cancers. b. Protects against associated with Epstein-Barr virus, which may contribute to Burkitt's lymphoma. c. Protects against hepatitis B virus, which may contribute to primary liver cancer. d. Protects against human lymphotropic virus, which may contribute to T-cell leukemia.

a. Protects against human papillomaviruses which are associated with genital cancers.

The patient has breast cancer with bone metastasis. Based on this information, which laboratory result would the nurse carefully monitor? a. Serum calcium level b. Serum blood glucose c. Serum potassium level d. Serum sodium level

a. Serum calcium level

What is the most typical schedule for radiation therapy? a. Small doses of radiation given on a daily basis for a set time period b. Large one-time dose of radiation given after completing chemotherapy c. Small doses of radiation given several days apart to minimize side effects d. Large doses administered monthly for a set period of months

a. Small doses of radiation given on a daily basis for a set time period

The nurse is caring for an older patient who is getting chemotherapy and filgrastim. Which intervention is the nurse most likely to use to facilitate the purpose of the filgrastim? a. Teach patient, family, and all visitors about meticulous hand hygiene b. Administer the filgrastim prior to chemo to prevent nausea c. Teach and assess for bleeding signs such as bruising or bleeding gums d. Assess the patient for fatigue and plan for periods of uninterrupted rest

a. Teach patient, family, and all visitors about meticulous hand hygiene

A patient is receiving radiation treatment by teletherapy. When does exposure to the patient create a risk for harmful radiation? a. The patient is never radioactive b. During the mechanical delivery of gamma rays c. For the first 24 to 48 hours after treatment d. Until the radiation source has decayed by one half-life

a. The patient is never radioactive

Which statement correctly describes metastatic tumors? a. They are caused by cells breaking off from the primary tumor b. They become less malignant over time c. They are usually less harmful than a primary tumor d. They become the tissue of the organ where they spread

a. They are caused by cells breaking off from the primary tumor

Which outcome statement indicates that the goal of cytoreductive surgery for cancer has been met? a. Tumor size has been decreased and chemotherapy is pending b. The noninvasive skin cancer was completely removed during surgery c. Subjective back pain has decreased since the removal of the tumor d. Incisional site of breast reconstruction shows no signs of infection

a. Tumor size has been decreased and chemotherapy is pending

What role does vascular endothelial growth factor (VEGF) have in the metastasis of cancer? a. VEGF triggers capillary growth to ensure blood supply to the tumor b. Use of VEGF helps to stop the growth and spread of the primary tumor c. VEGF is a carcinogen that activates when cancer cells reach the vascular system d. For cancers with a genetic link, VEGF must be present before metastasis occurs

a. VEGF triggers capillary growth to ensure blood supply to the tumor

The nurse reads in the patient's chart that the health care provider is concerned about myelosuppression. Which laboratory results will the nurse closely monitor and report to the provider? a. WBC count b. Serum potassium level c. RBC count d. Platelet count e. Serum sodium level f. Serum calcium level

a. WBC count c. RBC count d. Platelet count

The nurse hears in report that the patient is diagnosed with glioblastoma. Which question is the most important to ask the off-going nurse? a. What is the patient's current mental status? b. Does the patient have leg pain during ambulation? c. Is the patient able to eat a normal diet? d. Does the patient have trouble passing urine?

a. What is the patient's current mental status?

According to the American Cancer Society, what are the recommendations for early detection by screening for breast masses? a. Women aged 45-54 should have an annual mammogram, then every 2 years at age 55 and older b. High-risk women should have biannual mammograms and MRI c. High-risk women should be screened for breast cancer annually starting age 21 d. Women aged 60 years or older should have a mammogram every 10 years

a. Women aged 45-54 should have an annual mammogram, then every 2 years at age 55 and older

The nurse is preparing a brochure to inform patients about secondary prevention of cancer. Which information would be included? a. Yearly mammography for women starting at the age of 45 b. Chemoprevention with vitamin therapy c. Removing colon polyps for cancer prophylaxis d. Using sunscreen and had when outdoors

a. Yearly mammography for women starting at the age of 45

What is the minimum size for a detectable tumor? a. 1 millimeter b. 1 centimeter c. Depends on type of tumor d. Depends on site of tumor

b. 1 centimeter

An older patient is receiving epoetin alfa. Based on the knowledge that this med increases erythrocytes and many other types of blood cells, which abnormal assessment finding suggests the patient is experiencing an adverse effect of the medication? a. Temperature is 100.5*F b. BP is 160/90mmHg c. Patient has hemorrhagic cystitis d. Mucous membranes are dry

b. BP is 160/90mmHg

Which task is the nurse most likely to perform when caring for a patient with cancer? a. Informs a 36 yo woman about the initial diagnosis of breast cancer b. Explains recommendations for yearly mammograms to a 50 yo woman c. Suggests treatments based on staging of breast tumor to a 65 yo woman d. Advises a 23 yo woman to have surgery for breast cancer

b. Explains recommendations for yearly mammograms to a 50 yo woman

The health care provider informs the nurse that it is likely that the patient's cancer has invaded the bone marrow. Based on this information, the nurse will be vigilant for which signs and symptoms? Select all a. Nausea and vomiting b. Fatigue and weakness c. Decreasing WBCs d. Confusion with memory loss e. Bruises or other bleeding signs f. Tachycardia and shortness of breath

b. Fatigue and weakness c. Decreasing WBCs e. Bruises or other bleeding signs f. Tachycardia and shortness of breath

Which cancer patient is the most likely candidate for palliative surgery? a. Needs extensive cosmetic repair after treatment of neck cancer b. Has continuous vomiting because tumor is obstructing intestines c. Has a suspicious skin lesion that requires further investigation d. Has been treated for cancer and is currently asymptomatic

b. Has continuous vomiting because tumor is obstructing intestines

A young woman is suspected of having invasive breast cancer. Based on the types and frequencies, what is the most likely diagnosis? a, Fibrocystic breast condition b. Infiltrating ductal carcinoma c. Lobular carcinoma in situ d. Ductal carcinoma in situ

b. Infiltrating ductal carcinoma

What instructions will the nurse give to UAPs regarding the hygienic care of a patient with neutropenia? a. Do not enter the room unless absolutely necessary and then minimize time spent in the room b. Mouth care and washing of the axillary and perianal regions must be done during the shift c. If the patient seems very tired, assist with toileting but defer all other aspects of hygienic care d. Assist the patient to perform hygienic care according to the standard routine for all patients

b. Mouth care and washing of the axillary and perianal regions must be done during the shift

Which patient has a condition that is a significant contraindication for photodynamic therapy? a. Patient has a history of frequent sunburn and is at risk for skin cancer b. Patient has a known tumor involvement of a major blood vessel c. Patient needs treatments that would involve the upper airways d. Patient had surgery for breast cancer several years ago

b. Patient has a known tumor involvement of a major blood vessel

What is the most common side effect of radiation? a. Altered taste sensation b. Radiodermatitis c. Nausea d. Fatigue

b. Radiodermatitis

Benign cells have which characteristics? Select all a. Contain few pairs of chromosomes b. Resemble the parent tissue c. Growth is orderly with normal growth patterns d. Perform their differentiated function e. Invade other tissues f. Continue to make fibronectin

b. Resemble the parent tissue c. Growth is orderly with normal growth patterns d. Perform their differentiated function f. Continue to make fibronectin

The nurse is aware that the most common way for cancer to spread is bloodborne metastasis. In caring for a patient with cancer, what type of precautions would the nurse use? a. Contact isolation precautions b. Standard precautions c. Neutropenic precautions d. Droplet precautions

b. Standard precautions

Which patient circumstance would prompt the nurse to create a three-generation pedigree to more fully explore the possibility of genetic risk? a. Smoked for 20 years but quit 5 years ago b. Strong family history of colorectal cancer c. Male relatives with prostate problems d. Personal history of excessive sun exposure

b. Strong family history of colorectal cancer

Why do cancer cells spread throughout the body? Select all. a. They enrich nutrients at the original site b. They have loose adherence c. They readily slip through blood vessel walls and tissue d. They do not respond to contact inhibition e. They are fragile and easily break apart f. They readily respond to signals for apoptosis

b. They have loose adherence c. They readily slip through blood vessel walls and tissue d. They do not respond to contact inhibition

What laboratory result is the most important in relation to the nadir for a chemotherapeutic agent? a. RBC count b. WBC count c. Platelet count d. Serum calcium level

b. WBC count

Which patient circumstance represents the normal physiologic progress of mitosis and cellular regulation? a. A 25 yo woman is diagnosed with endometriosis b. A 45 yo woman notices several skin tags on her neck c. A 35 yo male has ulcer disease that is slowly resolving d. A 65 yo man has a benign tumor that seems to be enlarging

c. A 35 yo male has ulcer disease that is slowly resolving

A patient with colon cancer asks, "Why does everyone keep insisting that I eat so much? I'm not hungry and I've been overweight my whole life." What response is the most appropriate? a. What would you like to eat? I can get you something that you will enjoy b. The cancer may spread to your stomach so you should eat while you still can c. Cancer in the intestinal tract may increase metabolic rate and needs for nutrients d. Well, you don't have to eat if you don't want to, but eating will help your body to heal

c. Cancer in the intestinal tract may increase metabolic rate and needs for nutrients

Which information can be obtained from grading a tumor? a. Genetic linkage to the cancer b. Location and origin of metastasis c. Evaluating prognosis and appropriate therapy d. How long the cancer has been present

c. Evaluating prognosis and appropriate therapy

African Americans have the highest rate of cancer and the highest death rate from cancer. Which intervention targets the most likely explanation for this disparity? a. Increase local efforts to dispense cancer information to this vulnerable group b. Develop educational materials that are culturally sensitive toward African Americans c. Provide referral information to health care facilities that are affordable and accessible. d. Continue research that further clarifies the genetic or racial risk for cancer.

c. Provide referral information to health care facilities that are affordable and accessible.

A patient is being discharged with a prescription for tamoxifen to decrease the chance of breast cancer recurrence. Because of the common side effect, what does the nurse suggest to the patient? a. Have soda crackers and ginger ale on hand b. Install a hand rail around the bathtub c. Purchase a scale to monitor body weight d. Buy a soft-bristle toothbrush

c. Purchase a scale to monitor body weight

Which biologic process demonstrates the differentiated function of red blood cells (RBCs)? a. RBCs float freely through the circulatory system. b. RBCs die according to programmed cell death. c. RBCs make hemoglobin, which carries oxygen. d. RBCs are formed with 23 pairs of chromosomes.

c. RBCs make hemoglobin, which carries oxygen.

A patient is on a newer protocol, dose-dense chemotherapy. Which factor is most likely to contribute to patient noncompliance if the nurse fails to educate the patient and the family? a. Treatment is expensive and less likely to be covered by insurance b. Length of therapy is prolonged and progress is slow to manifest c. Side effects are likely to be more intense and unpleasant d. Medication administration is painful and pain does not respond to medications

c. Side effects are likely to be more intense and unpleasant

The nurse in the radiation department is caring for a patient who will receive stereotactic body radiotherapy. Which intervention is the nurse most likely to use in the care of this patient? a. Remind the patient that no pregnant visitors should come for several days b. Dispose of radioactive urine and stool so that self and others are not exposed c. Teach the patient about the need for exact positioning during the treatment d. Assess the patient for history of allergies to iodine or contrast media

c. Teach the patient about the need for exact positioning during the treatment

The nurse is talking to a young woman who "is using a tanning salon, because it is a safer way to get a tan". What is the best response? a. Even if you use a tanning salon, you should still use a sunscreen. b. Tanning salons are safer because exposure to radiation is very controlled. c. Ultraviolet radiation from sun exposure or tanning salons can cause skin cancer. d. Ionizing radiation is dangerous, but tanning salons use ultraviolet radiation.

c. Ultraviolet radiation from sun exposure or tanning salons can cause skin cancer.

The nurse is supervising a nursing student who is giving care to a patient with a sealed implant. The nurse would intervene if the student performed which action? a. Places a "caution: radioactive material" sign on the door b. Wears a dosimeter film badge at all times while caring for the patient c. Wears a lead apron while providing care and turns away from the patient d. Saves all dressings and bed linens in the patient's room

c. Wears a lead apron while providing care and turns away from the patient

What is the major side effect that limits the dose of chemo? a. nausea and vomiting b. peripheral neuropathy c. bone marrow suppression d. "chemo brain"

c. bone marrow suppression

Which cells would normally not produce fibronectin? a. normal nerve cells b. normal cardiac muscle cells c. normal red blood cells d. cells that are undergoing normal mitosis

c. normal red blood cells

role of RBC

carries O2 and nutrients to capillary refill and difuses into body tissues. filtered through the kidneys

Blood is classified as a tissue why?

cells are similar cells are easly associated preform similar function

aplastic anemia

characterized by an absence of all formed blood elements caused by the failure of blood cell production in the bone marrow

medical management for leukemia

chemo and radiation bone marrow transplant meds : luekeran, hydroxyurea, corticosteroids, cytoxan

systemic lupus erythematosus (SLE)

chronic autoimmune inflammatory disease of collagen in skin, joints, and internal organs Type 3 hypersensitivity

rheumatoid arthritis (RA)

chronic systemic disease characterized by autoimmune inflammatory changes in the connective tissue throughout the body

Thrombocyopenia

condition when the platelets is reduced below 100,000

aplastic anemia primary?

congential

From a primary prevention perspective, what is the most important information that the nurse should emphasize when teaching patients about tobacco and cancer risk? a. Risk for cancer increases when tobacco and alcohol are both used. b. Tobacco use is linked to many different types of cancer. c. Risk for cancer depends on the amount of tobacco used. d. Tobacco is the single most preventable source of carcinogenesis.

d. Tobacco is the single most preventable source of carcinogenesis.

4 sympotms realted to pernicious anemia

extreme weakness parasthia hypoxia redden smooth tounge

Two cateogories of WBC

granulocytes and agranulocytes

what is hemopoiesis ?

production of blood cells

medical management for hodgkins disease

stage 1-2 radiation stage 3-4 chemo or chemo/ rad

cell-mediated immunity

type of immunity produced by T cells that attack infected or abnormal body cells

The nurse knows which factors contribute to the development of wounds and lead to delays in wound healing? (Select all that apply.) A.) A patient who has diabetes. B.) A patient with COPD. C.) A patient with on bed rest who is repositioned. D.) A patient who is obese and sweats excessively. E.) A patient on long-term steroid therapy.

A, B, C, D, and E. Factors that contribute to the development of wounds and lead to delays in wound healing include comorbidities such as vascular disease, which impacts the skin's ability to obtain required oxygen and nutrients, or diabetes, which affects not only the microvasculature, but also the skin's normally acidic pH; malnutrition involving inadequate proteins, cholesterol and fatty acids, and vitamins and minerals; medications such as steroids, nonsteroidal, anti-inflammatories, and anticoagulants; excessive moisture from sweating; and external forces such as pressure, shear, and friction that occur when turning and repositioning the patient in bed.

The nurse assessing a patient using the SPIRIT framework would ask which questions? (Select all that apply.) A.) "Do you follow a particular religion?" B.) "How involved in your church are you?" C.) "Are there any practices I can help you with?" D.) "How will your religion affect your care?" E.) "What gives you hope in bad situations?"

A, B, C, and D. SPIRIT stands for Spiritual belief system, personal spirituality, integration and involvement in a spiritual community, ritualized practices and restrictions, implications for medical care, and terminal events planning. Hope is a good thing to assess but is more related to the HOPE framework.

The nurse is using the Braden scale to assess the patient's risk for a pressure ulcer. Which risk categories are associated with the Braden scale? (Select all that apply.) A.) Activity B.) Friction and shear C.) Moisture D.) Sensory perception E.) Cognition

A, B, C, and D. The Braden scale ranks the patient on the risk categories of sensory perception, moisture, activity, mobility, nutrition, and friction and shear. The scale does not include cognition.

The nurse has assessed a patient and determined that the patient has a sexual issue that needs to be addressed. What actions by the nurse are most appropriate? (Select all that apply.) A.) Use information from multiple sources to help plan care. B.) Collaborate with other health professionals to develop the plan. C.) Involve the patient and significant other in the process. D.) Use standard care plans to limit patient embarrassment. E.) Examine one's own biases before implementing the plan.

A, B, C, and E. A good care plan uses information from multiple sources, involves other disciplines as appropriate, and involves the patient and significant other. In matters related to sexuality, the nurse also must examine his/her own biases so they do not limit the ability of the nurse to work with the patient. Using standard care plans does not address the patient's unique needs.

A nurse has finished examining a patient. What actions does the nurse take next? (Select all that apply.) A.) Document all findings. B.) Provide privacy for dressing. C.) Provide any hygiene material needed. D.) Tells the patient he/she can leave. D.) Cleans the room after the patient leaves.

A, B, C, and E. After finishing the exam, the nurse provides the patient with privacy for changing back into street clothes and any needed hygiene material. The nurse also documents the findings and cleans the room before the next patient is seen. The nurse does not simply tell the patient he/she may leave. The nurse should indicate what will happen next before the patient leaves (i.e., providing written material summarizing the visit, scheduling the next appointment).

The nurse is caring for a patient who has just died. Which assessment findings by the physician and nurse are used to confirm that death has occurred? (Select all that apply.) A.) The patient was incontinent of bowel and bladder. B.) The patient's pupils are fixed and dilated. C.) The provider does not hear a heartbeat. D.) The patient's extremities are cool and mottled. E.) The patient has no palpable peripheral pulses. F.) The patient's face is relaxed and the mouth is open

A, B, C, and E. Assessment findings that confirm death has occurred include lack of pulse/heartbeat and fixed dilated pupils. Cool, mottled extremities, relaxed muscles, and incontinence of bowel and/or stool are common assessment findings in patients who are dying.

The nurse manager of a busy oncology unit is concerned about compassion fatigue among the nursing staff. Which signs and symptoms would alert the nurse to this problem? (Select all that apply.) A.) Nurses become very emotionally upset without an apparent cause. B.) Nurses start to avoid caring for certain patients. C.) Nurses start to call in sick more often. D.) Nurses begin working more overtime. E.) Nurses have difficulty showing empathy for patients.

A, B, C, and E. Compassion fatigue occurs when deeply caring and empathetic nurses become overwhelmed by the constant needs of patients and families. Symptoms include mood swings, avoidance of working with some patients, frequent sick days, irritability, reduced memory, poor concentration, and a decreased ability to show empathy.

A nurse conducting the general survey of a patient includes which items? (Select all that apply.) A.) Hygiene and grooming B.) Affect and mood C.) Sex and gender orientation D.)Sexual preferences and practices D.) Age

A, B, C, and E. Components of the general survey include age, race, hygiene and grooming, affect and mood, clothing, sex and gender orientation, age, and safety. Sexual preferences and practices are not included.

The nurse is planning dietary education for the patient. What food labeling considerations should the nurse be aware of when planning that education? (Select all that apply.) A.) Ask patient if food labels are read routinely. B.) Assess patient's level of understanding of food labels. C.) Encourage patient to read the food labels. D.) Explain to patient all food labels are different. E.) Assess patient's understanding of recommended daily allowance

A, B, C, and E. Evidence indicates a consistent link between eating healthier foods and reading nutrition labels. Patients should be asked if they read food labels when shopping for groceries or food products. Evaluate their understanding of the main elements of a nutrient label (i.e., calories, fats, carbohydrates, sugar, and serving size). Assess patient understanding of the percentages of recommended daily allowances of fats, proteins, and carbohydrates listed on food labels. Uniform nutrition labeling for packaged food was introduced in the United States in 1994, as part of the Nutrition Labeling and Education Act (NLEA), to increase consumer awareness about the nutritional content of food and improve dietary practices; therefore, all labels are the same.

The nurse is assessing factors that affect sexual function in patients with chronic diseases. What topics does the nurse include in the assessment? (Select all that apply.) A.) Fatigue B.) Medications C.) Pain D.) Occupation E.) Physical impairment

A, B, C, and E. Fatigue, medications, pain, and impairments all can have direct effects on sexuality. Lifestyle is another factor, but occupation does not in itself influence sexuality.

The nurse understands that which are important in the process of developing a cultural identity? (Select all that apply.) A.) School B.) Church/religious institution C.) Family D.) History E.) Community

A, B, C, and E. Many institutions and groups, both formal and informal, assist an individual in developing a cultural identity, including school, religious institutions, family, and community.

The nurse will include which interventions to help improve sleep quality during hospitalization on all patients' care plans? (Select all that apply.) A.) Maintaining sleep routines B.) Minimizing disruptions C.) Providing light snacks D.) Using sleep medications E.) Using relaxation measures

A, B, C, and E. Medications would be used carefully and do not always improve sleep. Addressing the sleep environment, maintaining sleep routines, providing light snacks if allowed, and instituting relaxation measures will all improve sleep.

A nurse is planning an educational event on safer sex. What topics does the nurse include? (Select all that apply.) A.) Proper use of condoms B.) Avoidance of risky behaviors C.) Need for routine examinations D.) Avoidance of homosexual activity E.) Symptoms of common STDs

A, B, C, and E. Safe sex education includes proper use of condoms, avoidance of risky behaviors (and what those are), the need for routine examinations, and symptoms of common STDs. The nurse should not include judgmental comments about sexual practices; all people need information on safer sex practices.

The nurse is using Giger and Davidhizar's Transcultural Assessment Model to gain information about a patient from an unfamiliar culture. What questions does the nurse ask that are relevant to this mode? (Select all that apply.) A.) "Who would you like present to help answer questions?" B.) "What do you believe caused your current illness?" C.) "How important is planning for the future to you?" D.) "Why don't you want to shake my hand?" E.) "What activities would you do to control your health?"

A, B, C, and E. The Giger and Davidhizar Transcultural Assessment Model looks at communication, space, social orientation, time, environmental control, and biological variation. The questions all address these factors; however, asking why the patient does not want to shake the nurse's hand sounds judgmental and "why" questions are a communication barrier.

When does the nurse assess patients' spirituality? (Select all that apply.) A.) Upon admission B.) New diagnosis C.) Life-changing diagnosis D.) When the chaplain makes rounds E.) When facing treatment decisions

A, B, C, and E. There are many times at which a spiritual assessment is necessary. All patients should have their spirituality assessed upon admission at a minimum. Other assessments should be conducted at times when the patient is at risk for spiritual distress. Assessment should be done based on patient need, not when the chaplain is available.

The nursing student learns that the purpose of measuring a patient's vital signs includes which of the following rationale? (Select all that apply.) A.) Monitor body systems functioning. B.) Identify early signs of problems. C.) Evaluate effectiveness of interventions. D.) Determine if a cure has been obtained. E.) Provide a baseline to compare against.

A, B, C, and E. Vital signs give information on the functioning of body systems, can lead the nurse to identify early signs of problems, can be used to evaluate the effectiveness of interventions, and provide a baseline to compare against subsequent readings. They are not used to solely determine if a disease has been cured.

The nurse is obtaining preoperative information for a patient who will be having emergency surgery shortly for a ruptured appendix. Which information is crucial for the nurse to assess? (Select all that apply.) A.) All medications that the patient is taking B.) Use of tobacco, alcohol, or recreational drugs C.) Allergies to medications, foods, or other substances D.) Date of last tetanus shot and flu vaccination Insurance coverage and preauthorization requirements E.) Possibility of pregnancy

A, B, C, and F. Priority assessment must be completed prior to emergency surgery, including use of medications, alcohol, tobacco, or recreational drugs because these may interact with anesthesia medications. Allergies must be identified to prevent reactions in the operating room. Special precautions may be taken if the patient is pregnant, so this must also be determined preoperatively. Asking the patient about vaccinations or insurance coverage is not a priority prior to surgery.

A nurse is assessing social determinants of health. Which does the nurse include in the assessment? (Select all that apply.) A.) Vaccination compliance B.) Family structure C.) Communication patterns D.) Roles for women E.) Education

A, B, D, and E. Income, education, health literacy, where people live or work, early childhood development, social exclusion, family structure, the status and role of women, and vaccination adherence are just some of the social determinants of health recognized worldwide. Communication patterns often are important to assess in culturally diverse individuals, families, and communities, but this is not considered a social determinant of health care.

The nurse is educating the patient about the risk of heart disease from metabolic syndrome and describes a cluster of which symptoms? (Select all that apply.) A.) Elevated blood glucose B.) High waist circumference C.) History of smoking D.) Hypertension E.) Elevation serum cholesterol

A, B, D, and E. Metabolic syndrome is a cluster of medical conditions characterized by insulin resistance and the presence of obesity, abdominal fat, elevated blood glucose, triglycerides, serum cholesterol, and hypertension. Smoking is not part of the syndrome.

Which patient-specific factors does the nurse include when assessing pulse? (Select all that apply.) A.) Age B.) Gender C.) Religion D.) Exercise D.) Medications

A, B, D, and E. The nurse should consider several patient-specific factors when assessing the pulse,age, gender, exercise, presence of fever, medications, fluid volume status, stress, and underlying disease processes. Religion is not an appropriate response.

The nurse is caring for a patient who has pain following abdominal surgery. Which actions are independent nursing interventions that can be used to make the patient more comfortable? (Select all that apply.) A.) Encourage the patient to relax and imagine resting on a tropical beach. Provide headphones so that the patient can listen to favorite music. B.) Increase pain medication dosage if prescribed regimen is ineffective to manage pain. Teach the patient to take pain medication before discomfort becomes severe. C.) Switch the patient from IV to oral pain medication when bowel sounds return. D.) Demonstrate the use of relaxation breathing before painful procedures.

A, B, D, and F. Independent nursing interventions may be carried out without an order from the provider. Changing medication orders must be done by the provider; increasing pain medication dosage and switching the patient to PO pain medications are not independent nursing interventions.

The nurse is educating a patient about including more omega-3 fatty acids in her diet. Which of the following food sources should be included? Select all that apply A.) Salmon B.) Flaxseed C.) Mackerel D.) Steak

A, B, and C

Health care providers are required to supply patients with written information regarding their rights to make medical decisions and implement advance directives, which consist of three documents. The nurse knows which items are considered "advanced directives"? (Select all that apply.) A.) Living will B.) Durable power of attorney C.) Health care proxy D.) Patient's Bill of Rights E.) The Uniform Anatomical Gift Act

A, B, and C Advance directives consist of three documents: (1) living will, (2) durable power of attorney, and (3) health care proxy, commonly referred to as a durable power of attorney for health care. The Patient's Bill of Rights informs consumers of health care about specific privileges of which they should be aware. Patients should expect: (1) excellent care, (2) a safe environment, (3) participation in planning their care, (4) privacy, (5) help with discharge arrangements, and (6) assistance with fulfilling financial responsibilities. The Uniform Anatomical Gift Act was approved to allow people over the age of 18 to donate their bodies or body parts after death for transplantation, deposit in tissue banks, or research.

The nurse is performing a respiratory assessment on a patient. Which assessment findings indicate to the nurse that the patient has a history of long-standing chronic respiratory disease? (Select all that apply.) A.) All the patient's fingernails are noticeably clubbed. B.) The patient needs to sleep on at least four to five pillows at night. C.) The patient's chest has equal antero-posterior and transverse diameters. D.) The patient's lower legs have large areas of brownish spotted discoloration. E.) The patient reports puffiness of both feet when standing for long periods. F.) The patient's forced vital capacity test result is 3.8 L of air.

A, B, and C. Clubbing of fingernails, the need to sleep in an upright position, and a barrel chest are all indicative of long-standing chronic respiratory disease like COPD. Brownish spotted discoloration is indicative of venous insufficiency. Edema can be seen in renal and heart failure. Forced vital capacity of almost 4 L is found in patients with good respiratory function.

The nurse is educating a patient about including more omega-3 fatty acids in the diet and knows which food sources should be included? (Select all that apply.) A.) Salmon B.) Flaxseed C.) Mackerel D.) Steak E.) Crayfish

A, B, and C. Dietary sources of omega-3 include fatty fish, such as salmon, tuna, mackerel, and lake trout, as well as nuts, seeds, and oils; flaxseed oil contains the highest amount of total omega-3 fatty acids. Steak and crayfish do not contain omega-3 fatty acids.

The nurse understands that Florence Nightingale is noted to have provided the initial basis for evidence-based practice (EBP) by doing which action? (Select all that apply.) A.) Basing her work in trial and error as well as observation B.) Using statistical data as a basis for improvements C.) Applying statistical methods such as "pie charting" to display results D.) Focusing on bedside care and ignoring nursing education F.) Publishing the first EBP journal

A, B, and C. Florence Nightingale, in her Notes on Nursing in 1859, outlined basic principles of nursing science. Nightingale's method of nursing included rigorous monitoring of the effectiveness of interventions and treatments. This provided the initial basis for EBP. Her work was based on trial and error, careful observation, discussion with patients, and clinical experience. She used statistical data to improve sanitation, health, nursing education, and health administration. Nightingale applied a statistical approach to the study of public health and mortality data and used a pie chart to display research findings. However, nursing did not publish its first EBP journal, Evidence-Based Nursing, until 1998.

Based on research on aging, the nurse knows that improper nutrition may result in the onset of which specific diseases? (Select all that apply.) A.) Type 2 diabetes B.) Atherosclerosis C.) Osteoporosis D.) Rheumatoid arthritis E.) Chronic asthma

A, B, and C. Improper nutrition may result in the onset of specific diseases of the endocrine, cardiovascular, gastrointestinal, and musculoskeletal systems, such as diabetes type 2, atherosclerosis, diverticulosis, osteoporosis, and some cancers. Rheumatoid arthritis is an inflammatory autoimmune disorder. Asthma is a respiratory disorder not related to poor nutrition.

The nurse is teaching a patient about ways to decrease risk of bone fractures. Which statements by the patient indicate a good understanding of decreasing this risk? (Select all that apply.) A.) "I should do weight-bearing exercises." B.) "I should get adequate intake of calcium and vitamin D." C.) "I should exercise regularly." D.) "I need to do yoga exercises." E.) "I wish I could reduce my risk but I can't do anything."

A, B, and C. Inadequate dietary intake of calcium and vitamin D or impaired calcium metabolism may result in osteoporosis, which increases bone fragility and may lead to fractures. Decreased physical exercise and lack of weight-bearing exercise also contribute to bone fragility, deterioration, and loss of strength. Any type of exercise will help; it does not need to be yoga, but it does need to include weight-bearing exercise.

The nurse is caring for a patient who is undergoing a liver biopsy. Which interventions will be included in the patient's care plan for the diagnosis of risk for infection: r/t invasive diagnostic procedure? (Select all that apply.) A.) Monitor for and report redness, warmth, discharge, or fever promptly to the physician. B.) Carefully maintain the sterile field during the biopsy procedure. C.) Teach patient how to care for the biopsy site when procedure is completed. D.) Provide a supportive, caring presence to minimize patient anxiety. E.) Provide information about the pathophysiology and treatment options for liver cancer. F.) Consider using healing touch and other mind-body-spirit interventions.

A, B, and C. Interventions for the Nursing diagnosis of risk for infection involve monitoring for signs and symptoms of infection, preventing contamination of supplies by maintaining a sterile field during the procedure, and teaching the patient how to care for the site afterward. Providing a caring presence, providing information about liver cancer, and using healing touch may be helpful for the patient but will not minimize the risk of infection.

On completion of assessment, a nursing diagnosis relevant to the educational needs of the patient or caregiver can be determined. The nurse recognizes that diagnoses specifically related to patient education include which responses? (Select all that apply.) A.) Deficient knowledge B.) Readiness for enhanced knowledge C.) Noncompliance D.) Pain E.) Alteration in elimination

A, B, and C. On completion of assessment, a nursing diagnosis relevant to the educational needs of the patient or caregiver can be determined. Diagnoses specifically related to patient education include deficient knowledge, readiness for enhanced knowledge, and noncompliance.

The home health care nurse educates patients on which goals of hospice care? (Select all that apply.) A.) Relieve suffering. B.) Support the patient and family. C.) Provide grief support. D.) Keep patients out of the hospital. E.) Lower medical expenses.

A, B, and C. The goals of hospice care include relief of suffering, supporting the family and patient, and providing grief support after the patient dies. Goals do not include keeping patients out of the hospital or lowering medical costs.

The nurse is caring for a patient who is recovering from bowel resection surgery. Which assessment findings indicate to the nurse that the patient no longer needs to remain NPO and may progress to oral intake of food and fluids? (Select all that apply.) A.) The patient passed flatus while ambulating this morning. B.) The patient's abdomen is soft with active bowel sounds × 4 quadrants. C.) The patient denies nausea or vomiting and states that he feels hungry. D.) The patient's abdominal incision is clean, dry, and intact with staples. E.) The patient ambulated in the hallway with a slow, steady gait. F.) The patient's urinary catheter is patent with clear, yellow urine.

A, B, and C. The patient may indicate readiness for oral intake when passing flatus and relating feelings of hunger. The absence of nausea and vomiting along with active bowel sounds in a soft abdomen also indicate that the patient's GI tract is ready for oral feedings.

The nurse recognizes which statements by the student nurse regarding handwashing indicate a need for further education? (Select all that apply.) A.) Wash hands first, then wrists. B.) Rinse from fingertips to wrists. C.) Dry using a scrubbing motion. D.) Turn off faucet with clean, dry paper towel. E.) Dry the hands in the same order as washing them.

A, B, and C. When washing hands, first wet the wrists and hands; with fingers pointing downward, first wash the wrists and then the hands below the wrists. Then apply soap, lather, and rub using a circular motion for 15 to 20 seconds. When rinsing, rinse from wrist to fingertips, keeping hands with fingers pointing downward. Using clean paper towels, dry thoroughly in the same order (from wrists to fingers) using a patting motion. Turn off the faucet with a clean, dry paper towel.

The nurse is planning care for an elderly patient. The nurse recognizes the patient is at risk for respiratory infections based on which factors? (Select all that apply.) A.) Decreased cough reflex B.) Decreased lung elasticity C.) Increased activity of the cilia D.) Abnormal swallowing reflex E.) Increased sputum production

A, B, and D The elderly are at an increased risk for respiratory infections because of decreased cough reflex, decreased elastic recoil of the lungs, decreased activity of the cilia, and abnormal swallowing reflex. They do not generally have increased sputum production.

The nurse is working with a nursing assistant to care for a patient with a new tracheostomy. Which tasks may the nurse delegate to the assistant? (Select all that apply.) A.) Obtaining masks, gloves, and suction supplies from the utility room B.) Helping to reassure the patient before, during, and after suctioning C.) Changing the Velcro or twill ties used to secure the tracheostomy D.) Transporting sputum specimens to the lab for culture and sensitivity testing E.) Assessing need for suctioning of the oropharynx or tracheostomy F.) Teaching the patient how to remove and clean the inner cannula

A, B, and D. Care of a new tracheostomy may not be delegated to a nursing assistant. Obtaining supplies needed for care, helping to reassure the patient, and bringing specimens to the lab are tasks that may be assigned to the assistant.

The nurse recognizes that cold therapy is contraindicated in which conditions? (Select all that apply.) A.) Edema B.) Shivering C.) Bleeding D.) Circulatory problems E.) Advanced age

A, B, and D. Cold should not be used if any of the following is present: edema (cold application slows reabsorption of the fluid), circulatory pathophysiology (cold application causes vasoconstriction, further reducing circulation to the area), and shivering (this is a comfort concern). Bleeding is contraindicated in heat therapy. Advanced age would require frequent observation due to thin skin.

The nurse is caring for a patient who will be having a colonoscopy the following morning. Which items must be removed from the patient's dinner tray since they are not allowed prior to the test? (Select all that apply.) A.) Cherry-flavored gelatin B.) Cream of chicken soup C.) Glass of apple juice D.) Coffee with cream and sugar E.) Lemon-flavored Italian ice F.) Can of ginger ale

A, B, and D. Patients who will undergo colonoscopy testing should have a clear liquid diet the day before the exam, so cream of chicken soup and coffee creamer should not be consumed. Foods with red food coloring should also be avoided prior to colonoscopy.

The nurse knows that during rapid eye movement (REM) sleep, which activities occur? (Select all that apply.) A.) Memories are stored B.) Increase in cerebral blood flow C.) Slow rhythmic scanning eye movements D.) Release of epinephrine E.) Repair of brain cells

A, B, and D. Rapid eye movement (REM) sleep occurs during deep sleep and is manifested by quick scanning movements of the eyes that are associated with dreaming. REM sleep is associated with memory storage, learning, increased cerebral blood flow, and epinephrine release. Repair of brain cells occurs during non-REM sleep.

When assigning tasks to other health care providers, the nurse understands that each task must be delegated using which guidelines? (Select all that apply.) A.) The task must be within the scope of the person to whom it is being delegated. B.) The task is one that can be delegated to other health care providers. C.) The task can be delegated whenever assessments are required. D.) The task may be re-delegated by the person to whom it was first delegated. E.) The task may require the nurse to procure resources to complete the task.

A, B, and E. Through quality improvement, the nurse appreciates the value of what each team member can do to improve patient care. When delegating to other health care providers, the nurse understands that the task must be within the scope and abilities of the person to whom it is being delegated. The nurse must know if the task is something that can be delegated. The RN is responsible for assessment of patients even if certain tasks are delegated to others. The person to whom the assignment was delegated cannot delegate that assignment to someone else. Adequate resources must be made available to the delegatee to complete the task.

The nurse is caring for a patient who just died after a lengthy illness. Which portions of postmortem care may be delegated by the nurse to the nursing assistant? (Select all that apply.) A.) Gently washing the body and closing the patient's eyes B.) Offering support and empathy to the patient's family members C.) Documenting the patient's time of death in the medical record D.) Notifying all of the patient's consulting providers of the patient's death E.) Removing the patient's hospital ID band, IV lines, and urinary catheter F.) Gathering the patient's belongings so they may be taken home by the family

A, B, and F. The nurse assistant can gently wash the patient's body, close the patient's eyes, and gather the patient's belongings. Offering support and empathy to the patient's family members would be done by all of the involved members of the nursing staff. Documenting the time of death in the chart and notifying all of the patient's providers is performed by the nurse. The nurse assistant can remove the patient's IV lines and urinary catheter if allowed by policy, but the hospital ID band would be left in place.

The nurse is caring for a patient with receptive aphasia. Which interventions will assist the nurse in communicating with the patient? (Select all that apply.) A.) Use simple phrases. B.) Speak louder than usual. C.) Stand in front of the patient. D.) Use a picture board. E.) Be patient and unrushed.

A, C, D and E. A patient with receptive aphasia cannot understand written or spoken language. Using simple phrases and talking either softly or loudly will not assist that patient. The sensory pathways are intact, but the words do not make sense. A picture board could be used by the nurse when assessing needs. As the patient participates in speech therapy, the ability to understand simple phrases may develop. Standing in front of the patient when talking may give non-verbal clues to the message.

The nurse understands that which factors can increase blood pressure? (Select all that apply.) A.) Head injury B.) Decreased fluid volume C.) Increasing age D.) Recent food intake E.) Pain

A, C, D, and E Head injury, increasing age, recent food intake, pain, and increased (not decreased) fluid volume all can increase blood pressure.

In preparing to teach the patient, the nurse must consider which concepts? (Select all that apply.) A.) Background B.) Race C.) Pain level D.) Emotional status E.) Readiness to learn

A, C, D, and E. Consideration must be given to the patient's background, readiness to learn, and current condition before education can occur. A patient's ability to read, write, and comprehend health care materials enhances health literacy. Race, by itself, is not a factor.

The nurse knows that a deficiency in vitamin C can result in which conditions? (Select all that apply.) A.) Stiff joints B.) Osteopenia C.) Petechiae D.) Loose teeth D.) Bleeding gums

A, C, D, and E. Deficiencies of vitamin C interfere with normal tissue synthesis and may result in gingivitis, which produces swollen and bleeding gums with loosened teeth, and painful, stiff joints. Other problems associated with malabsorption include anemia (a deficiency of red blood cells), excessive bleeding, petechiae (bleeding under the skin), poor wound healing, and neural tube defects. Osteopenia results from poor absorption of calcium.

Which actions by a nurse constitute spiritual care? (Select all that apply.) A.) Baptizing a critically ill child per the parent's request B.) Leaving the room, giving the patient and family privacy for prayer C.) Considering developmental stage when planning care D.) Notifying the hospital chaplain of a patient's request E.) Praying with patients and families when requested

A, C, D, and E. Many activities fall into the realm of spiritual nursing care, including baptizing an infant in an emergency, notifying the chaplain or other religious leader of patient requests for service, and praying with the patient and family. The nurse always considers the patient's developmental level when planning or providing any type of care. The patient and/or family may or may not want privacy for prayer; the nurse should assess the situation and not just leave.

The nurse manager is concerned about nursing staff who are working the night shift. What interventions can the manager suggest to help the nurses overcome shift-related sleep disturbances? (Select all that apply.) A.) Power nap before leaving for the first night shift. B.) Get a minimum of 4 hours of sleep. C.) Wear dark glasses when driving home from work. D.) Seek exposure to bright light when waking. E.) Maintain a regular sleeping schedule when working and on nights off.

A, C, D, and E. Obtain a minimum of 6 hours of sleep. Maintain a regular sleep schedule when working and on nights off. Wear dark glasses that block blue light when driving home after night work. Seek exposure to bright light (sunlight is best) as soon as possible after waking. Before the first night shift, power-nap 30 to 90 minutes before leaving for work.

The nurse is completing a nutrition assessment on a patient. What are some important considerations? (Select all that apply.) A.) The nurse should include the patient's cultural influences in the assessment. B.) The food diary accuracy is better for a 24-hour recall than a 3 to 5 day food journal. C.) The nurse should be nonjudgmental in the nutritional review. D.) A consultation with a registered dietitian may be indicated. E.) A gathering of anthropometric measurements may be necessary.

A, C, D, and E. When collecting data, the nurse should take into consideration the patient's culture and ethnicity. Recognizing these influences on the patient's nutritional intake allows the nurse to make informed decisions. The data analysis may reveal the need to refer the patient to a registered dietitian for further evaluation of nutritional status. The 24-hour recall is dependent on the ability of the patient to remember consumption of foods and their quantities from the previous day. It is vital to remember that the patient's recall may not be factual and the intake may not be that of a typical day. The other means of assessing a patient's usual dietary pattern is to have the patient keep a written journal of food intake for a certain amount of time. The food diary should encompass entries for 3 to 5 days and includes dietary intake for a typical weekend. Anthropometric measurements may be needed for a full assessment.

The nurse notes the following findings when assessing a patient with COPD. Which require prompt nursing intervention? (Select all that apply.) A.) The patient is unable to speak without gasping. B.) The patient's fingernails are noticeably clubbed. C.) The patient's sputum has turned from yellow to greenish-brown. D.) The patient has stridor with wheezes heard in all lung fields. E.) The patient's forced vital capacity has increased from 2.8 to 3.4 L. F.) The patient has become confused and mildly disoriented.

A, C, D, and F. A patient who is unable to speak without gasping is indicative of poor airflow through the airways, which must be addressed promptly. Greenish-brown sputum may indicate pneumonia requiring antibiotic treatment. Stridor and wheezes are indicative of an acute asthma attack. Confusion and disorientation in a patient with COPD may indicate retention of carbon dioxide. Clubbed fingernails are indicative of a chronic respiratory condition. Increased forced vital capacity is a positive sign.

he nurse is working with a new nursing assistant who is providing care to patients with urinary difficulties. Which actions by the nursing assistant indicates that additional teaching is required? (Select all that apply.) A.) The length of the urinary catheter is cleaned up to the patient's perineum. B.) A urine sample is obtained from the drainage bag immediately after catheter insertion. C.) A fresh condom catheter is applied every other day following careful perineal care. D.) Zinc oxide barrier cream is applied liberally to the perineal area for incontinent patients. E.) The catheter drainage bag is disconnected in order to put pants on the patient. F.) Clean technique is used to obtain a urine specimen for culture and sensitivity from the catheter.

A, C, E, and F. The urinary catheter must be cleaned from the urinary meatus down toward the drainage bag rather than up toward the perineum. A fresh condom catheter must be applied daily. The catheter drainage bag should not be disconnected to put pants on the patient. The drainage bag can be threaded through the pants leg before putting pants on the patient. Sterile technique should be used to obtain samples from the catheter.

The nurse recognizes which skills that are needed to be an effective manager? (Select all that apply.) A.) Understand the concepts of budgeting. B.) Run a unit efficiently without regard to cost. C.) Be able to staff the unit effectively. D.) Be adept at information management. E.) Achieve desired outcomes in any way possible.

A, C, and D An effective manager must have business skills and a business sense. Part of quality care is ensuring that the care the patient receives is cost effective. The nurse manager must understand concepts of budgeting, staffing, marketing, and information management. An understanding of human resource management is equally important. The skillful nurse manager understands the way these elements interact and their influence in achieving expected outcomes in an economically responsible manner.

Nurses use new information in their practice. In the process of implementing EBP, the nurse carries out which actions? (Select all that apply.) A.) Develops clinical questions. B.) Creates workshops and in-services. C.) Seeks answers to support the clinical decision. D.) Applies finding to patients. E.) Publishes a bulletin.

A, C, and D Workshops and in-service or focused training may be necessary if the plan involves a comprehensive change in care or it affects the entire health care agency or community. A bulletin can be provided that lists several safe practice concerns with rationales in the form of a safe practice alert. The fifth phase in the EBP process requires implementation of the change by applying the evidence. Nurses use the new information in their practice. In the process of implementing EBP, the nurse develops a clinical question, seeks answers to verify and support a clinical decision, and ultimately applies the findings to patients.

Which of the following are demographic data? Select all that apply. A.) Gender B.) Medications C.) Educational Level D.) Spiritual Preferences E.) Date of last TB test F.) Use of tobacco and alcohol

A, C, and D.

A nurse is planning sexuality education programs. Which topics are important to each age-group? (Select all that apply.) A.) Adolescents: contraception B.) Adolescents: infertility C.) Young adults: conception D.) Middle adulthood: sexual dysfunction E.) Old age: decreased sexuality

A, C, and D. Adolescents need education on contraception and avoidance of unwanted pregnancy, STDs, HIV infection, sexual abuse, sexual orientation, and good decision making. Young adults particularly need information on conception and infertility. Middle adults need education on emotional and physical changes that occur during this age span including the empty nest syndrome and sexual dysfunction. Older adults need education on physical changes brought by age and encouragement that sexuality normally continues throughout life.

The nurse is teaching a patient about the impact of obesity and a high body mass index (BMI). The nurse identifies that as the BMI increases, so does the risk for which conditions? (Select all that apply.) A.) Increase in blood pressure B.) Increase in HDL C.) Increase in total cholesterol D.) Development of atherosclerosis E.) Decrease in triglycerides

A, C, and D. As BMI levels rise, blood pressure and cholesterol levels also rise and the average high-density lipoprotein (HDL), or good, cholesterol levels decrease. Hyperlipidemia (elevation of plasma cholesterol, triglycerides, or both) or low HDL levels contribute to the development of atherosclerosis (the buildup of fat deposits on arterial vessel walls). Obesity contributes to higher triglycerides.

The nurse is providing dietary education to the patient to assist with inclusion of more complex carbohydrates in the diet. The nurse knows which foods would be beneficial to include? (Select all that apply.) A.) Green peas B.) Bananas C.) Beans D.) Potatoes E.) Apples

A, C, and D. Complex carbohydrates provide the body with vitamins and minerals. Food sources include bread; rice; pasta; legumes such as dried beans, peas, and lentils; and starchy vegetables such as corn, pumpkin, green peas, and potatoes. Bananas and apples are fruits, which are simple carbohydrates.

The nursing student learns which facts about religion and spirituality? (Select all that apply.) A.) Spirituality focuses on the meaning of life to people. B.) Religion and spirituality are mutually exclusive. C.) Religion implies an organized way of worship. D.) Religion provides the structure by which to understand spirituality. E.) Spirituality is an individual practice that does not include others.

A, C, and D. Spirituality focuses on the meanings of life, death, and existence. Religion is an organized and structured method of practicing or expressing one's spirituality, so they are interconnected and not mutually exclusive. Religion provides the structure for expressing spirituality. Spirituality can be expressed through relationships with others.

A nurse wants to create a community action plan for health problems related to air pollution from a nearby factory. Which stakeholders does the nurse consult as the priority? (Select all that apply.) A.) Factory owners B.) Stock shareholders C.) Community residents D.) Local health care providers E.) Factory employees

A, C, and D. Stakeholders have a significant interest in a topic. The priority stakeholders the nurse would want to consult for this project include the factory owners, community residents, and health care providers. The stockholders would probably not be consulted. The employees could be a significant stakeholder if the action plan affected employment.

The nurse is providing education to a patient who is being discharged home on antibiotic therapy. Which statement(s) by the patient indicates further education is needed? (Select all that apply.) A.) "I should take antibiotics every time I am sick." B.) "I should take all antibiotics as prescribed." C.) "I should save all unused antibiotics." D.) "I should stop taking antibiotics when I feel better." E.) "If I develop a rash while taking these I will call the provider."

A, C, and D. The overuse of antibiotics and inappropriate use, such as not completing prescriptions and sharing antibiotics, has led to increased resistance. Taking antibiotics as prescribed helps to ensure the infection will be treated correctly. A rash may indicate an allergic reaction and the patient needs to report this to the provider.

The nurse appropriately delegates care of the unit's patients to the properly trained UAP when that UAP is assigned which tasks? (Select all that apply.) A.) UAP assigned to reposition the patient. B.) UAP assigned to complete the MORSE falls risk scale. C.) UAP assigned to provide range-of-motion exercises. D.) UAP assigned to ambulate the patient in the hallway. E.) UAP assigned to time the patient on a TUG test.

A, C, and D. UAPs provide hands-on care for immobilized patients under the direct supervision of registered nurses. Turning and positioning of patients, range-of-motion exercises, transfers, and assistance with ambulation may be delegated to properly trained UAP. UAPs may not assess patients because that is a nursing responsibility. The MORSE falls risk scale is a risk assessment as is the Timed Up and Go (TUG) test.

Which of the following actions are considered a primary prevention? Select all that apply. A.) Use of a seat belt in a car B.) Having a skin test for TB C.) Getting immunized against the mumps D.) Receiving physical therapy for arthritic knees E.) Wearing a bicycle helment F.) Taking insulin daily as ordered

A, C, and E.

Which of the following are core beliefs of individuals' perception in Rosenstock's Health Belief Model? Select all that apply. A.) Perceived susceptibility of their risk of getting a condition B.) Perceived anxiety over the treatment plan C.) Perceived severity of the seriousness of the condition D.) Perceived conflict with the healthcare providers E.) Perceived benefits of the positive consequences of adopting the behavior.

A, C, and E.

The nurse identifies that knee-high SCD (Sequential Compression Device) sleeves are correctly placed on the patient when which conditions are met? (Select all that apply.) A.) Both sleeves are connected to the SCD device. B.) Two fingers fit inside when the SCDs are inflated. C.) There are no kinks in the tubing. D.) The ankle pressure is 55 to 65 mm Hg. E.) The cooling control is on.

A, C, and E. Proper positioning of the SCD sleeve allows proper fit and application, which decreases the risk of constricting the blood flow or diminishing optimal outcomes. Wrap the sleeve around the leg and fasten it with Velcro straps. Verify that two fingers fit between the leg and the sleeve when the sleeve is not inflated. Connect the sleeves to the device, ensure that there are no kinks in the tubing, and turn on the cooling and set it to 35 to 55 mm Hg.

According to the Healthy People 2020 initiative, health information and the associated access issues have become more complicated. There are many considerations when determining whether an individual has proficient health literacy. The nurse acknowledges that the patient should be able to do what actions? (Select all that apply.) A.) Read and identify credible health information. B.) Recognize abnormalities on an x-ray. C.) Navigate complex insurance programs. D.) Evaluate EKG findings. E.) Advocate for appropriate care.

A, C, and E. The patient should be able to exhibit certain competencies such as reading and identifying credible health information, understanding numbers in the context of the patient's health care, making appointments, filling out forms, gathering health records and asking appropriate questions of physicians, advocating for appropriate care, navigating complex insurance programs (Medicare or Medicaid, and other financial assistance programs), and using technology to access information and services. Interpreting EKGs and X-rays is beyond this scope.

The nurse knows that when patients are experiencing stress, which physiologic changes can be seen in their signs and symptoms? (Select all that apply.) A.) Increase in heart rate B.) Flaccid muscles C.) Pupil dilation D.) Decrease in blood pressure E.) Increase in respiratory rate

A, C, and E. The physiologic response to stress, whether physical or psychological, is activation of the autonomic nervous system, resulting in an increase in heart rate, blood pressure, and respirations along with pupil dilation and muscle tension and decreased blood flow to the skin.

The nurse is caring for a male patient who will be performing intermittent self-catheterization at home. Which actions by the patient indicate the need for additional teaching about this procedure? (Select all that apply.) A.) Patency of the balloon is tested prior to insertion of the catheter. B.) The catheter is inserted another 2 inches after urine is seen in the tubing. C.) The catheter is carefully secured to the leg to prevent accidental removal. D.) The foreskin is returned to its natural position after the catheter is removed. E.) Catheterization is performed regularly before the bladder becomes distended. F.)Water-soluble lubricant is generously applied along the length of the catheter.

A, C, and F. Only 5 to 8 inches of the catheter tip are covered with water-soluble lubricant. Patency of the balloon is only checked when indwelling catheters are inserted. Intermittent catheters need not be secured to the patient's leg because they will be removed after the bladder is drained. The other actions are correct.

The nurse is caring for a postoperative patient who had a colostomy placed 2 days ago. The appliance needs to be changed for the first time. Which ostomy care actions can the nurse delegate to the nursing assistant? (Select all that apply.) A.) Gently cleaning the stoma with warm water and a washcloth B.) Assessing the stoma and incision for signs of infection or ischemia C.) Obtaining needed supplies from the clean utility room D.) Teaching the patient how to care for the ostomy after discharge E.) Determining which type of ostomy appliance to use F.) Application of skin protectant to the area surrounding the stoma

A, C, and F. The nursing assistant can gently clean the stoma with warm water and a washcloth, obtain needed supplies, and apply skin protectant. The nurse is responsible for assessment, teaching, and determining which ostomy appliance to use.

The nurse is caring for a patient who needs to collect a 24-hour urine specimen at home. Which steps of specimen collection may be delegated to the assistant? (Select all that apply.) A.) Label the urine container and lab slips with the patient's name and information. B.) Assess the patient's ability to collect the specimen as required. C.) Explain the procedure to the patient. D.) Obtain the urine container from the utility room or laboratory. E.) Transport the specimen to the laboratory once it is collected. Ensure that the correct test is ordered and collected.

A, D and E. The assistant may label the container and lab slips, obtain the urine container from the utility room, and transport the specimen to the lab. These are tasks that do not require nursing judgment. Assessment of the patient is always done by the nurse, as well as explaining the procedure to the patient and ensuring that the correct test is performed.

The nurse identifies which factors that center on the childhood stress related to school experiences? (Select all that apply.) A.) Goal achievement B.) Family dissolution C.) Life changes D.) Test anxiety E.) Competition

A, D, and E. Childhood stress related to the school experience centers on competition, goal achievement, and test anxiety. Family dissolution and life changes are not related to the school experience.

The student nurse learns that during non-rapid eye movement (NREM) sleep, which activities occur? (Select all that apply.) A.) Brain waves slow B.) Slow rhythmic scanning eye movements C.) Dreaming D.) Drop in blood pressure E.) Conservation of energy

A, D, and E. During non-rapid eye movement (NREM) sleep, in which REM does not occur, physiological activity is reduced, brain waves, breathing and heart rate slow, and blood pressure drops. Slow scanning eye movements do not occur in either REM or NREM. Dreaming occurs in REM.

The nurse has a question regarding scope of practice and delegation. Where should the nurse seek clarification? (Select all that apply.) A.) The state's nurse practice act B.) Theory X management C.) Nurse's Code of Ethics D.) The NCSBN website E.) NCSBN journal articles

A, D, and E. Nurses must have knowledge of the nurse practice act in the state where they are licensed. Each state's nurse practice act defines the RN scope of practice and discusses appropriate delegation. A second resource in delegation is the use of the organization's policy and procedure manual. Employers must have job descriptions for each job class that outline the responsibilities and limitations of each position. The National Council of State Boards of Nursing (NCSBN) website and journal articles are other resources for understanding delegation. Nurses are expected to follow personal and professional ethics, as outlined in the American Nurses Association (ANA) Code of Ethics for Nurses to maintain integrity. Theory X—style managers believe that the average person dislikes work and will avoid it if given the opportunity to do so.

The preceptor is working with a new nurse to provide care for a patient with a new tracheostomy. Which actions by the new nurse indicate need for additional teaching about the procedure? (Select all that apply.) A.) The outer cannula is cleaned with the brush and half-strength H2O2. B.) The new tracheostomy holder is secured before the old soiled one is removed. C.) A Yankauer suction catheter is used to remove secretions from the patient's mouth. D.) Sterile gloves are applied before the soiled dressing is removed from the tracheostomy. E.) Half-strength H2O2 is used to remove crusted secretions around the tracheostomy site. F.) Pain medication is administered to the patient prior to suctioning.

A, D, and E. Only the inner cannula of the tracheostomy is removed for cleaning. The outer cannula stays in the trachea to maintain airway patency. Clean gloves are applied before the soiled dressing is removed. Normal sterile saline is used to remove secretions that have built up on the inner cannula and also is used to clean the patient's skin as needed.

The student nurse learns that spirituality consists of practices that lead to connection to which items? (Select all that apply.) A.) Other people B.) Nature C.) Religious institutions D.) Oneself E.) Higher power

A, D, and E. Spiritual practices generally promote three categories of activity: connection with oneself, with others, and with a higher power.

The nurse is caring for a patient who just had knee replacement surgery. Which factors will affect how the patient experiences pain after this surgery? (Select all that apply.) A.) The patient has had rheumatoid arthritis for the last 16 years. B.) The patient is allergic to aspirin and strawberries. C.) The patient owns a business and is self-insured. D.) The patient has been a vegetarian for the last 8 years. E.) The patient had the other knee replaced 2 years ago. F.) The patient was a marathon runner in high school and college.

A, E, and F The patient's history of rheumatoid arthritis, previous knee replacement surgery, and marathon running indicate that the patient has had significant experience dealing with pain, which will affect how he or she experiences pain after this surgery. The other factors will not affect how the patient experiences pain.

Florence Nightingale's theory focuses on which of following? A.) Environmental Adaptation B.) Interpersonal Processes C.) Energy Fields D.) Levels of Systems

A.

You observe the new staff nurse and see that her uniform is wrinkled and soiled, her hair is unkept, and she is wearing a lot of makeup and perfume. A.) How may this nurse's appearance influence communication with her patients? B.) What communication strategy would you use in this situation?

A. Nonverbal communication is powerful. Patients perceive this symbolic expression as the nurse being unprofessional and unprepared to provide safe care. This will interfere with the development of a helping trusting relationship. B.) As the observer, you can approach the nurse to ask how she is doing and whether there is anything that she needs. This may stimulate a conversation about concerns that be be occurring in her personal life. It is not your role to be her counselor, but you can provide support and help her to obtain appropriate assistance. If this is not effective, you may need to discuss your observations with the nurse manager on the unit. You demonstrate caring by going to the new nurse first and offering yourself.

You have just come onto the unit and been given a multiple-patient assignment. Many medications and treatments have been ordered. A.) How can you apply critical thinking to this assignment? B.) What specific actions will you take to maintain patient safety?

A.) - -Obtain as thorough a hand off report as possible. - Review the patient assignment in advance and determine the priorities for the shift on the basis of the nursing diagnosis and goals. - Determine what care will be delegated - Schedule your activities for the shift, as much as possible, such as medication administration, patient instruction, and admissions/discharges. B.) - To maintain patient safety, you will want to know which patients should be seen first and what interventions will be required to prevent injury. Knowledge of the treatments, medications, is essential in order to provide safe and effective care. Seek assistance to perform techniques that you have done before that may require more than one person to complete. Share assessment findings that are outside of the expected range with your colleagues. Involve the patient in your care.

For the following patient assessments, identify possible nursing diagnosis. A.) The patient is not drinking enough fluids and has dry mucous membranes and poor skin turgor. B.) The patient has difficulty emptying the bladder after lower abdominal surgery. C.) With an inner ear problem, the patient has been experiencing some dizziness during ambulation.

A.) -Insufficient fluid volume B.) - Urinary Retention C.) - Potential for injury

What is role of each of the following in improving critical thinking? A.) Concept Maps B.) Simulation C.) Discussion with colleagues D.) Role Playing

A.) Concept maps help to provide a way to organize and visualize data in order to identify relationships and solve problems. B.) Simulation experiences enable the student to apply previously learned content in a safe and realistic environment that allows time for questioning, clarifying, and feedback. This builds confidence that can be applied to actual patient care experiences. C.) Discussion with colleagues allows for the sharing of best practices and creative ideas for patient care. A cognitive process takes place that identifies and corrects knowledge gaps, erroneous assumptions and biases.

Your neighbor asks you what kind of education is necessary to become a registered nurse. He says that the nurse in the clinic does much of what the doctor used to do for him, including writing his prescriptions. A.) What is your response regarding the education needed for registered nursing? B.) What educational preparation does the nurse in the clinic appear to have?

A.) Education to become an RN may be completed with a diploma school associated with a hospital and/or college (not as common today), an associates degree (usually a 2 year community college), or a 4 year institution for a baccalaureate degree. B) The nurse in the clinic appears to be an advanced practice nurse/nurse practitioner. This level of practice requires a minimum of a master's degree or doctoral degree.

You are visiting a friend who has experienced a heart attack. You notice that he is acting differently than usual, being quiet and withdrawn. He tells you, "I have to change my whole life - my diet, medicine, and cardiac rehab." He follows by saying that he is not happy about any of it. A.) Using the stages of illness model, in which stage would you place this individual? What behaviors support this assessment? B.) As a nurse, how would you assist this individual to cope with his illness and the need to adapt his lifestyle?

A.) Most likely stage IV since they are able to recognize that he is in a patient role and requires treatment. He knows he needs to incorporate treatment regimen into his daily life but he's not happy about it. B.) Provide emotional support and education about the disease and treatments. Provide strategies for coping, including relaxation techniques, imagery, exercise, massage and music therapy. Support group meetings.

For the National Patient Safety Goals, indicate an example of how nurses can achieve each goal. A.) Identify patients correctly B.) Prevent Infection C.) Improve the safety of using medications

A.) Patients should be identified at least two ways - name band and asking them to state their names and birthday. B.) Hand washing is the main way to prevent infection, along with careful medical and surgical asepsis. C.) Nurses need to know the patient's medications and how they should be administered and monitored. For example, they must identify the correct route for giving a medication and be alert to potential adverse effects or contraindications for use.

Identify how each of the following statements made by the nurse can become therapeutic communication. A.) "I don't know why you don't take your medication, Mr. Jones." B.) "Try not to forget to use your call button if you need me." C.) "Don't be concerned. I'm sure that the test results will be fine." D.) "Honey, it's time for your bath."

A: "I noticed that you haven't been taking your medication's. Are you having any questions about them that I can answer for you?" B: "Here is the call bell. If you push this button the aid or I will come." C: "It sounds like you're concerned about the test results." D: "Mrs. Jones, I'm here to help you with your bath. Is there a particular way that you would like to have me help you?"

Do not use abbreviations

-U (unit) -IU (international unit) -QD (daily) -QOD (every other day) -Trailing zero (X.0 mg) -Lack of leading zero (.X mg) -MS -MSO4 & MgSO4

neutropenia clinical manifestations

-depressed WBC count or immature WBCs are present -fever -C/o sore throat and dysphagia -ulcerative lesions of pharyngeal and buccal mucosa -diarrhea -rectal tenderness -vaginal itching or discharge -SOB -nonproductive cough -pain

What is Regression?

Response to stressful situations in which the child temporarily loses previously learned skills such as bed wetting.

Hodgkin disease

inflammatory or infectious process that develops in to neoplasm affects males twice a as frequently as females Reed Sternberg cells

Normal range of neutrophils

2200-7700

The nurse on the surgical unit has a multiple-patient assignment. On beginning the shift, the nurse determines that the first patient to see in the morning is the individual who? A.) Has a blood pressure of 90/50 mm Hg. B.) Requires instruction for wound care. C.) Needs to be transferred from the bed to the chair. D.) Received pain medication 5 minutes ago.

A

Risk of disease expression based on hereditary factors

Genetic Vulnerability

The nurse identifies which sequence to be the usual progression of sleep? A.) NREM 1-3 then REM, then back through NREM 1 and 2 B.) REM then NREM 1-4, then back through NREM 2 and 3 C.) NREM 1-3 then back through NREM 3 and 2 then REM D.) REM then NREM 1-4 then back through NREM 3

C The usual sleep sequence for a person is a fairly rapid progression through NREM 1 through 3, back through NREM 3 and 2, and then into REM sleep. There is no NREM stage 4.

What about Preschoolers (Ages 4-6) Theories of Development?

Initiative vs Guilt (Erikson) Preoperations (Piaget) Phallic (Freud)

MAR

Medication Administration Record

What resources are available to nurses to obtain evidence-based practice information?

Recent literature.

Systemic anaphylaxis (anaphylactic shock)

Results when an individual sensitized to an antigen is exposed to it again May result in circulatory collapse and death Treated with epinephrine

hemophilia B

factor IX or chirstmas diease

Von Williebrand diease

inherited bleeding disorder by abnormally slow coagulation of blood

2 types of agranulocytes

lymphocytes monocytes

DIC

overstimulation of clotting and anticlotting process

medical management for sickle cell anemia

oxygen rest fluids analgesic bone marrow transplant

Provide an example of a question that a nurse can ask to determine the patients level of knowledge about the medical diagnosis.

"Have you been told about a low salt diet?" "How long have you had diabetes?" "What do you do to treat your arthritis?"

Provide an example of a question that could be used during a review of systems.

"Have you ever had any difficulty with breathing/chest pain/urination, etc?" "Are you able to walk on your own?" "Do you wear glasses?"

Who is Piaget?

"Stages of Cognitive Development" For our purposes, we are concerned with concrete and formal operational stages Studies - only 33% of 8th graders were consistently capable of formal operational thinking Believed children develop schema - or mental models - connecting new knowledge to existing knowledge How the brain organizes information Piaget believed that conservation develops in the concrete operational stage. Conservation refers to the ability to determine that a certain quantity will remain the same even if the container, shape, or apparent size is changed.

The student using the FICA Spiritual Health Assessment will consider which factors? (Select all that apply.) A.) Faith and belief B.) Focused practices C.) Importance of faith D.) Faith community involvement E.) Address spirituality in care

A, C, D, and E. FICA stands for faith and belief, importance of faith, faith community involvement, and address spirituality in care.

The nurse is using the critical thinking trait of integrity when she does which of the following? A.) Feels certain about being able to perform the skill B.) Provides factual and honest information to the patient C.) Considers all of the information before moving forward with the plan of care. D.) Follows an orderly approach to completing the required interventions.

B

The nurse identifies which medication that has the highest potential for abuse? A.) Methylphenidate (Ritalin)—schedule II B.) Alprazolam (Xanax)—schedule IV C.) Acetaminophen & codeine (Tylenol #3)—schedule III D.) Diphenoxylate & atropine (Lomotil)—schedule V

A According to the Controlled Substances Act, drugs that have the potential for abuse/dependency are classified as schedule I-V. Schedule I drugs have no approved medical applications in the United States. Schedule II drugs have high potential for abuse/dependency and have multiple restrictions for prescriptions. Schedule III, IV, and V have lower risks of dependency/abuse and fewer restrictions for prescriptions. Methylphenidate has the highest risk of abuse in this selection.

The nurse is working with a patient who has the following signs and symptoms: weight gain, edema to the lower extremities, increase blood pressure, and abdominal distention. On the basis of this information, which of the following is the most appropriate nursing diagnosis? A.) Inadequate nutritional intake B.) Increase fluid volume C.) Urinary retention D.) Potential for trauma

B

What are Lawrence Kohlberg's stages of moral development?

3 levels of moral reasoning, divided into sublevels: Preconventional, Conventional, and Post conventional.

The nurse is providing care for a patient who has had a stroke recently and has multiple self-care deficits. The nurse is coordinating care with in-home agencies and arranging for the delivery of needed equipment. Which ethical concept is the nurse applying? A.) Advocacy B.) Confidentiality C.) Autonomy D.) Accountability

A Supporting or promoting the interests of others or doing so for a cause greater than ourselves defines advocacy. Confidentiality is the ethical concept that limits sharing private patient information. Autonomy, or self-determination, is the freedom to make decisions supported by knowledge and self-confidence. Accountability is the willingness to accept responsibility for one's actions.

The nurse is to teach an 84 year old spanish speaking patient newly diagnosed with diabetes how to self-administer insulin. The patient has hearing and visual impairments. In order to be effective as a teacher, the nurse should: Select all that apply A.) Assess reading level and learning style B.) Determine readiness to learn C.) Use family members as interpreters D.) Provide written instructions in English E.) Place the patient in group classes

A and B

A nurse is conducting a physical examination using palpation. Which assessments might the nurse note? (Select all that apply.) A.) Rebound tenderness B.) Crepitation C.) Guarding D.) Turgor E.) Consistency

A, B, C, D, and E. Crepitation is crackling or rubbing felt (and perhaps heard) during palpation. Turgor is the amount of tension in body tissues caused by fluid content. Consistency compares organs for their location and size related to the norms. Rebound tenderness occurs after the stimulation is discontinued but is elicited with palpation. Guarding is positioning to prevent movement of a painful body part. In this scenario, the patient would guard to prevent the nurse from palpating a painful area.

What is the inflammatory response?

A localized response resulting in pain, heat, and swelling. Mast cells are activated in damages tissue and release histamines. These dilate blood vessels, raising the temperature and thus preventing pathogens from reproducing. They also make the blood vessels more leaky, causing swelling. Mast cells release cytokines, too, which attract phagocytes.

What is one major challenges that nurse face in providing health information to patients?

A lot of patients are health illiterate. They have difficulty in completing routine health tasks such as understanding a drug label or vaccination table.

anaphylactic shock

A severe reaction that occurs when an allergen is introduced to the bloodstream of an allergic individual. Characterized by bronchoconstriction, labored breathing, widespread vasodilation, circulatory shock, and sometimes sudden death.

Paradigm

A way the nurse views the world

What are the five rights to delegation?

A. Right task B. Right circumstance C. Right person D. Right direction or communication E. Right supervision

The patent is seen in the clinic for her first prenatal visit. It is determined that, by the first visit, the patient should be able to identify 5 symptoms that indicate a possible problem with the pregnancy. On her return to the clinic, the patient can state 3 symptoms. The evaluation for this patient goal is which of the following? A.) Goal met; patient able to state sufficient symptoms B.) Goal partially met; patient able to state 3 of the 5 symptoms C.) Goal not met; patient is unable to state all 5 symptoms D.) Goal not met; patient identifies three of the symptoms

B

The patient expresses a desire to learn methods to be independent regarding self-care. Based on this, the most appropriate nursing diagnosis would be? A.) Ineffective health maintenance B.) Readiness for enhanced self-care C.) Hygiene self-care deficit D.) Disturbed body image

B

The physical assessment technique to use a determine skin moisture and temperature is? A.) Inspection B.) Palpation C.) Percussion D.) Auscultation

B

Which assessment question should the nurse use for the intellectual standard of clarity? A.) "What are the most important things you need to know about your diet?" B.) "Am I correct that you take two medications at home for your blood pressure?" C.) "Have we talked about all of the issues that you have with wound care?" D.) "Can you be more specific about the location of the discomfort?"

B

Who is Linda Richards?

America's first trained nurse

Consideration of the ethical and legal consequences for failure to identify areas of concern requiring treatment

American Nurses Association's Nursing: Scope and Standards of Practice (2015)

ADPIE

Assessment Diagnosis Planning Implementation Evaluation

The nurse is caring for a patient who is undergoing treatment for cancer. The nurse states, "You look down today." What does the nurse's communication best indicate? A.) The nurse shows respect for the patient. B.) The nurse exhibits concern for the patient. C.) The nurse actively listens to the patient. D.) The nurse gives knowledge to the patient.

B

What is the wear and tear theory?

As time goes by, repeated use and abuse of the body's tissues cause it to be unable to repair all the damage. Organs and organ systems worn down by toxins in the diet and in the environment; by the excessive consumption of fat, sugar, caffeine, alcohol, and nicotine; ultraviolet rays of the sun.

A threat of bodily harm or violence

Assault

Identify at least two of the factors associated with compassion

Attentiveness, listening, confronting, involvement, helping, presence, and understanding.

What happens in 12 to 36 months?

Autonomy vs Shame/Doubt (Erikson 24 months) Preoperations (Piaget) Anal Stage (Freud)

An example of holistic measure can be incorporated into the patients care plan in the use of: A.) Analgesic medication B.) Therapeutic touch C.) Wound debridement D.) Passive ROM

B

Assessment of the patient reveals some negative health behaviors. Which of the following indicates a lifestyle risk factor? A.) Hypertension B.) Sunbathing C.) Overcrowded living conditions D.) Manufacturing-based occupation

B

BCMA

Bar Code Medication Administration

Equipment that has latex (besides gloves)

Blood pressure cuffs Stethoscopes Anesthetic masks Tourniquets IV tubing Syringes/Medicine vials Electrode pads O2 masks Tracheal tubes Colostomy/ileostomy pouches Adhesive tape Urinary Catheters Medicines that contain/are stored in natural rubber should be avoided (vials/syringes)

Type III hypersensitivity reaction

Complement-mediated immune disorders

Self-Efficacy

Confidence in one's ability to take action Change strategies - Provide training and guidance in performing action. Use progressive goal setting. Give verbal reinforcement. Demonstrate desired behaviors.

Identify at least four essential traits of critical thinking

Critical thinking traits include thinking independently, fairness, responsibility, and accountability, risk taking, discipline, perseverance, creativity, curiosity, integrity, and humility.

The nurse identifies what physiological response occurs with the onset of darkness and in preparation for sleep? A.) Cortisol levels peak B.) Cortisol levels increase C.) Core body temperature increases D.) Melatonin levels increase

D Melatonin levels increase and core temperature and cortisol levels decrease with the onset of darkness. Cortisol levels peak at 6 a.m.

What are DIMPLE schoolagers?

D - Doers I - Industry vs Inferiority M - Modesty P - Peers L - Loss of Control E - Explanation of Procedures

What is the purpose of continual patient reassessment?

Detects noticeable changes in the patient's condition, requiring adjustments to interventions outlined in the plan of care.

Who is Lawrence Kohlberg?

Developed theory of moral development

Autoimmunity

Disturbance in the immunologic tolerance of self-antigens

Who is Freud?

Psychoanalytic. When you hear "Freud" think: unconscious, id, ego, superego, defense mechanisms, dream, free association..

What is Veracity?

Duty to tell the truth

What are the four D's of Negligence?

Duty, Derelict, Direct Cause, Damage

Values

Enduring ideas about what an individual considers is good, best and the right thing to do.

Determination of whether the patient's goals are met and examination of the effectiveness of interventions

Evaulation

How often do we get vital signs on a stable patient?

Every 4 to 8 hours.

What does Cephalocaudal mean?

Head to Tail

Conclusions

Inferences

ISBAR

Introduction Situation Background Assessment Recommendation

Negligence committed by a person functioning within a professional role

Malpractice

Provide an example of a value conflict that a nurse may have in caring for a patient.

Many value conflicts can arise between the nurse patient. One may be that the patient values home remedies, while the nurse feels strongly about the available prescription medications. The patient may be having a termination of a pregnancy, and the nurse could have a religious background that bans the practice.

Pts on beta blockers to have anaphylaxis?

May be resistant to treatment with epinephrine •Can develop refractory hypotension and bradycardia Administer glucagon •Has inotropic and chronotropic effects that are not mediated through β-receptors

RA clinical manifestations

Mild to debilitating Symmetrical joints Pain, stiffness Redness, heat, swelling Decreased mobility

Standards of Practice

Minimum set of criteria to deliver quality care

Indicate the standard of care being demonstrated in practice or performance for each of the following: The primary nurse discusses the goals of treatment with the patient, family, and respiratory therapist.

Outcomes identification/Collaboration

During an assessment, along with physical data, the nurse will collect information from the patient about?

Nurse will collect information about the patient's emotional and psychological state, sociocultural background and practices, as well as other personal family, employment and financial concerns.

Explain the difference between assertive and aggressive communication

Overly assertive nurses may be perceived as aggressive if they do not respect the rights and opinions of others. Nurses who communicate aggressively tend to receive negative or defensive responses from patients, family members, and health care team members.

Provide an example of how the nurse and patient may have conflicting priorities for care.

Patient may have financial concerns addressed before learning injection technique.

Tapping the patient's skin with short, sharp strokes that cause a vibration to travel through the skin and to upper layers of the underlying structures.

Percussion

What was identified by The Joint Commission as one of the top five causes of sentinel events from 2005-2017?

Poor communication.

Who is Clara Barton?

Practiced nursing in the Civil War and established the American Red Cross

What would be an outcome identification for the following goal? The pressure ulcer will be free of infection during the healing of treatment.

Pressure ulcer clean, dry with reduction in size by 1 cm and approximation of wound edges within 24 hours.

Is Development Quantitative or Qualitative?

Qualitative

4 components of blood

RBC WBC Platelets Plasma

Iron deficiency anemia

RBC contain decreased levels of hemogloubin excessive iron loss

The nurse enters a patient's room to obtain the informed consent and discovers the patient has no idea about the possible complications of surgery. What should the nurse do?

Request that the physician come and speak to the patient.

Which tumor, node, metastasis (TNM) staging classification would indicate the best prognosis for the patient's survival? a. T(IS)N(0)M(0) b. T(x)N(X)M(x) c. T(2)N(1)M(0) d. T(2)N(3)M(1)

a. T(IS)N(0)M(0)

Healthy people 2020 is designed to track which personal risk behaviors?

Risk factors and personal behaviors related to physical activity, access to health services, tobacco use, substance use, responsible sexual behavior, mental health, immunizations, and injury and violence prevention.

Identify at least 3 health promotion activities that the nurse can discuss with a patient

Routine exercise, meeting nutritional and vitamin requirements, maintaining BMI within acceptable range, good sleep habits and stress reduction.

For the acronym SOLER, identify what actions or skills are used

S - Sit facing the patient O - Open stance when listening L- Lean toward the speaker E - Eye contact R - Relax

Provide an example of how the nurses can use positive self talk for a patient.

The nurses can use positive self-talk to assist a patient to get through painful procedures or treatments or reduce anxiety.

Identify one concept included in the theories from each of the following nurse theorist: Roy

The human being as an adaptive open system.

Message

The information that is transmitted during communication

How can the nurse arrange the environment so that a patient interview can be the most effective?

The interview should be conducted in an area that is free from as many distractions as possible, ensuring that the patient is comfortable and relaxed. The patient should feel safe because the questions raised may cause stress and anxiety. When feasible, the nurse and the patient should be seated at eye level with each other.

How is the nursing process defined?

The nursing process is the foundation of professional nursing practice. It is the framework within which nurses provide care to patients in an organized and effective manner. The nursing process is the scientific method through which professional nurses systematically identify and address actual or potential patient problems.

What are the characteristics of the nursing process?

The nursing process requires you to think critically. It is analytical, dynamic, organized, outcome orientated, and collaborative, and it is universally adaptable to various healthcare settings.

Platelets

Thrombocytes controls blood loss

What circumstances can lead the nurse to refer a patient to a specialized care?

When more specific patient conditions arise such as extreme anxiety, dietary requests, and discharge/home care needs.

Which factors determine the type of therapy for cancer? Select all. a. Type and location of cancer b. Overall health of the patient c. Whether the cancer has metastasized d. Previous lymph node biopsy e. Patient's gender f. Family history and genetics

a. Type and location of cancer b. Overall health of the patient c. Whether the cancer has metastasized f. Family history and genetics

Latex allergies

assess for allergies to bananas, apricots, cherries, grapes, kiwis, passion fruit, avocados, chestnuts, tomatoes and peaches

medical management for hemophilia

avoid trama relieve pain transufsions

why is urine output carefully monitored with a pt who has hypovolemic anemia?

because it correlates with the amount of blood loss

clinical assessment of DIC

bleeding dyspnea diaphoresis clod, mottled digits purpura petechiae

Aplastic anemia

bone marrow is replaced with fatty deposits can produce RBC

clinical assesments of multiple myeloma

bone pain infection anemia bleeding hyperclamcia renal failure

The patient is prescribed a biologic response modifier, leukine. Which outcome statement about the medication therapy reflects the concept of immunity? a. Electrolyte levels are improving and there is no edema b. Erythrocytes are increased and fatigue is resolving c. Platelet count is normalizing and there are no signs of bleeding d. White cell count is improving and there are no signs of infection

d. White cell count is improving and there are no signs of infection

clinical manafestations and assessment for sickle cell anemia

dehydration change in oxygen tension in the body loss of appetite irratbility weakness abdominal enlargemt joint and back pain edema in extremities

clinical assessment for iron definicy anemia

pallor fatigue shortness of breath angina burning tongue headache paresthesia

clinical assessment for lymphangitis

penicillin mosit heat elevate extremity

The ability to focus on facts and ideas directly related and pertinent to the subject at hand

Relevance

The patient is unconscious after a motor vehicle accident. How does the nurse obtain information on the patient?

Rely on friends or family members (secondary sources)

The nurse is assessing a patient who is undergoing outpatient therapy for breast cancer. Which patient report causes the greatest concern because of possible metastasis to a common site? a. "I don't seem to have a very good appetite" b. "My ribs hurt but I haven't had any injuries" c. "My skin is dry and it feels itchy and irritated" d. "I feel like I need to urinate all of the time"

b. "My ribs hurt but I haven't had any injuries"

The nurse hears in report that the patient has cachexia. Which assessment will the nurse plan to perform? a. Ability to ambulate independently b. Appetite and nutritional intake c. Mental status and cognition d. Sensation and pulses in extremities

b. Appetite and nutritional intake

What role do normal hormones and proteins such as insulin and estrogen play in the development of cancer? a. They prolong or delay the growth of cancer cells b. They can promote frequent division of cells c. They act like carcinogens under certain conditions d. They turn off the suppressor genes

b. They can promote frequent division of cells

Which patient report should be investigated as one of the seven warning signs of cancer? a. Soreness and stiffness to joints in the morning b. Abdominal pain related to irregular meals c. Redness to skin with pain after sun exposure d. Sore on nipple present for several months

d. Sore on nipple present for several months

Which features are specific to cancer cells? a. They grow very slowly but eventually harm the body b. They have a small, fragile nucleus that is easily damaged c. They produce fibronectin, which strengthens the cell wall d. They have an unlimited life span and spread easily

d. They have an unlimited life span and spread easily

medical management for iron definicy anemia

iron 900 mg daily oral or injection Ztrack diet high in iron

What is the superego?

its our moral conscience, and is found as part of the conscious and unconscious mind.

Characteristics of collaboration among health care professionals include...:

- Clinical competence and accountability - Common Purpose - Interpersonal competence and effective communication - Trust and mutual respect - Recognition and valuation of diverse complementary knowledge and skills - Humor

Type II hypersensitivity reaction

-Cytotoxic and Cytolytic Reactions -IgM -Ex: Hemolytic transfusion reactions, Goodpasture Syndrome

The nurse is caring for a patient who is to have a noncontrast MRI scan performed. Which assessment finding leads the nruse to report that the patient may not be able to have the test? A.) The patient has an implanted insulin pump B.) The patient is breastfeeding her newborn infant C.) The patient is severely allergic to iodine and latex D.) The patient has profound hearing loss

A

Which one of the following best meets the criteria for a goal statement? A.) Respiratory rate will remain within 20 to 24 breaths per minute through discharge. B.) Patient will ambulate in the hallway frequently C.) Treatment regimen will be understood D.) Patient will describe activity restrictions

A

The nurse is caring for a patient who just underwent laparoscopic appendectomy. The patient complains of severe postoperative pain between the shoulder blades. Which term best describes the pain that this patient is having? A.) Referred pain B.) Phantom pain C.) Neuropathic pain D.) Psychogenic pain

A Referred pain is pain that occurs when discomfort is felt in a different area than the source of the pain. Phantom pain occurs in amputees when pain is felt in the missing limb. Neuropathic pain occurs in the nervous system and often feels like burning or tingling. Psychogenic pain is discomfort felt by the patient that has no physical cause.

The nurse recognizes which leadership theory that assumes that leaders are born with certain leadership skill that few people possess? A.) Trait theory B.) Behavioral theory C.) Situational theory D.) Transformational theory

A Trait theories assume that leaders are born with the personality traits necessary for leadership, which few people are thought to possess. Behavioral theories assume that leaders learn certain behaviors. These theories focus on what leaders do, rather than on what characteristics they innately possess. Situational theories suggest that leaders change their approach depending on the situation. Transformational leaders use methods that inspire people to follow their lead. Transformational leaders work toward transforming an organization with the help of others.

Indicate whether the following are classified as acute or chronic illnesses. A.) Appendicitis B.) Multiple Sclerosis C.) Diabetes Mellitus D.) Arthritis E.) Influenza

A - Acute B - Chronic C - Chronic D - Chronic E - Acute

Identify levels of prevention for the following. A.) Showing a patient how to use a cane B.) Teaching new parents about the use of a car seat C.) Having a colonoscopy D.) Receiving nutrition information

A - Tertiary B - Primary C - Secondary D - Primary

The nurse is caring for a patient who is taking medication that is toxic to the liver. Which laboratory test results will be reviewed by the nurse to ensure that the patient's liver is tolerating the medication without damage to the organ? (Select all that apply.) A.) Alanine aminotransferase (ALT) B.) Alkaline phosphatase (ALP) C.) Blood urea nitrogen (BUN) D.) Anti-nuclear antibody (ANA) E.) Erythrocyte sedimentation rate (ESR) F.) Fibrin degradation products (FDP)

A and B. Alanine aminotransferase (ALT) and alkaline phosphatase (ALP) are indicators of liver function, and increased levels indicate liver damage from a variety of causes. BUN, ANA, ESR, and FDP are not indicators of liver function.

Which of the following are included in the orientation/introductory phase of the patient interview? Select all that apply. A.) Establishing the name by which the patient prefers to be called. B.) Providing personal information to the patient C.) Using all open-ended questions D.) Being seated at eye level with the patient E.) Asking all of the family members to remain

A and D.

The nurse knows that standard precautions are indicated for which group(s) of patients? (Select all that apply.) A.) All patients B.) Patients with HIV C.) Patients with MRSA D.) Patients with tuberculosis E.) Patients who are bleeding

A and E. The nurse can take steps at any link in the chain to halt the spread of infection. Standard precautions are used with all patients to limit direct exposure to blood and body fluids. The other choices are additional precautions such as airborne precautions are used with patients who have diseases such as tuberculosis and contact precautions with patients who have MRSA.

Who is protected under ADA?

A disabled individual is defined for purposes of ADA as person who meets one of the following criteria: Has a physical/mental impairment that substantially interferes with one or more major life activities Has a record of such impairment Is regarded as having such an impairment

For chronic illnesses: A.) How do individuals with chronic illnesses need to adapt their lives? B.) When general strategies should the nurse employ when helping patients with chronic illnesses?

A) They need to adapt by incorporating medication therapy, health care provider visits, lab testing, dietary intake, activity restrictions, and other ongoing treatments (colostomy care, diabetic foot care) B) The nurse can use strategies to help patients cope with feelings of anger, frustration, and depression and provide measures to promote comfort and symptom management.

The nurse recognizes which personality factors that have been shown to buffer the impact of stress? (Select all that apply.) A.) Resilience B.) Sense of coherence C.) Gender D.) Hardiness E.) Coping style

A, B and D. Personality factors such as resilience, hardiness, and sense of coherence can buffer the impact of stress, reducing the negative consequences. Gender is not a personality factor. Coping style refers to a pattern of measures taken to relieve stress but is not a personality factor.

The nurse conducting a sleep workshop in the community would identify which patients to be at risk for obstructive sleep apnea (OSA)? (Select all that apply.) A.) Deviated septum B.) Recessed chin C.) Alcohol use D.) Large neck E.) Recent tonsillectomy

A, B, C, and D. Risk factors for OSA include obesity, large neck circumference, smoking, alcohol use, and a family history of OSA. Structural abnormalities such as a recessed chin, abnormal upper-airway structures, deviated septum, nasal polyps, or enlarged tonsils can predispose a person to OSA.

The nurse is providing education to a community group on environmental safety. Which safety measures are effective in improving their environmental safety? (Select all that apply.) A.) Use of night-lights throughout the home B.) Illumination of stairwells and pathways C.) Installation of motion-activated lighting on the exterior of the home D.) Application of wax to all floors to increase shine E.) Staying indoors when air pollution is high

A, B, C, and E. Inadequate lighting presents safety concerns in home, work, community, and health care environments. For an individual to safely and successfully navigate pathways and perform various activities while avoiding potential obstacles and hazards, the environment must be well illuminated. Well-lit, glare-free halls, stairways, rooms, and work spaces help to reduce the risk of tripping, slipping, and falling. Night-lights reduce the risk of injuries to children, guests, and older adults. Lighting the exterior of the house will also reduce the risk of falling. Staying indoors during episodes when air pollution is high can help prevent chronic lung disease. Waxed floors are slippery.

The student learns that which are cycles in the female sexual response cycle? (Select all that apply.) A.) Excitement B.) Orgasm C.) Resolution D.) Detumescence E.) Plateau

A, B, C, and E. The female sexual response cycle includes the phases of excitement, plateau, orgasm, and resolution. Detumescence is when the penis is no longer erect after orgasm.

Regarding perineal care, which nursing actions are appropriate? (Select all that apply.) A.) The nurse applies gloves prior to performing perineal care. B.) The nurse ignores the erection of a male patient during perineal care. C.) The nurse documents the perineal care. D.) The nurse only completes perineal care with daily bathing. E.) The nurse can delegate perineal care.

A, B, C, and E. The nurse uses standard precautions (gloves) whenever contact with body fluids is expected. A male patient may have an erection during care, which is a normal response with tactile stimulation. The care provider can ignore the erection and continue with the procedure or return later to complete the care, depending on the comfort level and the situation. Documentation is part of hygienic care. Note any redness, drainage, odor, edema, or skin changes. Perineal care is provided during a bath or shower but may be necessary more frequently, especially in incontinent patients. Perineal care can be delegated.

The nurse is performing a health history to determine the patient's sensory status. Which questions will be best suited to elicit the information needed? (Select all that apply.) A.) "Do you ever lose your balance?" B.) "Do you wear glasses?" C.) "Do you like to read the newspaper?" D.) "Can you feel the difference between hot and cold water?" E.) "Do you wear a hearing aid?"

A, B, D and E. Balance, eyesight, hearing, and sensation are all sensory function. Asking if the patient likes the newspaper does not specifically address vision.

The nurse is caring for an elderly patient whose dementia has become worse over the last 24 hours. The nurse suspects that the patient may have developed a urinary tract infection and obtains a urine sample. Which assessment findings prompt the nurse to contact the provider to obtain an order for urine culture and sensitivity testing? (Select all that apply.) A.) Urinary dipstick testing is positive for nitrates. B.) The urine appears cloudy with a foul odor. C.) The urine is concentrated and dark amber in color. D.) The urine smells faintly like sweet fruit. E.) The patient is urinating more frequently than usual. F.) The patient is normally continent but has been incontinent twice.

A, B, E, and F. Concentrated dark urine indicates dehydration rather than infection of the urinarytract. Urine that smells of sweet fruit contains ketones from high blood sugar. Urine that is cloudy with a foul odor and positive for nitrites is most likely due to urinary tract infection. Frequent urination and incontinence are signs of urinary tract infection in the elderly.

The nurse who incorporates the HOPE framework assesses a Native American patient for which of the following? (Select all that apply.) A.) Desire for shaman to be present B.) Personal use of herbs and prayers C.) Desire to create a living will D.) Power of storytelling for healing E.) Involvement in church activities

A, B, and D. Native Americans often use shamans; prayers, songs, and dances; storytelling; and herbs in health care. The HOPE framework assesses sources of hope, meaning comfort, strength, peace, love, and connection; organized religion; personal spirituality and practice; and effects on medical care and end-of-life issues. The nurse who knows about both topics will assess this patient for the desire for a shaman to be present, the personal use of herbs and prayers, and storytelling. A living will is more accurately assessed with the SPIRIT framework. Involvement in church activities can be best assessed using either the SPIRIT or FICA framework.

In an emergency situation, the nurse will pay particular attention to?

ABCs. Airway, Breathing, and Circulation.

The concept of being answerable for one's actions

Accountability

What are three types of nursing diagnoses?

Actual nursing diagnoses identify existing problems or concerns of a patient. Risk nursing diagnoses apply when there is an increased potential or vulnerability for a patient to develop a problem or complications. Health-promotion nursing diagnoses are used in situations in which patients express interest in improving their health status through a positive change in behavior.

What are the ABCs of life support?

Airway, Breathing, and Circulation

Identify at least three of the criteria that identify a profession.

Altruism, a body of knowledge and research, accountability, higher education, autonomy, a code of ethics, professional organizations, licensure, and diversity.

What is incivility?

Any form of socially harmful behavior, such as: Aggression Interpersonal deviance Social undermining Interactional justice Harassment Abusive supervision Bullying

Indicate the standard of care being demonstrated in practice or performance for each of the following: The RN admits a patient to the unit and collects comprehensive data on his health status.

Assessment

How is critical thinking used in the following steps of the nursing process? A.) Assessment B.) Implementation

Assessment - Use of critical thinking skills help the nurse to identify data gaps and focus on relevant information. Skills can also be used to determine the best way in which to obtain critical patient data. Implementation - Nurses can use critical thinking skills to provide the best type of care for individual patients. They can also adapt care to need specific needs or when current interventions are not effective.

A subdimension of Swanson's process of caring, Knowing, involves which of the following? A.) Sharing feelings with the patient B.) Performing thorough assessments C.) Focusing on the patient's experiences D.) Offering realistic optimism

B

The nurse has delegated the feeding of a patient who has recently had a stroke to the UAP. Which procedure that the UAP performs would demonstrate a need for further education? A.) Uses thickened liquids. B.) Puts the bed at 25 degrees. C.) Encourages slow eating. D.) Has the patient alternate between food and sips of fluid.

B During feeding, the head of the bed needs to be elevated at 30 to 45 degrees or higher. Liquids are thickened, and patients are encouraged to use slow-eating habits and to alternate between bites of food and sips of fluids to facilitate swallowing.

The nurse is caring for a patient who is receiving vancomycin (Vancocin) to treat a severe infection. The next dose is due to be administered at 10:00 a.m. What time will the nurse draw the vancomycin serum trough level? A.) 7:30 a.m. B.) 9:30 a.m. C.) 11:30 a.m. D.) 1:30 p.m.

B The trough is the lowest serum level of the medication. Serum trough levels are to be drawn just prior to the administration of the medication.

When the nurse is caring for a patient receiving enteral feedings, which tasks can that nurse delegate to the UAP? (Select all that apply.) A.) Verify tube placement B.) Perform oral care C.) Administer tube feeding D.) Obtain vital signs and report results E.) Measure oxygen saturation

B, C, D, and E. Administering an enteral feeding may be delegated, at the nurse's discretion, to UAP in accordance with state regulations and facility policies and procedures. The nurse should verify tube placement and assess the patient prior to delegating this procedure. The UAP can perform oral care and obtain vital signs, including oxygen saturation, and report results.

The nurse is caring for a diabetic patient who has had a long history of poor glucose control. For what complications is the patient at risk? (Select all that apply.) A.) Sudden loss of consciousness B.) Diabetic retinopathy C.) Stroke D.) Peripheral neuropathy E.) Memory loss

B, C, D, and E. Long-term complications of hyperglycemia may contribute to cognitive and sensory deficits such as memory loss. They also can lead to diabetic retinopathy, peripheral neuropathy, and stroke. Loss of consciousness is usually seen with hypoglycemia in diabetics.

The nurse is delegating taking vital signs to an unlicensed assistive personnel (UAP). What instructions does the nurse provide the UAP? (Select all that apply.) A.) "Let me know if Mr. Smith's blood pressure is low." B.) "Take Mrs. Jones' blood pressure every 15 minutes." C.) "Call me if Ms. Walsh's systolic blood pressure drops to under 100 mm Hg." D.) "Do you want me to demonstrate using the electronic blood pressure cuff?" E.) "I'll take Mr. Derby's blood pressure since he is not stable."

B, C, D, and E. The nurse can delegate measuring vital signs to UAPs if the patient is stable. The nurse must ensure the UAP knows the proper technique for taking vital signs and knows which readings must be reported. Telling the UAP to report a blood pressure that is "too low" is too vague.

The nurse who is aware of spirituality practices of major religions knows that which religions view health and illness as a process of balance or imbalance? (Select all that apply.) A.) Catholicism B.) Native American C.) Hinduism D.) Greek Orthodox E.) Buddhism

B, C, and E. Native American, Hindu, and Buddhist practitioners believe that health and illness are a matter of balance or imbalance in the body.

The nursing student learns that which are correct regarding acculturation and assimilation? (Select all that apply.) A.) Assimilation is forced entry into a different culture. B.) Acculturation depends on first-hand contact between groups. C.) Acculturation results in changes to the minority culture only. D.) Assimilation can occur at the group or individual level. E.) Assimilation causes a minority group member to blend into the majority group.

B, D, and E. Acculturation occurs from first-hand contact between a minority group and the majority cultural group and can result in changes to one or both cultures. Assimilation occurs when members of a minority group blend into the majority group and can occur at the group or individual level. Assimilation is not a forced change.

In writing goal statements, it is important to use measurable verbs. Identify the verbs that are best to use in developing measurable and patient-orientated goal statements. Select all that apply. A.) Understand B.) Verbalize C.) Know D.) Perform E.) Think F.) List

B, D, and F.

Identify an ethical consideration related to the cost of health care.

Balancing the need to assist the patients to receive optimum care while considering the cost of the care.

Actual physical harm caused to another person

Battery

Jean Watson

Belief that health care needs to move from a total disease-cure focus based solely on scientific inquiry to a more holistic approach that incorporates values, beliefs, intentions, and the caring consciousness. Care that is not just task based but incorporates the overall patient, such as making time for the patient to talk about their thoughts about the value of continuing chemotherapy with a late-stage cancerous growth.

Perceived Susceptibility

Beliefs about the chances of getting a condition or disease. Change Strategies - Define populations at risk. Tailor risk information to the individual's characteristics or behaviors. Help the individual develop an accurate perception of his or her own risk.

Provide examples of nonverbal communication.

Body language (posture, stance, gait), facial expressions, and eye movement, touch, gestures, and symbolic expression (appearance).

What is the difference between EMR and EHR?

Both EMR and EHR are digital records of patients health information. An EMR is best understood as a digital version of a patients chart while an EHR contains the patients records from multiple doctors and provides a more long term view of a patients health

Identify the levels of Maslow's hierarchy of needs.

Bottom going up: Physiologic needs: Oxygen, food, water, elimination, temperature control, sex, movement, rest, and comfort. Safety and Security: From a physiologic and psychological threat; and protection, stability, and lack of danger. Love and Belonging: Affection, intimacy, support, and reassurance Self-Esteem: Self-worth, self-respect, independence, privacy, status, dignity, and self-reliance. Self-Actualization: Recognition and realization of one's potential, growth, health, and autonomy.

Generalization

Broad statement or idea about people or things

The nurse recognizes which goal to be appropriate for the nursing diagnosis of Anxiety? A.) The patient will attend a weekly support group. B.) The patient will discuss possible coping strategies during weekly office visits. C.) The patient will report increased ability to concentrate on care instructions before discharge. D.) The patient's family will use respite care once a week for the next month.

C Attending a weekly support group is an appropriate goal for Difficulty coping. An appropriate goal for Ineffective coping would be to discuss possible coping strategies during weekly visits. Using respite care once a week for the next month is an appropriate goal for Caregiver stress.

The nurse is caring for a patient who has diarrhea and identifies which priority nursing diagnosis for this patient? A.) Lack of knowledge related to prescribed diet modifications B.) Impaired nutritional intake related to poor appetite C.) Diarrhea related to excessive loss of fluid through stool D.) Anxiety related to incontinence with loose stools and need for clothing change

C Dehydration is the priority nursing problem for this patient, so diarrhea is the most important Nursing diagnosis. Impaired nutritional intake, lack of knowledge, and anxiety can be addressed once fluid balance is restored.

The nurse is preparing to discharge a patient home. In providing instruction about the patient's medications, the nurse should make which statement? A.) "Before taking Metoprolol, you need to take your BP and rate." B.) "MS should be taken only when needed for pain." C.) "Take 1 baby aspirin by mouth every morning." D.) "Take your water pill bid and you should be fine."

C Do not use abbreviations or medical terminology when providing patients with instructions.

When explaining delegation to student nurses, what statement by the nurse educator aligns to the ANA regarding delegation? A.) A transfer of authority to a less-qualified individual B.) The nurse transferring accountability to the delegate C.) The transfer of tasks by the nurse while retaining accountability D.) Transferring responsibility for assessments and planning

C For patient care to be completed in a safe and timely manner, it is sometimes necessary for the nurse to delegate tasks to other health care providers. The National Council of State Boards of Nursing (NCSBN) offers support in this process. In their joint statement (ANA and NCSBN, 2005), the ANA describes delegation as the transfer of responsibility, and the NCSBN calls it a transfer of authority. This transfer gives a competent individual the authority to perform a selected nursing task in a selected situation. The nurse retains accountability for the delegation. Any significant findings during the care such as alterations in skin integrity, shortness of breath, or changes in a patient's condition should be reported to the nurse. The nurse is then responsible for assessing the alterations and addressing them in the plan of care.

The nurse is caring for a patient who takes furosemide (Lasix) daily to treat congestive heart failure. The nurse will watch for which electrolyte imbalance that may occur due to this therapy? A.) Hypocalcemia B.) Hypernatremia C.) Hypokalemia D.) Hyperphosphatemia

C Furosemide is a loop diuretic that causes loss of potassium through the urine. Patients taking this medication are at risk for hypokalemia, so the nurse should check the patient's electrolyte values closely, particularly the serum potassium level.

A nurse is caring for a victim of domestic violence. What charting by the nurse is most appropriate? A.) Patient allegedly beat up by her boyfriend. B.) Patient has several bruises on the legs. C.) Patient states, "My boyfriend hit me with a hammer." D.) Patient claims she was assaulted last night.

C Good charting is objective and detailed. Using the patient's own words, in quotation marks, is the most accurate example of documentation. The nurse should not use words like "allegedly" or "claims" because they seem to cast doubt on the patient's story. The bruises on the legs need to be measured and described more fully.

The nurse is caring for a patient who has a history of congestive heart failure with generalized pitting edema. Which laboratory results will the nurse expect to find in the patient's chart? A.) Glycosylated hemoglobin 12% B.) Platelet count 450,000/mm3 C.) Hematocrit 32% D.) Prothrombin time 8.8 seconds

C Hemodilution is a common finding when patients are in fluid overload caused by congestive heart failure. A normal hematocrit result is 42% to 52% for a male and 37% to 47% for a female, so the patient's 32% hematocrit level is markedly low. The other laboratory results are not expected due to congestive heart failure or fluid overload.

The ER nurse is triaging a patient with suspected poisoning. Who should the nurse anticipate contacting first? A.) Family services B.) Radiology C.) Poison Control Center D.) Respiratory

C If poisoning is suspected, the National Poison Control Center should be contacted immediately. This information will be needed to determine treatment. Respiratory may be needed, and radiology and family services may also be needed, but that will be determined after the treatment plan is determined.

A nurse notes a patient has abnormal vital signs. What action by the nurse is best? A.) Document the findings. B.) Notify the provider. C.) Compare with prior readings. D.) Retake the vital signs in 15 minutes.

C Individual vital signs are not as important as the trends. For instance, a patient may have a blood pressure higher than "normal" that is normal for the patient. Trends give more useful information than a single reading. Documentation is important, but the nurse needs to do more. If the readings are significantly abnormal, the provider should be notified. The nurse may retake the vital signs if he/she is not confident of the first set of measurements, but should not wait for time to pass.

The nurse is educating the family of a patient on falls risk precautions. Which statement by the family indicates a need for further education? A.) "I should keep the wheelchair locked unless using it to move Mom." B.) "I should leave the bathroom light on as she does at her home." C.) "I should leave her slippers by the wheelchair." D.) "I should keep her cell phone close to her bed."

C Leave lights on or off at night, depending on the patient's cognitive status and personal preference. Keep the wheels of any wheeled device (e.g., bed, wheelchair) in the locked position. Keep patient belongings (e.g., tissues, water, urinals, personal items) within the patient's reach. If the patient is ambulatory, require the use of nonskid footwear (socks or shoes).

A patient admitted after abdominal surgery has a Nursing diagnosis of risk for infection. The nurse identifies which goal to be most appropriate? A.) Patient will ambulate length of hallway this shift. B.) Patient will consume 20% of meals by the end of the week. C.) Patient's incision will be without signs or symptoms of infection at discharge. D.) Patient will verbalize need to stop antibiotics medication when symptom free.

C Maintaining skin integrity is an appropriate goal for this patient to ensure the patient does not develop a wound infection. Ambulating will assist in preventing skin breakdown be getting the patient out of bed, but it is not the priority goal for a patient with an incision. Consuming only 20% of meals will not ensure adequate nutrition and verbalizing the end of antibiotic administration to be when symptoms end is inappropriate. Antibiotics should be taken until the prescription is complete.

A nurse in the emergency department wants to screen a patient for domestic violence, but the woman's partner won't leave. What action by the nurse is best? A.) Ask the questions anyway. B.) Tell the partner to leave. C.) Go with the patient to the bathroom. D.) Skip the abuse assessment.

C Nurses are required to screen for domestic abuse. The nurse needs to provide complete privacy during this assessment. If the partner won't leave, the nurse can go with the patient to the bathroom under the guise of obtaining a urine sample and ask the questions there. Telling the partner to leave will most likely increase the partner's vigilance. Skipping the assessment is not an option.

The nurse is explaining the purpose of occlusive dressings to the student nurse. Which statement by the student nurse indicates a lack of understanding? A.) "Occlusive dressings are used for autolytic debridement." B.) "Hydrocolloids are a type of occlusive dressing." C.) "Occlusive dressings can be used on infected wounds." D.) "Occlusive dressings support the most comfortable form of debridement."

C Occlusive dressings such as hydrocolloids and transparent films are used for autolytic debridement and are contraindicated in infected wounds. It is the most comfortable form of debridement for the patient.

When the nurse is wearing sterile gloves, which action would result in the gloves becoming nonsterile? A.) Fold gloved hands until procedure begins. B.) Change a dressing using aseptic technique. C.) Place sterile gloved hands below waist. D.) Use correct protocol when donning sterile gloves.

C Once the hands have been placed below the waist, they can longer be considered sterile or free from organisms. Asepsis refers to freedom from disease-causing contamination. All other choices maintain asepsis.

The nurse is attempting to open an occluded PEG tube. Which intervention by the nurse requires re-education? A.) Flushes the tube with a small amount of air. B.) Flushes the tube using a 50- to 60-mL syringe and warm water. C.) Reinserts the stylet to break up the clot. D.) Flushes the tube with a special enzyme solution.

C Once the stylet is removed, it is never reinserted because it can puncture the intestine. If the tube becomes occluded, flush it with a small amount of air. If this is unsuccessful in removing the occlusion, flush the tube using a 30 to 60 mL syringe and warm water. If flushing the tube with water is ineffective, research now suggests using special enzyme solutions or declogging devices rather than carbonated beverages or juices.

A nurse is caring for a patient who has orthopnea. What action by the nurse is most appropriate? A.) Encourage deep breathing and coughing. B.) Medicate the patient for pain as needed. C.) Keep the head of the bed elevated. D.) Monitor the length of time the patient doesn't breathe.

C Orthopnea is difficulty breathing in positions other than sitting up. To assist the patient who has orthopnea, the nurse keeps the head of the bed elevated to ease breathing.

The nurse knows what goal to be appropriate for a patient with a stage 3 pressure ulcer with the nursing diagnosis impaired physical mobility? A.) Patient will remain free of wound infections during the hospitalization. B.) Patient will report pain management strategies and reduce pain to a tolerable level. C.) Patient will be able to assist with position changes using over bed trapeze within 1 week. D.) Patient will consume adequate nutrition to meet nutritional requirements within 1 week.

C Patient will be able to assist with position changes using over bed trapeze within 1 week is an appropriate goal for impaired mobility. The patient remaining free of wound infections during the hospitalization is an appropriate goal for impaired tissue integrity. The patient reporting pain management strategies to reduce pain to a tolerable level is an appropriate goal for acute pain. The patient consuming adequate nutrition to meet nutritional requirements within 1 week is an appropriate goal for Impaired nutritional status.

A patient wishes to review his medical record. What response by the nurse is best? A.) "I'm sorry, we don't allow you to look at your chart." B.) "Let me check to see if we can allow you to do that." C.) "Yes, I can sit with you while you look at it, so you can ask questions." D.) "Yes, all patients can review their charts at any time they wish."

C Patients have the right to look at their records. It is best if a health care provider is present to answer any questions the patient may have or to help interpret any information found within the record.

The nurse is seeing a patient during a follow-up visit after discharge in which the patient had a nursing diagnosis of Difficulty coping. Which statement by the patient would be a cause for concern? A.) "I am sleeping better most nights." B.) "I feel less anxious." C.) "I do not need to do the relaxation exercises anymore." D.) "I am continuing my exercises every day."

C Patients need to continue using the stress-reduction techniques to maintain a feeling of well-being. Once stress decreases, patients typically report feeling better, sleeping more soundly, and feeling less anxious. Continuing their positive activities such as exercising is good.

The nurse understands that which set of vital signs most likely indicates infection? A.) T: 98.6 °F (37.0 °C), P: 75 beats/min, R: 18 breaths/min, BP 120/80 mm Hg B.) T: 99 °F (37.2 °C), P: 80 beats/min, R: 18 breaths/min, BP: 110/70 mm Hg C.) T: 100.5 °F (38 °C), P: 96 beats/min, R: 22 breaths/min, BP: 150/100 mm Hg D.) T: 98.9 °F (37.1 °C), P: 66 beats/min, R: 18 breaths/min, BP: 98/62 mm Hg

C With infection, temperature will rise and blood pressure will increase along with pulse and respiratory rate.

A patient, who is an adherent Muslim, is in a burn unit with severe burns. The patient has high caloric requirements but is refusing to eat during Ramadan. What action by the nurse is best? A.) Insert a feeding tube and provide enteral feedings. B.) Ask the provider about Total Peripheral Nutrition. C.) Call the patient's religious leader for advice. D.) Tell the patient he has to eat to get better.

C With permission, the nurse should consult with the patient's religious leader on this situation. There may be exceptions to the rule to fast during Ramadan for medical conditions. The other options ignore the patient's religious preferences, and both the tube feeding and parenteral nutrition have potential serious side effects.

The nurse is caring for a patient who is postoperative day one from an abdominal surgery. When the patient complains of a "popping sensation" and a wetness in the dressing, the nurse immediately suspects which complication? A.) A wound infection B.) The stitches came loose C.) Wound dehiscence D.) Wound crepitus

C Wound dehiscence, which usually occurs in connection with surgical incisions, is the partial or complete separation of the tissue layers during the healing process. This is an emergency situation. Stitches can come loose, but there is no popping sensation. Wound infections are characterized by redness, warmth, and drainage, and crepitus is air trapped under the skin.

The nurse is caring for a patient who is postoperative day one from abdominal surgery. The patient complains of a "popping sensation" and wetness in her dressing. The nurse immediately suspects: A.) A wound infection B.) The stitches came loose C.) Wound dehiscence D.) Wound crepitus

C Wound dehiscence, which usually occurs in connection with surgical incisions, is the partial or complete separation of the tissue layers during the healing process. This is an emergency situation. Stitches can come loose, but there is no popping sensations. Wound infections are characterized by redness, warmth, and drainage, and crepitus is air trapped under the skin.

Which of the following actions fall with an excepted ethical guidelines? Select all that apply. A.) Sharing the nurses personal information with a patient B.) Posting photos of the patients on social media site C.) Refusing to provide health information on a patient to a caller from a newspaper D.) Meeting with the patient for a social event after his or her hospitalization E.) Speaking with other members of the healthcare team about the patient's status F.) Telling the patient's roommate about the outcome of the patient surgical procedure

C & E

The patient is on protective precautions. The nurse knows which statements are true regarding these precautions? (Select all that apply.) A.) A positive-pressure room with a HEPA filtration system is required. B.) Special respirator masks should be available and one size fits all. C.) No live plants are allowed in the room. D.) The patient may eat any foods desired. E.) Everyone entering the room wears a mask.

C and E Protective precautions may require a positive-pressure room. No live plants, fresh flowers, fresh raw fruit or vegetables, sushi, or blue cheese may be brought into the room because they may harbor bacteria and fungi. The patient cannot eat just any foods because some are restricted. A mask is required for anyone entering the room and for the patient if leaving the room.

In the body systems model for data organization, identify how the following signs and symptoms should be organized. A.) Episodes of chest pain B.) Decreased ROM to the left knee C.) Periodic epigastric distress after eating

Chest pain - Cardiovascular Decreased ROM - Musculoskeletal Periodic Epigastric Distress - Gastrointestinal

Governs unjust acts against individuals, rather than federal or state crimes.

Civil Law

The ability to focus and filter clinical data in order to recognize what is most and least important, so the nurse can identify if an actual problem is present

Clinical Reasoning

What are clinical pathways, protocols, and standing orders, and how are they used by the nurse?

Clinical pathways - Sometimes referred to as care pathways, care maps or critical pathways, are multidisciplinary resources designed to guide patient care. Protocols - Written plans that can be generalized to groups or patients with the same or similar clinical needs and do not require a physician order. Outlines procedures for admitting patients or handling routine care situations. Since they are general, they often included in critical pathways. Standing Orders - Written by physicians and list specific actions to be taken by a nurse or other health care provider in instances when access to a physician is not possible or when care is common to a certain situation.

Indicate the standard of care being demonstrated in practice or performance for each of the following: The RN documents the care provided to the patient and gives a report to the nurse on the next shift.

Communication

What is The Moral of Development Theory?

Complicated process involving the acceptance of the values and rules of society in a way that shapes behavior. Sequential but people do not automatically go from one level to another as they mature.

What are the three kinds of physical assessments that can be performed by the nurse, and when are they usually conducted?

Comprehensive/Complete, Clinical/Focused, and Emergency.

Provide at least 2 examples of cultural, ethnicity and religious diversity considerations for a patient assessment.

Cultural and ethic norms affect the willingness of patients to speak openly about health concerns. Nurses should explain the need for information that may be considered intimate in nature.

The patient has a nursing diagnosis of Risk for Falls. The nurse identifies which goal to be most important? A.) Patient will ambulate twice a day. B.) Patient will have no symptoms of infection. C.) Patient will perform activities of daily living. D.) Patient will have no injuries during hospital stay.

D All the goals except lack of infection are appropriate for a patient with a Risk for Falls diagnosis; however, the most important goal is for the patient to have no injuries during the hospitalization.

A student nurse is caring for a patient who is a refugee. The patient will take his own blood glucose readings and will self-administer a set dose of insulin but will not follow a sliding scale regimen in which the patient has to choose what dose of insulin to give. What action by the student nurse is best? A.) Ask the provider to prescribe only a set insulin regimen. B.) Instruct the patient on the benefits of sliding scale insulin. C.) Teach the patient that strict carbohydrate limits are needed. D.) Ask the patient to explain the meaning of making this decision.

D The patient may have a more fatalistic world view than is common in Western societies. The patient may follow "orders" from an authority figure but may feel like it is not his place to determine his insulin dose, or the patient may not feel competent in making that decision. Many explanations are possible. The student needs to determine what the patient feels related to this type of decision making before doing anything else.

The nurse is providing care to a post-stroke patient on the rehabilitation floor with a nursing diagnosis of Impaired health maintenance. Which goal is most appropriate on day one? A.) Patient will ambulate independently twice a day. B.) Patient will perform all own ADLs. C.) Patient will consume 75% of all meals. D.) Patient will begin to perform 25% of own ADLs.

D The patient needs to work toward achieving as much independence in self-care as possible; starting with 25% in a post-stroke patient on day one is more achievable than 100%. Ambulating and eating meals are not goals for a problem with self-care.

For the following patient information, identify how they may be clustered into a diagnostic label: - States "I am very thirsty." - Reports weight loss of 5 pounds over the past 3 days - Complains of feeling tired - Has increased hematocrit and creatinine - Has decreased blood pressure with a slight increase in body temperature

Decrease fluid volume

What is Thelarche?

Development of breast buds (8-13 years)

Provide examples of direct and indirect nursing care.

Direct nursing care involves having a personal contact with patients. Such as cleaning an incision, administering an injection, completing teaching at the bedside. Indirect nursing care are interventions that are performed for the benefit of the patients but not face-to-face. Such as making the change to end-of-shift report, communication and collaboration with members of the interdisciplinary team, and ensuring availability of needed equipment.

Indicate the standard of care being demonstrated in practice or performance for each of the following: The nurse attends a conference and attends workshops on new techniques for patients with cardiac pathological conditions.

Education

Identify what emotions can be conveyed by patients through inflection.

Emotions are conveyed through inflection may include anger, joy, depression, anxiety, tension, and confusion/questions.

Initial anaphylaxis interventions

Ensure patent airway .Intubation if evidence of impending obstruction •Remove insect stinger if present. •Establish IV access •Epinephrine (1mg/mL preparation): Give 0.3-0.5mg IM preferably in the mid-outer thigh. Can repeat every 5-15 min •Administer high-flow O2 (8-10L/min) via face mask. Can give up to 100% as needed •Nebulized albuterol (Proventil) for bronchospasm resistant to epinephrine •Diphenhydramine (Benadryl) IV for urticaria and itching. •Corticosteroids: methylprednisolone (Solu-Medrol)

Red blood cells

Erythrocytes - carries O2 and nutrients Disc shape no nucleus , very flexible

Cues to Action

Factors that activate readiness to change Change strategies - Provide how-to information, promote awareness, and use reminder systems.

Who is Westerhoff?

Faith development. 4 styles: Experienced, Affilitative, Searching, and Owned.

True/False: If a patient achieves the identified goals, the nurse should continue to address them in the care plan.

False

True/False: It was found that only 20% of internet users searched for health information online

False

A more serious crime that results in the perpetrator being imprisoned in a state or federal facility for more than 1 year

Felony

Kristen Swanson

Five processes characterize caring: Knowing, Being with, Doing For, Enabling, and Maintaining belief. Care that incorporates actions that demonstrate caring, such as providing a soothing bath to a patient confined in the bed.

Identify the essential qualities of patient goals.

Focused, realistic and measurable.

Identify communication strategies that may be used for a patient with expressive aphasia.

For a patient with expressive aphasia, the nurse can work with a patient to have a series of gestures or use of the photo/alphabet board, I notepad, laptop/tablet tablet, or an erasable whiteboard to convey messages

Who is Florence Nightingale?

Founder of Modern Nursing

How would you evaluate the patient achievement of the goal on the basis of the observations noted? Goal - Patient will report a pain level of 2 to 3 out of 10 during morning care within 24 hours. During morning care, the patient indicated that the pain level was about 5 on a scale of 10. It was noted that he was protecting the surgical site and his apical pulse was 108 bpm.

Goal not met. Patient experiencing a pain level above 3 and demonstrating objective signs of discomfort.

What is utilitarianism?

Greatest good for the greatest number of people

Caring

Having concern or regard for which affects the welfare of another.

Who is Dorothea Dix?

Head of US Sanitary Commission

Polycythemia vera signs and symptoms

Headache Vertigo Dizziness Tinnitus Plethora *red face* Generalized pruritis Painful burning/redness of hands/feet Angina, HF, intermittent claudications, thrombophlebitis Complications: Stroke 2ndary to thrombosis Hemorrhagic phenomena Hepatomegaly Splenomegaly Uric acid increased (gout)

A state of complete physical, mental, and social well-being and not merely the absence of disease or infirmity

Health

Includes all pertinent information that can guide the development of a patient centered plan of care

Health History

Concepts focus on the interrelatedness of the physical body and the mind, with aspects of spirituality, emotional security, and the environment

Holistic Health

Identify one concept included in the theories from each of the following theorist: Rogers

Human beings and their environments are interacting in continuous motion as infinite energy fields.

What are adolescent (12-18 years) theories of development?

Identity vs Role Confusion (Erikson) Formal Operations (Piaget) Genital Stage (Freud) - Time for sexual energy thwarts notion of child as part of family.

How does the nurse prepare for a physical assessment?

If test results are were ordered before the patient is seen, the results are reviewed by the nurse. Privacy for the patient is ensured; good lighting established and the equipment and instruments needed are gathered before the physical examination is started. Hand hygiene is performed, and clean gloves are worn if contact with body fluids is anticipated.

Provide examples for each type of nursing intervention. A.) Independent (nurse initiated) B.) Dependent C.) Collaborative

Independent - Providing patient education or emotional support, assessing status, instituting preventative measures. Dependent - Administering medications, performing wound care. Collaborative - Physical therapy, hospice care, spiritual or financial counseling.

Self-concept is the way individuals see themselves in relation to?

Individuals see themselves in relation to social character or abilities, physical appearance, body image, and ways of thinking. This entails the person's mental image of themselves in relation to others.

Uses specific facts or details to make conclusions and generalizations; proceeding from specific to general

Inductive Reasoning

What are School Age (7-11 years) Theories of Development?

Industry vs Inferiority (Erikson) - Competitive use of skills Concrete operations (Piaget) - Real vs Magical Latency (Freud) - Time of no particular sexual expression

How does the nurse use information gathering in critical thinking process?

Information gathering assist the nurse to collect relevant, precise, and accurate data. Because clinical decisions are often based on such data collection, it is important that the nurse utilize critical thinking skills during these assessments.

What can affect pulse oximetry reading?

Injured extremities, peripheral edema, jaundice, movement, nail polish, and cold temperatures.

The use of vision, hearing, and smell to closely scrutinize physical characteristics of a hole person and individual body systems

Inspection

Identify two differences between the licensed practical nurse and the registered nurse.

LPNs complete an educational program consisting of 12 to 18 months of training, and upon completion they must past the National Council Licensure Examination for Practical Nurses (NCLEX-PN) to practice as an LPN/LVN. LPNs are under supervision of an RN in most institutions and are able to collect data, but usually they cannot perform an assessment requiring decision making, cannot formulate a nursing diagnosis, and cannot initiate a care plan. They may update the care plans and administer medications, with the exception of certain IV medications.

What is preconventional morality?

Lack of internalized standards about right and wrong; making decisions based solely on what is best for oneself

Socialization

Learning the theory and skills for the nursing role

Written forms of defamation of character

Libel

First order beliefs are derived from

Life experiences and respect authorities

How do the following influence patient care? A - Life Span B - Gender C - Culture

Life span - Intervention must always be age or developmental appropriate. Encouraging patients of all ages to actively participate in their care provides them with control, even in the most serious circumstances. Gender - Gender roles may impact care delivery. Some may prefer care from nurses of the same gender. Culture - Important to know if eye contact or physical contact should be avoided. Proper explanation of procedures is essential. Asking patients to share their understanding of a procedure that has been explained.

Angioedema

Localized areas of swelling beneath the skin, often around the eyes and lips, but it can also involve other body areas as well. Progresses to airway Atopic Suddenly or over several hours Lesions

When do we not use an electronic device for vitals?

Low BP, irregular heart rates, shivering, and seizure activity

What is a nadir?

Lowest point of neutropenia in a pt tx with chemo

What is MAGIC preschoolers?

M - Mutilation - Such as cutting ones own hair A - Associate Play, Abandonment G - Initiative vs Guilt I - Imagination, Imaginary playmate C - Curious

Rheumatoid arthritis education

Maintain joint in neutral position / use biggest joints primarily Distribute weight over many joints instead of a few Change positions frequently Avoid repetitive movements Modify chores to avoid stress on joints (avoid heavy lifting, sit on a stool during meal prep - avoid standing for long periods of time) Nutrition Heat/cold applications - nonpharm pain relief

What is the difference between nursing diagnosis and a medical diagnosis?

Medical diagnosis are labels for diseases, whereas nursing diagnoses consider the patients response to a medical diagnoses and life situations as well as make clinical judgements based on a patient's actual medical diagnoses and conditions.

Provide at least 3 examples of what a nurse may teach a patient and/or family.

Medications, wound care, use of assistive devices, diets, postoperative cautions, pain relief measures, and other areas for physical and mental health.

Creating a risk to others by failing to do something that a "reasonable person" would ordinary do or doing something that a "reasonable person" would ordinarily not do.

Negligence

Who gets a rectal temperature?

Newborns, patients with neutropenia, spinal cord injuries, diarrhea, rectal diseases/surgery, and quadriplegia.

Data are analyzed, validated, and clustered to identify patient problems

Nursing diagnosis

The identification of actual or potential health problems/life processes and responses to a problem

Nursing diagnosis

Madeleine Leininger

Nursing is a transcultural care profession, and the concept of care is at its center. Care related to the recognition of and accommodation for the individual's cultural background and practices, such as praying at particular times during the day.

What is the advantage of the EHR in planning and implementing care?

Often include care plan formats that enhance nurse's ability to formulate individualized plans of care and facilitate coordination of care. Documentation of evaluative criteria and data in patient's EHR allows nurses and other health care providers to collaborate electronically while responding to changes in a patient's health status.

What are Freud's psychosexual stages?

Oral (0-2) - Infant achieve gratification through oral activities such as feeding, thumb sicking and babbling. Anal (2-3) - The child leans to respond to some of the demands of society (such as bowel and bladder control). Phallic (3-7) - The child leans to realize the differences between males and females and becomes aware of sexuality. Boys imitate their father to get the attention of their mother and vs versa with girls. Latency (7-11) - The child continues his or her development but sexual urges are relatively quiet. Genital (11 to adult) - The growing adolescent shakes off old dependencies and leans to deal maturely with the opposite sex.

What are PAIRS Adolescents?

P -Peer group, Puberty A - Altered body image I - Identity R - Role Confusion S - Separation from Peers

The use of touch to assess body organs and skin texture and thickness

Palpation

What is the ego?

Part of unconscious and conscious. it mediates between the superego and the Id. seeks LONG TERM gratification. Reality principle.

Provide examples of prevention-orientated nursing interventions.

Patient education, immunization programs, cleaning a wound to prevent infection, and placing infants on their backs to sleep.

How does involving the patient/or family in planning help improve goal attainment?

Patient is more likely to accept goals and treatment plan, which can contribute to achieving the expected outcomes

Who may be involved in collaborative care for a patient?

Patient, family and nurses; PCP; medical or surgical specialists, dietitian, PT, OT, music or spiritual adviser; social worker.

Identify a diversity consideration for communication based on the patients culture or ethnicity.

People born in more densely populated areas typically less personal space for comfort. English-speaking people typically prefer at least 18 inches of distance between themselves and others when conversing. In contrast, people of various cultural backgrounds may be comfortable standing bath code while communicating. Direct eye contact may be perceived as aggressive or impolite by individuals from Arab, Indo-Chinese or American Indian cultures. Special communication challenges exist between migrants and nurses that must be addressed to avoid health inequities.

POMR

Problem Oriented Medical Records that divide records into four sections - the database, problem, treatment, and progress

How to treat hypotension r/t anaphylaxis

Recumbent position with legs elevated NS bolus 1-2L Maintain BP with fluids, volume expanders, vasopressors (e.g., dopamine [Intropin]

How is polycythemia vera treated?

Remove blood - 1 pt per week until normal.

Identify the different nursing roles (e.g., advocate) that are identified with the following situations: Nurses determine care concerns and ask questions about nursing practices.

Researcher

Information shared by family members, friends or other members of the health care team

Secondary Data

Confidence in one's ability to take action

Self-efficacy

What are the stages of Piaget's Cognitive Development?

Sensorimotor (Birth to 2 years) - Identifies object performance, the object still exist when out of sight. Recognition of ability to control and act intentionally. Preoperational (2 to 7 years) - Begins to you use language. Egocentric thinking difficulty seeing things from other viewpoints. Classified objects by single feature such as color. Concrete Operational - (7 to 11) - Logical thinking. Recognizes conservation of numbers, mass, and weight. Classifies objects by several features and can place them in order. Formal Operational - (11 years and onward) - Logical thinking about abstract propositions. Concerned with the hypothetical and the future. Create hypotheses and test.

What is the first step in the planning phase of the nursing process?

Set priorities among nursing diagnosis

Identify the difference between long- and short-term goals.

Short-term goals are usually achievable within an immediate time frame for less than 1 week approx. Long-term goals take longer, sometimes weeks or months.

When should discharge planning begin for the patient in acute care?

Should begin when the patient is admitted to acute care facility and continue until he or she goes home.

For each of the sense, identify what information the nurse can gather. Sight - Hearing - Touch - Smell -

Sight - Physical appearance, movement, gait, coloration, skin integrity, fluid balance - moisture/dryness/edema, facial expressions, posture. Hearing - Inflection in the voice, wheezing, coughing, crying, cardiopulmonary and abdominal sounds Touch - Skin temperature and moisture, edema, discomfort, abnormal growths Smell - Body and breath odors associated with hygiene or pathology, bodily elimination.

SBAR

Situation Background Assessment Recommendation

For the widely accepted method of hand-off communication, SBAR, identify what the acronym means.

Situation: What is happening right now? Background: What led up the current situation? Assessment: What is the identified problem, concern, or need? Recommendations: What actions or interventions should be initiated to alleviate the problem?

Oral defamation of character

Slander

Provide an example of a risk factor reduction activity

Smoking cessation, beginning an exercise regimen, eating less salt and saturated fats, stress reducing activities, avoiding excessive alcohol consumption.

Determination of a patient's ability to swallow is commonly delegated to which member of the health care team?

Speech Therapist

What are the stages of Illness Model?

Stage 1 (Symptom experience) - A clinical manifestation of disease is experienced, and the person acknowledges that something is wrong and seeks a cure. Stage 2 (Assumption of sick role) - The person decides that the illness is genuine and that care is necessary Stage 3 (Medical Care Contact) - Professional advice from health care providers is sough by the individual. A professional health care provider identifies and validates the illness and legitimizes the sick role. Stage IV (4) (Dependent Patient Role) - The person, who is designated as a patient, usually undergoes treatment Stage V (5) (Perceived Recovery) - The patient abandons the sick role and resumes usual tasks and the roles to the greatest degree possible.

Iron administration nursing

Take 1 hour before meals with something acidic (vitamin C or orange juice) Stools will become tarry/blackened Drink liquid iron with a straw *stains teeth* Z-track method for parenteral iron Side effects: heartburn, constipation, diarrhea

What should the nurse do to prepare the patient and environment for the physical assessment?

The nurse should explain to the patient what will happen during the assessment, then prepare the environment for patient comfort, including privacy, appropriate temperature, and safe and comfortable examination area/table.

The nurse recognizes that the stethoscope most correctly represents which possible link in the chain of infection? A.) Source B.) Portal of exit C.) Portal of entry D.) Mode of transmission

The stethoscope would be a means for the pathogen to travel from source to host. The source is the reservoir or host. The portal of exit is where the pathogen escapes from the reservoir of infection, and the portal of entry is where the microorganism enters the susceptible host.

Encode

Translate their thoughts and feelings into communication with a receiver

Holistic

Treating the patient's physical, mental, emotional, spiritual, and social self

True/False: The Joint Commission requires the patient care plan be evaluated on a continual basis.

True

Gathering information to determine whether the information or data collected are factual and true

Validation

What is an action of carcinogens? a. Damage the DNA b. Increase the migration of cells c. Turn off oncogenes d. Stimulate viral activity

a. Damage the DNA

For a patient undergoing external radiation therapy, what do the nurse's instructions include? Select all a. Do not remove the markings b. Use lotions liberally to keep skin soft and moist c. Avoid direct skin exposure to sunlight for up to a year d. Use mild soap and water on the affected skin e. Gently rub treated areas to stimulate circulation f. Avoid wearing belts or clothing that binds the irradiated area

a. Do not remove the markings c. Avoid direct skin exposure to sunlight for up to a year d. Use mild soap and water on the affected skin f. Avoid wearing belts or clothing that binds the irradiated area

What is a sentinel event?

an unexpected occurrence involving death or serious physical or psychological injury, or the risk thereof

iron deficiency anemia

anemia resulting when there is not enough iron to build hemoglobin for red blood cells

If apoptosis is occurring within a patient's body, what is the expected outcome of this physiologic process? a. Rapid growth of malignant tumor metastasizing throughout the body b. Organs have an adequate number of cells at their functional peak c. Normal tissue continues to function in an abnormal place d. Cells will initially resemble parent cells but will rapidly mutate

b. Organs have an adequate number of cells at their functional peak

The patient has thyroid cancer and will be treated with injection of the radionuclide iodine-131 (brachytherapy). Which guideline is the most relevant when instructing unlicensed assistive personnel on how to assist the patient with hygiene and activities of daily living? a. Oncology Nursing Society practice guidelines b. American Cancer Society treatment guidelines c. Institutional evidence-based policies for infection control d. Institutional policies for handling body fluids and wastes

d. Institutional policies for handling body fluids and wastes

When is the best time to perform BSE? a. Day before menstrual flow is due b. Third day after menstrual flow starts c. Whenever ovulation occurs d. One week after menstrual period

d. One week after menstrual period

Polycythemia vera

general increase in red blood cells

what does it mean when a pt has a high neutrophil count?

infection

lymphangitis

inflammation of one or more lympahatic vessles

clinical assessment for hemophilia

internal and external bleeding hemarthrosis excessive blood loss from small cuts and dental procedures

Multiple Myleoma

malignant neoplastic immunodeficiency diease of bone marrow

clinical assessment of lymphedema

massive edema and tightness of effected extremeity and pain

How can critical thinking be used to meet the Quality and Safety Education for nurses standard for patient safety?

- Optimal patient management requires critical thinking through collaboration with all disciplines involved in the patient's care. Interdisciplinary clinical rounds are an effective approach to management of complex patient problems related to discharge planning, end-of-life decisions, and other ethical issues. - Critical thinking is used by RN to guide decisions related to delegation of assignments and tasks. Before delegation of a task, the nurse must be knowledgeable about the role, scope of practice, and competency of the person who is to receive the delegated task. - When developing preoperative plans of care, nurses use critical thinking, collaboration, and communication to develop individualized plans of care. Critical thinking helps the nurse to identify missing data.

What are the five key elements that are considered for nursing interventions?

- Patient assessment findings indicating signs and symptoms that have resulted from or in response to an illness or life experience. - The underlying etiology or related factor identified in each nursing diagnosis - Realistic patient outcomes in light of patient's health status and resources for improvement - Evidence based interventions signed with patient acceptance and practicality - Expertise of nurses and other health care professionals and agencies involved in the patient's care.

What considerations should be made before implementing patient care?

- Patient's education and health literacy level - Relevant cultural, religious and ethnic factors and limitations - Potential communication or language barriers - Patient abilities and condition status - Patient resources (support system and insurance coverage or ability to pay for treatment) - Patient treatment preferences - Nurse and health care provider experience and competency - Best practice research findings - Scope and standards of practice - Institutional resources

How does technology help in the development of critical thinking?

- Research indicates that students feel more confident and safer when they are permitted to use referencing software during clinical experiences. - Using technology allows students to connect related information, perceive its meaning, and guide nursing intervention planning and implementation. - Handheld technology supports clinical competency and helps students in the development of clinical decision-making skills.

Type IV hypersensitivity reaction

-Delayed hypersensitivity reactions -Ex: transplant rejection, tubercular caseous necrosis, contact dermatitis.

A group of nurses are planning to conduct free health check-up camps on weekends in rural areas with the support of a social welfare organization. Which characteristic of the nursing profession are these nurses exhibiting? A.) Altruism B.) Accountability C.) Autonomy D.) Ethical Compliance

A

A patient expresses to the nurse that she is fearful of the upcoming surgery. Which caring behavior is being used by the nurse this time? A.) Listening B.) Touch C.) Predictability D.) Consistency

A

An antibiotic mediation is administered to the patient. Shortly afterward, the patient develops itching and redness of the skin. If an antihistamine is given to counteract the antibiotics effect, the nurse is: A.) Compensating for adverse reactions B.) Preparing for a special procedure C.) Assisting with ADLs D.) Using preventative measures

A

For the patient who has chronic obstructive pulmonary disease with an excess of secretions in the bronchioles, which nursing diagnosis is most appropriate? A.) Incomplete airway clearance B.) Ineffective respiratory pattern C.) Potential for asphyxia D.) Difficulty maintaining spontaneous ventilation

A

What should the nurse include when writing an actual nursing diagnosis? A.) Diagnosis label, related factors, and defining characteristics B.) Diagnosis label and risk factors C.) Diagnosis label and related factors D.) Diagnosis label and defining characteristics

A

Which of the following is the best example of the measurable patient goal? The patient will: A.) Ambulate independently at least 20 feet in the hallway by the end of the week. B.) Be seen by the nurse for regular monitoring of blood pressure. C.) Increase intake of potassium-rich foods D.) Have less pain and anxiety

A

Which of the following patient behaviors specifically demonstrates self-efficacy? A.) Believing that the injections can be given each day B.) Denying that a medical condition exists C.) Living up to a perceived image as an ill person D.) Keeping a personal journal of health questions

A

A nurse is planning primary prevention activities. Which activity would the nurse include in this plan? A.) Safer sex education for teens B.) Mammogram screening C.) Medication compliance D.) Annual physical exams

A Primary prevention includes activities designed to prevent a disease or condition from occurring in the first place. Examples of primary prevention activities include vaccinations, wellness programs, good nutrition for health, and safer sex programs. Mammograms and physical exams are secondary prevention measures. Medication compliance would be tertiary prevention.

The nurse is caring for a patient who is admitted to the hospital with diabetic ketoacidosis. Which assessment finding indicates an attempt made by the patient's body to correct the pH? A.) The patient's respirations are very deep and rapid. B.) The patient's urine is dark and concentrated. C.) The patient's skin is pale, cool, and diaphoretic. D.) The patient is sleepy and difficult to arouse.

A The patient with diabetic ketoacidosis is in a state of metabolic acidosis. The body will attempt to compensate for the acidosis by blowing off extra amounts of carbon dioxide through deep, rapid respirations. Since carbon dioxide is converted to carbonic acid, removal of carbon dioxide will help shift the body's pH to a less acidotic state.

The nurse will be caring for a patient who is severely malnourished. Laboratory test results show that the patient's albumin level is critically low. What assessment finding will the nurse expect to note when assessing the patient? A.) The patient has generalized 3+ pitting edema. B.) The patient is confused and disoriented. C.) The patient's urine is dark and very concentrated. D.) The patient lung sounds are very diminished.

A The patient's low albumin level will lead to generalized pitting edema because there isn't enough protein in the blood to keep water within the bloodstream. Lack of oncotic pressure from low serum albumin leads to edema. The other findings are not related to malnutrition.

The nurse is caring for a patient with a history of left-sided congestive heart failure who is acutely short of breath. The nurse hears fine crackles throughout both lung fields and notes that the patient's pulse oximetry is only 88% on 4 L of oxygen. What is the priority intervention of the nurse? A.) Administer the ordered intravenous diuretic. B.) Prepare for insertion of a chest tube. C.) Suction secretions from the patient's respiratory tract. D.) Have the patient use the ordered incentive spirometer.

A The patient's respiratory distress is due to pulmonary edema and fluid overload from left-sided congestive heart failure. A diuretic will pull the excess fluid out of the body through the urine and relieve the patient's distress. A chest tube is not needed as the fluid is within the alveoli rather than between the lung and chest wall. Suctioning and use of an incentive spirometer will not address fluid overload or improve the patient's symptoms.

The nurse is caring for a patient who is taking narcotic pain medication after surgery. Which breakfast choices will help prevent constipation and promote return to regular bowel function? A.) Raisin bran with skim milk, fresh fruit, and wheat toast B.) Pancakes with maple syrup, bacon, and coffee with cream C.) Omelet with cheddar cheese, green pepper, and onions D.) Bagel with cream cheese, and strawberry nonfat yogurt

A The postoperative patient taking narcotic pain medications is at risk for developing constipation. A high-fiber diet with plenty of liquids will help prevent this from occurring. Raisin bran, fruit, and wheat bread are all good sources of fiber.

The nurse is caring for a patient with an indwelling urinary catheter caused by severe prostate enlargement. Which is the priority nursing diagnosis for this patient? A.) Risk for infection r/t indwelling urinary catheter B.) Disturbed body image r/t presence of catheter C.) Risk for contamination r/t potential leakage of urine on clothing D.) Impaired urination r/t blockage of bladder outlet

A The presence of an indwelling urinary catheter puts the patient at high risk for urinary tract infection, and this is the highest priority diagnosis for the patient. Disturbed body image is not as important as the risk of infection. Risk for contamination is not a nursing diagnosis. Impaired urination was corrected by placement of the urinary catheter.

The nurse has identified a research problem and knows what to be. What is the next step with this problem? A.) Conduct a literature review. B.) Address ethical procedures. C.) Collect data. D.) Analyze data.

A The research process involves many different components. The literature review is conducted after a research problem is identified. Ethical procedures must be addressed before the study begins. Data are then collected and analyzed before discussion of the research results.

The nurse understands the unique ability of the patient to understand and integrate health-related knowledge is known by which term? A.) Health literacy B.) Formal patient education C.) Informal patient education D.) Primary education

A The unique ability of a patient to understand and integrate health-related knowledge is known as health literacy. Formal patient education is delivered throughout the community in the form of media, in a variety of educational and group settings, or in a planned, goal-directed, one-on-one session with a patient in the acute care setting. Informal education is usually learner or patient directed. Many health care consumers begin receiving information as children through their primary education. Handwashing, proper dental care, and nutrition are examples of early instructions.

The nurse is preparing to teach a patient for the first time and needs to evaluate the health literacy of the patient. The nurse uses the VARK assessment to gather what information? A.) Assess the learning styles of the patient. B.) Find the one method that the patient uses to learn. C.) Be sure that the patient is a unimodal learner. D.) Reduce the need for creating a collaborative learning plan.

A Tools have been developed to help health care workers evaluate the health literacy of their patients. One such tool is the VARK (verbal, aural, read/write, kinesthetic) assessment of learning styles of people who are having difficulty learning. Individuals typically learn through more than one method. For example, a patient's VARK assessment may indicate learning through VAR or ARK. When the use of more than one style facilitates learning, the individual is considered a multimodal learner, meaning that the person does best when more than one teaching strategy is used or that the person is able to adapt to a variety of teaching strategies on the basis of what is being presented. Understanding how patients learn best makes collaborative learning plans most effective. It is good practice to provide multiple means of learning, because most individuals learn through more than one style and repetition enhances learning.

The nurse manager of a unit is sharing the most recent results of a patient satisfaction survey to motivate staff. This approach is a characteristic of what type of nursing leader? A.) Transformational B.) Transactional C.)Situational D.) Autocratic

A Transformational leaders use methods that inspire people to follow their lead. Transformational leaders work toward transforming an organization with the help of others sharing survey results may work to inspire staff. Transactional leaders use reward and punishment to gain the cooperation of followers. The authoritarian or autocratic leader exercises strong control over subordinates. Situational theories suggest that leaders change their approach depending on the situation

The nurse recognizes which statement by the patient indicates a teaching need? A.) "I use bobby pins to remove excessive ear wax." B.) "I use soap and a warm cloth to clean the outside of my ear." C.) "My doctor sometimes gives me oil drops for my ears." D.) "I never use Q-Tips."

A Washing the ear with a washcloth and soap is sufficient in most patients. If the patient has a buildup of wax, or cerumen, the health care provider may order special oil drops to soften the wax before irrigating the ear canal. Do not try to remove the wax using a cotton-tipped applicator because this can push the wax farther into the ear canal. Caution patients to never insert anything sharp into the ear, such as bobby pins. Sharp objects can rupture the tympanic membrane.

The nurse knows that a patient with a compromised cardiopulmonary system has a diminished capacity for exercise because of which conditions? (Select all that apply.) A.) Decreased tissue perfusion B.) Loss of sensation C.) Hemiparesis D.) Diminished respiratory capacity E.) Muscle weakness

A & D. Compromised cardiac function, decreased tissue perfusion, and diminished respiratory capacity directly affect a person's ability to perform activities of daily living (ADLs) and exercise. Hemiparesis and loss of sensation are associated with nervous system disorders. Muscle weakness can be from a number of causes.

The nurse is providing discharge education for the patient who is going home with a walker. Which statements by the patient indicate a good level of understanding of safety in the home? (Select all that apply.) A.) "I need to remove the throw rugs." B.) "I should make sure I only take a bath." C.) "I cannot use the stairs." D.) "I need to place a nonskid mat in front of the kitchen sink." E.) "I wish I had two ways of leaving the house."

A & D. To ensure patients do not have hazards that can cause falls at home, the nurse should evaluate where the living quarters are. If the patient has stairs, they need to be able to safely learn how to use the stairs. They need to remove throw rugs that are a trip hazard and place nonskid mats in front of sinks, tubs, and showers. They can shower with a bench or chair in the shower for sitting. Patients need a clear the exit so they can get out of the house quickly in case of an emergency, but do not specifically need two different exits because of the walker.

A nurse is studying intrinsic factors that influence the development of asthma in a community. What factors does the nurse assess? (Select all that apply.) A.) Socioeconomic status B.) Genetics C.) Pollution in the area D.) Water cleanliness E.) Immunization status

A & E Host, or intrinsic factors are individual variables such as genetics, age, gender, ethnic group, immunization status, and human behavior that impact a person's health. The other options are all extrinsic factors, which pertain to environmental characteristics.

A new nurse is conducting a patient interview. What behaviors observed by the experienced nurse require education on this process? (Select all that apply.) A.) Typing intently on a keyboard when asking questions B.) Allowing family to accompany the patient as requested C.) Using gestures and eye contact to demonstrate interest D.) Closing the door to the room to ensure privacy E.) Providing nonverbal cues to negative thoughts

A and E. During the interview process, the nurse needs to demonstrate interest in the patient by leaning slightly toward him/her, allowing requested family or friends to accompany the patient, and closing the door to the room to ensure privacy. Typing intently when the patient is talking can be interpreted as lack of interest. Providing nonverbal cues to the nurse's negative thoughts (such as scowling when the patient mentions something negative) does not promote comfort or trust.

A nurse understands that which characteristics of family dynamics impact a patient's sexuality? (Select all that apply.) A.) Religion B.) Age C.) Ethnicity D.) Culture E.) Geographic location

A, B, C and D. Religion, values, age, ethnicity, and culture all impact family dynamics, which in turn affect expressions of sexuality. Although geographic location can influence culture, it is not a specific family dynamic important to sexuality.

The nurse knows the following items should be included in the documentation of the patient on falls precautions. Select all that apply A.) History of any falls B.) Falls risk assessment scores C.) Patient and family education D.) Use of assist devices E.) Any fall or reported fall

A, B, C, D, and E

When the nurse is assisting patients with hygiene care, which tasks should be included? (Select all that apply.) A.) Bathing B.) Oral care C.) Perineal care D.) Foot care E.) Patient communication

A, B, C, D, and E. Hygienic practices include bathing, oral care, perineal care (cleansing of the genital area, urinary meatus, and anus), foot care, and shaving. During hygiene care the nurse communicates with the patient, assesses the skin, and observes for any abnormalities.

The nurse is completing documentation after feeding a patient with aspiration precautions. Which items should the nurse document? (Select all that apply.) A.) Episodes of coughing or gagging B.) Hesitation or fear of eating C.) Amount eaten D.) Aspiration protocol used E.) Respiratory status

A, B, C, D, and E. It is important to document thoroughly the patient's experience during the feeding so the other nursing staff will be aware of patient's needs including any episodes of coughing, gagging, or choking; respiratory status; hesitancy or fear of eating; and occurrences of nausea, vomiting, regurgitation, and/or reflux symptoms. The nurse would document the amount the patient ate, but this is not part of the required documentation for aspiration precautions.

The nurse manager of the unit is implementing a program to assist the nursing staff in managing compassion fatigue. Which interventions will be the most successful? (Select all that apply.) A.) Support group that nurses can participate in that meets on the unit B.) Exercise competitions to encourage nurse to exercise and log their time C.) Organized break times so nurses can get off the unit for breaks and lunches D.) Quiet area on the unit where the nurses can go during break E.) Promotion of work-life balance

A, B, C, D, and E. To care most effectively for others, nurses must first take time to care for themselves. Many of the stress reduction interventions incorporated into patient care plans can be effective in addressing the stressors faced by nurses. Exercise, balanced nutrition, and mindfulness therapy have been shown to help health care professionals in coping with the demands of patient care. Interventions designed specifically to prevent nurse burnout and address compassion fatigue include mentoring programs, quiet areas on a nursing unit for relaxation, availability of pastoral care, the sharing of feelings with trusted colleagues, and promotion of work-life balance.

In addressing patient education, the nurse recognizes that patient education is a process involving what components? (Select all that apply.) A.) Assessment B.) Diagnosis C.) Planning D.) Implementation and evaluation E.) Reliance on evidence-based practice (EBP)

A, B, C, and D. Assessment of health literacy occurs with each patient encounter. On completion of assessment, a nursing diagnosis relevant to the educational needs of the patient or caregiver can be determined. After working with the patient or caregiver to determine the appropriate nursing diagnosis, the next step is developing the patient education plan. In all patient education situations, a return demonstration by the patient (i.e., repeating what has been taught) helps the nurse to assess the level of learning that has taken place. Although evidence-based practice is important, it is sometimes insufficient when making patient care decisions.

The student studying culture learns that which are characteristics of all cultures? (Select all that apply.) A.) Integrated B.) Shared C.) Learned D.) Symbolic E.) Inherited

A, B, C, and D. Cultures are learned, symbolic, shared, and integrated. Since culture refers to patterns of beliefs, actions, values, and ways of life that are taught, they are not inherited.

When the nurse is performing a focused wound assessment on a patient, what information should be included in the documentation? (Select all that apply.) A.) Location and size B.) Characteristics of the wound bed C.) Patient's response to wound treatment D.) Patient's pain level E.) Presence of drainage

A, B, C, and E. A focused wound assessment includes an evaluation of the wound's location, size, and color; presence of drainage; condition of the wound edges; characteristics of the wound bed; and patient's response to the wound or wound treatment. The patient's pain level would be documented with his/her pain assessment.

The nurse is completing an assessment of an older adult and notices some cognitive impairment not normally associated with aging. Which of these alterations would prompt further follow-up? (Select all that apply.) A.) The patient does not remember where her son lives. B.) The patient is unable to balance her checkbook. C.) The patient got lost in a city she never traveled to before. D.) The patient often has difficulty remembering words. E.) The patient got lost going to her usual grocery store.

A, B, D, and E. Symptoms of cognitive impairment include disorientation, loss of language skills, loss of the ability to calculate, poor judgment, and memory loss. If a patient exhibits these symptoms, further investigation is needed. Some decline in cognitive function occurs with aging, such as the ability to navigate easily in new areas.

The nurse is caring for a patient who is hospitalized with cognitive impairment and recognizes which interventions will assist the patient in orientation? (Select all that apply.) A.) Keep a photo of the family in the room. B.) Use a clock on the wall. C.) Make sure the room is kept bright and well lit. D.) Avoid moving the patient from room to room. E.) Have each nurse introduce himself or herself to the patient.

A, B, D, and E. location or name of the hospital. Orientation to person, place, and time is ongoing. Staff members are always identified by name, both verbally and nonverbally (with a name tag). The patient's environment is kept as constant as possible and moving the patient from room to room is avoided. Some familiar objects, such as a family photo, are placed near the patient if the hospital stay is longer than a few days. The environment is kept free of distractions such as loud noises and bright lights. Natural lighting to provide the patient with orientation to time of day can be accomplished by opening blinds or curtains during the day and darkening the room at night.

Which of the following are being observed in society? Select all that apply, A.) A majority of countries indicate that they are experiencing a nursing and midwifery shortage. B.) The 65-year old and older population is rapidly increasing. C.) Developing countries are seeing the largest population growth. D.) People worldwide are living healthier lives. E.) Health care requirements of the world populations are decreasing.

A, B, and C.

The nurse assessing respirations understands that problems in what organs can directly affect the process of respiration? (Select all that apply.) A.) Brain B.) Lungs C.) Heart D.) Liver E.) Skeletal muscle

A, B, and C. Problems in the brain, heart, and lungs can directly lead to changes in respiratory rate and effort. Problems in the liver and skeletal muscle do not affect respirations directly.

The nurse should avoid soaking the feet of which patient population? (Select all that apply.) A.) Patients with peripheral vascular disease B.) Patients with a stroke C.) Patients with diabetes D.) Patients with arthritis E.) Patients who are malnourished

A, B, and C. Soaking the feet of patients with peripheral vascular disease, cardiovascular disease such as strokes and diabetes are contraindicated because it may cause skin breakdown or infection. Patient with arthritis or malnourished have no contraindications to having their feet soaked.

The nurse is preparing discharge instructions for a patient who has equilibrium alterations. Which instructions will the nurse include? (Select all that apply.) A.) Use grab bars in the tub and/or shower at home. B.) Keep rooms well-lit and focus ahead when walking. C.) Change positions quickly to avoid dizziness. D.) Use a cane or walker for stability. E.) Ride in the back seat of the car and look ahead.

A, B, and D. The patient experiencing dizziness or vertigo exercises caution when changing positions. The patient suffering from motion sickness needs to ride in the front seat of the car and look far ahead through the car windshield. Keeping rooms well-lit and focusing ahead when walking, using grab bars in the shower and/or tub, and using canes or walkers are all good safety measures. Changing positions quickly may lead to dizziness.

In addition to maintaining current professional practice knowledge, competent practice skills, and professional relationships with patients and their families, what additional actions should the nurse take to practice within the law? (Select all that apply.) A.) Maintain confidentiality. B.) Follow legal guidelines for sharing information. C.) Block document once per shift. D.) Change nursing procedures according to latest journal articles. E.) Meet licensure and continuing education requirements.

A, B, and E. In addition to maintaining current professional practice knowledge, competent practice skills, and professional relationships with patients and their families, nurses should follow guidelines to practice legally and avoid charges of malpractice, maintain confidentiality, follow legal and ethical guidelines when sharing information, document punctually and accurately, adhere to established institutional policies governing safety and procedures, comply with legal requirements for handling and disposing of controlled substances, meet licensure and continuing education requirements, and practice responsibly within the scope of personal capabilities, professional experience, and education.

Interpretation of nonverbal communication may require further validation with the patient. Which of the following should prompt further physical assessment? The patient: Select all that apply A.) Grimaces B..) Ambulates independently and purposely C.) Pulls away when touched D.) Responds incongruently to questions asked. E.) Sits with head down and shoulders slumped. F.) Maintains eye contact and nods during instruction.

A, C, D, and E.

The nurse is conducting a windshield survey. What items does the nurse assess? (Select all that apply.) A.) Types of housing available B.) Cars seen in parking lots C.) Recreational facilities D.) Health care facilities E.) Places of worship

A, C, D, and E. A windshield survey is a type of community health assessment. The nurse walks or drives through a neighborhood and notes the type of housing available, the presence and condition of recreational facilities, the presence of health care facilities, and places of worship among other items. Types of cars noted in the neighborhood are not one of the assessments.

The nurse recognizes which sleeping conditions are identified as dyssomnias? (Select all that apply.) A.) Difficultly getting to sleep B.) Nocturnal enuresis C.) Inability staying asleep D.) Being excessively sleepy E.) Falling asleep during the day

A, C, D, and E. Dyssomnias are disorders associated with getting to sleep, staying asleep, or being excessively sleepy. Nocturnal enuresis is a parasomnia.

The nurse is preparing some educational materials for her patient about the impact of obesity and a high body mass index (BMI). She knows that as BMI increases, so does the risk of these conditions. Select all that apply. A.) Increase in blood pressure B.) Increase in HDL C.) Increase in total cholesterol D.) Development of artherosclerosis

A, C, and D.

The nurse is helping a patient understand the difference between macronutrients and vitamins and minerals. The nurse identifies which items that should be included in the list of macronutrients? (Select all that apply.) A.) Water B.) Potassium C.) Starches D.) Fiber E.) Riboflavin

A, C, and D. The major nutrients, often referred to as macronutrients (nutrients that are needed in large amounts), include carbohydrates (sugar, starches, and dietary fiber). Water is also a macronutrient. Potassium is a mineral, and riboflavin is vitamin B2; these are micronutrients.

According to the healthy people 2020, health information and the associated access issues have become more complicated. There are many considerations when determining whether an individual has proficient health literacy. The patient should be able to: Select all that apply. A.) Read and identify credible health information B.) Recognize abnormalities on an x-ray C.) Navigate complex insurance program D.) Evaluate EKG findings E.) Advocate for appropriate care

A, C, and E

The nurse is working with a patient who does not speak English. And interpreters contacted and brought to the unit to relay information. Which of the following were correct when an interpreter is used for patient communication? Select all that apply A.) An interpreter should be obtained as soon as the need is identified B.) Interpreters for the deaf are not legally required under Title VI C.) Nurses need to follow hospital policy and procedures regarding interpreter use in patient care D.) Interpretation can never be provided by phone or video E.) Use short sentences and stop often to allow time for the interpreter to translate F.) Family members should always be used to interpret medical information

A, C, and E

Which statements by the nurse are correct regarding informed consent and someone who requires an interpreter? (Select all that apply.) A.) A professional interpreter is needed. B.) A family member may interpret when convenient. C.) Detailed medical information remains a priority. D.) Professional interpreters are not effective in providing medical information. E.) If necessary, family members can make decisions regarding informed consent.

A, C, and E If a patient is illiterate or requires an interpreter, the method of obtaining informed consent must be adapted appropriately. Use of a professional interpreter rather than a family member is essential to provide detailed medical information accurately. A patient whose culture prefers to allow other family members to make final health care decisions is inconsistent with nursing's ethical belief in autonomy. However, in this situation, the method of obtaining informed consent may need to be adapted to meet the patient's beliefs within the scope of the law.

A nurse is uncomfortable with a patient's comments, which are sexual in nature. Which actions by the nurse are most appropriate? (Select all that apply.) A.) Tell the patient to stop making sexual comments. B.) Try joking with the patient to establish rapport. C.) Tell the patient you are leaving and will return in a few minutes. D.) Inform the patient he/she can be sued for this behavior. E.) Explain to the patient how you feel about the comments.

A, C, and E. There are several steps a nurse can take when patients are displaying inappropriate sexual behavior. First, the nurse should tell the patient to stop what he/she is doing. The nurse can also tell the patient that he/she is leaving for a few minutes and will return when the patient can control this behavior. The nurse can also tell the patient how he/she feels about the comments or actions. The nurse should not joke with the patient or in any way participate in this behavior. The nurse should not threaten a patient with a lawsuit.

Which of the following nursing behaviors were found to be most important among critically ill intubated patients and their families and thought to contribute to recovery? Select all that apply. A.) Provision of information and reassurance. B.) Speed at completing skills. C.) Leaving them alone. D.) Offering guidance. E.) Use of a soothing tone of voice.

A, D, and E

Identify the appropriate interventions for the nurse who is assisting a patient through values clarification. Select all that apply. A.) Being aware of the protentional influence of the professional role. B.) Identifying the nurse's values in the situation. C.) Providing advice in difficult circumstances. D.) Sharing information to assist in decision making. E.) Avoiding direct confrontation and serious conflict. F.) Using a very stern and serious approach to identify consequences.

A, D, and E.

According to the Agency for Health Care Research and Quality, low health literacy is associated with which of the following? A.) Increased hospitalizations B.) Reduced emergency care use C.) Greater use of diagnostic procedures, such as mammography D.) Less hesitancy to receive vaccine

A.

On the basis of the following nurse diagnoses, identify a goal and outcome indicator(s) for each. A.) Nutritional intake greater than body requirements B.) Discomfort related to surgical incision

A. Body weight is the indicator. Patient will lose 1 pound per week. B.) Vital sign assessment - pain is the indicator. Patient will express a pain level of less than 3 on a scale of 10 following medication administration.

Which critical thinking error is present in the following situations? A.) Refusing to alter the procedure to accommodate the patient's preference for hygienic care B.) Performing a treatment without knowing about the technique C.) Deciding that the parents of a pediatric patient will be very meddlesome D.) Making an incorrect connection between the patient's respiratory problems and their cause

A.) Close Mindedness B.) Lack of Information C.) Bias and/or erroneous assumption D.) Illogical thinking

In relation to illness: A.) What factors influence an individual's response to illness? B.) What type of response may be exhibited by the patient

A.) Cognitive and emotional development, as well as the body's physiological reaction to stress. B.) Denial, anger, frustration, withdrawal, depression, and acceptance.

Your patient has been recently diagnosed with diabetes mellitus type 2. With no history of diabetes in his family, he is unfamiliar with his diet and treatments. He will require an oral hypoglycemic medication, as well as information about the prescribed diet. A.) What other information may be important to assess for this patient? B.) State a care map for this patient.

A.) Does the patient have any additional health problems? Will the patient be the person responsible for the medications and cooking? Is the patient motivated and able to learn? What is home environment like? Is the patient working? How will the change in diet and addition of daily medication influence his lifestyle? B.) Put the assessment from the situation into the map and show relationships between the data.

Revise the following statements to meet criteria for patient goals: A.) The patient will report feeling more comfort. B.) Urinary output will be satisfactory.

A.) Patient will verbalize a decrease in pain to less than 3 on a scale of 10 following administration of analgesic medication. B.) Urine output will be at least 30 mL/hour throughout the day.

Identify an example for each of the following caring behaviors: A.) Presence B.) Touch

A.) Presence - This is being with the patient/family at some of life's most important events such as a birth, death, in a preoperative room before surgery, or as they go through life-changing events. B.) Touch - The physical contact used by the nurse to demonstrate a connection and establish trust.

Safe practice requires that the nurse check the patients identity. What are two methods are commonly used to identify patients?

Asking the patient their name and birthday while cross-checking the armband.

Patient care data are gathered through observation and interviews

Assessment

A technique of listening with the assistance of a stethoscope, to sounds made by organs or systems such as the heart, blood vessels, lungs, and abdominal cavity.

Auscultation

According to Jean Piaget's theory, what does an infant in the sensorimotor stage do upon realizing that sucking results in a pleasing effect? A.) The infant stops sucking and starts kicking things and people. B.) The infant generalizes this action to other things like sucking fingers and blankets C.) The infant does not eat anything edible D.) The infant develops a good appetite.

B

During the storming face and small group development, the group leader? A.) Determines ground rules B.) Works with members to resolve conflicts C.) Encourages participation by all members. D.) Identifies the achievement of goals

B

For Baby Boomers, which of the following is a generational factor what may influence behavior? A.) Being slow to warm up B.) Expecting detailed information C.) Being very technologically literate D.) Having short attention span.

B

In a 5-tier triage system, what finding is designated as level 2 - emergent? A.) Cardiac arrest B.) Possible stroke C.) Dehydration D.) Abrasion

B

The nurse is concerned about aspiration precautions when feeding her patient who has recently suffered a stroke. Which of the following procedures that the nurse performs would demonstrate a need for further education? A.) The nurse uses thickened liquids B.) The nurse puts the bed at 30 degrees. C.) The nurse encourages slow eating D.) The nurse has the patient alternate between food and sips of fluid.

B

The nurse is concerned that the patient has developed atelectasis after surgery. Which of the following is an appropriate diagnostic label for this problem? A.) Insufficient airway clearance B.) Ineffective gas exchange C.) Diminished cardiac output D.) Lack of spontaneous ventilation

B

The nurse recognizes which action by the nursing student would be considered uncivil? A.) Prompt arrival to class B.) Texting during class C.) Attentive listening D.) Active participation in class

B Civility (i.e., acting politely) is essential in all interactions among faculty and nursing students. Respectful interaction between students and faculty members establishes professional communication patterns and affects the way in which students interact with patients. Texting in class is disrespectful and is an example of incivility. Arriving on time, listening attentively, and participating in class all show respect and civility.

The nurse identifies that The Code of Ethics for Nurses is defined in which terms? A.) Like the Constitution and not revisable B.) A succinct statement of ethical obligations C.) Required by entry level nurses only D.) A negotiable document dependent on individual conscience

B The current nursing code, the Code of Ethics for Nurses with interpretive statements, was published in 2015. The Code of Ethics for Nurses is "a succinct statement of the ethical obligations and duties of every individual who enters the nursing profession," the profession's "nonnegotiable ethical standard," and "an expression of nursing's own understanding of its commitment to society."

The nurse is caring for a patient who takes 6 tablets of methotrexate once every week on Fridays. How many mg of methotrexate does the patient take per dose? Trexall Methotrexate tablets, USP 2.5 mg tablets RX only A.) 10 mg B.) 15 mg C.) 20 mg D.) 25 mg

B 2.5 mg tablets × 6 = 15 mg.

A nurse is told in the hand-off report that a patient is afebrile. What assessment finding correlates with this statement? A.) Blood pressure 152/98 mm Hg B.) Temperature 98.4 °F (36.8 °C) C.) Apical pulse 82 beats/min D.) Respirations 16 breaths/min

B A temperature of 98.4 °F is normal. "Afebrile" means having a normal temperature. The other readings are not related to this term.

The nurse realizes that a medication error has been made. The nurse then reports the error and takes responsibility to ensure patient safety despite personal consequences. This nurse has exhibited what ethical concept? A.) Autonomy B.) Accountability C.) Justice D.) Advocacy

B Accountability is the willingness to accept responsibility for one's actions. Autonomy, or self-determination, is the freedom to make decisions supported by knowledge and self-confidence. Supporting or promoting the interests of others or doing so for a cause greater than oneself defines advocacy. To do justice is to act fairly and equitably.

During discharge teaching, the nurse is to give the patient a signed, dated, and timed prescription from the physician for medications to be taken at home. Which prescription drug order needs to be corrected before it is given to the patient? A.) Warfarin (Coumadin) 5 mg PO daily before dinner B.) Methotrexate (Trexall) 8 tablets PO once weekly on Saturdays C.) Levothyroxine (Synthroid) 137 mcg PO daily before breakfast D.) Zolpidem (Ambien) 5 mg PO at bedtime as needed for sleep

B All prescriptions must have the name of the drug to be administered along with dosage, route, and frequency. The methotrexate order does not contain a dosage for the drug, just the number of pills to be taken. The other orders are complete.

An emergency department (ED) manager wants to improve care for victims of sexual assault. What action by the manager is best? A.) Designate a private area of the ED for examinations. B.) Establish a SART team for the department. C.) Ask nurses to volunteer to be advocates for these patients. D.) Have victims examined immediately, rather than waiting their turn.

B Although all options would improve care for these patients, a SART (Sexual Abuse Response Team) approach offers comprehensive, evidence-based practice for these patients.

The nurse is caring for a patient recovering from knee replacement surgery. The patient complains of severe pain in the knee after receiving hydrocodone with acetaminophen (Vicodin) 2 hours previously. What is the nurse's best action? A.) Administer another dose of the medication. B.) Apply ice packs to the knee. C.) Apply heat packs to the knee. D.) Perform gentle range of motion.

B Application of cold decreases swelling and pain, produces local analgesia, and slows nerve conduction, which improves functioning. Examples of cold therapy are ice bags and cold compresses. The nurse should not administer another dose of medication without an order from the provider. Heat will increase blood flow to the area rather than reduce swelling. Gentle ROM will increase pain if done at this time.

A home health care nurse is working with the family of a patient who has Alzheimer disease and requires 24-hour care. What assessment by the nurse indicates the family is meeting an important goal for caregiver role stress? A.) Family eats dinner together every night. B.) Family uses respite care one night a week. C.) Family investigates research trials for patient. D.) Family verbalizes exhaustion from caregiving.

B Caregiver role stress can occur when the caregiver(s) is unable to meet obligations or unable to take care of personal needs. Using a respite caregiver once a week gives the family a little time off to accomplish needed tasks. The other observations are not tied to this diagnosis.

A nurse observes a student taking an adult patient's tympanic temperature. What action by the student requires the nurse to intervene? A.) Student washes hands prior to patient contact. B.) Student pulls the pinna of the patient's ear down and back. C.) Student explains the procedure to the patient. D.) Student pulls the pinna of the patient's ear up and back.

B For an adult, the correct procedure for taking a tympanic temperature includes pulling the pinna of the patient's ear up and back. Children's pinnae are pulled down and back. Washing hands and explaining the procedure are appropriate.

The nurse is caring for a patient who recently had a liver biopsy. To whom must the nurse give the results? A.) The patient B.) The patient's health care provider C.) The patient's insurance provider D.) The patient's spouse

B HIPAA protects the patient by requiring that testing results be shared only with health care professionals who need the information to provide treatment and with individuals designated in writing by the patient. The patient's provider will need the biopsy results to determine the patient's plan of care. The nurse does not give test results to the insurance company. The nurse may share the results with the patient or spouse, but it is not required, unless designated by the patient in writing.

The nurse is caring for a patient with diabetes who will be doing fingerstick blood glucose testing at home. What is the best way for the nurse to ensure that the patient can perform the procedure correctly? A.) Quiz the patient on the steps of the procedure. B.) Have the patient perform the procedure in front of the nurse. C.) Ask the patient if he has any questions about the test. D.) Use terminology that the patient can easily understand.

B Having the patient successfully perform the procedure in front of the nurse is an excellent way for the nurse to ensure that the patient knows how to do it correctly. Quizzing the patient about the procedure, asking the patient if he/she has questions, and using understandable terminology are fine, but only a return demonstration will assess the patient's ability to perform the procedure successfully and correctly.

The student nurse learns that which item is the most important symbolic aspect of culture? A.) Flags B.) Language C.) Art D.) Music

B Language is the most extensively used set of symbols in a culture. The other items are important symbols but are not as important as language because words are used to represent objects and ideas.

A nurse is completing an OASIS assessment on a patient. What data would be most important for the nurse to assess? A.) Presence of grocery stores nearby B.) Safety concerns within the home C.) Number and kind of pets D.) Proximity to a health care facility

B OASIS (Outcomes and Assessment Information Set) is a data set of outcome measures for adult home health care clients that is used to track outcome-based quality improvement. Factors that could potentially affect patient safety in the home are particularly important. The other options are not included in this assessment.

The nurse is caring for a patient who has a PCA pump following total hysterectomy surgery. The nurse sees the visitor push the PCA button while the patient is sleeping quietly. What is the best response of the nurse? A.) "Thank you for pushing the button for her to help keep her comfortable after surgery." B.) "Please do not push the button for the patient—she could receive more medication than she needs." C.) "You can push the button for her now, but please have her do it herself when she awakens." D.) "PCA pumps are great because she doesn't have to wait for me to administer her pain medication."

B Only the patient should operate the PCA and push the administration button. Family members and visitors should never activate the PCA pump for the patient because too much medication could be delivered, resulting in overdose and respiratory suppression.

A nurse is going to take an oral temperature on a patient who has just consumed a cup of coffee. What action by the nurse is best? A.) Have the patient drink room temperature water. B.) Return in 30 minutes to take the patient's temperature. C.) Take the patient's temperature rectally instead. D.) Document that temperature is unable to be obtained.

B Oral temperatures will be inaccurate if the patient has been drinking or eating hot or cold foods. The nurse instructs the patient not to continue drinking the coffee and returns in 30 minutes to take the temperature. Drinking room temperature water will not "even out" the patient's mouth temperature. The rectal route is not preferred by patients and should not be used in this situation. The nurse needs a temperature and so should not document that it was not obtained.

The nurse identifies which patient to be best suited for PCA analgesia? A.) A patient who is confused after a head injury B.) A patient recovering from total hysterectomy surgery C.) A patient who has severe psychogenic pain D.) A patient with arthritis who is unable to push the nurse call button

B Patients recuperating from surgery are often good candidates for PCA analgesia. Confusion, inability to push the PCA button, and psychogenic pain are all contraindications for PCA analgesia.

The student nurse is assessing a patient's pulses. What action by the student requires the nurse to intervene? A.) Assessing apical pulse between the fifth and sixth intercostal spaces B.) Assessing the dorsalis pedis pulse by palpating behind the patient's knee C.) Assessing the radial pulse on the patient's wrist D.) Assessing the brachial pulse on the patient's inner elbow

B The dorsalis pedis pulse is palpated on the top of the foot. The other assessment locations and pulses are correct.

The nurse learns that spermatozoa are produced in which sexual organ? A.) Scrotum B.) Testes C.) Glans D.) Prostate

B The male testes produce spermatozoa and the male hormone testosterone.

The nurse administers a medication to a patient. Shortly afterward, the patient develops an itchy rash over the entire body and reports feeling very unwell. What is the priority action of the nurse? A.) Leave the patient to notify the provider and the pharmacist. B.) Determine if the patient is having any difficulty breathing. C.) Document the reaction in the patient's chart. D.) Obtain an order for hydrocortisone cream to relieve the itching.

B The nurse must first determine if the patient is having any difficulty breathing, since the patient may be starting to have an anaphylactic reaction to the medication, which can lead to shortness of breath and airway swelling. After assuring that the patient is stable, the nurse can notify the appropriate personnel and request any treatments for the reaction.

The nurse is preparing to give a patient a complete bed bath. What area of the body should be bathed first? A.) Hands B.) Eyes C.) Face D.) Arms

B The nurse should start washing the patient's eye area, using a washcloth without soap, followed by the patient's face, hands, and arms.

The nurse is caring for a patient who will self-administer medication injections at home after discharge. How can the nurse best determine that the patient understands the technique and can administer the injections correctly? A.) Provide written instructions about how to administer the injections. B.) Watch the patient self-administer an injection. C.) Call the patient the next day to ask if there is any difficulty with administering the injections. D.) Ask the patient to express understanding as to how to administer the injections.

B The nurse should watch the patient self-administer an injection to make sure that the patient is doing it correctly. This will give the nurse an opportunity to point out and correct any mistakes and offer the patient reassurance about the technique.

The nurse is caring for a patient who is taking many prescription medications for various health problems. Which direction from the nurse will help the patient avoid dangerous drug interactions? A.) Only take over-the-counter medications. B.) Have all of the prescriptions filled at the same pharmacy. C.) Avoid taking generic preparations of prescribed medications. D.) Only take the medications that the patient feels are necessary.

B The patient's risk for dangerous drug interactions is increased when many medications are taken. Filling all the prescriptions at the same pharmacy will allow the pharmacist to check for possible interactions.

After administering an antibiotic to the patient, the nurse notes the patient complaining of feeling ill, is scratching and has hives. The patient soon starts having difficulty breathing and is hypotensive. What is the nurse's assessment of the situation? A.) The patient is having a mild allergic reaction and an antihistamine will make the patient feel better. B.) The patient is having an anaphylactic reaction and epinephrine should be administered right away. C.) The patient's infection is worsening and progressing to septic shock so blood cultures should be drawn. D.) The patient has developed toxic shock syndrome and the antibiotic orders must be changed right away.

B The patient's symptoms are indicative of anaphylaxis: a severe, life-threatening allergic reaction. The airways close, the throat swells closed, and the blood pressure drops dangerously low. The patient may go into shock and die. This is a medical emergency. Anaphylaxis can occur immediately after the administration of medication and can be fatal. Treatment includes immediate discontinuation of the drug and administration of epinephrine (an antagonist), intravenous (IV) fluids, steroids, and antihistamines while providing respiratory support. Patients may have very mild allergic reactions to medications and experience a rash or itching. This patient is not developing septic shock or toxic shock syndrome.

The nurse is caring for an elderly patient with a history of arthritis, urinary incontinence and poor perineal hygiene practices. The patient has had four urinary tract infections in the past year. Which is the priority goal for the nursing diagnosis Impaired health maintenance for this patient? A.) The patient will be provided with educational materials about risks of urosepsis. B.) The patient will allow family members to assist with daily bathing and perineal care. C.) The patient will discuss the possible consequences of frequent UTIs. D.) Regular home care nursing visits and follow-up telephone contact will be arranged.

B The priority for this patient is to improve personal hygiene and perineal care in order to reduce the risk of future urinary tract infections. The patient's agreement to allow family members to assist with bathing and perineal care will greatly reduce this risk. Providing educational materials about the risk of urosepsis, discussion of UTI consequences, and regular follow-up care are interventions rather than patient goals.

The nurse is caring for four patients. Which one should the nurse assess for spirituality needs as a priority? A.) New mother, older child at home. B.) Faces terminal diagnosis. C.) Needs to change medications. D.) Pleasant but quiet.

B There are many cues to alert the nurse that a patient might have unmet spiritual needs, including facing a terminal illness. The nurse should conduct spiritual assessments on all patients, but this one is the priority.

The nurse identifies which medications that are to be administered via parenteral routes? (Select all that apply.) A.) Bisacodyl (Dulcolax) 10 mg suppository daily PRN constipation B.) Prochlroperazine (Compazine) 10 mg IM q 6 hours PRN nausea C.) Brimonidine (Alphagan) 0.1% solution 2 drops to each eye daily D.) Proventil (Ventolin) inhaler 2 puffs as needed for shortness of breath E.) Fentanyl (Duragesic) 50 mcg transdermal patch apply every 72 hours F.) Insulin lispro (Humalog) insulin 15 units subcutaneously ac meals

B and F. Parenteral medications are administered by injection into tissue, muscle, or a vein rather than through the gastrointestinal or respiratory route.

The nurse is caring for a postoperative orthopedic patient who has two Hemovac drains in place. Which interventions will the nurse perform? (Select all that apply.) A.) Measure the amount of drainage in the device prior to emptying. B.) Label each drain and record them separately. C.) Recompress the device after emptying. D.) Secure the device to the patient's gown above the level of the wound. E.) Check for kinks in the tubing.

B, C and E. Use a marked, graduated measuring device to collect the drainage when emptying the reservoir to facilitate accurate measurement of the drainage. After emptying, recompress the device to maintain suction. Secure the container(s) to the patient's hospital gown below the level of the wound, avoiding tension on the tubing and making sure there are no kinks. If there are multiple drains, label them and document observations by the drain label.

The nurse is bathing a patient and notes reddened skin above the coccyx. Which action by the nruse is appropriate? Select all that apply A.) Apply a barrier cream and massage the area B.) Document and describe the area and report to the physician C.) Wash and dry the area and position without pressure on coccyx D.) Report the area to the charge nurse

B, C, and D.

A nurse holds the hands of a patient when talking to her during rounds. What might be the impact of the nurse's behavior on the patient? A.) The patient's caregivers feel embarrassed B.) It enhances the self-esteem and mental health of the patient. C.) The patient feels isolated D.) It enhances the patient's physical discomfort.

B.

For the following health models, identify the primary concept: Basic Human Needs Health Belief Model Holistic Health

Basic Human Needs - Interrelationship between the elements of basic requirements for survival and the desires that drive personal growth and development. The model is most often presented as a pyramid consisting of five levels - the lowest level is related to physiological need, whereas the upmost level is associated with self-actualization Human Belief Model - Explores the relationship between patient attitudes and beliefs and how those factors predict health behavior. There are three primary components 1) perception of susceptibility to illness 2) perception about the seriousness of the illness and 3) probability that the individual will act to prevent avoidable health risks Holistic health - Synergistic relationship exists between the body and the environment. Holistic are is not solely a method of healing but also an approach for how the therapies can be applied. Holistic concepts focuses on the inter relatedness of the physical body, and the psyche. Incorporating features of spirituality, emotional security, and environment into natural, holistic care supports the premise that the body knows how to heal itself given the proper support system needed to do so.

Perceived Benefits

Beliefs about the effectiveness of taking action to reduce risk or seriousness. Change strategies - Explain how, where, and when to take action and describe likely positive results.

Perceived Barriers

Beliefs about the material and psychological costs of taking action Change strategies - Offer reassurance, incentives, and assistance; correct misinformation or allegations.

Perceived Severity

Beliefs about the seriousness of a condition and it's consequences. Change strategies - Specify the consequences of a condition and recommend action or intervention.

In the stages of Illness Model, stage II is characterized by the individual: A.) Accepting medical treatment B.) Seeking medical advice C.) Deciding that care is necessary D.) Recognizing that something is wrong.

C

The nurse has assisted the patient to wash hands, face, axillae, and perineal area. What type of bath should the nurse chart? A.) Sink Bath B.) Complete Bed Bath C.) Partial Bed Bath D.) Shower

C

The nurse is educating the patient about the proper disposal of medications in the home. Which statement by the patient indicates she has a good understanding of the information? A.) "Remove the label from the bottle and throw in the trash." B.) "Flush the medication." C.) "Mix the medications with kitty litter and place the mixture in a jar and put the jar in the trash." D.) "Dissolve the medication in water and pour down the drain."

C

The nurse is explaining to the UAP that the patient is on a full-liquid diet. Which statement by the UAP indicates a need for reorientation? A.) "I can give the patient orange juice." B.) "I can give the patient yogurt." C.) "I can give the patient oatmeal." D.) "I can give the patient milk."

C

The nurse is preparing to teach a 90-year-old patient. In teaching an elderly patient, the nurse realizes that: A.) Most elderly patients are highly literate B.) Cognitive abilities always decline with age C.) Sensory alterations often occur with aging D.) Teaching methods are the same as for the middle aged

C

A patient asks the nurse to recommend a nonprescription contraceptive. What options does the nurse discuss? A.) Diaphragm B.) Cervical cap C.) Condom D.) Intrauterine device

C The condom is the only option listed that is nonprescription.

The nurse is educating the patient about the use of heat/cold therapy at home. Which statement by the patient indicates the need for further education? A.) "I should fill my ice bag 2/3 full of ice." B.) "I should use distilled water in my Aqua-K pad." C.) "I can warm up my hot pack in the microwave." D.) "I should check the order for how long to leave the compress on."

C Warm compresses and water for soaks should not be heated in the microwave unless the product and microwave are specifically designed for this type of heating. Ice bags are filled two-thirds full, distilled water is used in Aqua-K pads, and application time for heat is as stated in the PCP order (for cold, it is a maximum of 20 to 30 minutes).

The nurse is acting in the role of patient advocate. This role specifically includes: A.) Motivating others toward a common goal B.) Incorporating research into their practice. C.) Communicating with the patient's wishes to other health care providers. D.) Bringing about change in the legislation on health policy issues.

C.

What effect could a patient's disability have on determining goals?

Can influence the way or the time frame in which the goals can be achieved.

The nurse wants to know if there is fluid in the abdominal cavity. What physical assessment techniques should be used?

Can use inspection and palpation to see and feel whether the patient's abdomen is distended. Percussion may also be used to hear if there is fluid present.

Identify one concept included in the theories from each of the following theorist: Watson

Caring; with nurses dedicated to heath and healing.

How does certification differ from licensure?

Certification is a voluntary process in a specialty area. Licensure is mandatory in order to practice.

Organizing patient assessment data into groupings with similar etiologies

Clustering

Identify the errors that the nurse should avoid when formulating a nursing diagnoses.

Clustering of unrelated data, accepting erroneous data, using medical diagnosis as related factors within the nursing diagnosis statement, missing the true underlying etiology of a problem, and identifying multiple nursing diagnosis labels in one diagnostic statement.

Identify the different nursing roles (e.g., advocate) that are identified with the following situations: RNs, UAPs, LPNs, PCP, social workers, clergy, and therapist all interact productively to provide quality patient care.

Collaborator

Identify at least three common concepts from the definitions of nursing from the American Nurses Association, International Council of Nurses, and Virginia Henderson.

Common concepts include protection, promotion, and optimization of health and abilities; prevention of illness; and care of the ill, disabled, and dying.

A goal for a patient who is hypertensive is to return to expected limits. Which one of the following outcome indicators is most appropriate? A.) Patient expresses decreased discomfort q3h B.) Patient identifies two things that reduce stress C.) Patient will not experience headaches D.) Patient's blood pressure is between 120/80 mm Hg and 130/90 mm Hg.

D

A patient is admitted to an acute care area. The patient is an active business man who is worried about getting back to work. He has had severe diarrhea and vomiting for the last week. He is weak, and his breathing is labored. Using Maslow's hierarchy of needs, identify this patient's immediate priority. A.) Self-actualization B.) Esteem and self-esteem issues C.) Safety D.) Air, water and nutrition

D

According to the health promotion model, one of the focus points is: A.) Complementary therapy B.) Basic survival requirements C.) Interrelationship of the mind and body D.) Behavior-specific knowledge

D

Conveying acceptance may be indicated by the nurse responding with: A.) "Go on." B.) "What is your greatest concern?" C.) "You appear tired." D.) "I follow what you are saying."

D

During the termination phase of the patient interview, the nurse does which of the following? A.) Prepares the environment B.) Performs the physical examination C.) Sets goals with the patient for care D.) Summarizes and validates information from the patient

D

For a patient with a nursing diagnosis of Restricted Physical Mobility Related to Bilateral Arm Casts, the nurse should select which of the following direct care interventions? A.) Counseling B.) Teaching C.) Reassessment D.) Assisting with ADLs

D

For the health promotion nursing diagnosis Ability to Comprehend the Need For Enhanced Nutrition, which of the following is the most appropriate defining characteristic? A.) Inability to feed self B.) Diminished oral intake C.) Reduction in body mass and strength D.) Identification of healthy food choices

D

For the process of reflection, the nurse asks himself which of the following? A.) "How should I report the increase blood pressure reading?" B.) "Why is the patient having pain now?" C.) "Did the patient's respiratory status just change?" D.) "How should I have taught the patient to do self-injection more efficiently?"

D

The new nursing staff member is observed by the unit manager during a patient interview. Which of the following should the manager identify to the new nurse to avoid in the future? A.) Using a moderate tone of voice B.) Sitting close and leaning toward the patient. C.) Asking open-ended questions D.) Tapping her pen on the bedside table

D

The nurse is caring for a patient who will be receiving iodine-based contrast medium for a CT scan. Which allergy should be reported to the technician and radiologist before the test is performed? A.) Gluten and lactose B.) Strawberries C.) Peanuts and cashews D.) Shrimp and scallops

D

The nurse is caring for an elderly patient with dementia. Which laboratory finding indicates to the nurse that the patient is often forgetting to eat meals? A. Serum bilirubin 0.4mg/dL B.) PLT (platelet count) 425,000/mm^3 C.) Serum cholesterol 175mg/dL D.) Albumin 1.4g/dL

D

The nurse tells the patient he will stay until the physician comes back for the results of the diagnostic test. This is an example of which communication technique? A.) Using focused comments B.) Providing general leads C.) Validating D.) Offering self

D

The nurse is caring for an elderly patient with dementia. Which laboratory finding indicates to the nurse that that patient is often forgetting to eat meals? A.) Serum bilirubin 0.4 mg/dL B.) PLT (platelet count) 425,000/mm3 C.) Serum cholesterol 175 mg/dL D.) Albumin 1.4 g/dL

D Albumin level is an indicator of the patient's protein intake and nutritional status. Normal albumin level is 3.3 to 5 g/dL. It is an essential component of fluid balance, responsible for maintaining colloidal oncotic pressure in the vascular and extravascular spaces. Low levels of albumin may indicate malnutrition.

The patient is reportedly well educated and employed as an engineer but is struggling to comprehend terms found in health-related literature given to explain his disease process. The nurse recognizes that this is evidence of what issue? A.) Low literacy B.) Psychomotor dysfunction C.) Affective domain deficiency D.) Low health literacy

D Although low literacy and low health literacy are related terms, they are not interchangeable. Low health literacy is content specific, meaning that the individual may not have difficulty reading and writing outside the health care arena. These patients may struggle to comprehend the complicated, unfamiliar terms and ideas found in health-related materials or instructions. The psychomotor domain incorporates physical movement and the use of motor skills in learning. Teaching the newly diagnosed diabetic how to check blood sugar is an example of a psychomotor skill. Affective domain learning recognizes the emotional component of integrating new knowledge. Successful education in this domain takes into account the patient's feelings, values, motivations, and attitudes.

The nurse understands an institutional review board (IRB) is a review committee established to carry out what task? A.) Approve research involving animal subjects. B.) Approve research that is not government funded. C.) Function differently than scholarly journals do. D.) Protect the rights of human research subjects.

D An institutional review board (IRB) is a review committee established to help protect the rights and welfare of human research subjects. Regulations require IRB review and approval for research involving human subjects if it is funded or regulated by the federal government. Most research institutions, professional organizations, and scholarly journals apply the same requirements to all human research. The IRB must approve the research and procedure for data collection from human subjects.

The nurse is explaining to the patient why antibiotics are being administered. The answer would be correct if the nurse stated antibiotics are effective against which microorganism? A.) Viruses B.) Fungi C.) Parasites D.) Bacteria

D Antibiotics are effective against bacteria, and exact antibiotic sensitivity is tested so that appropriate antibiotics a re prescribed. Infections that are caused by fungi are treated with antifungal medications. Certain antiviral medications are used to manage the symptoms of a viral infection. These medications, if given during the early phases of illness, can decrease the amount of time that the patient has viral symptoms. Treatment for parasitic infections varies depending on type of parasite.

When providing the patient with routine hygienic care, which action would the nurse omit? A.) Massage the back with lotion B.) Oral care with a toothbrush C.) Shaving with a disposable razor D.) Ear hygiene with cotton-tipped applicators

D Cotton-tipped swabs or applicators should not be used in the ears for cleaning because this can push wax farther into the ears. A back massage may be given as part of a complete bed bath. Oral care is an essential nursing intervention that provides patient comfort, removes plaque and bacteria, reduces the risk of tooth decay, and decreases halitosis. Oral care includes brushing the teeth and tongue, flossing, rinsing the mouth, and cleaning dentures. Shaving a patient may be part of hygienic care and can be done with a disposable or electric razor.

The nurse is caring for a patient who has had many admissions and readmissions. The nurse believes that the patient keeps coming to the hospital because the patient "wants his drugs," and is "non-compliant" at home with diabetic therapy. To reduce the risk of slander against this patient, the nurse should carry out which action? A.) Write opinions in the medical record only. B.) Never share observations. C.) Make judgmental statements in private. D.) Avoid making judgmental statements.

D Defamation of character occurs when a public statement is made that is false and injurious to another person. Oral defamation of character is slander. Slander is spoken information that is untrue, causing prejudice against someone or jeopardizing that person's reputation. The nurse should not make opinionated, slanderous comments about patients, orally or in writing. Written forms of defamation of character are considered libel.

A nursing student wants to observe enculturation practices of an ethnic minority community. What action by the student is best? A.) Attend a community dance. B.) Learn to cook an ethnic meal. C.) Visit the group's worship service. D.) Observe a grandmother teaching a child.

D Enculturation is the process of passing a culture down from generation to generation. Culture can be taught directly, for instance, with the grandmother teaching the child. Culture can also be taught indirectly as when a child observes a role in the community. The student observing the grandmother teaching a child is the best example of enculturation.

What response would the nurse provide to correctly identify the most effective method to prevent hospital-acquired infections? A.) Use of sterile technique B.) Isolation protocols C.) Antibiotic use D.) Handwashing

D Handwashing is the most effective method to prevent hospital-acquired infections. Sterile technique is only used for certain procedures and isolation protocols are used for patients already infected or for protective isolation in immune-compromised patients and are not used for every patient. Antibiotics are used to treat infections.

The nurse displays an understanding of high-risk populations for MRSA when identifying which group as the lowest risk? A.) Prison inmates B.) College dorm residents C.) Team athletes D.) Food service workers

D High-risk populations for MRSA include those living in close quarters or those who have frequent skin-to-skin contact, including prison inmates, college dorm residents, and team athletes. Food service workers work together but do not generally live in close quarters or have skin-to-skin contact frequently.

What statement by the nurse is true regarding oral care of patients on anticoagulants? A.) Use an electric toothbrush daily. B.) Avoid oral care. C.) Use mouthwash only. D.) Use a soft-bristled toothbrush.

D Oral care is important regardless of medication, but a soft-bristled toothbrush should be used related to increased risk of bleeding for any patient on an anticoagulant. An electric toothbrush is too aggressive, and mouthwash is not adequate.

The nurse knows which method to be an appropriate way to tie restraints? A.) Knot tied to the bed frame B.) Quick-release knot tied to the side rail C.) Bow tied to the bed rail D.) Quick-release ties attached to the bed frame

D Restraints should never be tied in a knot because the knot may prohibit a quick exit in the event of an emergency requiring evacuation. Instead, use quick-release ties or mechanisms such as buckles. Restraints are never be tied to side rails because injuries may result when they are raised or lowered. They should be tied to a stable part of the bed such as the frame.

The nurse is caring for a patient who is about to have surgery. Which intervention will be included in the patient's care to meet the goals for risk for perioperative positioning injury related to immobilization during surgical procedure? A.) Use adequate assistance to move patient onto the OR table. B.) Watch for early signs of hypovolemia caused by patient's NPO status since midnight. C.) Use therapeutic touch and guided imagery to allay patient's fears of surgery. D.) Pad all bony prominences and avoid hyperextension of extremities.

D Risk for perioperative positioning injury is addressed by ensuring that the patient's skin and bony prominences are well padded during the surgery. In addition, hyperextension of extremities may lead to joint damage, so this should be avoided as well. The other interventions are appropriate for perioperative care but do not relate directly to the Nursing diagnosis of positioning injury potential.

The nurse manager would counsel the staff nurse for delegating which task to the UAP? A.) Personal hygiene B.) Assistance with eating breakfast C.) Assistance with toileting D.) Interpretation of abnormal vital signs

D The RN must remember to delegate tasks that do not require nursing judgment. Interpretation of abnormal vital signs requires assessment skills possessed by the RN only. Only tasks that are routine and do not require variation from a standardized procedure, such as providing hygiene, assisting with eating, and toileting, should be delegated.

The nurse has implemented a community-wide immunization program for seasonal influenza. Once the program has ended, what action by the nurse is best? A.) Begin planning for next year's program. B.) Send mail surveys to participants. C.) Determine financial gains or losses. D.) Evaluate the program and outcomes.

D The last step of the nursing process is evaluation. The nurse should evaluate the program to see if interventions had the desired effect. Evaluation could include surveys or looking at financial outcomes, but those are only limited aspects of the process. Planning for next year's event should not occur until after evaluation has been completed.

Which collaborative team member would be most effective in assisting the nurse to identify medication alternatives that are less likely to cause drowsiness and dizziness to reduce the risk of falls in the elderly patient? A.) Nursing case manager B.) Charge nurse C.) Physical therapist D.) Pharmacist

D The nurse collaborates with the pharmacist and health care provider to identify and implement safe medication alternatives for older adults to minimize side effects such as drowsiness, dizziness, and orthostatic hypotension, which can increase fall risk. Although case managers and charge nurses might have some experience in this area, pharmacists are educated to focus on medication. Physical therapists evaluate the patient's ability to perform and maintain balance during routine activities such as sitting, standing, and walking.

The nurse anticipates correctly that what medication category would be ordered to treat athlete's foot? A.) Antiviral B.) Antibiotic C.) Antihelminth D.) Antifungal

D The nurse would expect to treat athlete's foot with an antifungal because it is a fungal infection. An antibiotic treats bacterial infections, antivirals treat viral infections, and antihelminth treats parasitic worms.

The nurse is caring for a patient who was brought to the ED after overdosing on narcotic pain medication. The patient was found unresponsive with no respirations. Arterial blood gases were drawn shortly after the patient's arrival to the hospital. Which results will the nurse expect to see? A.) pH 7.56, PaCO2 32 mm Hg, HCO3 32 mEq/L, PaO2 90 mm Hg B.) pH 7.35, PaCO2 45 mm Hg, HCO3 26 mEq/L, PaO2 70 mm Hg C.) pH 7.45, PaCO2 38 mm Hg, HCO3 28 mEq/L, PaO2 80 mm Hg D.) pH 7.27, PaCO2 58 mm Hg, HCO3 24 mEq/L, PaO2 60 mm Hg

D The patient who overdosed on narcotic pain medication will be in respiratory acidosis due to respiratory suppression. Low pH of 7.27 and elevated PaCO2 are consistent with respiratory acidosis as insufficient carbon dioxide is removed from the blood. The low 60 mm Hg PaO2 is due to insufficient oxygen intake.

The nurse is working with a diabetic patient and is attempting to teach psychomotor skills. This is occurring when the nurse has the patient complete what action? A.) Verbally describe his feelings about diabetes. B.) Answer three of five true-or-false questions about diabetes. C.) Identify three positive lifestyle changes to manage blood sugar. D.) Draw up and self-inject insulin correctly.

D The psychomotor domain incorporates physical movement and the use of motor skills in learning. Teaching the newly diagnosed diabetic how to check blood sugar is an example of a psychomotor skill. Learners in the cognitive domain integrate new knowledge through first learning and then recalling the information. They then categorize and evaluate, making comparisons with previous knowledge that result in conclusions related to the new content. Affective domain learning recognizes the emotional component of integrating new knowledge. Successful education in this domain takes into account the patient's feelings, values, motivations, and attitudes.

A patient is scheduled to have an MRI and has a metal religious icon pinned to his gown, which can't go in the scanner. What action by the nurse is best? A.) Take the icon off the patient's gown until she returns. B.) Give the icon to the patient's family for safekeeping. C.) Pin the icon to the patient's pillow so it can go to radiology. D.) Explain the restriction and ask the patient's preference.

D The religious icon has profound significance for the patient and should not be removed by the nurse. Since the icon cannot go into the MRI scanner itself, the nurse should explain the situation to the patient and get the patient's opinion of various options. All other options are possibilities, but it should be the patient's determination.

A hospitalized patient complains of bilateral leg pain and asks the nurse to massage her legs. One calf is noticeably larger than the other and is warm and slightly reddened. What action by the nurse is best? A.) Only massage the leg with normal assessment findings. B.) Massage the front of both legs and avoid the posterior surfaces. C.) Perform a Homan's test to both legs prior to massaging either of them. D.) Educate the patient on why a massage would be contraindicated.

D This patient has manifestations of a deep vein thrombosis, and the nurse should not massage the patient's legs. The nurse should inform the patient of why this is contraindicated. The other actions are not warranted.

The nurse is caring for a patient with high cholesterol who has been prescribed atorvastatin (Lipitor). Which laboratory result indicates that the patient has been taking the medication as ordered and following the physician's dietary recommendations? A.) Serum triglyceride level 325 mg/dL B.) High-density lipoproteins (HDL) 56 mg/dL C.) Low-density lipoproteins (LDL) 155 mg/dL D.) Total cholesterol level 185 mg/dL

D Total cholesterol levels should be less than 200 mg/dL, so a cholesterol level of 185 mg/dL indicates that the patient has been compliant with the prescribed therapy. The other laboratory results are abnormal and would not indicate compliance.

The nurse knows the following types of wounds heal by tertiary intention: A.) An acute wound in which the patient has sutures placed when it happened. B.) A pressure ulcer that was treated with dressing changes and healed C.) An acute wound in which surgical glue was used to close the wound D.) A wound that was left open initially and closed later with sutures.

D When a delay occurs between injury and closure, the wound healing is said to happen by tertiary intention. Wounds such as surgical incisions or traumatic wounds in which the edges of the wound can be approximated (brought together) to heal are examples of acute wounds. This type of wound is said to heal by primary intention. When a wound heals by secondary intention, the new tissue must fill in from the bottom and sides of the wound until the wound bed is filled with new tissue such as a pressure ulcer.

Identify all of the following behaviors that are associated with stage IV in the stages of illness model. Select all that apply. A.) Recognition that something is wrong B.) Resumption of usual tasks C.) Acceptance of the sick role D.) Performance of treatments E.) Validation of the illness F.) Requirement of emotional support

D and F.

Which of the following statements made by the nurse are therapeutic? Select all that apply. A.) "Everything will be okay." B.) "I don't agree with what you want to do." C.) "It's for your own good." D.) "Have you made a decision about accepting treatment?" E.) "You seem happy about being discharged today." F.) "So, there are two areas that you want more information about - your medications and diet."

D, E and F.

SLE Nursing Management

Daily I&O 24 hour urine (measure protein/creat clearance) Assess for s/s of bleeding Neuro checks (including psychosis r/t steroids)

What are the usual components of a health history

Demographic data, which is collected during the orientation phase of the interview; a patients chief complaint, or reason for seeking health care; the history of present illness; allergies; medications; adverse reactions to medications; past medical history; family and social history; and health promotion practices

Heparin-induced thrombocytopenia (HIT)

Development of IgG antibodies against heparin- bound platelet factor 4 (PF4). Antibody-heparin-PF4 complex activates platelets leading to thrombosis and thrombocytopenia.

What is nonmaleficence?

Do No Harm

What causes hypoventilation?

Drug overdose, obesity, COPD, and cervical spine injury.

Identify the different nursing roles (e.g., advocate) that are identified with the following situations: Patients need to be informed about their medications, procedures, diagnostics, and health promotion measures.

Educator

Identify one concept included in the theories from each of the following nurse theorist: Florence Nightingale

Environmental adaptation with appropriate noise, hygiene, light, comfort, socialization, hope, nutrition and conservation of patient energy.

HIPAA

Health Insurance Portability and Accountability Act

What are major components of the EHR?

Health information Diagnostic test results Order-entry system Decision support

What are Erikson's stages of psychosocial development?

Infancy (Birth to 18 months) - Trust vs Mistrust - Feeding - Children develop a sense of trust when caregivers provide reliability, care, and affection. A lack of this will lead to mistrust. Early Childhood (2 to 3 years) - Autonomy vs Shame and Doubt - Toilet Training - Children need to develop a sense of person control over physical skills and a sense of independence. Success leads to feelings of autonomy and failure results in feelings of shame and doubt. Preschool (3 to 5) - Initiative vs Guilt - Exploration - Children need to begin asserting control and power over their environment. Success in this stage leads to a sense of purpose. Children who try to exert too much power experience disapproval, resulting in a sense of guilt. School Age (6-11 years) - Industry vs Inferiority - School - Children need to cope with new social and academic demands. Success leads to a sense of competence, while failure results in feelings of inferiority. Adolescence - Identity vs Role Confusion - Social Relationships - Teens need to develop a sense of self and personal identity. Success leads to an ability to stay true to yourself, while failure leads to role confusion and weak sense of self. Young Adulthood (19 to 40 years) - Intimacy vs Isolation - Relationships - Young adults need to form intimate, loving relationships with other people. Success leads to strong relationships, while failure results in loneliness and isolation. Middle Adulthood (40 to 65) - Generativity vs Stagnation - Work and Parenthood - Adults need to create or nurture things that will outlast them, often by having children or creating a positive change that benefits other people. Success leads to feelings of usefulness and accomplishment, while failure results in shallow involvement in the world. Maturity (65 to death) - Ego integrity vs Despair - Reflection on Life - Older adults need to look back on life and feel a sense of fulfillment. Success at this stage leads to feelings of wisdom, while failure results in regret, bitterness, and despair.

Describe the characteristics that make goals more effective.

Mutually acceptable by the nurse, patient and family. - Appropriate in terms of nursing and medical diagnoses and therapy - Realistic in terms of patient capabilities, time energy and resources. - Specific enough to be understood clearly by patient and other nurses. - Measurable enough to facilitate evaluation

Why is patient advocacy so important in the perioperative setting?

Patients who are going under general anesthesia or sedated are not able to advocate for themselves. Perioperative nurses are the anesthetized patient's representatives to ensure that the patient's needs are being met and that values are being honored.

Delegation

Process of entrusting or transferring the responsibility for certain tasks to other personnel

What is Beneficence?

Promoting the well being of individuals and the public (do good) Ex- doing dental screenings

Is Growth Quantitative or Qualitative?

Quantitative

Ethics

Standards of right and wrong behavior

Philosophy

Statement about the beliefs and values of nursing in relation to a specific phenomenon, such as health

Per Quality and Safety Education for Nurses (QSEN), effective interdisciplinary communication is critical. Which communication and collaborative strategies can be implemented to promote patient safety?

Strategies to promote patient safety include clear and concise written and verbal reporting, regular patient rounds or conferences, and prompt referrals in notification of changes in patient status.

A subjective indication of a disease or a change in condition as perceived by a patient.

Symptom

A unified language classification system

Taxonomy

Type 1 hypersensitivity

The 1st exposure/reaction to a sting may be mild. •Allergic reactions can get worse with each sting. •The next reaction may be more severe or even deadly

Boykin & Schoenhofer

The nursing situation in which the nurse and patient share the lived experiences of caring. By caring for the patient, the nurse is part of the caring, such as feeling better or more positive when the patient demonstrated an improvement in their health status.

How does gender influence goal setting and literacy?

Women have higher health literacy than men. 16% of men and 12% of women had below basic health literacy in most recent National Assessment of Adult Literacy (NAAL) in the US. Time to educate all patients during the goal setting process is valuable for achieving positive outcomes.

What is a living will?

a written statement detailing a person's desires regarding their medical treatment in circumstances in which they are no longer able to express informed consent, especially an advance directive.

The nurse reads a laboratory report indicating that the tissue sample of a patient is essentially neoplastic. How does the nurse interpret this report? a. Cell growth is abnormal and not needed for tissue replacement. b. The tissue specimen shows malignant cell growth. c. The parent cell was abnormal, but new growth is benign. d. Early cell death is inevitable because the morphology is abnormal.

a. Cell growth is abnormal and not needed for tissue replacement.

For women with genetic risk factors for breast cancer, which intervention would address one of the modifiable risk factors? a. Discuss strategies to avoid weight gain and obesity b. Encourage frequent genetic testing for tumors c. Have testing for BRCA1 and BRCA2 gene mutations d. Consider hormone replacement therapy

a. Discuss strategies to avoid weight gain and obesity

The nurse is designing a teaching plan for a patient who had surgery for breast cancer. What information does the nurse include in the plan? Select all a. Do not use lotions or ointments on the area b. Delay using deodorant until healing is complete c. Swelling and redness of the scar itself are normal and permanent d. Report any increased heat or tenderness to the area e. Wear loose pajamas at home for 6-8 weeks f. Begin active ROM exercises 1 week after surgery

a. Do not use lotions or ointments on the area b. Delay using deodorant until healing is complete d. Report any increased heat or tenderness to the area f. Begin active ROM exercises 1 week after surgery

Sickle Cell Anemia

an abnormal cresent shaped RBC

clinical assessment for leukemia

anemia thrombocytopenia enlarged lymph nodes splenomegaly

The American Cancer Society reports that the cancer incidence and survival rate are related to which factors? a. Gender of patient and gender of family caregiver b. Availability of and access to health care services c. Belief that cancer is a chronic disorder d. Age at initiation of lifestyle modification

b. Availability of and access to health care services

Each chemo agent has a specific nadir. What is important for the nurse to do when giving combination therapy? a. Give two agents with similar nadirs b. Avoid giving agents with similar nadirs at the same time c. Watch for first agent's nadir and then give second agent d. Give two agents from different drug classes

b. Avoid giving agents with similar nadirs at the same time

Which areas of the body contain cells that grow throughout the life span? Select all a. Heart b. Hair c. Brain d. Bone marrow e. Skin f. Lining of intestines

b. Hair d. Bone marrow e. Skin f. Lining of intestines

After surgery, a female has been told her breast tumor contains estrogen receptors. What is the clinical significance of this information and how will this type of cancer be treated? a. This is a triple negative breast cancer and additional surgery is the best option b. This type of cancer has a better prognosis and usually responds to hormonal therapy c. This tumor is localized; therefore, radiation therapy should effectively eradicate the cancer d. There are metastases, so long-term survival rate is low; systemic therapy is the only option

b. This type of cancer has a better prognosis and usually responds to hormonal therapy

Which lunch tray represents a diet that would decrease the risk of cancer? a. Plain chicken breast on white bread b. Vegetable plate with a bran muffin c. Grilled cheese sandwich with fruit salad d. Bacon cheeseburger with french fries

b. Vegetable plate with a bran muffin

Which patient with cancer has the greatest risk for infection? a. Recently diagnosed with breast cancer b. With neutropenia from leukemia c. With lung cancer who has a persistent cough d. Diagnosed with prostate cancer 3 years ago

b. With neutropenia from leukemia

Cancer surveillance for high-risk women is used to detect cancer in its early stages and is referred to as what kind of prevention? a. primary b. secondary c. tertiary d. prophylactic

b. secondary

Which woman would be the most likely candidate to consider removal of "at risk" breast tissue? a. Has a family history of breast and colon cancer and eats a high-fat diet b. Has large breasts that make self-examination difficult and smokes cigarettes c. Has mutations in the BRCA1 and BRCA2 genes and sister had breast cancer d. Has mammogram results that suggest an immediate biopsy is needed

c. Has mutations in the BRCA1 and BRCA2 genes and sister had breast cancer

Why does the nurse wear a dosimeter when providing care to a patient receiving brachytherapy? a. Indicates special expertise in radiation therapy b. Protects the nurse from absorbing radiation c. Measures the nurse's exposure to radiation d. Ensures that the radiation dosage is accurate

c. Measures the nurse's exposure to radiation

The nurse hears in shift report that a 32 yo patient had prophylactic oophorectomy. What subjective symptoms does the nurse anticipate that the patient would report? a. Swelling of upper arms b. Tenderness of breasts c. Menopausal symptoms d. Nausea related to medications

c. Menopausal symptoms

A patient is taking oprelvekin. Which assessment data finding indicates that the therapy is working? a. Weight has increased by 2 lbs b. Nausea and vomiting are relieved c. Platelet count is increasing d. Hemoglobin level is normalizing

c. Platelet count is increasing

If a primary tumor is located in a vial organ, what happens? a. Cancer is more likely to spread to other sites b. The organ stops producing normal cells c. There is interference with organ function d. Function of the organ is initially increased

c. There is interference with organ function

Function of hemoglobin

contains Fe , and is carried by RBC, carries O2 and exchanges it for Co2 at capillary level

hypovolemic anemia medical management

control bleeding treat shock O2 elevate lower extremeties and keep warm monitor vital signs

medical management for thrombocytopenia

corticosteroid splenecotmy gama globulin immunosuppressive drugs platelet transfusion avoid trauma

medical management for willibrand diease

cryoperciptate fibrinogen fresh plasma DDAVP avoid trama no asprin!!

The nurse is talking to a young athlete who needs lung removal for treatment of lung cancer. Which statement best indicates that the patient is coping with the uncertainty of cancer and long-term impact on his physical activities? a. "If I delay the surgery, I could still compete for a couple of months" b. "My coach says I might be able to compete even with one lung" c. "Competing in sports is important to me, and eventually I will recover" d. "I love to compete in sports, but I like to do a lot of other things too"

d. "I love to compete in sports, but I like to do a lot of other things too"

Which person has the greatest risk for developing cancer? a. 10 yo African American with allergic asthma b. 32 yo Asian immigrant with low income c. 23 yo white American who has type 1 diabetes d. 62 yo African American who had an organ transplant

d. 62 yo African American who had an organ transplant

Which cancer patient has the highest risk to develop sepsis? a. 34 yo patient who has received high dose radiation to the upper chest area b. 66 yo patient with hypercalcemia and dehydration c. 53 yo patient with small cell lung cancer and hyponatremia d. 82 yo patient with neutropenia and a low grade fever

d. 82 yo patient with neutropenia and a low grade fever

A patient diagnosed with bone cancer reports fatigue, loss of appetite, and constipation. Which lab result does the nurse report immediately? a. Potassium 4.2 b. Magnesium 2.0 c. Sodium 140 d. Calcium 10.5

d. Calcium 10.5

Which biologic process demonstrates that there is a problem with cellular regulation? a. Living cells spend most of their time in G0 state b. Mitosis occurs to replace damaged tissue c. Cyclin activity is balanced by suppressor genes d. Cells continue to divide without contact inhibition

d. Cells continue to divide without contact inhibition

The nurse is counseling a woman who was recently diagnosed with breast cancer. Which factor has the most influence on the choice for her treatment? a. Age at the time of diagnosis b. Overall health status c. Personal choice and self-care capacity d. Extent and location of metastases

d. Extent and location of metastases

Based on the "inverse square law" for radiation exposure, which patient received the smallest radiation dose? a. Received radiation dose at a distance of 0.5 meter b. Received radiation dose at a distance of 1 meter c. Received radiation dose at a distance of 2.5 meters d. Received radiation dose at a distance of 3 meters

d. Received radiation dose at a distance of 3 meters

The nurse is caring for a 56 yo woman who had a modified mastectomy for breast cancer. The woman jokes, "That breast was too saggy anyway. Good riddance to it." Later, the nurse sees the woman crying. What should the nurse do first? a. Encourage the woman to accept body changes by looking at the surgical site. b. Suggest participation in a support group sponsored by the american cancer society c. Invite a breast cancer survivor who successfully coped with mastectomy d. Sit with the woman and encourage her to express her feelings and concerns

d. Sit with the woman and encourage her to express her feelings and concerns

clinical assessment with agranulocytosis

infection ulcerations of mucus membrane bronchial pneumonia UTI

What is Menarche?

initial appearance of menstruation about 2 years after breast development

Leukemia

malligant disorder of the hemopoetic system , excess leukocytes accumulate in the bone marrow and lymph node

What is postconventional morality?

morality of a mature adult; less concerned about how others might see them and more concerned about how they see and judge themselves over the long run

hemophilia A

most common VIII - 80 %

3 kinds of granulocytes

neutrophils esoinophills basophils

malignant lymphoma

non Hodgkin lymphoma tumors usually start lymph nodes and spread to liver , spleen , bone marrow and GI tract

medical management of multiple myeloma

not cureable radiation chemo corticorsteroids IV fluids

Type I hypersensitivity reaction

o IgE mediated o Against environmental antigens (allergens) o IgE finds to Fc receptors on surface of mast cells (cytotropic antibody) o Histamine release - H1 and H2 receptors occurs - Antihistamines in rxn o Manifestations: Itching, urticaria, conjunctivitis, rhinitis, hypotension, bronchospasm, dysrhythmias, GI cramps and malabsorption Ex: Hay fever

medical management for agranulocytosis

remove cause of bone marrow depression prevent or treat infections handwashing strict asepsis

humoral immunity

specific immunity produced by B cells that produce antibodies that circulate in body fluids

Which of the following information is classified by the nurse as subjective data from the patient? A.) "I feel dizzy." B.) There is a red area on the abdomen C.) An oral temperature reading of 99. D.) The hematocrit is less than the expected level.

A

For the patient with a nursing diagnosis of potential for aspiration, the nurse anticipates that there will be goals and interventions related to safety observations during: A.) Eating B.) Bathing C.) Ambulating D.) Transferring

A

On entering the room, the experienced nurse has a sense that the patient's status has changed. The nurse is using the critical thinking skill of? A.) Intution B.) Validation C.) Inference D.) Inductive Reasoning

A

The nurse is ambulating her patient back from the bath when the patient begins to have a seizure. Which of the following should the nurse do first? A.) Lower the patient to the floor if standing B.) Move sharp or hard objects away from the patient C.) Turn the patient to their side to prevent aspiration D.) Attempt to place a tongue blade to prevent choking

A

The nurse is caring for a patient who has severe acid reflux. Which test will allow the physician to directly check for damage to the esophagus? A.) Upper GI endoscopy B.) MRI scan with contrast C.) Abdominal ultrasound D.) PET scan

A

The nurse is caring for a patient who will be undergoing flexible sigmoidoscopy testing to screen for colon cancer. What goal will the nurse include in the patient's plan of care? A.) Patient will verbalize understanding of pre-procedure preparation to be completed at home the day before the test. B.) Patient will feel comfortable about the upcoming test and have trust in the healthcare providers. C.) Patient will learn common side effects of the medications used to prepare the GI tract for endoscopy testing. D.) Patient will realize how important regular sigmoidscopy testing is in the prevention of colon cancer.

A

The nurse is working with a patient to modify his risk factors. Which one of the following areas can be addressed by the nurse? A.) Diet B.) Family history C.) Developmental Level D.) Gender-oriented susceptibility

A

Another nurse does not accurately document that the patient is a allergic to a medication. A.) What is the possible consequence of this omission? B.) What should have been done and when?

A - If the nurse does not document the patient's allergy, the information will not be available to the rest of the healthcare team. The patient could receive a drug and suffer severe adverse reaction. B - The assessment of the patient's drug allergy should have been recorded/reported immediately

What happens at each point of the helping relationship? A.) Orientation phase B.) Working Phase C.) Termination Phase

A - Orientation phase. Making introductions, establishing professional boundaries (formally or informally, as needed) and expectations, and clarifying the role of the nurse. Observing, interviewing, and assessing the patient, followed by validation of perceptions. Identifying the needs and resources of the patient. B - Working Phase. Development of a contractor plan of care to achieve identified patient goals. Implementation of the care plan or contract. Collaborative work among the nurse, patient, and other healthcare providers, as needed. Enhancement of trust and rapport between the nurse and the patient. Reflection by the patient and emotional aspects of illness. He's a therapeutic communication with a nurse to keep interactions focus on the patient. C - Termination Phase - alerting the patient to impending closure of the relationship. Evaluating outcomes of cheap during the interaction. Concluding the relationship and transitioning patient care to another caregiver, as needed.

Which patients would benefit from preoperative teaching about splinting of incisions to minimize discomfort? (Select all that apply.) A.) Patient having coronary bypass graft surgery B.) Patient having open breast biopsy C.) Patient having total hip replacement surgery D.) Patient having lumbar spine decompression surgery E.) Patient having surgery to repair retinal detachment F.) Patient having total abdominal hysterectomy

A and F. Postoperative splinting is done by supporting the abdominal and chest muscles to minimize the pain of coughing and deep breathing after surgery. Patients who have just had heart or abdominal surgery will benefit from splinting. The other surgical procedures do not affect the chest or abdomen, and these patients would not benefit from teaching about splinting.

Which nursing activity is found in a tertiary healthcare environment? A.) Working the triage desk in the emergency department B.) Administering influenza immunizations at the senior independent-living facility C.)Admitting a patient following open heart surgery to the cardiovascular intensive care unit D.) Providing well-baby care in the clinic run by the local community health department

C

A nurse is educating women on breast cancer risk reduction. What topics does the nurse include in the presentation? (Select all that apply.) A.) Exercise B.) Limiting alcohol C.) Low-fat diet D.)Breast self-exams E.) Milk intake

A, B, and C. Reducing breast cancer risk can be accomplished by getting regular exercise, limiting alcohol, and eating a low-fat diet. Performing breast self-examinations will not reduce risk but may help women find abnormalities early. Milk intake is not related.

What is the difference between a conceptual model and a theory?

A model is a collection of interrelated concepts that provides direction for nursing practice, research, and education. A nursing theory represents a group of concepts that can be tested in practice and can be derived from a conceptual model.

Which of the following actions can be safely delegated to an unlicensed assistive personnel? A.) Assessment of a patient who just had surgery B.) Determination of the patient's level of stability when using crutches C.) Measurement of vital signs on a stable patient D.) Provision of analgesic medications

C

Which of the following actions is considered indirect patient care? A.) Providing assistance with ambulation B.) Starting an IV line C.) Making a change-of-shift report D.) Teaching the patient about the prescribed medications

C

The nurse explains to the patient that which services will be covered under Medicare? (Select all that apply.) A.) Infusion therapy B.) Ostomy management C.) Renal dialysis D.) Chemotherapy E.) Grocery shopping

A, B, C, and D. Medicare will reimburse for professionally rendered services provided by a licensed health care provider. Grocery shopping would not be covered. If homemaker services are provided to a patient also receiving skilled care, then they too are reimbursed.

A nurse educator is delivering a lecture on "nursing as a profession" to a group of nursing students who have recently joined the baccalaureate nursing degree course. Which statements are true? Select all that apply. A.) "Care delivery by the nurses is only based on the orders given ny healthcare providers." B.) "Nurses have to follow a specific code of ethics while delivering care." C.) "A Nurse is responsible and accountable to her patients." D.) "Nursing education involves learning the care giving techniques without any theoretical body of knowledge." E.) "A nurse is responsible to provide specific health care to the patients."

A, B and E

The nurse is admitting a patient to the general medical-surgical unit. What should the nurse assess as part of a routine sleep assessment? (Select all that apply.) A.) Usual sleeping and waking times B.) Bedtime routines C.) Sleeping environment preferences D.) Medications used for sleep E.) Any current life events

A, B, C, D, and E. A sleep assessment should be completed when a patient is admitted to a health facility. The nurse assesses the patient's usual sleeping and waking times, medications, illnesses, bedtime routines, and sleeping environment preferences and incorporates the information into the plan of care when possible. The nurse should also assess current life events and emotional status.

The nurse knows which items are included in the documentation for a patient on fall precautions? (Select all that apply.) A.) History of any falls B.) Falls risk assessment scores C.) Patient and family education D.) Use of assist devices E.) Any fall or reported fall

A, B, C, D, and E. The nurse should document the general assessment, include the patient's medical history, subjective and objective data, medication review, musculoskeletal status, and history of falls. Falls assessment and reassessment, patient family education and use of assist devices are also documented. Thoroughly document a fall or reported fall.

The nurse recognizes that the cause of pressure ulcers includes which factors? (Select all that apply.) A.) Intensity of the pressure B.) Duration of the pressure C.) Tissue's ability to tolerate the pressure D.) Person's age E.) Person's nutritional status

A, B, C, D, and E. The primary cause of pressure ulcers is, as the name suggests, pressure. However, it is more than just pressure; it is the intensity of the pressure, the length of time that the tissue is subjected to the pressure, and intrinsic and extrinsic factors that affect the tissue's ability to withstand or tolerate that pressure. Intrinsic and extrinsic factors can include nutrition status and age.

Provide examples for diversity considerations for the following areas. A.) Life Span B.) Culture, ethnicity, and religion C.) Disability

A.) Families and cultures have attitudes about what and how they eat that they transmit in the form of values. Some of the strategies include limiting the purchase of unhealthy foods, involving children in shopping and mean preparation, and engaging children in ongoing conversation about healthful eating and the value of weight control. B.) In some cultures, parents may arrange marriages for their children. Pharmaceutical treatment may be rejected by individuals from other cultures on the basis of traditional beliefs and values. Exploring the implementation of alternative or complementary therapies may help to meet patient needs by demonstrating respect. C.) People with disabilities note that a problem is often the feeling of being excluded by society. Nurses demonstrate respect for individuals with disabilities by including them in their care as much as possible and seeking to understand what works best for the individual rather than generalizing treatment modalities.

You have been assigned to the same patient each day you have worked during the past week. You have developed a good relationship with the patient. When you walk into the room to tell the patient that you will once again caring for her during this shift, the patient does not seem to be her normal self. You ask the patient if she's okay. A.) What type of caring process or behavior are you demonstrating? B.) What will you do next in this situation?

A.) Knowing-Assessing B.) Find more out about how the patient is feeling and whether she is having specific difficulty (pain, emotional issues.)

What does the nurse consider when writing a health-promotion nursing diagnosis? A.) The response to a current need B.) The desire for lifestyle changes C.) The chance of complications D.)The patient's current need

B

Which statement by the nurse indicates comprehension of ethical issues? A.) Ethical issues are rare occurrences but take a great deal of time to resolve. B.) Ethical issues have required The Joint Commission to mandate ethics committees. C.) Ethical issues most frequently lead to legal intervention in patient care matters. D.) Ethical issues lead to ethics committees made up entirely by nurses.

All nurses are faced with ethical decisions each day in practice, and some choose to obtain further education and experience in the field of bioethics and participate on institutional ethics committees along with physicians, ethicists, attorneys, and academicians. Ethics committees are required by The Joint Commission to respond to ethical challenges related to patient care requiring consultation. The work of the ethics committees in health care institutions helps to prevent unnecessary legal intervention in patient care matters. Ethics committee members come from all areas of health care, not just nursing. If acceptable resolutions are not achieved through consultation with the ethics committee, patients, families, and health care providers, the legal system may become involved.

Which of the following nursing diagnoses best meets the criteria for a diagnostic statement? A.) Potential for diarrhea associated with the possible side effect of antibiotic therapy B.) Potential for heart disease with the risk factor of smoking C.) Potential for urinary retention following catheter removal D.) Potential for pneumonia

B

Which one of the following is the main goal of Healthy People 2020? A.) Health care cost reduction B.) Access to health services C.) Elimination of intrinsic risk factors D.) Identification of morbidity rate thresholds

B

You are caring for an 80-year-old woman, and you ask her a question while you are across the room washing your hands. She does not answer. What is your next action? A.) Repeat the question in a loud voice, speaking very slowly B.) Move to her bedside, get her attention, and repeat the question while facing her C.)Bring her a communication board so she can express her needs D.) Leave the room quietly since she evidently does not want to be bothered right now

B

The nurse and UAP are making an occupied bed together. Which action by the nurse is incorrect? A.) The nurse asks and assists the patient to turn toward the UAP and loosens the fitted sheets and rolls it toward the patient. B.) The nurse rolls linens to the side then places the linens on the floor while finishing. C.) The nurse tucks the clean bottom sheet under the cleaner underside of the dirty linens. D.) The nurse wears gloves to remove dirty linens.

B

The nurse appropriately delegates care to the UAP when she: A.) Instructs the UAP to assess the patient's skin during a bath B.) Instructs the UAP to reposition the patient using a trapeze C.) Instructs the UAP to assess the patient's ability to perform ROM exercises D.) Instructs the UAP to notify the health care provider of any changes.

B

The nurse is caring for a confused, combative patient. Which action would be considered last by the nurse to control behavior of the patient? A.) Orient the patient frequently B.) Apply restraints C.) Move the patient to a room close to the nurse's station D.) Encourage the family to spend time with the patient

B

The nurse is caring for a diabetic patient who will be doing fingerstick blood glucose testing at home. What is the best way for the nurse to ensure that the patient can perform the procedure correctly? A.) Quiz the patient on the steps of the procedure B.) Have the patient perform the procedure in front of the nurse C.) Ask the patient if he has any questions about the test D.) Use termniology that the patient can easily understand

B

The nurse is providing education to a patient about the difference between a simple and complex carbohydrates. Which statement by the patient indicates a need for further education? A.) "Simple carbohydrates give me quick energy." B.) "Complex carbohydrates come from fruit." C.) "Complex carbohydrates take longer to break down." D.) "Simple carbohydrates come from milk products."

B

The nurse is to administer 15 mg of morphine liquid to the patient. How much morphine liquid will the nurse draw up to administer to the patient? Morphine sulfate oral solution (CONCENTRATE)100 mg/5 mL(20 mg/mL)CII only A.) 0.5 mL B.) 0.75 mL C.) 1.3 mL D.) 1.5 mL

B

The nurse is working with a student nurse to teach her about restraint use in patients. Which statement by the student nurse indicates a learning need regarding restraints? A.) "Having all 4 side rails up on the bed is considered a restraint." B.) "The use of restraints has been shown to decrease fall-related injuries." C.) "Death has been associated with the use of restraints." D.) "Medications administered to control behavior are considered a chemical restraint."

B

The nurse manager knows a Magnet hospital is characterized by which? (Select all that apply.) A.) Excellent medical outcomes B.) A high level of nursing job satisfaction C.) A low number of grievances D.) Nursing care leading excellent patient outcomes E.) Evidence-based environment support

B, D, and E. A Magnet hospital is characterized by excellent patient outcomes resulting from nursing, a high level of nursing job satisfaction with a low nurse turnover rate, and appropriate resolution of any grievances. The Magnet Recognition Program supports an evidence-based environment, which includes the nurses' autonomy to improve quality of care through research utilization (ANCC, 2018). Research and EBP must, therefore, become a part of the nurses' care of the patients.

In reviewing the history of nursing, advances in health care and the role of nurses have been associated with which of the following? A.) Weather disasters B.) Military Conflicts C.) Women's rights movements D.) Economic growth periods

B.

A nurse is asked to perform a complex intervention for a patient. However, the nurse is unable to perform it successfully. What would be the appropriate action by the nurse? A.) Continue until the intervention is successfully completed. B.) Modify the intervention to make it easy to perform. C.) Ask for assistance from a senior nurse D.) Tell the patient about the inability to perform it.

C

An appropriate goal for the patient who is postoperative day one from abdominal surgery and on bed rest with the nursing diagnosis Impaired Skin Integrity is: A.) The patient will ambulate twice a day B.) The patient will eat 50% of meals C.) The patient will have no further skin breakdown D.) The patient will interact with others

C

Excessively dry skin can lead to cracks and openings in the integumentary system. Based on this, what is the most applicable nursing diagnosis for a patient with excessively dry skin? A.) Imbalanced Nutrition: Less than body requirements B.) Deficient fluid volume C.) Risk for infection D.) Acute pain

C

For the nursing diagnosis Altered Speech Associated with recent neurological disturbances as seen by an inability to speak coherently, the cause is attributed to? A.) Altered Speech B.) As evidenced C.) Recent neurological disturbances D.) Inability to speak coherently.

C

For the patient with the nursing diagnosis Activity Intolerance, the nurse expects that the patient will specifically demonstrate: A.) Elevated body temperature B.) Disinterest in diversional activties C.) Dyspnea on exertion D.) Erythema

C

In Rosenstock's Health Belief Model, focus is placed on which of the following? A.) Basic human survival needs B.) Functioning of the individual in all dimensions C.) Relationship of perceptions and compliance D.) Multidimensional nature of patients in the environment

C

In the working phase of the helping relationship, the nurse expects to: A.) Observe and assess the patient B.) Establish professional role boundaries C.) Implement the care plan or contact D.) Evaluate the outcomes achieved.

C

The nurse is caring for a patient who is NPO with a new PEG (percutaneous endoscopic gastrostomy) tube. Which of the patient's medications can the nurse administer through the tube? (Select all that apply.) A.) Zolpidem tartrate (Edluar) sublingual tablet 5 mg nightly at bedtime B.) Ondansetron (Zofran) oral disintegrating tablet 8 mg q 8 hours PRN nausea C.) Cefaclor (Ceclor) for oral suspension 250 mg q 6 hours D.) Oxymorphone hydrochloride extended release (Opana ER) 40 mg q 12 hours E.) Phenytoin (Dilantin) chewable tablet 100 mg q 12 hours F.) Potassium chloride oral solution 20 mEq daily

C, E, and F. Extended-release, oral disintegrating, and sublingual tablets may not be administered through feeding tubes. Suspensions and oral solutions are ideal for feeding tube administration. Chewable tablets may be crushed and dissolved in liquid for administration through feeding tubes.

The nurse is caring for a patient with advanced COPD who reports feeling short of breath. The nurse notes that the patient's lung sounds are diminished bilaterally and the patient's pulse oximetry is 91% on 2 L/min oxygen via nasal cannula. What actions will the nurse take to make the patient more comfortable? (Select all that apply.) A.) Increase the patient's oxygen to 4 L/min via nasal cannula. B.) Suction the patient's airway using sterile technique. C.) Maintain eye contact and provide calm reassurance. D.) Turn the patient onto the side for postural drainage. E.) Administer the ordered nebulized bronchodilator. F.) Elevate the head of the patient's bed to fully upright.

C, E, and F. Patients who are acutely short of breath due to advanced COPD will benefit from nebulized bronchodilator medication to open the airways. Elevating the head of the bed will prevent pressure on the diaphragm from the abdominal contents. A caring demeanor with eye contact will help the patient remain calm until the medication begins to work and the shortness of breath is eased. Patients with COPD should be kept on low-flow oxygen to maintain pulse oximetry of more than 90%.

The nurse has delegated the hygienic care of a patient to the nursing assistant. It is important that the assistant treats the patient in a caring manner. Additional instruction is required if the nurse observes the nursing assistant doing which of the following? A.) Asking the patient what preferences he has for personal care. B.) Having the patient assist with parts of the bath as able. C.) Leaving the patient uncovered while the bathwater is discarded D.) Telling the patient that she will return shortly with new linens.

C.

The nurse knows an appropriate goal for a patient with a stage III pressure ulcer with the nursing diagnosis Impaired Mobility is: A.) The patient will remain free of wound infections during the hospitalization B.) The patient will report pain management strategies and reduce pain to a tolerable level. C.) The patient will turn self in bed using over trapeze every two hours using assistance when needed D.) The patient will consume nutrition to meet nutritional requirements within 1 week.

C.

The nurse is working with a diabetic patient, and is attempting to teach psychomotor skills. This is occuring when the nurse has the patient: A.) Verbally describe his feelings about diabetes. B.) Answer 3 of 5 true-or-false questions about diabetes. C.) Identify 3 positive lifestyle changes to manage blood sugar. D.) Draw up and self-inject insulin correctly.

D

The patient has a nursing diagnosis of risk for falls. Which goal is most important? A.) Patient will ambulate twice a day B.) Patient will no symptoms of infection C.) Patient will perform activities of daily living D.) Patient will have no injuries during hospital stay

D

The nurse is acting in the planning function as a manager. The nurse knows which stage should be completed first? A.) Set the plan. B.) Assess the situation and future trends. C.) Convert plan into action statement. D.) Set the goals.

D The planning function of a manager is comparable to the assessment, diagnosis, and planning portions of the nursing process. It includes four stages: (1) setting goals, (2) assessing the current situation and future trends, (3) setting the plan, and (4) converting the plan into an action statement.

The nurse is providing discharge instructions to the parents of a toddler about sleeping habits. Which statement indicates further education is needed? A.) "Sleep needs may change during growth spurts." B.) "Children sleep 12 hours a day." C.) "Toddlers will often resist going to bed." D.) "The bedtime routine can vary."

D The regular bedtime routine should be consistently followed. Children need 11 to 14 hours of sleep a day, and toddlers and preschoolers may exhibit resistance to going to bed. Sleep needs fluctuate with growth spurts.

A nurse is told in hand-off report that a patient opens eye spontaneously, is confused but able to answer questions, and demonstrates purposeful movement to painful stimuli. What does the nurse calculate the patient's Glasgow Coma Scale to be? A.) 7 B.) 9 C.) 11 D.) 13

D This patient's eye opening would be scored 4, verbal response would be 4, and motor response would be scored at 5; this equals a score of 13.

The nurse is preparing to perform suctioning on a new tracheostomy with the potential for forceful expulsion of secretions and identifies what PPE (personal protective equipment) should be worn? A.) Gloves and eyewear B.) Gloves, gown, and mask C.) Eyewear and gown D.) Eyewear, mask, gown, and gloves

D Use gloves routinely when blood or body fluid might be present. If splashing is possible, use your nursing judgment about what other PPE might be necessary. Forceful expulsion of secretions would require all PPE—gown, mask, eyewear, and gloves—to provide adequate protection.

Clustered supporting data

Defining characteristics

You have performed an assessment on the patient and found the following: The patient has an active social life, spending time with friends after work. His caloric intake is appropriate for his size. He has no difficulty with breathing and no cardiovascular symptoms. His vision and hearing are active. - Organize the data to functional health patterns.

Health perception and health management - Active social life; spending time with friends Nutrition and metabolism - Caloric intake is appropriate for his size Activity and Exercise - No difficulty breathing; No cardiovascular symptoms Cognition and Perception - Vision and hearing are acute

Includes initiating specific nursing interventions and treatments

Implementation

What actions are involved in implementing the nursing process?

Implementation consist of both the performance of a nursing task and the documentation of the intervention.

What are the different methods that may be used by the nurse for patient assessment?

Observation, Patient Interview including the completion of a health history and review of systems; and physical examination

The nurse observes the patient is unable to independently move to the examination table. What action should be taken by the nurse?

Obtain assistance from a co-worker to assist the patient to move to the examination table safely

What is the primary purpose of a State's Nurse Practice Act?

Provide a scope of practice defined by each state or jurisdiction and are the legal limits of nursing practice.

Indicate the standard of care being demonstrated in practice or performance for each of the following: Care is taken by the nurse to open and use only the supplies necessary for the dressing change.

Resource utilization

When discussing stage 3 pressure ulcers with the student nurse, which description would the staff nurse include? A.) A pressure ulcer that involves exposure of bone and connective tissue. B.) A pressure ulcer that does not extend through the fascia. C.) A pressure ulcer that does not include tunneling. D.) A partial-thick wound that involves the epidermis.

Stage 3 pressure ulcers are full-thickness wounds that extend into the subcutaneous tissue but do not extend through the fascia to muscle, bone, or connective tissue. There may be undermining or tunneling present in the wound. Stage 4 pressure ulcers involve exposure of muscle, bone, or connective tissue such as tendons or cartilage. Stage 2 pressure ulcers are partial-thickness wounds that involve the epidermis and/or dermis.

Identify the general concepts that are addressed in a conceptual model for nursing.

The concepts are optimal functioning of the person, or patient, how people interact with the environment, illness, and health promotion, and nursing's role.

What are the essential components of professional nursing communication?

The essential components of professional nursing communication are respect, assertiveness, collaboration, delegation and advocacy.

Identify one concept included in the theories from each of the following theorist: Orem

The goal of nursing care is to help patients perform self-care by increasing their independence.

How is health care access a challenge to people in rural areas?

Time needed to travel to the nearest health care provider or facility, delaying treatment for treatable diseases before they become severe, life threatening systematic infections that require hospitalizations and IV drug therapy. It's not uncommon to address multiple problems of differing levels of severity that have been saved up for the trip to a distant provider.

SLE drug therapy

Topical cortisone drugs Plaquenil (antimalerials = hydroxychloroquine) Tylenol or NSAIDs Chronic steroid therapy Methotrexate Immune Suppressives (lophosphamide, mycophenolate mofetil [CellCept])

Wrongs committed against another person that do not involve a contract

Tort

What theories happen in Infancy (Up to 1 year)

Trust vs Mistrust (Erikson) Sensorimotor (Piaget) Oral (Freud) First 3 months - Recliners (Sleeps 20 hours a day) 3-6 months - Sitters 6-9 months - Bouncers, crawlers 9-12 months - Crawler, Cruiser

Thrombocytopenia

a condition in which there is an abnormally small number of platelets circulating in the blood

A patient is lying in bed after a mastectomy. How does the nurse position the patient? a. Head of the bed up at least 30* with the affected arm elevated on a pillow b. Supine body position with the affected arm positioned straight by the side c. Any position that is the most comfortable to the patient d. Side-lying position with the unaffected side down toward the mattress

a. Head of the bed up at least 30* with the affected arm elevated on a pillow

The nurse hears in report that the patient has xerostomia. Which teaching point does the nurse plan to review with the patient? a. Regular dental visits are essential because of increased risk for dental caries b. Use mild soap and apply unscented moisturizers to reduce itching sensation c. Avoid rigorous sports because bones are more prone to pathologic fractures d. Avoid direct sun exposure for at least 1 year because skin will be sensitive

a. Regular dental visits are essential because of increased risk for dental caries

what is a hemocytoblast?

all blood cells develop from this stem cell

What happens during a differential wbc count?

different kinds of wbc are counted and reported as a percentage of total examined

medical management of lymphedema

diuretic antibiotics compression pump elastic stocking restricted sodium avoid contrictive clotting skin care

clinical assesments for hodgkins disease

enlargement of lymph nodes anorexia weight loss prutius low grade fever anemia WBC

SLE symptoms

fatigue (80-100%) joint aches (hands) and myalgias (90%) eye: keratoconjunctivitis sicca butterfly rash painless oral and nasal ulcers (12-45%) renal involvement (50%) leukopenia anemia thrombocytopenia peptic ulcers (any organ can be affected)

3 possible causes of hypovolemic anemia

hemorage surgical problems GI bleeding

Hemophilla

hereadtary coagulation disorder by a disturbance of clotting factor

Sickle cell gene?

homozygous

Hypovolemic Anemia?

large amount of blood are loss, means there is a decreasing RBC

Severe range of neutrophils **medical emergency**

less than 500 Temps >100.4 (38C)

clinical assessment lymphoma

painless lymph nodes fever weight loss anemia prutius fatigue malaise

clinical assessment for thrombocytopenia

petechiae ecchymoses platelets below 100000 bleeding from mucus membranes

medical management of aplastic anemia

platelet transfusion steroids or androgens bone marrow transplant

lymphedema

primary or secondary disorder, accumulation of lymph in the soft tissue

medical management for lymphoma

radiation chemo bone marrow transplant tumor necrosis factor

reticulocyte is an immature form of

rbc


संबंधित स्टडी सेट्स

PSM 1 - SCRUM, Scrum Master Certified, Scrum Master training + Prepare for the PSM I® certification, SCRUM PSM I Assessment, PSD1, Scrum, Scrum, PSM I Assessment, PSM I Assessment, Scrum, PSM I Assessment, PSM I Assessment

View Set

Chapter 21: The Medical Assistant as Clinic Manager

View Set

Rate of Change and Intro to Slope

View Set

Spelling 5de leerjaar afspraakstukjes [d/t verlengingsregel] en onthoudstukjes [c (als k)], [c (als s)], [Franse a] & [i] WP9

View Set

3/7 to 3/8 我的课程My Courses

View Set